Download as pdf or txt
Download as pdf or txt
You are on page 1of 262

454 Questions

Que. 1 Consider the following statements:


Assertion: The Hindustan Socialist Republican Association (HSRA) was formed to overthrow British rule in India through violent
revolution.
Reason: HSRA believed that armed struggle was the only way to achieve complete independence from British domination.

1. Both A and R are individually true and R is the correct explanation of A.


2. Both A and R are individually true but R is not the correct explanation of A.
3. A is true but R is false.
4. A is false but R is true.

Correct Option - 1
The correct answer is Option 1.
Key Points
HSRA:
The Hindustan Socialist Republican Association (HSRA) was formed in 1928 by revolutionaries like Bhagat Singh and Chandra Shekhar
Azad.It was a successor to the Hindustan Republican Association (HRA), founded in 1924, with a stronger emphasis on socialist
ideals alongside armed revolution.

HSRA, founded in 1928, was a revolutionary organization that aimed to achieve Indian independence through armed struggle.
Statement 1 (Assertion) is correct
HSRA believed that non-violent methods were insufficient to overthrow British rule. They saw violent revolution as the only path to
complete freedom. Statement 2 (Reason) is correct and explains Statement 1

Therefore, both Assertion (A) and Reason (R) are true, and Reason (R) correctly explains Assertion (A)
Additional Information
The HSRA aimed to overthrow British rule in India through armed struggle and establish a socialist republic. HSRA's activities
exposed the economic exploitation of India under British rule.

They carried out daring acts like the Kakori Train Robbery (1925) and the Central Assembly Bombing (1929) to cripple British
authority and inspire others. These actions, though controversial, kept the fight for freedom alive and challenged British power.
While Mahatma Gandhi's non-violent movement dominated the freedom struggle, HSRA presented a radical alternative. Their
violent methods sparked debate, forcing Indians to consider different paths to achieving freedom.
HSRA primarily comprised of young men from middle and upper classes, limiting their connection with the masses.
After the execution of Bhagat Singh and other key members in 1931, HSRA's influence faded.
Though short-lived, HSRA's legacy lives on. Their militant spirit influenced future revolutionary movements in India. Additionally,
their emphasis on social equality resonated with socialist thinkers and political parties later in Indian history.

Que. 2
Consider the following statements

1.The Swadeshi Movement primarily focused on boycotting British goods and promoting self-reliance within India.

2: The movement resulted in the decline of indigenous industries due to competition from cheaper foreign goods

3: The National Council of Education was established during the Swadeshi Movement to promote a national education system.

How many of the statements given above is/are correct?

1. One only
2. Two only
3. All three
4. None

Correct Option - 2
The correct answer is Option 2
Key Points
Swadeshi Movement :
The Swadeshi Movement, sparked by the partition of Bengal in 1905, aimed at a self-reliant India. People boycotted British goods,
especially textiles like Manchester cloth.

1/262
Swadeshi (meaning "of one's own country") aimed to encourage the use of Indian-made goods and reduce dependence on British
imports. Hence, Statement 1 is correct.
While the Swadeshi Movement faced challenges due to established British industries, its goal was to revive indigenous industries,
not lead to their decline. Hence, Statement 2 is incorrect.
The National Council of Education, established in 1906, aimed to create an alternative education system independent of British
control. Hence, Statement 3 is correct.

Additional Information

Swadeshi encouraged the revival of indigenous industries, particularly handloom weaving. Handloom production rose sharply, with
estimates suggesting a 20% increase between 1905 and 1908.
The movement transcended economics, fostering a strong sense of national consciousness.
Women played a significant role in the Swadeshi Movement. They actively participated in boycotts, picketing shops selling foreign
goods, and promoting the use of khadi (handspun cloth).
In response to the Swadeshi Movement and the growing desire for an independent identity, the National Council of Education (NCE)
was established in 1906. It aimed to create an alternative education system free from colonial control.
The NCE curriculum emphasized Indian history, culture, and values, fostering a sense of national identity among students. This
contrasted with the British curriculum that focused on British history and instilled a sense of loyalty to the empire.
Though short-lived, the NCE served as a significant initiative. It highlighted the need for an indigenous education system and inspired
future efforts to reform Indian education in the context of nationalism

Que. 3 Consider the following statements

Statement 1: The Champaran Satyagraha was the first major Satyagraha movement led by Mahatma Gandhi in India.

Statement 2: The movement aimed to address the unfair treatment of indigo farmers by British planters in the Champaran district of Bihar.

1. Both Statement 1 and Statement 2 are individually true and Statement 2 is the correct explanation of Statement 1.
2. Both Statement 1 and Statement 2 are individually true but Statement 2 is not the correct explanation of Statement 1.
3. Statement 1 is true but Statement 2 is false
4. Statement 1 is false but Statement 2 is true

Correct Option - 2
The correct answer is Option 2
Key Points
Champaran Satyagraha: The Champaran Satyagraha in 1917 marked the first major Satyagraha movement led by Mahatma Gandhi in
India. This non-violent resistance campaign became a defining tactic in the fight for independence.

The Champaran Satyagraha in 1917 marked the first major Satyagraha (non-violent resistance) movement led by Mahatma Gandhi
in India.Statement 1 (Assertion) is true.
The movement specifically addressed the Tinkathia system, where peasants were forced to cultivate indigo on a specific portion of
their land at low prices. This exploitative system by British planters led to immense hardship for the farmers in Champaran.Statement
2 (Reason) is true, but it doesn't explain Statement 1.

Additional Information
The movement aimed to address the plight of indigo farmers in the Champaran district of Bihar. British planters enforced the
exploitative Tinkathia system, forcing farmers to cultivate indigo, a cash crop, on a portion of their land at meager prices. This system
resulted in economic hardship for the farmers.
Gandhi led a fact-finding mission, documented the farmers' grievances, and demanded an official inquiry. The movement's pressure led
to a government commission being formed to investigate the Tinkathia system. Based on the findings, the system was abolished, and
compensation was provided to the farmers.
The movement showcased Gandhi's leadership and his philosophy of Satyagraha. His methods of non-violent protest, non-
cooperation, and civil disobedience inspired future movements across India.
The Champaran Satyagraha marked a shift towards mass mobilization. Gandhi successfully mobilized the local population to
participate in the movement, demonstrating the power of collective action. This success fueled further Satyagraha movements in the
years to come.

Que. 4 Which of the following statements is/are true regarding Particularly Vulnerable Tribal Groups (PVTGs) in India?

Statement I: PVTGs are identified based on specific criteria such as pre-agricultural level of technology, stagnant or declining population, low
literacy rates, and a subsistence level of the economy.

Statement II: The Ministry of Tribal Affairs is responsible for the welfare and development of PVTGs in India.

Statement III: As of 2022, there are 75 PVTGs identified across various states in India.

2/262
1. One only
2. Two only
3. All three
4. None

Correct Option - 3
The correct answer is option 3.
Key Points
PVTG's
PVTGs are identified based on specific criteria such as pre-agricultural level of technology, stagnant or declining population, low
literacy rates, and a subsistence level of the economy. These criteria help in identifying the most vulnerable tribal groups in need of
special attention and support.
The Ministry of Tribal Affairs is indeed responsible for the welfare and development of PVTGs in India. It formulates various
schemes and programs aimed at their upliftment and protection.
As of 2022, there are indeed 75 PVTGs identified across various states in India. These groups are spread across different regions, each
facing unique challenges that require tailored interventions for their upliftment and preservation of their cultural heritage.
Some examples of PVTGs in India include the Jarawa tribes of Andaman and Nicobar Islands, the Sentinelese people of the Sentinel
Islands (known for their minimal contact with the outside world), and the Gujjar-Bakerwal community spread across the Himalayas.
Development projects like dams and mining often lead to the displacement of PVTGs from their ancestral lands, impacting their
traditional way of life and livelihood.
The Scheduled Tribes and Other Traditional Forest Dwellers (Rights Recognition) Act, 2006: This act aims to recognize and
protect the rights of forest-dwelling communities, including PVTGs, over their habitat and traditional resources.

Que. 5 Which of the following statements regarding the Lucknow Pact of 1916 is incorrect?

1. The Lucknow Pact was a significant event in the Indian nationalist movement, marking a rare instance of Hindu-Muslim unity
2. It was signed between the Indian National Congress and the All India Muslim League.
3. The pact demanded separate electorates for Muslims, along with reserved seats in the provincial legislatures.
4. The Lucknow Pact addressed the issue of separate electorates for other religious minorities.

Correct Option - 4
The correct answer is Option 4.
Key Points
Lucknow Pact

The Lucknow Pact of 1916 was a crucial event in the Indian freedom struggle, as it witnessed the coming together of the
Indian National Congress and the All India Muslim League, setting aside their differences and presenting a united front
against British colonial rule.

The pact also included a weightage system, granting additional seats to Muslims in some provinces like Punjab and Bengal, even
though they formed a minority of the population. This aimed to further address Muslim anxieties about political power.
One of the key provisions of the Lucknow Pact was the demand for separate electorates for Muslims. This meant that Muslim voters
would elect their representatives separately from other communities. Additionally, the pact called for reserved seats for Muslims in the
provincial legislatures based on their population size.
While the Lucknow Pact primarily focused on addressing the political demands of the Muslim community, it did not completely
ignore the interests of other religious minorities. However, it did not extend the provision of separate electorates to other religious
groups such as Sikhs or Christians.
Though the pact fostered temporary unity, it also sowed the seeds of future discord. The concept of separate electorates solidified
religious divisions and hindered efforts to build a truly unified national movement.

Que. 6 Consider the following statements

Statement 1: The Standstill Agreements were signed between the British Raj and the princely states of India to ensure a smooth transition of
power after independence.

Statement 2: These agreements aimed to maintain continuity in essential services like trade and communication, and allowed the princely
states to remain independent after 1947.

1. Both Statement 1 and Statement 2 are individually true and Statement 2 is Correct explanation of Statement 1.
2. Both Statement 1 and Statement 2 are individually true but Statement 2 is not the correct explanation of Statement 1.
3. Statement 1 is true but Statement 2 is false.
4. Statement 1 is false but Statement 2 is true.
3/262
Correct Option - 3
The correct answer is Option 3.
Key Points

The Standstill Agreements were signed between the newly formed dominions of India and Pakistan and the princely states of
British India, not with the British Raj. These agreements aimed to maintain a smooth transition of power and avoid disruption in the
administration of princely states. Hence, Statement 1 is true.
The agreements focused on maintaining continuity in essential services like trade and communication during a short period (typically
one year). They did not grant the princely states permanent independence. After the Standstill Agreement period, the states were
expected to decide on their accession to India, Pakistan, or remain independent. Hence, Statement 2 is false.

Additional Information

The most well-known example is the Standstill Agreement signed between the Indian government and the Nizam of Hyderabad in
1947. This agreement eventually broke down, leading to Indian military action and the annexation of Hyderabad in 1948.
The Standstill Agreements played a crucial role in the immediate aftermath of independence, ensuring some degree of stability and
continuity.
However, they also contributed to the complexities of the accession process and, in some cases, led to conflicts between princely
states and the newly formed dominions.

Que. 7 Consider the following statements

Statement 1: The Desai Liaqat Pact, aimed to establish a coalition government in future independent India, representing both Hindus and
Muslims

Statement 2: The pact aimed to address the issues of Hindu-Muslim unity and constitutional reforms in British India.

Statement 3: The Pact proposed the creation of a separate Muslim nation within a united India.

How many of the statements given above are correct?

1. One only
2. Two only
3. All three
4. None

Correct Option - 2
The correct answer is Option 2.
Key Points
Desai-Liaqat Pact :The Desai-Liaqat Plan, also known as the Desai-Liaqat Pact, was a proposal formulated in 1945 to bridge the growing
divide between the Indian National Congress and the All-India Muslim League during the Indian independence movement.

The Desai-Liaquat Pact, also known as the Delhi Proposals, was an attempt to form a power-sharing agreement between the Indian
National Congress and the All-India Muslim League. Hence, Statement 1 is correct.
The proposal aimed to address the ongoing friction between Hindus and Muslims in British India. A united government with fair
representation was seen as a way to bridge the gap and promote Hindu-Muslim unity. Hence , statement 2 is correct.
The Desai-Liaquat Pact did not propose the creation of a separate Muslim nation within a united India. This concept gained traction
with the Muslim League under Muhammad Ali Jinnah, ultimately leading to the partition of India and the creation of Pakistan. Hence,
statement 3 is incorrect

Additional Information

The plan aimed to forge a coalition government for an independent India that would represent both Hindus and Muslims. This was a
crucial attempt to address the intensifying Muslim anxieties about their political and social standing in a future independent India.
The plan represented an attempt at a negotiated settlement to address the political deadlock between Hindus and Muslims.
The plan offered significant concessions to Muslims. It proposed granting them one-third representation in the central government
council, exceeding their population percentage at the time.
The primary aim of the Desi Liaqat Pact was to address the communal issues between Hindus and Muslims, focusing on achieving
Hindu-Muslim unity. However, it did not directly concern constitutional reforms in British India
Despite its intentions, the plan faced opposition within both parties. The Congress balked at the high level of Muslim representation,
while some Muslim League members continued to advocate for a separate Muslim state. Ultimately, the plan was never formally
endorsed and failed to gain widespread acceptance.

4/262
The pact never proposed the creation of a separate Muslim nation within a united India.

Que. 8 The Super-pressure Balloon-borne Imaging Telescope (SuperBIT) is a balloon-borne telescope designed for astronomical
observations. What is the primary scientific goal of SuperBIT?

1. Studying the life cycle of stars.


2. Mapping the distribution of dark matter in the universe.
3. Observing the atmospheres of exoplanets.
4. Measuring the expansion rate of the universe.

Correct Option - 2
The correct answer is Option 2.
Key Points

SuperBIT is a balloon-borne telescope operating in the stratosphere (40 km above Earth) using NASA's super-pressure balloon
for space-quality imaging.
Its primary scientific goal is to map the distribution of mysterious dark matter within galaxy clusters and across the universe's large-
scale structure.
SuperBIT's data collection is expected to be comparable to the Hubble Space Telescope, complementing other observatories
like James Webb and Nancy Grace Roman Space Telescope.
This diffraction-limited telescope with a wide field-of-view achieves incredible stability for precise measurements, rivaling space
telescopes at a fraction of the cost.
The innovative super-pressure balloon system allows SuperBIT to stay aloft for over 30 days, crucial for gathering enough data
for weak gravitational lensing studies of dark matter.


Additional Information
HUBBLE SPACE TELESCOPE:

Launched in 1990, the Hubble Space Telescope was the first major optical telescope to be placed in space. This revolutionized
astronomy by allowing scientists to observe the universe without the interference of Earth's atmosphere. ​
Because it sits above the atmosphere, Hubble is not affected by atmospheric distortion, which blurs starlight.
Hubble's observations have helped us understand the age and size of the universe, how stars and galaxies form, and the existence
of dark matter and dark energy.
Hubble has been in operation for over 3 decades, far exceeding its original design life. Even though the James Webb Space Telescope
has now launched, Hubble continues to make valuable observations.

Que. 9 Which of the following statements accurately describes the All India States’ People’s Conference?

1. It was an organization established to advocate for the rights and interests of princely states in British India.
2. The All India States’ People’s Conference played a significant role in the Indian independence movement by mobilizing support for
civil disobedience campaigns.
3. The conference was primarily composed of leaders from the Indian National Congress and the All India Muslim League.
4. Its formation marked a unified effort by princely states to resist British interference and demand greater autonomy.

Correct Option - 1
The correct answer is Option 1.
Key Points
All India State People's Conference

The All India States’ People’s Conference was indeed established in 1927 to represent the interests of the princely states in British
India. It aimed to address the concerns of these states and advocate for their rights within the framework of British colonial rule.
The conference provided a platform for the leaders of princely states to voice their grievances and demand greater autonomy.
While the Indian independence movement involved various organizations and movements advocating for freedom from British rule,
the All India States’ People’s Conference focused specifically on the concerns of princely states rather than playing a direct role
in civil disobedience campaigns.
The All India States’ People’s Conference consisted primarily of representatives from the princely states and their leaders. While
there may have been some interactions or collaborations with leaders from the Indian National Congress or the All India Muslim
League, the conference was not primarily composed of members from these political parties.
While the formation of the All India States’ People’s Conference aimed to address the issue of British interference in princely states
and advocate for greater autonomy, it did not necessarily signify a unified effort by all princely states.
The degree of participation and cooperation varied among different princely states based on their individual circumstances and
interests.

5/262
Que. 10 Consider the following statements about the All India Tilak swaraj Fund:
1.It was set up by the British government to win over moderate nationalists in India.
2.The fund aimed to raise money for the activities of the Non-Cooperation Movement.
3.The target amount for the Fund was not met, reflecting a lack of public support for the independence movement.
How many of the statements given above is/are correct?

1. One only
2. Two Only
3. All three
4. None

Correct Option - 1
The Correct answer is Option 1.
Key Points
All India Tilak Swaraj Fund:
The All India Tilak Swaraj Fund was a significant initiative launched during the Indian independence movement .Established in 1921 by
Mahatma Gandhi in memory of Bal Gangadhar Tilak, a prominent nationalist leader, on his first death anniversary.

The British government wouldn't raise funds for the Indian independence movement. Hence. Statement 1 is incorrect.
The Tilak Swaraj Fund was launched by Mahatma Gandhi to support the Non-Cooperation Movement. Hence, Statement 2 is correct.
The fund successfully exceeded its target, showcasing widespread public support for the movement. Hence, Statement 3 is incorrect.

Additional Information
Aimed to raise funds for the Non-Cooperation Movement, a nationwide campaign advocating for self-reliance and resistance to
British rule.
Demonstrated Gandhi's ability to mobilize mass support and contributions from across India.
Highlighting Tilak's legacy: The fund associated the movement with Tilak's ideals of self-rule and assertive nationalism.
The ambitious goal was to collect ₹1 crore (approximately ₹150 crore in today's value).
The nationwide campaign received a tremendous response, exceeding the target within a set deadline, showcasing the widespread
support for the Non-Cooperation Movement.

Que. 11
Consider the following statements about the Singpho uprising:
1. The primary cause of the uprising was resentment against British policies that restricted their hunting practices.
2. The uprising achieved its objective of expelling the British from the Tirap region.
3. The uprising is a significant example of tribal resistance against British rule in Arunachal Pradesh.
How many of the statements given above is/are incorrect?

1. One only
2. Two only
3. All three
4. None

Correct Option - 2
The correct answer is Option 2.
Key Points
​Singpho uprising: An uprising of the Singhpos took place in 1843 against the Britishers in Tirap, Arunachal Pradesh. It was an act of
retaliation to reclaim the Singhpo lands that were encroached on by the East India Company.

While there could have been grievances related to restrictions on traditional practices, land encroachment and exploitative policies
were more likely the primary cause of the uprising. Hence Statement 1 is Incorrect.
The British militarily suppressed the uprising. The Singphos did not achieve their objective of driving out the British. Hence,
Statement 2 is incorrect.
The Singpho uprising is indeed a notable instance of tribal resistance against British colonialism in Arunachal Pradesh. Hence,
Statement 3 is correct.

Additional Information
All the Singhpo chiefs came together to fight against the common enemy, including those who had signed an agreement of support
with the Britishers earlier. The Singhpos attacked soldiers posted at the Beesa post,for which they were captured and held captive
by the garrisoned soldiers.
Later the sepoys surrendered when they were informed about the capture of Nigroo and Koojoo by the Singphos. This incident
claimed the lives of many British soldiers. A marching troop wassent to suppress all Singhpo activity.

6/262
During the attack, the Singhpos were assaulted severely and their villages were sabotaged and burnt. Prominent leaders of the
Singhpos like Beesa Gaum and Nigroola had no choice but to finally surrender to the advanced British artillery

Que. 12
Consider the following statements about the Bibighar massacre:
1. It was a massacre of Indian sepoys by the British East India Company during the Sepoy Mutiny.
2. The massacre involved the killing of a large number of British women and children by rebel forces.
3. The Bibighar massacre occurred during the Siege of Lucknow in 1857.
How many of the statements given above is/are correct?

1. One only
2. Two only
3. All three
4. None

Correct Option - 1
The correct answer is Option 1.
Key Points
Bibighar Massacre:
The Bibighar Massacre occurred in Cawnpore (present-day Kanpur), India, in 1857, during the Sepoy Mutiny (also known as India's First
War of Independence) against the British East India Company's rule.

The massacre involved the killing of a large number of British women, children, and some men who were captured during the
Mutiny. Hence, Statement 1 is incorrect.
The massacre targeted British women, children, and some men who were taken captive during the rebellion. Hence, Statement 2 is
correct.
The Bibighar massacre occurred during the Siege of Cawnpore (now Kanpur) in 1857. Hence, Statement 3 is Incorrect.

Additional Information
British civilians and soldiers, including women, children, and some wounded men, were captured and killed by rebel forces.
Estimates suggest hundreds were massacred. The specific perpetrators remain a subject of debate, with theories pointing towards rebel
sepoys, civilians, or irregular forces.
The British garrison in Cawnpore surrendered to a rebel force led by Nana Sahib under a promise of safe passage. However, the British
were ambushed and massacred during their evacuation.
Women, children, and some injured men were taken captive and held in a house called the Bibighar.After a few days, the captives in
the Bibighar were brutally murdered. Accounts suggest some sepoys refused to participate in the killings.
The Bibighar Massacre became a symbol of the rebellion's brutality. Widely publicized by the British, it influenced public opinion
and military actions, leading to a hardening of British attitudes and acts of retribution.The exact number of victims and the specific
perpetrators remain debated by historians.

Que. 13 Consider the following statements about Trutiya Ratna:


1.Trutiya Ratna is regarded as one the first modern and social plays written in Bengali.
2.The play critiques the caste system in 19th century India.
3.The play revolves around a poor farmer and his pregnant wife.
How many of the statements given above is/are correct?

1. One only
2. Two only
3. All three
4. None

Correct Option - 2
The correct answer is Option 2.
Key Points
Trutiya Ratna: Meaning "The Third Jewel" in Marathi is a Marathi language play written by Jyotirao Phule in 1855.

Trutiya Ratna is regarded as one of the first modern, social, and independent plays written in Marathi. Hence, Statement 1 is incorrect.
The play critiques the caste system prevalent in 19th-century India, particularly the exploitation of lower castes by Brahmins, the
highest caste. Hence, Statement 2 is correct.
The play depicts how a certain class of society robs another superstitious, rustic, poor class of society based on the sole capital of
being high by birth. The play tells the story of a rural couple. A farmer and his pregnant wife fall prey to religious deception by a local
Brahmin priest. Hence, Statement 3 is correct.
7/262
Additional Information
The title "Trutiya Ratna" (Third Jewel) refers to knowledge, seen as the third essential element in life alongside truth and dharma
(righteousness). The play emphasizes the importance of education for empowerment.
The play revolves around a poor farmer and his pregnant wife. They are deceived by a Bhat (Brahmin priest) who exploits their
ignorance and superstitions for personal gain. A Christian missionary intervenes, exposing the Bhat's trickery and offering the couple
alternative knowledge and guidance.
Trutiya Ratna reflects Jyotirao Phule's progressive ideas and his fight against social evils like caste discrimination and exploitation of
the underprivileged.
It showcases the potential of theatre as a medium for social commentary and raising awareness.
The play continues to be studied for its historical and social relevance, prompting discussions about social justice and the fight for
equality.

Que. 14 Consider the following statements

Statement: The term "diku" is used exclusively in urban areas of India to refer to wealthy individuals.

Assertion: Diku is a term used in tribal regions of India, often with a negative connotation towards outsiders.

1. Both the statement and assertion are individually true and the assertion is the correct explanation of the statement.
2. Both the statement and assertion are individually false
3. The statement is true but the assertion is false.
4. The statement is false but the assertion is true

Correct Option - 4
The correct answer is Option 4.
Key Points
Diku: The term "Dikus" refers to outsiders and nontribals. The tribal people suffered greatly as a result of the Dikus' actions, which turned
them into dependents. Hindu landowners, missionaries, moneylenders, traders, and the British were all considered outsiders and referred to as
Dikus.

"Diku" is primarily used in tribal regions, not urban areas, and doesn't solely refer to wealth. Hence, Statement is False.
"Diku" is used in tribal regions and often carries negative connotations towards outsiders who are perceived as exploitative and
lacking cultural understanding. Hence, assertion is correct.

Additional Information

The term "diku" is used in India, particularly in tribal regions, to refer to outsiders or foreigners, but it can carry a negative
undercurrent.
"Diku" primarily refers to people who are not part of a particular tribal community. It signifies someone who comes from outside
the close-knit tribal social group.
Dikus are often seen as having greater access to resources and economic opportunities compared to tribal communities.
There can be a concern that outsiders exploit tribal resources and people for their own benefit.
"Diku" can imply a lack of understanding or respect for the traditions and customs of the tribal community. Outsiders might be seen
as insensitive to local practices and beliefs.
The meaning and connotation of "diku" can vary depending on the specific context and region. In some situations, it might be a
neutral term simply referring to someone from outside the immediate community. However, the potential for negative connotations
exists.

Que. 15 The Tenancy Act of 1903, implemented in British India, aimed to address which of the following issues?

1. Regulation of inter-state water disputes


2. Exploitation of peasants by landlords in Punjab
3. Urban housing reforms for the working class
4. Streamlining tax collection procedures

Correct Option - 2
The correct answer is Option 2.
Key Points

Tenancy Act of 1903

The Tenancy Act of 1903 was specific to the Punjab province and aimed to regulate the relationship between landlords and tenants.

8/262
The Act offered limited protection to tenants due to the influence of powerful landlords in its creation and the presence of
loopholes that allowed landlords to continue exploitation.
The Act's shortcomings sparked discontent among peasants, indirectly contributing to the Indian freedom struggle. This discontent
stemmed from a desire for better living conditions and freedom from exploitation, both by landlords and the British Raj.
Though limited in effectiveness, the Tenancy Act is recognized as one of the earliest attempts at agrarian reform in colonial India,
paving the way for future legislation aimed at improving the condition of farmers.
The Act, despite its limitations, exemplifies the growing discontent with British rule and the yearning for a more just society, a
sentiment that fueled the Indian independence movement.

Que. 16 Consider the following statements about the Matangeshwar Temple:


1. It is located in Khajuraho, Madhya Pradesh.
2. It is the only actively used Hindu temple among the Chandela-era monuments.
3. The temple is famous for a massive Shivalinga that is believed to grow in size miraculously.
How many of the statements give above is/are correct?

1. Only one
2. Only two
3. All three
4. None

Correct Option - 3
The correct answer is Option 3.
Key Points
Matangeshwar Temple: The Matangeshwar Temple is a Shiva temple located in Khajuraho, Madhya Pradesh, India.The temple was built
by the Chandela dynasty ruler Chandra Dev in the early 10th century AD. The name Matangeshwar comes from the fact that Lord Shiva is
considered as the venerated sage Matang.​

The Matangeshwar Temple is situated in Khajuraho, Madhya Pradesh. Hence, Statement 1 is correct.
While Khajuraho is famous for its exquisite Chandela era temples, Matangeshwar Temple is the only one that remains actively used for
worship. Hence , statement 2 is correct.
The Matangeshwar Temple houses a large Shivalinga, and local beliefs hold that it grows in size on Kartik Purnima. Hence,
Statement 3 is correct.

Additional Information
The Matangeshwar Temple is a square-plan temple with a plain exterior and interior, unlike other Khajuraho Temples which are
known for their elaborate sculptures.
The Matangeshwar Temple is considered to be one of the holiest temples in central India and is visited by thousands of pilgrims,
especially during Mahashivaratri.
According to legend, the Shivalinga's height increases by a fraction of an inch every year on Kartik Purnima, the full moon day in the
Kartik month of the Hindu calendar. Temple authorities apparently take measurements to validate this claim.
Situated just outside the western group of temples in Khajuraho, the Matangeshwar Temple is easily accessible to visitors exploring
the Khajuraho temple complex.

Que. 17 George Joseph was a freedom fighter known for:

1. Leading the armed rebellion against British rule in Bengal.


2. actively participated in the Vaikom Satyagraha against untouchability.
3. Championing the cause of Indian revolutionaries abroad.
4. Leading negotiations with the British Raj during the Round Table Conferences.

Correct Option - 2
The correct answer is Option 2.
Key Points
George Joseph: George Joseph (1887-1938) was a prominent lawyer and a dedicated Indian independence activist. He was one of the
earliest and most notable Syrian Christians from Kerala to actively participate in the freedom struggle.

Joseph played a crucial role in the Vaikom Satyagraha of 1924, a protest against untouchability restrictions on access to a public road
near a temple. Hence, Option 2 is correct.

Additional Information
After his studies in London, Joseph established himself as a successful lawyer in Madurai, Tamil Nadu. He was drawn to the Home
Rule Movement led by Annie Besant and became a vocal advocate for self-governance.

9/262
His commitment to the freedom struggle led him to become an All-India Congress leader by 1920. He actively participated in various
movements and campaigns.
Joseph's strong voice found expression through his editorial work. He edited the nationalist newspaper "The Independent" in
Allahabad and later succeeded C. Rajagopalachari as the editor of Mahatma Gandhi's influential publication "Young India." This
platform allowed him to spread awareness and inspire others in the freedom struggle
He recognized the importance of worker's rights and played a key role in setting up a trade union movement in Madurai to advocate
for better wages and working conditions for textile mill workers.

Que. 18
Consider the following statements about the Guruvayur Satyagraha:
1. It was a non-violent protest movement against the denial of entry into the Guruvayur Temple for Dalits.
2. The movement resulted in the immediate opening of the Guruvayur Temple to all Hindus.

1. Both statements 1 and 2 are correct.


2. Both statements 1 and 2 are incorrect.
3. Statement 1 is correct; Statement 2 is incorrect.
4. Statement 1 is incorrect; Statement 2 is correct.

Correct Option - 3
The correct answer is Option 3.
Key Points
Satyagraha in Guruvayur: The Satyagraha in Guruvayur (1931-1932) was a non-violent protest movement launched in Kerala, India, as
part of the wider Indian freedom struggle. It specifically fought against the social evil of untouchability and aimed to secure the right of entry
into the Guruvayur Temple for all Hindus, regardless of caste.

The satyagraha aimed to secure temple entry for all castes. Hence , statement 1 is correct.
While the movement paved the way for future reforms, the temple did not open its doors immediately. It happened in 1947 with the
Temple Entry Bill. Hence, statement 2 is incorrect.

Additional Information
The Satyagraha followed the principles of non-violent resistance popularized by Mahatma Gandhi. Protestors, consisting of upper-
caste and Dalit volunteers, would attempt to enter the temple grounds while chanting devotional songs (sankirtana).
The Satyagraha lasted for almost a year and garnered national attention. Kelappan undertook a crucial 12-day hunger strike to
highlight the cause. While the temple did not open its doors immediately, the movement significantly pressured the authorities and
paved the way for future social reforms.
Eventually, due to Gandhi's intervention and changing political landscapes, the temple opened its doors to all Hindus in 1947 with the
passing of the Temple Entry Bill by the Madras Government.
The Guruvayur Satyagraha was a significant step in dismantling the discriminatory practice of untouchability in Kerala and India.
The movement was led by K. Kelappan, a prominent figure in the Kerala Provincial Congress Committee.

Que. 19
Consider the following statements about the Kalaram Temple Entry Satyagraha:
1. Dr. B. R. Ambedkar led a Satyagraha demanding entry for Dalits into the Kalaram Temple in Nashik.
2. The protest resulted in immediate legislation granting Dalits access to all Hindu temples in India.
3. The Kalaram Temple Satyagraha is a significant event in the history of the Indian independence movement.
How many of the statements given above is/are correct?

1. Only one
2. Only two
3. All three
4. None

Correct Option - 1
The correct answer is Option 1.
Key Points
Kalaram Temple Entry: The Kalaram Temple in Nashik, Maharashtra, India, holds a significant place in India's history due to the Kalaram
Temple Entry Satyagraha. The Kalaram Temple, a Hindu temple dedicated to Lord Rama, traditionally denied entry to Dalits.

Led by: Dr. B. R. Ambedkar, a prominent figure in the fight against caste discrimination and the upliftment of Dalits (untouchables) in
Indian society. Hence, statement 1 is correct.
While the protest raised awareness, it did not lead to immediate legislative changes. The fight for temple entry rights for Dalits
continued for many years. Hence, statement 2 is incorrect.
10/262
The Satyagraha was a fight against caste discrimination, not directly related to Indian independence. It did, however, contribute to the
larger movement for social justice in India. Hence, statement 3 is incorrect.

Additional Information
Following the principles of non-violent resistance (Satyagraha) championed by Mahatma Gandhi, Dr. Ambedkar led a protest with
thousands of Dalit followers. They surrounded the temple, sang devotional songs, and demanded the right to enter and worship.
The protest did not achieve immediate success, but it sparked national attention and highlighted the plight of Dalits facing
untouchability.
The Kalaram Temple Satyagraha is a crucial event in the history of the Dalit movement in India. It symbolized the fight for equal
rights and access to religious institutions, paving the way for future social reforms
It serves as a reminder of the ongoing fight against caste discrimination and the pursuit of equal rights for all citizens.

Que. 20 Which of the following statements is correct about the Aravipuram Movement:

1. It was a movement led by Mahatma Gandhi to promote Hindu-Muslim unity.


2. Sri Narayana Guru defied caste restrictions by consecrating an idol at Aravipuram.
3. It led to the immediate abolition of the caste system in Kerala.
4. The movement aimed to establish separate temples for the Ezhava community.

Correct Option - 2
The correct answer is Option 2.
Key Points
Aravipuram movement: The Aravipuram Movement, also known as the Aravipuram Prathishta (consecration), was a significant
event in Kerala's history and the fight against caste discrimination.
Sri Narayana Guru himself consecrated (performed the installation rituals) of a Shiva idol at a newly built temple on Shivaratri day
in 1888.Hence , Option 2 is correct
This act challenged the prevailing social norms. Traditionally, people from lower castes, including Sri Narayana Guru's Ezhava
community, were forbidden from performing such rituals or even entering some temples.
The act symbolized Sri Narayana Guru's message of "One Caste, One Religion, One God," advocating for a society free from caste-
based discrimination.
The Aravipuram Prathishta marked the beginning of a larger social reform movement led by Sri Narayana Guru. He established
educational institutions, temples open to all castes, and promoted self-help initiatives for his community.
The movement instilled a sense of self-respect and social empowerment within the Ezhava community, historically considered a lower
caste. It inspired similar movements challenging caste discrimination in Kerala and other parts of India.

Que. 21 Consider the following statements on Antiquities and Art Treasures Act, 1972

1: The Antiquities and Art Treasures Act, 1972 allows for the unrestricted export of antiquities older than 100 years.

2: The Act aims to prevent the smuggling and illegal trade of culturally significant items in India.

3.The government can acquire any artwork, regardless of age, as an art treasure for preservation.

How many of the statements given above is/are correct?

1. Only one
2. Only two
3. All three
4. None

Correct Option - 1
The correct answer is Option 1.
Key Points
Antiquities and Art Treasures Act, 1972: The Antiquities and Art Treasures Act, 1972 is an Indian law aimed at protecting the country's
cultural heritage by regulating the trade and ownership of antiquities and art treasures.

The Act aims to preserve India's cultural heritage. Unrestricted export would contradict this goal. The Act regulates the export of
antiquities and art treasures. Only the government or authorized bodies can export such items under specific conditions. Hence,
Statement 1 is incorrect.
One of the main objectives of the Act is to curb illegal activities related to antiquities and art treasures. Smuggling and fraudulent
dealings deprive India of its cultural wealth and can damage these precious items. Hence, Statement 2 is correct.
The Act empowers the government to declare any work of art, irrespective of age, as an "art treasure" if it deems it has exceptional
cultural importance and needs protection. This broader category allows the government to preserve a wider range of India's artistic
heritage. Hence, Statement 3 is correct.

11/262
Additional Information
The Act came into effect in 1976.It was enacted to comply with the UNESCO 1970 Convention on combating the illegal import,
export, and transfer of ownership of cultural property.
Only the Indian government or its authorized bodies can export such items. This aims to prevent India's cultural wealth from
leaving the country permanently.
The Act discourages smuggling and fraudulent dealings in antiquities.
It allows the government to acquire important antiquities and art treasures for preservation in public places like museums, ensuring
these items are accessible to the public.
The Act empowers the Indian government to declare any work of art, not necessarily an antiquity, as an art treasure for protection
purposes.

Que. 22
Statement: The Kattunayakan tribe, residing primarily in the Western Ghats, is classified as a Particularly Vulnerable Tribal Group
(PTG) due to their deep-rooted forest-based way of life.

Assertion: This classification highlights the challenges faced by the tribe in adapting to modern development initiatives.

Choose the correct option:

1. Both Statement and Assertion are true, and Assertion is the correct explanation of the Statement.
2. Both Statement and Assertion are true, but Assertion is not the correct explanation of the Statement.
3. Statement is true, but Assertion is false.
4. Statement is false, but Assertion is true.

Correct Option - 1
The correct answer is Option 1.
Key Points
Kattunayakan tribe:
The Kattunayakan tribe, also known as Jennu Kurumbas, are an indigenous community residing in parts of South India, primarily in the
states of Tamil Nadu and Kerala.Their name translates to "king of the jungle" in Tamil and Kannada, reflecting their deep connection to
the forest.

The Kattunayakans are an indigenous tribe who have traditionally relied on the forest for their survival. They practice hunting,
gathering, and honey collection, which are all activities deeply linked to their forest environment. Hence , statement is true.
The PVTG classification signifies that the Kattunayakans face specific challenges due to their traditional way of life.Modern
development projects like wildlife conservation or infrastructure development might restrict access to the forest, impacting their ability
to hunt, gather, and collect honey. Hence, assertion is true and correct explanation of the statement.

Additional Information
Traditionally, the Kattunayakans have been dependent on the forest for their livelihood. They practice hunting, gathering of forest
products, and honey collection, with honey being a key source of income.
They prefer living in remote areas within the Western Ghats. Their religion is deeply rooted in nature, with worship of animals,
birds, trees, and other natural elements. Ancestral reverence is also practiced.
They speak a language influenced by various Dravidian languages.
Restrictions on forest access for conservation and development projects can affect their traditional way of life. They are classified as
one of India's Particularly Vulnerable Tribal Groups (PTGs) due to factors like low literacy rates and limited access to modern
amenities.
The recent Oscar win for the documentary "Elephant Whisperers," which features a Kattunayakan family caring for orphaned
elephants, might have brought renewed interest in the tribe and their connection to the forest.

Que. 23 Which of the following statements is/are CORRECT regarding the National Mission for Cultural Mapping (NMCM) in India?

1. It is a joint initiative of the Ministry of Culture and Ministry of Tourism.


2. It aims to create a digital inventory of folk arts, artists, and cultural heritage across the country.
3. Launched in 2021, it is spearheaded by Indira Gandhi National Centre for the Arts (IGNCA).
4. To provide financial support to individual artists.

Correct Option - 2
The correct answer is Option 2.
Key Points
NMCM: The National Mission for Cultural Mapping (NMCM) is an initiative by the Ministry of Culture in India. Launched in 2017, it aims
to create a comprehensive database of the country's vast and diverse cultural heritage.

A core objective of NMCM is to establish a central digital platform. This platform will act as a national repository for information on
various cultural aspects like art forms, artists, traditions, and heritage sites. This data will be crucial for research, policy making, and
https://lms.testbook.com/genpdf/fromques.php?qids=65f93cf52fe08b6c70dc4d11,65f93e6eb5431ddfb31dfe15,65f96fd18acd632ab8e39282,65f9712de8d50… 12/262
future preservation efforts. Hence , Statement 2 is Correct.

Additional Information
By documenting art forms, artists, and cultural resources across the nation, the NMCM works to safeguard these traditions for
future generations.
The mission strives to create a central platform with information on various cultural aspects, facilitating research, planning, and
preservation efforts.
Support artists and art forms: The NMCM aims to empower the artist community by providing them with better opportunities and
recognition.
Revitalize cultural vibrancy: By fostering awareness and appreciation for India's cultural wealth, the mission seeks to strengthen
the country's cultural identity.
NMCM was launched in 2017 by the Ministry of Culture. However, after facing initial delays, the initiative was handed over to
IGNCA in 2021 for more focused implementation.
The Mission does not contain any provision for financial assistance.

Que. 24 The Mera Gaon Meri Dharohar (MGMD) survey is a significant initiative because it:

1. Promotes standardization of agricultural practices across different villages


2. Provides real-time data for monitoring rural development programs.
3. Creates a centralized platform for showcasing India's diverse cultural heritage.
4. Identifies poverty-stricken villages for targeted government interventions.

Correct Option - 3
The correct answer is Option 3.
Key Point
MGMD:The Meri Gaon Meri Dharohar (MGMD) survey is a project by the Indian government's Ministry of Culture to create a massive
record of India's village culture . The main goal is to document and preserve the unique cultural identity of each village in India.

The survey creates a central digital repository where information on various cultural aspects of Indian villages is stored. This data can
be accessed by the public, researchers, and cultural institutions. Hence, statement 3 is correct.

Additional Information
The Mera Gaon Meri Dharohar (MGMD) survey is an initiative launched by the Government of India under the National Mission for
Cultural Mapping (NMCM).

OBJECTIVES:

MGMD aims to create a comprehensive record of India's diverse village cultures. This includes capturing information on local art
forms, traditions, festivals, historical sites, and prominent personalities.
By actively involving villagers in the survey process, MGMD fosters a sense of ownership and pride in their cultural heritage.
The collected data will be used to develop a national database that can be accessed by the public. This can potentially boost cultural
tourism by showcasing the unique aspects of different villages.

Process:
Collaboration with CSCs: The Ministry of Culture partners with Common Service Centres (CSCs) which are village-level digital service
centers.
Village Level Entrepreneurs (VLEs): Trained personnel from the CSCs, called VLEs, conduct meetings with villagers to gather
information.
Data collection: VLEs document details about the village's cultural aspects through discussions with residents

Que. 25 Consider the following statements about the Global Buddhist Summit 2023:
(i) It was the first international summit on Buddhism.
(ii) The summit focused on addressing social and environmental challenges.
(iii) The summit aimed to strengthen India's cultural ties with Southeast Asian nations.
How many of the statements given above are correct?

1. Only one
2. Only two
3. All three
4. None

Correct Option - 3
The correct answer is Option 3.

13/262
Key Points
GLOBAL BUDDHIST SUMMIT:

The two-day Global Buddhist Summit 2023 concluded successfully in New Delhi with the adoption of the New Delhi Declaration.
It saw the participation of delegates from nearly 30 countries, including Taiwan, Myanmar, Thailand, Vietnam, Sri Lanka, and
Mongolia.
It was the first-ever Global Buddhist Summit organized by the Ministry of Culture in collaboration with the International Buddhist
Confederation (IBC).
The focus of the Global Buddhist Summit was on modes of disseminating and internalizing universal values and finding ways to
work together, to address the burning challenges both within and globally and offer a sustainable model for the future of the
world.
The summit aimed to find solutions to global problems based on Buddha's teachings.
The texts, doctrines and philosophy of Buddha Dhamma are the best guide for inter-faith dialogue, harmony and universal peace.

Significance of Buddhism for India

Importance of India in Buddhism: in Buddhism: Because Buddhism was founded in India, this global summit will highlight the
significance and importance of India in Buddhism.
Strengthening cultural and diplomatic ties: Additionally, this summit will serve as a vehicle for strengthening cultural and
diplomatic ties with other nations, particularly those that adhere to the Buddhist ethos.
Cultural Significance: Buddhism is an integral part of India’s cultural heritage. It played a significant role in shaping India’s art,
architecture, literature, and philosophy. Today, India’s rich heritage in Buddhism attracts millions of tourists from all over the world.

Que. 26 Consider the following statements about the Pushkaram festival:


I. It is a pilgrimage held every 6 years along a specific river based on astrological positions.
II. The most recent Pushkaram on the Ganges river was celebrated in 2021.
III. Devotees believe taking holy dips during Pushkaram washes away sins and brings good fortune.
How many of the statements given above is/are correct?

1. Only one
2. Only two
3. All three
4. None

Correct Option - 1
The correct answer is Option 1.
Key Points
Pushkaram festival:
Pushkaralu is a sacred pilgrimage for Hindus, occurring every 12 years along a specific river based on astrological alignments. In 2023, it
was the turn of the Ganges River.

Pushkaram occurs every 12 years, not 6 years. The time interval is based on astrological calculations linked to Jupiter's movement.
Hence, Statement 1 is incorrect.
The most recent Ganges Pushkaram happened in 2023, not 2021. It's crucial to link information from the prompt to the questions.
Hence, Statement 2 is incorrect.
Devotees believe holy dips during Pushkaram cleanse sins and bring blessings, aligning with the festival's spiritual significance.
Hence, Statement 3 is correct.

Additional Information

The celebration happens annually, once in 12 years along each river.


Each river is associated with a zodiac sign, and the river for each year’s festival is based on which sign Bruhaspathi (Jupiter) was in
at the time.
It is believed that bathing in the sacred river during Pushkaram cleanses the devotees of their sins.
This year festival involves worshipping ancestors and the river Ganga and is being organised in Kashi after a gap of 12 years.
In India, we have 12 major rivers such as Ganga, Yamuna, Narmada, Saraswathi, Godavari, Krishna, Kaveri, Bhima, Tapati,
Tungabhadra, Sindhu, Pranhita for which the Pushkaram festival is celebrated for each river according to the zodiac sign of that river.
The 12 days after the Jupiter enters that particular zodiac sign will be the Pushkaram/Pushkaralu which is a 12-day festival for that
river.
According to legend, after severe penance, the devotee Pushkara was blessed by Lord Shiva with the ability to live in water and purify
holy rivers
On a request from Bruhaspati (Jupiter), Pushkara decided to enter one of the 12 sacred rivers, including Ganga, Yamuna, Godavari,
and Krishna, among others

Que. 27 Akka Mahadevi, a significant figure in ancient Indian literature, was known for:

1. Epic poetry celebrating the lives of kings and warriors.


14/262
2. Devotional poems expressing intense spiritual longing, defying societal expectations.
3. Philosophical treatises on logic and epistemology.
4. Instructive poems emphasizing the importance of social hierarchy.

Correct Option - 2
The correct answer is Option 2.
Key Points
Akka Mahadevi (12th century): A Veerashaiva saint and poet from Karnataka, Akka Mahadevi is considered one of the greatest mystical
poets of all time. Her poems, written in Kannada, focus on intense devotion and spiritual longing, breaking away from conventional ideas of
femininity.

Akka Mahadevi is renowned for her Vachana poems, known for their passionate devotional expression and mystical longing for Lord
Shiva. She challenged societal norms by expressing her spirituality openly, breaking away from conventional ideas of femininity at the
time. Hence , statement 2 is correct.
Akka Mahadevi didn't focus on epic poetry glorifying kings and warriors. Her work belonged to the Bhakti movement, emphasizing
personal devotion.Hence , statement 1 is incorrect.
While some women in ancient India might have explored philosophical themes, Akka Mahadevi's primary focus wasn't logic or
epistemology (the study of knowledge).Hence , statement 3 is incorrect.
Akka Mahadevi didn't write poems promoting social hierarchy. Her poems often transcended societal structures and emphasized a
direct connection with the divine.Hence , statement 4 is incorrect.

Additional Information
Akka Mahadevi's Vachanas are known for their mystical and intense expression of love and longing for Lord Shiva. She often used
metaphors of a woman yearning for her beloved to describe her spiritual quest. This defied societal norms of the time, where women
were expected to prioritize domesticity and social roles.
By expressing her spirituality so openly and defying societal expectations, Akka Mahadevi became a symbol of feminist resistance
within Indian literature. Her poems questioned the limitations placed on women and highlighted their capacity for profound spiritual
experiences.
Akka Mahadevi is revered as a saint and a poet within the Lingayat tradition and beyond. Her Vachanas continue to inspire readers
with their passionate devotion, bold imagery, and timeless message of spiritual longing.

Que. 28 Consider the following statements about Tungnath Temple:


1.It is the highest Shiva temple in the world.
2.It is located in the Pithoragarh district of Uttarakhand.
3.It is part of the Panch Kedar pilgrimage circuit.
How many of the statements given above is/are correct?

1. Only one
2. Only two
3. All three
4. None

Correct Option - 1
The correct answer is Option 1.
Key Points
The Tungnath Temple is one of the highest Shiva temples in the world, located at an elevation of 3,680 meters (12,073 ft) in the
Rudraprayag district of Uttarakhand, India.

Tungnath Temple holds the title of one of the highest Shiva temples in the world, but it's not definitively the absolute highest. Hence,
Statement 1 is incorrect.
Tungnath Temple is situated in the Rudraprayag district of Uttarakhand, not Pithoragarh. Hence, Statement 2 is incorrect.
Tungnath Temple is indeed a revered part of the Panch Kedar pilgrimage, a holy circuit dedicated to Lord Shiva in the
Himalayas. Hence, Statement 3 is correct.

Additional Information
It is also the highest of the five Panch Kedar temples, which are dedicated to Lord Shiva.
The temple is believed to be over 1,000 years old and is said to have been built by the Pandavas, the heroes of the Hindu epic
Mahabharata.
The temple is open from April or May, depending on the date set by the Badro Kedar Temple Committee, to November every year.
During winters, the idol of Lord Shiva is moved to a lower temple location called Mukunath due to extreme weather conditions.

Que. 29 Which of the following statements about Juna Khatiya, an archaeological site in Gujarat, India, is true?

15/262
1. It belongs to the later Harappan period.
2. It is located in the Amreli district of Gujarat.
3. It is known for being one of the largest Harappan necropolises discovered in India.
4. It primarily contains artifacts made of bronze.

Correct Option - 3
The correct answer is Option 3.

Key Points
Juna Khatiya: Juna Khatiya is an archaeological site located in the Kutch district of Gujarat, India. It has gained significance for being one
of the biggest Harappan (Indus Valley Civilization) necropolis (cemetery) discovered so far.

It belongs to Early Harappan (3200 BCE to 2600 BCE) period and located in Lakhpat Taluka, Kutch district, Gujarat.
Significance: Largest Harappan cemetery documented in India as of now. Other prominent Harappan sites in Gujarat include
Dholavira, Lothal, and Surkotada.
Findings: Excavations have revealed around 500 graves spread over a 16-hectare area. These graves contain:
Skeletal remains: Offering insights into the physical characteristics and burial practices of the Harappan people.
Burial structures: Made of dressed sandstone, with rectangular shapes being most common, followed by oval or circular ones.
Grave goods:
Pottery fragments with similarities to those found in Sindh and Balochistan, suggesting trade links.
Stone knives and other tools.
Shell bangles and beaded jewelry.
Animal bones, indicating the possible inclusion of animal offerings in the burials.
The discovery of Juna Khatiya has provided valuable information about the death rituals and social practices of the Harappan
civilization. It sheds light on their belief systems, social stratification (presence/absence of grave goods), and craftsmanship.
The site was accidentally discovered in 2016 and has been under excavation since 2018.
The findings from Juna Khatiya challenge the notion of a uniform Harappan culture, suggesting regional variations in burial
practices.

Que. 30 Consider the following statements about Gongadi shawls:


(a) They are traditional woolen shawls made by Kuruma communities in Telangana from the wool of Deccani sheep.
(b) They are known for their softness and are often dyed with vibrant colors.
(c) They are produced organically and are said to become stronger and darker with age.
How many of the statements given above is/are correct?

1. Only one
2. Only two
3. All three
4. None

Correct Option - 2
The correct answer is Option 2.
Key Points
Gongadi Shawls: Gongadi, (known also as Kambal) is the traditional woolen blanket woven by the indigenous Kuruma pastoralist
communities from wool of the indigenous Deccani sheep (known locally as Nalla gorrae) breed found in the Deccan Plateau region
including the Indian state of Telangana.

Gongadi shawls are traditional woolen shawls made by the Kuruma communities in Telangana using the wool of Deccani sheep.
Hence, statement 1 is correct.
Gongadi shawls as having coarse wool, not soft. Additionally,they are produced organically without dyes, so vibrant colors is not a
characteristic. Hence, statement 2 is incorrect.
The unique quality of Gongadi shawls to become stronger and darker with age. Hence, statement 3 is correct.

Additional Information
The coarse woolen blanket is famous for its durability and versatility.
The tough gongadi usually lasts for more than a decade and acquires this unique quality from the craft of hand weaving.
One of the unique nature of gongadi is that it does not fade but grows darker in time.
The indigenous Kuruma weavers say that gongadi is so strong that you can lift a fully grown bull off the ground with it.
The weaving craft of Kurumas is not limited to weaving just gongadi but also includes weaving carpets, bedsheets, stolls, scarfs, bags,
etc. from wool.
The traditional gongadi is produced organically, without using any dyes either natural or synthetic. Sizing of the strings is done using
the paste of soaked and cooked tamarind seeds. The border/hem (Kada) of traditional gongadi is woven in many designs.

16/262
Deccani breed

It derives its name from the region Dakkan (Deccan) corresponding to its original spread across the Semi arid & Deccan plateau of
Telangana, Parts of AP, Northern Karnataka, Maharashtra, and parts of Northern Tamil Nadu.
Sheep found to the North of the Tungabhadra River are called “Deccani” and those to the South of it, “Bellary” Deccani is a recognised
meat breed.
The Deccani breed is the only pure black coarse wool breed in India.
It is reared mainly for lamb production Locally in Telangana it is called as "Nalla Gorre" or Black Sheep.

Que. 31 Consider the following statements


Statement 1: Cham dances are performed by male Buddhist monks wearing elaborate costumes and masks representing deities and mythical
beings.

Statement 2: The specific movements and themes of each Cham dance vary depending on the region and monastery.
Which one of the following is correct in respect of above statements?

1. Both statements 1 and 2 are individually true and together imply a cause-and-effect relationship.
2. Both statements 1 and 2 are individually true but do not together imply a cause-and-effect relationship.
3. Statement 1 is true; Statement 2 is false.
4. Statement 1 is false; Statement 2 is true

Correct Option - 2
The correct answer is Option 2.
Key Points
Cham Lama dance :The Cham dance, also known as the Lama dance, is a vibrant masked and costumed dance associated with Tibetan
Buddhism and Buddhist festivals.

Cham dances involve male Buddhist monks wearing specific costumes and masks depicting deities and mythical figures. Hence,
Statement 1 is true.
There are various Cham dances, each with its own story and symbolism. Additionally, details like costumes and music can vary
depending on the region and monastery. Hence , statement 2 is correct.

While the costumes and masks might reflect the specific theme of a particular Cham dance (e.g., Black-Hat Dance with a wrathful protector),
the statement doesn't suggest that regional variations in costumes cause the variation in themes. They are likely both independent results of
the rich cultural heritage and traditions practiced in different Buddhist communities.
Additional Information
Originated in Tibet, possibly as early as the 8th century CE. Performed during specific Buddhist festivals like Losar (Tibetan New
Year) and Tsechu (religious festivals).Believed to bring merit to both participants and observers.
Performed by male Buddhist monks who train extensively in the dance and mask-wearing traditions. Dancers wear elaborate
costumes and masks depicting various deities, wrathful protectors (Dharmapalas), and other characters from Buddhist mythology.
The dance involves specific sequences of movements, often symbolic and requiring strength and agility due to the heavy masks.
Accompanied by traditional Tibetan musical instruments like drums, cymbals, and horns.

Types of Cham Dances:


There are numerous Cham dances, each with its own story and symbolism. Some of the most common include:
Black-Hat Dance: Represents the vanquishing of evil by a wrathful protector deity.
Skeleton Dance: Represents the impermanence of life and death.
Guru Dance: Reenacts the life story of Guru Padmasambhava, who is credited with introducing Buddhism to Tibet.
The Cham dance is a UNESCO Intangible Cultural Heritage of Humanity. The specific details of the dance (costumes, masks, music) can
vary depending on the region and monastery.

Que. 32 The Govind Swarup Lifetime Achievement Award is given in the field of:

1. Literature
2. Public service
3. Astronomy and Astrophysics
4. Medicine

Correct Option - 3
The correct answer is Option 3.

17/262
Key Points
The Govind Swarup Lifetime Achievement Award is presented by the Astronomical Society of India (ASI) to recognize eminent
Indian astronomers for their lifelong contributions to the field of astronomy and astrophysics in India.
The award is named after Prof. Govind Swarup (1929-2020), who is considered the pioneer of radio astronomy in India. He played a
pivotal role in designing and building two of India's most important astronomical observatories - the Ooty Radio Telescope and the
Giant Metrewave Radio Telescope (GMRT).
The inaugural recipient of the Govind Swarup Lifetime Achievement Award in 2022 was Prof. Jayant Narlikar, a renowned
astronomer and the founding director of the Inter-University Centre for Astronomy and Astrophysics (IUCAA).

Nomination Process:
The Astronomical Society of India (ASI) typically announces the nomination process a few months before the award ceremony.
Nominations are invited from eminent astronomers and institutions within India.
A selection committee formed by ASI members evaluates the nominations and selects the awardee based on their lifetime
contributions to astronomy and astrophysics research in India.
Selection criteria:
While lifetime contributions are crucial, the committee might also consider factors like:
Pioneering research leading to significant discoveries in astronomy or astrophysics.
Mentorship and inspiration provided to younger generations of astronomers, fostering future talent in the field.
Service to the astronomical community through leadership roles in institutions or national committees, promoting collaboration and
advancement in the field.

Que. 33 Consider the following statements:


1. The Cholas divided their empire into large provinces called ' gramas'.
2:.The 'Nattar' officials were in charge of sub-divisions within 'Valanadus'.
3.The Cholas encouraged public participation through assemblies called 'Sabhas'.
How many of the give above statements is/are correct?

1. Only one
2. Only two
3. All three
4. None

Correct Option - 2
The correct answer is Option 2.
Key Points
CHOLAS: The Cholas were a powerful dynasty that ruled South India for over four centuries, from the 9th to the 13th centuries AD. They
are known for their efficient and well-organized administration, which contributed significantly to their success.

The Cholas actually divided their empire into large provinces called Mandalams. ' gramas' is a smaller unit referring to villages.
Hence statement 1 is Incorrect.
The Nattar officials were indeed responsible for managing sub-divisions within Valanadus. Valanadus were smaller administrative
units than Mandalams. The Nattar played a crucial role in collecting taxes, maintaining law and order, and overseeing local affairs
within their jurisdiction. Hence, statement 2 is correct.
The Cholas actively encouraged public participation in local administration through village assemblies known as Sabhas. These
assemblies consisted of elected representatives from the villages and played a vital role in managing village affairs, resolving disputes,
maintaining social order, and overseeing irrigation and temple management. Hence, statement 3 is correct.

Additional Information

Key Features of Chola Administration:


Centralized Monarchy: The Chola empire had a centralized monarchy, with the king at the head. The king held absolute power and was
considered the source of all authority.
Hierarchical Administration: The empire was divided into various administrative divisions, each with its own officials. The major
divisions were:
Mandalams: These were large provinces, usually ruled by members of the royal family or high-ranking officials.
Valanadus: Subdivisions of mandalams, headed by the 'Periyanar'.
Nadus: Smaller units within valanadus, administered by the 'Nattar'.
Villages: The basic unit of administration, with a high degree of autonomy. Villages were self-governing units with their own
assemblies responsible for various local matters.
Strong Bureaucracy: The Cholas had a well-developed bureaucracy to manage the vast empire. Officials were appointed based on merit
and were responsible for maintaining law and order, collecting taxes, and overseeing public works.
Land Revenue System: The Cholas had a well-defined land revenue system. Land was classified into different categories, and taxes were
levied based on the type and productivity of the land.
Local Assemblies: The Cholas encouraged the participation of people in local administration through village assemblies. These
assemblies, known as Sabhas, Urs, and Nagaram, played a crucial role in managing village affairs, resolving disputes, and maintaining

18/262
social order.
Military The Cholas maintained a strong military consisting of infantry, cavalry, elephants, and a powerful navy. The navy played a vital
role in their maritime expeditions and helped them establish a vast maritime empire.

Que. 34 Which among the following is not a product from Uttar Pradesh that has been awarded a GI tag ?

1. Amroha Dholak
2. Baghpat Home Furnishings
3. Lanjia Saura Painting
4. Mainpuri Tarkashi

Correct Option - 3
The correct answer is Option 3.
Key Points
GI TAG : A geographical indication (GI) tag is a name or sign used on certain products that correspond to a specific geographical location or
origin.
For example, Darjeeling Tea, Kanchipuram Silk, etc.
The GI tag ensures that only the authorised users or those residing in the geographical territory are allowed to use the popular product name.
GI tags UP :Uttar Pradesh, known for its rich cultural heritage and traditional crafts, has recently seen seven of its distinctive products being
granted Geographical Indication (GI) tags by the Geographical Indications Registry in Chennai.

Lanjia Saura Painting:


The art form belongs to the Lanjia Saura community, a PVTG largely residing in the Rayagada district, Odisha. These
paintings are in the form of exterior murals painted on the mud walls of homes. White paintings figure over a crimson-
maroon background. Hence , Option 3 is incorrect.

Additional Information
Seven Products from Uttar Pradesh have Received the GI Tag:

Amroha Dholak: A Musical Marvel The Amroha Dholak is a musical instrument crafted from natural wood. Preferred wood
choices include mango, jackfruit, and teakwood. Animal skin, usually goatskin, is meticulously fitted to create the drum's surface.

Baghpat Home Furnishings: Baghpat and Meerut are renowned for their exclusive handloom home furnishing products.
The weaving process involves cotton yarn and is predominantly done on frame looms.

Barabanki Handloom Product: Barabanki and its surrounding areas are home to around 50,000 weavers and 20,000 looms. The
annual turnover of the Barabanki cluster is estimated to be ₹150 crore.

Kalpi Handmade Paper: Kalpi is recognized for handmade paper manufacturing. Munnalal 'Khaddari,' a Gandhian, introduced
the craft in the 1940s, although its roots in Kalpi's history may extend further.

Mahoba Gaura Patthar Hastashlip: Mahoba Gaura Patthar Hastashlip represents the unique stone craft of Mahoba. The stone
used, scientifically known as the 'Pyro Flight Stone,' is a soft and radiant white-coloured stone predominantly found in the region.
Mainpuri Tarkashi: Mainpuri Tarkashi is a popular art form characterised by brass wire inlay work on wood. Traditionally used
for khadaous (wooden sandals), Mainpuri Tarkashi has been a household necessity.

Sambhal Horn Craft:

Sambhal Horn Craft utilises raw materials procured from deceased animals and this craft form is entirely handmade

Que. 35 Consider the following statements about the Ambubachi Mela:


1.It is a festival celebrating the harvest season in Assam.
2.It is associated with the Kamakhya Temple in Guwahati.
3.The temple remains closed for three days during the festival.
How many of the statements given above is/are correct?*

1. Only one
19/262
2. Only two
3. All three
4. None

Correct Option - 2
The correct answer is Option 2.
Key Points
Ambubachi Mela: The Ambubachi Mela is a unique and significant Hindu festival held annually at the Kamakhya Temple in Guwahati,
Assam.The festival lasts for four days, typically falling in the Assamese month of Ahaar (around mid-June) during the monsoon season.

The festival Celebrates the yearly menstruation cycle of the presiding deity, Goddess Kamakhya, who embodies the Mother Shakti
(feminine divine energy). Hence, Statement 1 is incorrect.
Ambubachi Mela is a unique and significant Hindu festival held annually at the Kamakhya Temple in Guwahati, Assam. Hence ,
statement 2 is correct.
During this period, the temple doors are closed to devotees for three days. This symbolizes the menstruation cycle of the goddess.
Hence, statement 3 is correct.

Additional Information

Represents feminine power, fertility, and renewal.Considered one of the largest religious gatherings in eastern India.
Daily worship and agricultural activities are suspended during the festival. Devotees, particularly Tantric practitioners, observe
austerities and rituals outside the temple.
On the fourth day, the temple reopens for a ceremonial washing and purification. This signifies the completion of the menstrual cycle.

Unique Aspects:
The reddening of the Brahmaputra River near the temple during the mela is a natural phenomenon attributed to the monsoon but
associated with the festival's symbolism.
The festival is not just a religious event but also a cultural extravaganza. Folk dances, music recitals, and traditional performances create
a vibrant atmosphere.
Controversies: Some aspects of the festival, particularly the involvement of Tantric rituals, have been criticized for their perceived
association with animal sacrifice and social practices considered regressive.

Que. 36 Consider the following statements regarding the Gandhi Peace Prize :
1. It was awarded posthumously to Bangabandhu Sheikh Mujibur Rahman, the founder of Bangladesh.
2. The recipient in 2021 , Gita Press, is a publishing house known for promoting communal harmony and social justice.
3. The award is solely decided by the President of India.
How many of the statements given above is/are correct?

1. Only one
2. Only two
3. All three
4. None

Correct Option - 2
The correct answer is Option 2.
Key Points
Gandhi Peace Prize: Gandhi Peace Prize Awards for Social, Economic and Political transformation through Non-violence was
instituted in the year 1995, on the occasion of the 125th birth anniversary of Mahatma Gandhi, as a tribute to his ideals and contributions
to humanity.

It was awarded posthumously to Bangabandhu Sheikh Mujibur Rahman, the founder of Bangladesh in 2020. Hence, Statement 1 is
correct.
Gita Press, Gorakhpur, a 100-year-old institution that publishes Hindu religious texts and promotes Gandhian ideals of peace and
social harmony, has been awarded the Gandhi Peace Prize for 2021 by the Government of India. Hence, statement 2 is correct.
The award is overseen by a jury headed by India's Prime Minister. Hence , statement 3 is incorrect.

Additional Information

The award carries an amount of Rs 1 crore, a citation, a plaque and a traditional handicraft or handloom item. The
award is conferred by the President of India at a function in Rashtrapati Bhavan.

This award is given to individuals, associations, institutions or organizations who have worked selflessly for peace, non-violence
and amelioration of human sufferings.

The award is open to all persons regardless of nationality, race, language, caste, creed or gender.

20/262
Que. 37 Consider the following statements about Raja Mihir Bhoja:
1.He inherited a flourishing Pratihara empire and further expanded its territory.
2.He is credited with establishing a strong central administration that promoted cultural development.
3.He was a devout follower of Buddhism and actively promoted its teachings.
How many of the statements given above is/are correct?

1. Only one
2. Only two
3. All three
4. None

Correct Option - 1
The correct answer is Option 1.
Key Points
Mihir Bhoja: Mihir Bhoja, also known as Bhoja I, was a powerful emperor who ruled the Pratihara dynasty in India from 836 to 885 CE.

He inherited a weakened kingdom from his father, Ramabhadra. Through his military prowess and strategic thinking, he
transformed the Pratihara Empire into a vast and prosperous realm. Hence, statement 1 is incorrect.
He is known for establishing a strong administration that fostered growth. Hence, statement 2 is correct.
Mihir Bhoja was a devotee of Lord Vishnu, not Buddha. Hence, statement 3 is incorrect.

Additional Information

Military Campaigns:

He is considered a great general and empire builder.


He successfully expanded his territory by conquering regions to the south, including Malwa, Deccan, and Gujarat.
He faced challenges from rival empires like the Palas in the east and the Rashtrakutas in the south.
Religious Beliefs:
A devotee of Lord Vishnu, he adopted the title of Ādivarāha (the Primal Boar, an avatar of Vishnu).

Legacy :

He is remembered as one of the most outstanding political figures of 9th century India.
He is credited with establishing a strong and stable administration that fostered economic growth and cultural development.
There's some debate among historians regarding his ethnicity. Some claim he belonged to the Rajput community, while others believe
he was from a Gurjar background.

Que. 38 The Gurjara-Pratihara dynasty rose to prominence in medieval India by:

1. Establishing control over the lucrative spice trade routes in the south.
2. Successfully repelling Arab invasions from the west.
3. Forming a strategic alliance with the Pala dynasty in the east.
4. Subjugating the powerful Rashtrakuta empire in the Deccan.

Correct Option - 2
The correct answer is Option 2.
Key Points
Gurjara- Pratihara dynasty : The Gurjara-Pratihara dynasty was a significant power in North India during the medieval period,
ruling from roughly the mid-8th century to the 11th century CE.

While the exact origin of the Gurjaras is debated, their kingdom gained prominence under the leadership of Nagabhata I (730-760
CE). Nagabhata I is credited with successfully halting the Arab advance. He defeated Arab armies led by Junaid and Tamin,
effectively stopping their eastward expansion. This victory not only secured the Gurjara-Pratihara territories but also protected a
significant part of North India from Arab domination. Hence Option 2 is correct.

Additional Information

In the 8th century CE, the Arab armies were on a major expansion spree, conquering vast territories in the Middle East, North Africa,
and parts of Central Asia.Their sights then turned towards India, particularly the lucrative Indus Valley region.
This is where the Gurjara-Pratiharas emerge as a significant force. Their rise to power coincides with the Arab attempts to push
eastward into India.

Geographical Reach:
Their empire initially centered around Ujjain and later shifted to Kannauj, a strategic city vying for control in North India.

21/262
At its peak, their territory stretched from Gujarat and Kathiawar in the west to parts of Bengal in the east.

Important Rulers:
Nagabhata I (730-760 CE): Known for halting Arab advancements.
Mihir Bhoja (836-885 CE): Considered the most powerful ruler, expanding the empire and promoting cultural achievements.
Mahendrapala I (885-910 CE): Consolidated the empire's power and prosperity.

Military Prowess:
The Gurjara-Pratiharas were renowned for their military strength, successfully defending against Arab invasions and engaging in power
struggles with rival dynasties like the Palas and Rashtrakutas.

Administration and Legacy:


They established a strong centralized administration that facilitated trade and economic growth.
They were patrons of art, architecture, and literature, contributing to the development of the "Pratihara style" of temple architecture.
Their decline began in the 10th century due to internal conflicts and external pressures from other rising dynasties.

Que. 39 Consider the following statements


1. The Sangam academies were centers of learning and literature that flourished under the patronage of the Chola kings.
2. Sangam literature, written in Old Tamil, provides valuable insights into the social and political life of the Tamils during this period.
Which one of the following is correct in respect of above statements?

1. Both statements are individually true and statement 2 is correct explanation of statement 1.
2. Both statements are individually true but statement 2 is not the correct explanation of statement 1.
3. Statement 1 is true; Statement 2 is false.
4. Statement 1 is false; Statement 2 is true

Correct Option - 4
The correct answer is Option 4.
Key Points
Sangam age :The Sangam age, also referred to as the Sangam period, was a flourishing era in the history of ancient Tamil Nadu, Kerala
(then known as Tamilakam), and parts of Sri Lanka, roughly spanning from the 3rd century BCE to the 3rd century CE.

The Sangam academies are traditionally believed to have flourished under the patronage of the Pandya kings, not the Cholas. Hence,
statement 1 is false.
Sangam literature, written in Old Tamil, is indeed a valuable source of information about the social and political life of the Tamils
during the Sangam period. Hence, Statement 2 is true.

Additional Information
It was named after the legendary Sangam academies of poets and scholars believed to be centered in the city of Madurai.
The Sangam age was a time when the Tamil region was ruled by the three dynasties - the Cheras, Cholas, and Pandyas - along with
some independent chieftains known as the Velir.
The most important source of information about the Sangam age is the Sangam literature, a vast collection of poems written in Old
Tamil. These poems offer insights into the social, political, and cultural life of the Tamils during this period .
The Sangam academies are believed to have flourished under the patronage of the Pandya kings. Scholars and poets gathered at these
academies to discuss and refine their works.
The exact dating of the Sangam period is a topic of debate among historians, but it is generally considered to be a period of great
cultural and literary significance in the history of South India.

Que. 40 Consider the following statements about Anubhava Mantapa:


1. It was established by Basavanna in the 12th century.
2. It served as a platform for open discussions on spiritual matters, irrespective of caste or gender.
3. It was a temple dedicated to the worship of Shiva.
How many of the statements given above is/are correct?

1. Only one
2. Only two
3. All three
4. None

Correct Option - 2
The correct answer is Option 2.
Key Points
Anubhav mantapa : "Anubhav Mantapa" is a term that originates from Indian culture and philosophy, specifically from the teachings of
Basavanna, a 12th-century philosopher and social reformer who is considered one of the most important saints in the Lingayat tradition.

https://lms.testbook.com/genpdf/fromques.php?qids=65f93cf52fe08b6c70dc4d11,65f93e6eb5431ddfb31dfe15,65f96fd18acd632ab8e39282,65f9712de8d50… 22/262
Anubhava Mantapa, also known as the "Hall of Spiritual Experience," was an institution established by Basavanna in the 12th century
in the kingdom of Kalyana (present-day Basavakalyan in Karnataka, India). Hence , statement 1 is correct.
The Anubhava Mantapa functioned as a forum for open discussions on spiritual matters, where participants from all backgrounds could
engage freely regardless of caste or gender. This concept of inclusivity was revolutionary for its time. Hence , statement 2 is correct.
The Anubhava Mantapa wasn't a temple dedicated solely to Shiva worship. While Basavanna was a Veerashaiva saint devoted to
Shiva, the purpose of the Mantapa was broader. It focused on spiritual experiences and discussions, not specific rituals. Hence ,
statement 3 is incorrect.

Additional Information
Basavanna believed in the importance of individual spiritual experience (anubhava) over traditional rituals and dogma. Anubhava
Mantapa played a crucial role in fostering a sense of equality and social justice by challenging the prevailing caste system and
advocating for the dignity and rights of all individuals.

The Anubhava Mantapa was instrumental in spreading the teachings of Basavanna and the Lingayat philosophy, which emphasized
devotion, social equality, and ethical living.

It contributed significantly to the socio-religious reformation in medieval Karnataka and continues to be revered as a symbol of
progressive thought and inclusivity in Indian history and philosophy.

Lingayat tradition:

Lingayats revere Shiva as the supreme being and wear the Ishta Linga, a consecrated representation of Shiva, throughout their lives.
Daily devotion and rituals are important aspects of the tradition.

Lingayats reject the caste system and believe in the equality of all individuals. They believe everyone is capable of spiritual
attainment regardless of birth. This concept of equality is central to the Lingayat philosophy.

The Lingayat tradition emphasizes honest work (Kayak) as a form of worship. Productive labor and fulfilling one's duties are seen as
ways to serve God and achieve spiritual progress.

Living an ethical life (Achara) is a core principle in Lingayatism. This includes principles like truthfulness, non-violence, non-
stealing, sexual purity, and social responsibility.

Que. 41 Which of the following statements about Bastille Day is most accurate?

1. It celebrates the establishment of a new dynasty in France.


2. It is a day to remember the sacrifices made for absolute monarchy.
3. It marks the beginning of a period of increased social and political inequalities.
4. It reflects the fight for liberty, equality, and fraternity during the French Revolution.

Correct Option - 4
The correct answer is Option 4.
Key Points
Bastille Day: This event, which took place on July 14, 1789, is considered a pivotal moment in the French Revolution. The Bastille, a
prison in Paris, symbolized the oppressive rule of the monarchy. By storming it, revolutionaries signaled their defiance and marked the
beginning of a period of radical change.
Bastille Day reflects the ideals of the Revolution: Liberty, Equality, and Fraternity became the rallying cries of the French Revolution.
The storming of the Bastille represented a fight for these ideals – liberty from oppression, equality before the law, and fraternity
(brotherhood) among citizens.
Bastille Day celebrates the overthrow of the Ancient Régime: The "Ancient Régime" translates to "Old Regime" and refers to the social
and political system in France before the revolution. This system was characterized by absolute monarchy, rigid social hierarchies (estates),
and widespread poverty. The storming of the Bastille became a symbol of overthrowing this system and paving the way for a more egalitarian
society.

Causes of the French Revolution:


Social Inequality: The Ancien Régime placed a heavy burden on the common people (Third Estate) through high taxes and limited social
mobility.
Economic Hardship: Widespread poverty and food shortages fueled discontent among the masses.
Enlightenment Ideals: Philosophers like Montesquieu, Voltaire, and Rousseau challenged the absolute power of the monarchy and
advocated for reason, liberty, and individual rights. These ideas inspired revolutionary fervor.

Legacy of Bastille Day:


French National Identity: Bastille Day remains a significant symbol of French national identity, celebrating the nation's commitment to
liberty and equality.
Global Impact: The French Revolution's ideals inspired revolutions and independence movements worldwide, promoting the concepts of
democracy and human rights.

23/262
Que. 42 Consider the following statement
1. The Mylara cult, prevalent in South India, emphasizes inclusivity and welcomes devotees from all social backgrounds,
challenging the rigidities of the caste system.
2.Syncretic religious movements in India typically promote social equality and inclusivity.
Which one of the following is correct in respect of the above statements?

1. Both Statement-I and Statement-II are correct, and Statement-II is the correct explanation for Statement-I.
2. Both Statement-I and Statement-II are correct, and Statement-II is not the correct explanation for Statement-I.
3. Statement-I is correct, but Statement-II is incorrect.
4. Statement-I is incorrect, but Statement-II is correct

Correct Option - 2
The correct answer is Option 2.
Key Points
Mylara cult: The Mylara cult was a religious tradition in the Deccan region during the medieval period. The cult is centered around the
worship of a goddess named Mylara, believed to be a form of Lord Shiva.

The Mylara cult is known for its emphasis on social equality and inclusivity. It welcomes devotees from all backgrounds, regardless of
caste, which was a radical concept during its time. Hence, Statement 1 is correct.
Some syncretic religious movements in India do promote social equality and inclusivity. However, this is not always the case. Some
syncretic movements might simply blend rituals and beliefs without necessarily challenging social hierarchies. Hence , statement 2 is
correct but not correct explanation of Statement 1.

Additional Information
Origin and Deities:

Emerged around the 12th century CE, associated with the Lingayat saint and philosopher Siddharama.
Focuses on the worship of Mylara, a folk deity believed to be a manifestation of Lord Shiva. Mylara is also known by various names
like Mailara, Khandoba, and Khandnatha.
Geographical Spread:
Primarily prevalent in the southern regions of India, encompassing Karnataka, Maharashtra, Andhra Pradesh, and Tamil Nadu.
Unique Aspects:
Syncretic nature: Blends elements of Hinduism with local folk traditions, creating a distinct devotional practice.
Social equality: Unlike the caste system, the Mylara cult emphasizes inclusivity and welcomes devotees from all backgrounds.
Historical Significance: Recent discoveries of sculptures in Basrur, Karnataka, shed light on the cult's presence in coastal regions
during the medieval period. These sculptures depict a warrior figure associated with Mylara

Que. 43 Consider the following statements about Seethakali folk art:


1. It is a traditional dance-drama performance from Kerala, focusing on the Ramayana.
2. It is performed exclusively by Brahmin priests during religious ceremonies.
3. The costumes and makeup used in Seethakali are known for their vibrant colors and elaborate designs.
How many of the statements given above is/are correct?

1. Only one
2. Only two
3. All three
4. None

Correct Option - 2
The correct answer is Option 2.
Key Points
Seethakali folk art : Seethakali is a centuries-old folk art form originating from Kerala, India. It's a captivating blend of storytelling,
dance, and drama, specifically focusing on the epic Ramayana.

Seethakali is indeed a traditional folk art form from Kerala that revolves around the Ramayana epic. Hence , statement 1 is correct.
Seethakali is not performed exclusively by Brahmin priests. Traditionally, it is performed by the Vedar and Pulaya communities of
Kerala. Hence , statement 2 is incorrect.
Seethakali performances are known for their vibrant costumes and elaborate makeup, adding to the visual appeal of the
storytelling. Hence , statement 3 is correct.

Additional Information

Origin and Performance:

24/262
Believed to have originated around 150 years ago in Perinad, Kollam district.
Traditionally performed by the Vedar and Pulaya communities during the harvest festival Onam.
Currently, there's only one registered Seethakali performing group – Perinad Seethakali Sangham, recognized by the Kerala Folklore
Akademi.

Storytelling Through Performance:


The central narrative revolves around the Ramayana, particularly the struggles and triumphs of Sita, Rama, and Lakshmana.
Performances portray key events like Sita's abduction by Ravana, Rama's search, and the epic battle between Rama and Ravana.
The focus isn't strictly chronological, but rather on highlighting themes of devotion, perseverance, and the ultimate victory of good over
evil.

Visual Spectacle:
Vivid makeup and costumes enhance the visual appeal. Characters like Rama and Lakshmana are often adorned in green, a color
symbolizing gods and goddesses in Kathakali, another prominent Keralan art form.
Elaborate headgear and jewelry add to the visual drama.
Natural materials like bamboo and palm leaves are used in the creation of props and costumes, showcasing artistic ingenuity.

Music and Movement:


Accompanied by captivating folk music and songs, passed down through generations.
The music creates a dynamic atmosphere, heightening emotions throughout the performance.
Rhythmic movements and gestures by the performers enhance storytelling and emotional expression.

Significance and Revival Efforts:


Seethakali serves as a testament to Kerala's rich cultural heritage and storytelling traditions.
It showcases the artistic abilities and cultural identity of the Vedar and Pulaya communities.
In recent times, efforts have been undertaken to document, preserve, and promote this unique art form, ensuring its survival for future
generations.

Seethakali Today:
While facing challenges of modernization and changing performance landscapes, Seethakali continues to hold cultural significance in
Kerala.
Increased recognition and support can ensure its continued practice and appreciation, not just within Kerala, but also on a wider stage.

Que. 44 Consider the following statements about the Hoysala dynasty and its artistic contributions:
1. Hoysala temples are known for their star-shaped plans and intricate carvings on the exterior walls.
2.The Hoysala style is a distinct departure from the earlier Dravidian temple architecture of South India.
3.Soapstone was the preferred material for Hoysala sculptures due to its coarse texture and durability.
How many of the statements given above is/are correct?

1. Only one
2. Only two
3. All three
4. None

Correct Option - 1
The correct answer is Option 1.
Key Points
Hoysala dynasty : The Hoysala dynasty (1026 CE – 1343 CE) ruled a significant part of southern India and left behind a remarkable
legacy in art and architecture, particularly focusing on intricate temple design

Hoysala temples are indeed recognized for their intricate carvings and distinctive star-shaped plans on the outer walls. Hence,
statement 1 is correct.
The Hoysala style actually developed from the earlier Western Chalukya style, which itself had roots in Dravidian architecture. The
Hoysala style incorporated core Dravidian principles while adding unique elements.Hence, statement 2 is incorrect.
Soapstone was preferred by Hoysala sculptors precisely for the opposite reason mentioned. Soapstone (steatite) has a fine texture that
allowed for incredibly detailed and intricate carvings. Hence, statement 3 is incorrect.

Additional Information
Architectural Style:
Star-shaped Temples: Hoysala temples are known for their trikuta (three-shrined) and ekashika (single-shrined) layouts, often with
a star-shaped plan on the outer walls. This intricate design creates a sense of dynamism and visual complexity.
Exuberant Decorations: The most striking feature of Hoysala temples is their exuberant sculptural detail. Every inch of the outer walls
is adorned with intricate carvings depicting deities, mythological scenes, geometric patterns, and everyday life.
Famous Examples:

Chennakeshava Temple, Belur: This temple exemplifies the Hoysala style with its intricate carvings, detailed pillars, and a hoysala
mantapa (hall with a unique pillared roof).
https://lms.testbook.com/genpdf/fromques.php?qids=65f93cf52fe08b6c70dc4d11,65f93e6eb5431ddfb31dfe15,65f96fd18acd632ab8e39282,65f9712de8d50… 25/262
Hoysaleswara Temple, Halebidu: This unfinished masterpiece showcases the grandeur of Hoysala architecture, with intricate carvings of
deities, animals, and mythical creatures.
Kesava Temple, Somanathapura: Known as the "Star-shaped Temple" due to its plan, this temple is renowned for its well-preserved
sculptures and balanced proportions.
The Sacred Ensembles of the Hoysala, the famed Hoysala temples of Belur, Halebid and Somanathapur in Karnataka have been added
to the United Nations Educational, Scientific and Cultural Organization (UNESCO) World Heritage list in 2023 . This inclusion marks
the 42nd UNESCO World Heritage Site in India.

Sculptural Techniques:

Narrative Reliefs: Temple walls were often adorned with narrative reliefs depicting stories from Hindu epics like the Ramayana and
Mahabharata, alongside scenes from everyday life.
Sensuality and Dynamism: The sculptures are known for their sensuality, capturing the grace and beauty of celestial beings.
Additionally, the artists infused a sense of dynamism into the figures, often portraying them in action poses.
Significance of Hoysala Art:

Last Great Temples of the Deccan Plateau: The Hoysala temples represent the culmination of Dravidian temple architecture in the
Deccan Plateau region, before the rise of Islamic influence.
Fusion of Styles: Hoysala art reflects a fusion of Dravidian and North Indian styles, showcasing a unique artistic blend.

Que. 45 Consider the following statements


1. The Ramon Magsaysay Award is the highest honor bestowed in Asia and is often referred to as the Asian Nobel Prize.
2. The award is presented in fixed categories that recognize exceptional service in various fields.
Choose the correct answer

1. Both statements I and II are true.


2. Both statements I and II are false.
3. Statement I is true; Statement II is false.
4. Statement I is false; Statement II is true

Correct Option - 3
The correct answer is Option 3.
Key Points
Ramon Magsasay award: Established in 1957 as Asia's highest honour and premier prize. It celebrates individuals who demonstrate
exceptional spirit in serving the people of Asia, regardless of their background. The award is presented annually on August 31st, which
coincides with the birthday of Ramon Magsaysay, the third president of the Republic of the Philippines, who inspired its creation.

Ramon Magsaysay Award is considered Asia's highest honor and is internationally recognized as the Asian equivalent of the Nobel
Prize. Hence statement 1 is correct.
The award initially featuring six categories, it clarifies that after 2009, the award is no longer given in fixed categories except for the
"Emergent Leadership" category. Hence statement 2 is false.

Additional Information
The award initially featured six categories, including “Government Service”, “Public Service”, “Community Leadership”, “Journalism,
Literature, and Creative Communication Arts”, “Peace and International Understanding”, and “Emergent Leadership”.
However, post-2009, the Ramon Magsaysay Award is no longer being given in fixed Award categories, except for Emergent
Leadership.
Recently, Surgical oncologist Padma Shri Dr R. Ravi Kannan, Director of the Cachar Cancer Hospital and Research Centre (CCHRC) in
Assam, received the prestigious Ramon Magsaysay Award for 2023.

He won the award for revolutionising the treatment of cancer in Assam through his people-centered and pro-poor programmes (offering free
treatment, food, lodging, and employment for caregivers) .

Que. 46 Consider the following statements about the architectural style of the Old Parliament House (Samvidhan Sadan):
1. It is built in the Gothic Revival style with pointed arches and stained-glass windows.
2. It features a neoclassical design with red sandstone walls and a white dome.
3. The building layout incorporates elements inspired by the Indian mandala.
How many of the statements given above is/are correct?

1. Only one
2. Only two
3. All three

26/262
4. None

Correct Option - 2
The correct answer is option 2.
Key Points
Samvidhan sabha : The old house of the Indian Parliament, also known as the Samvidhan Sadan, is a circular building designed by
British architects Edwin Lutyens and Herbert Baker . It was built between 1921 and 1927 as part of the construction of New Delhi as the new
capital of British India.

The building features a neoclassical architectural style, with red sandstone walls and white dome accents. Hence, Statement 1 is
Incorrect.
This style draws inspiration from the architecture of ancient Greece and Rome. It emphasizes symmetry, proportion, and the use of
classical elements like columns, domes, and pediments. Red sandstone walls and a white dome, both of which are commonly used
materials in neoclassical architecture. Hence, Statement 2 is correct.
The building has a circular design, which was inspired by the Indian mandala. The mandala is a spiritual and ritual symbol that
represents the universe in Hinduism and Buddhism. Hence, Statement 3 is correct.

Additional Information
Central Chamber: At the center of the building is a circular Central Chamber. This chamber was originally used for meetings of the
Chamber of Princes, which represented the various princely states of India. Today, it is used for special occasions.
Surrounding Halls: Surrounding the Central Chamber are three semicircular halls. These halls were originally used for the sessions of
the Central Legislative Assembly (later used for the Lok Sabha), the State Council (later used for the Rajya Sabha), and the Chamber of
Princes (now used as the Library Hall).
Columns and Dome: The building is surrounded by 144 columns on the outside. The central dome is topped with a chhatri, a decorative
structure commonly found in Indian architecture.
Gardens and Jali: The former parliament is surrounded by large gardens and the perimeter is fenced off by sandstone railings with
intricate jali work. Jali is a perforated screen made from stone or wood that allows for ventilation and light while still providing privacy.

Que. 47 Consider the following statements


1. The Chausath Yogini Temples are a group of Hindu temples dedicated to the 64 manifestations of Lord Shiva.
2 . These temples are known for their unique circular design with individual shrines for each Yogini.
Choose the correct answer:

1. Both statements I and II are true.


2. Both statements I and II are false.
3. Statement I is true; Statement II is false.
4. Statement I is false; Statement II is true.

Correct Option - 4
The correct answer is option 4.
Key Points
Yoginis: The Yoginis are a group of 64 female deities associated with tantric practices and aspects of the Divine Mother. They are believed
to be fierce protectors and possess the power to grant spiritual boons to devotees.

The Chausath Yogini Temples are dedicated to the Yoginis, a group of 64 female deities associated with tantric practices and the
Divine Mother, not Lord Shiva. Hence, statement 1 is false.
The unique circular design of these temples with individual shrines or niches for each Yogini statue. Hence, statement 2 is correct.

Additional Information
Architectural Style:

The most striking characteristic of these temples is their circular layout. This circular wall typically houses individual niches or
shrines for each Yogini statue. The central area within the circle might contain a larger shrine dedicated to a specific deity, often Shiva.

Building materials vary depending on location and age, with sandstone and granite being commonly used. Some temples might have a
slightly different number of Yogini sculptures, but the essence remains the same.

Examples of Chausath Yogini Temples:

Chausath Yogini Temple, Bhedaghat (Madhya Pradesh): This well-preserved temple is one of the most famous, featuring 81
shrines. The additional shrines could signify other deities or be a mark of royalty associated with its construction.
Chausath Yogini Temple, Mitaoli (Madhya Pradesh): This 11th-century temple with 65 chambers is another notable example,
known for its historical significance

Que. 48 The Archaeological Survey of India (ASI) collaborates with other organizations for various projects. Which of the following is the
LEAST likely collaborator for the ASI?
27/262
1. Universities and research institutions for archaeological research projects.
2. State governments for the maintenance of historical sites under their jurisdiction.
3. International organizations dedicated to cultural heritage preservation
4. Private companies for the development of tourism infrastructure near historical sites.

Correct Option - 3
The correct answer is option 3.
Key Points
Archaeological Survey India: The Archaeological Survey of India (ASI) plays a crucial role in safeguarding and understanding India's rich
cultural heritage.The ASI is responsible for identifying, protecting, and maintaining thousands of monuments and archaeological sites across
India.

International organizations dedicated to cultural heritage preservation, like UNESCO (United Nations Educational, Scientific and
Cultural Organization), often work on broader initiatives and policy frameworks. They might collaborate with the ASI on specific
projects of international significance, but such collaborations wouldn't be a regular occurrence in the ASI's day-to-day operations.
Hence , Statement 3 is correct.

Additional Information
Importance of ASI:

Documentation and Research: The ASI conducts excavations, surveys, and research projects at various sites. Reports document these
activities, providing valuable information about past civilizations, cultural practices, and artistic expressions. These reports serve
as a primary source for historians, archaeologists, and researchers studying India's past.
Understanding Cultural Evolution: By analyzing artifacts, architecture, and inscriptions, ASI reports contribute to our
understanding of how Indian culture has evolved over time. They shed light on social structures, economic systems, religious beliefs,
and artistic developments in different periods.
Tourism and Education: ASI reports form the basis for developing informative content for museums, signage at historical sites, and
educational materials. This fosters public awareness and appreciation for India's cultural heritage.

Importance of ASI Reports:


Credible Source: ASI reports are prepared by trained professionals following established archaeological methods. They provide reliable
and verifiable information about historical sites and discoveries.
Reference Material: These reports serve as a reference point for further research by academics and students interested in Indian history
and archaeology.
Planning and Conservation: ASI reports inform decisions related to conservation efforts, restoration projects, and the management of
historical sites. They provide insights into the condition of monuments and recommend suitable preservation strategies.
Promoting Public Interest: Reports can spark public interest in archaeology and the importance of preserving cultural heritage. They can
raise awareness about the threats historical sites face and encourage public support for conservation efforts.

Que. 49 The Tankai method, recently in news for revival efforts, is a traditional technique associated with:

1. Weaving intricate silk textiles


2. Constructing irrigation canals
3. Building earthquake-resistant structures
4. Stitching wooden planks for shipbuilding

Correct Option - 4
The correct answer is option 4.
Key Points
Tankai method: The Tankai method is a fascinating 2,000-year-old shipbuilding technique from India known for its use of stitching instead
of nails to construct ships. Hence, option 4 is correct.
Historical Significance:
The Tankai method played a crucial role in India's maritime history, enabling the construction of sturdy and reliable ships for trade,
exploration, and cultural exchange.
This technique was particularly valuable for small local fishing boats, where flexibility and resilience in challenging coastal waters were
essential.

Current Status:
While the Tankai method faced decline with the arrival of European shipbuilding techniques that relied heavily on nails, there's a
renewed interest in reviving this traditional method.
The Ministry of Culture and the Indian Navy have signed a Memorandum of Understanding (MoU) to revive the Tankai method,
recognizing its potential benefits and historical significance.

Construction Technique:
Unlike conventional shipbuilding that relies on nails or bolts, the Tankai method involves stitching wooden planks together.
Strong threads or even coir (coconut fiber) ropes are used for stitching, creating a flexible and robust hull.

28/262
Advantages:
Flexibility: Stitched construction allows the hull to flex slightly in rough waters, potentially reducing stress and damage compared to rigid
nailed ships.
Durability: The stitching technique, if done well, can create a watertight seal and offer long-lasting durability, especially beneficial in
areas with frequent repairs due to harsh conditions.

Que. 50 Consider the following statements


1. The Sangeet Natak Akademi primarily focuses on promoting contemporary and experimental forms of music, dance, and drama.
2. The Akademi's Fellowship (Ratna Sadsya) awards prioritize recognizing young, upcoming artists with exceptional talent.
3. The Akademi's initiatives contribute significantly to fostering intercultural dialogue and exchange within the performing arts.
How many of the statements given above is/are correct?

1. Only one
2. Only two
3. All three
4. None

Correct Option - 1
The correct answer is Option 1.
Key Points
Sangeet Natak Akademi : The Sangeet Natak Akademi is the national level academy for performing arts set up by the Government of
India. The Akademi awards the prestigious Sangeet Natak Akademi Awards, considered the highest national recognition for practicing artists,
gurus, and scholars in these performing arts.

While the Akademi might support some contemporary forms, its emphasis is on preserving and promoting India's traditional
performing arts heritage. Hence, statement 1 is incorrect.
Fellowships are awarded to distinguished individuals for their lifetime contributions, not just young talent. Hence, statement is 2
incorrect.
The Akademi's exchange programs and collaborations with artists from other countries promote intercultural dialogue. Hence,
statement 3 is incorrect.

Additional Information
Preservation and Promotion: Established in 1953, the Akademi's primary mission is to preserve and promote India's vast and
diverse heritage in the fields of music, dance, and drama.
Fellowships and Awards: The Akademi also confers Fellowships (Ratna Sadsya) to distinguished individuals for their lifetime
contributions. Additionally, the Ustad Bismillah Khan Yuva Puraskar recognizes and encourages young talent under 40 years of age.
Documentation and Research: The Akademi maintains a vast archive of recordings, photographs, films, and other materials
related to performing arts. This serves as a valuable resource for researchers and scholars.
Promotion and Outreach: The Akademi organizes performances, workshops, seminars, and conferences to promote public
appreciation and understanding of Indian performing arts.

Que. 51 Consider the following statements about Rashtriya vigyan puraskar


1.The Vigyan Ratna Award is presented annually on National Science Day. Statement
2. The Rashtriya Vigyan Puraskar includes categories specifically for young scientists.
Choose the correct answer

1. Both Statement 1 and Statement 2 are individually true and Statement 2 provides the correct explanation of Statement 1.
2. Both Statement 1 and Statement 2 are individually true but Statement 2 does not provide the correct explanation of Statement 1.
3. Statement 1 is true; Statement 2 is false.
4. Statement 1 is false; Statement 2 is true

Correct Option - 4
The correct answer is option 4.
Key Points
Rashtriya vigyan puraskar : The Rashtriya Vigyan Puraskar (RVP) is a set of national awards in India that recognize and honor
outstanding contributions in the field of science, technology, and innovation. It was established by the Government of India in 2023.

The Vigyan Ratna Award, part of the Rashtriya Vigyan Puraskar awards, is presented annually, but not on National Science Day
(February 28th).The Award Ceremony for all categories of awards will be held on 23rd August (National Space Day). Hence,
Statement 1 is false.
The Rashtriya Vigyan Puraskar does indeed include a category specifically for young scientists - the Vigyan Yuva Puraskar. Hence,
Statement 2 is true

Additional Information
29/262
The Rashtriya Vigyan Puraskar shall be one of the highest recognitions in the field of science, technology, and innovation in India.
Scientists/ technologists/innovators working in government, private sector organizations or any individual working outside any
organization, who have made distinguished contributions in terms of path-breaking research or innovation or discovery in any field of
science, technology, or technology- led innovation shall be eligible for the awards.
People of Indian Origin staying abroad with exceptional contributions benefiting the Indian communities or society shall also be
eligible for the awards. The awards shall be given in following four categories:-
Vigyan Ratna (VR) award will recognize lifetime achievements & contributions made in any field of science and technology.
Vigyan Shri (VS) award will recognize distinguished contributions in any field of science and technology.
Vigyan Yuva-Shanti Swarup Bhatnagar (VY-SSB) award will recognize & encourage young scientists up to the age of 45 years who
made an exceptional contribution in any field of science and technology.
Vigyan Team (VT) award to be given to a team comprising of three or more scientists/researchers/innovators who have made an
exceptional contribution working in a team in any field of science and technology.
Scientists, technologists, and innovators working in any field of science in government or private organizations who have
contributed or carried out a path breaking research, technology led innovation or discovery in any field of science or development of
innovative technologies/products having significant societal impact shall be eligible for the award.
Scientists, technologists, and innovators of Indian origin abroad with exceptional contributions benefitting the Indian communities or
society at large shall also be eligible.
The Rashtriya Vigyan Puraskar shall be given in the 13 domains, namely Physics, Chemistry, Biological Sciences, Mathematics &
Computer Science, Earth Science, Medicine, Engineering Sciences, Agricultural Science, Environmental Science, Technology &
Innovation, Atomic Energy, Space Science and Technology, and Others. The representation from each domain/field, including gender
parity will be ensured.

Que. 52 Consider the following statements


1. Rural tourism in India is a well-established industry with a significant share of the national tourism market.
2.The Government of India launched the “Incredible Rural India” campaign to promote awareness and development of rural destinations.
Choose the correct answer

1. Both Statement 1 and Statement 2 are individually true and Statement 2 provides the correct explanation of Statement 1.
2. Both Statement 1 and Statement 2 are individually true but Statement 2 does not provide the correct explanation of Statement 1.
3. Statement 1 is true; Statement 2 is false.
4. Statement 1 is false; Statement 2 is true.

Correct Option - 4
The correct answer is option 4.
Key Points
Rural Tourism : Rural tourism, also known as village tourism, is gaining traction in India.India boasts a vast rural landscape rich in
culture, heritage, natural beauty, and unique experiences. Homestays, farm stays, and eco-tourism projects are creating new avenues for
travelers seeking authenticity and connection with nature.

While rural tourism holds immense potential, it's still a developing sector in India. It doesn't currently hold a significant share of the
national tourism market. Hence statement 1 is false.
The "Incredible Rural India" campaign is a government initiative to promote rural tourism development. Hence statement 2 is true.

Additional Information
Potential:
Economic Booster: Rural tourism can empower rural communities by creating jobs in hospitality, handicrafts, local guides, and
homestays. This can generate income, promote local businesses, and contribute to rural development.
Sustainable Tourism: Rural tourism can promote sustainable practices like responsible waste management, water conservation, and
preservation of local flora and fauna.
Government Initiatives: Recognizing the potential, the Indian government launched the "Incredible Rural India" campaign to promote
rural destinations. Additionally, schemes like "Swadesh Darshan" and "Heritage homestay scheme" provide financial assistance and
training for rural tourism development.

Que. 53 Consider the following statements


1. The UNWTO recently awarded Dhordo village in Gujarat the distinction of "Best Tourism Village.

2. The UNWTO is an intergovernmental organization with a focus on promoting responsible tourism practices.

3. The UNWTO has four official languages, including English, French, spanish and Arabic
How many of the following statements above is/are correct?

1. Only one
2. Only two
3. All three
30/262
4. None

Correct Option - 2
The correct answer is option 2.
Key Points
UNWTO: The United Nations World Tourism Organization (UNWTO) is the United Nations agency responsible for promoting responsible,
sustainable, and universally accessible tourism.

Recently, the Prime Minister of India hailed the Dhordo village in Kutch district of Gujarat for being awarded as the Best Tourism
Village by the United Nations World Tourism Organisation (UNWTO). Hence statement 1 is correct.
The UNWTO's focus on promoting responsible tourism practices aligns with its mission of sustainable and ethical tourism
development. Hence statement 2 is correct.
The UNWTO actually has six official languages: Arabic, Chinese, English, French, Russian, and Spanish. Hence statement 3 is
incorrect.

Additional Information
UNWTO's key functions:

Global Forum: It acts as a platform for discussing and addressing tourism policy issues on a global scale.
Sustainable Practices: It encourages the implementation of the Global Code of Ethics for Tourism, promoting ethical and sustainable
tourism practices worldwide.
Knowledge & Research: It serves as a source of tourism research and knowledge, aiding informed decision-making in the tourism
sector.
It was established in 1975. Membership of the UNWTO includes 159 countries.

Dhordo village :

Dhordo village, located in the Kutch district of Gujarat, India, achieved international recognition in 2023 by being awarded the
prestigious title of "Best Tourism Village" by the United Nations World Tourism Organization (UNWTO).
This recognition highlights Dhordo's unique contributions to sustainable tourism practices.
Tourism in Dhordo is largely community-driven, with villagers directly involved in hosting tourists and showcasing their culture and
traditions.
Dhordo offers tourists a glimpse into the rich cultural heritage of the Kutchi people. Visitors can experience traditional Kutchi cuisine,
music, dance, and art forms.

Que. 54 Mauryan Period


Functions
Officers
Bodyguards or personal attendants to the
Amatyas
emperor
Sannidhatas Superintendents of specific departments
Rajukas Revenue collectors in specific regions

How many of the above pairs are correctly matched ?

1. Only one
2. Only two
3. All three
4. None

Correct Option - 1
The correct answer is option 1.
Key Points
Mauryan Administration: The Mauryan Empire boasted a well-organized administrative structure with various officials overseeing
different aspects of governance.
Here are some terms used for officers during the Mauryan period:
Amatyas: These were civil servants who managed the day-to-day administration of the empire. They can be considered similar to
present-day bureaucrats. They oversaw various departments like agriculture, trade, and revenue collection.
Mantrins: This term referred to the council of ministers who advised the king. Important members included the Purohita (chief priest),
Mahamantri (chief advisor), Senapati (commander-in-chief), and Yuvaraja (crown prince).
Tirtha: According to Kautilya's Arthashastra, an influential ancient Indian text, Tirtha referred to a group of 18 high-ranking officials
who held significant responsibility within the Mauryan administration.
Adhyakshas: These were superintendents or officials responsible for specific departments or regions. They oversaw areas like
agriculture, trade, and revenue collection.
Sannidhatas: These officers likely served as bodyguards or personal attendants to the Mauryan emperor.
Rajukas: These officials were responsible for overseeing revenue collection in specific regions of the empire.
31/262
Dharma-Mahamatra: This officer functioned as the Chief Justice, ensuring the upholding of justice and adherence to the legal code.

Que. 55 Consider the following statements about Kautilya:


1. He is best known for his work, the Arthashastra, which deals with statecraft, economics, and military strategy.
2. He served as a key advisor to Chandragupta Maurya, the founder of the Mauryan Empire.
3. His political philosophy emphasized the importance of morality and ethics in governance.
Which of the statements given above are correct?

1. Only one
2. Only two
3. All three
4. None

Correct Option - 2
The correct answer is option 2.
Key Points
Kautilya :Kautilya, also known as Chanakya or Vishnugupta, was a prominent figure in ancient India. He is believed to have lived around
the 3rd century BCE. He is widely recognized for his contributions in several fields. He is considered a pioneer in the fields of political
science and economics in India.

His most famous work, the Arthashastra (Science of Polity), is an ancient Indian treatise that delves into statecraft, economic policy,
and military strategy. Hence, statement 1 is correct.
Kautilya served as the chief advisor and strategist to Chandragupta Maurya, the founder of the Mauryan Empire. Hence, statement 2 is
correct.
The Arthashastra focuses more on practical strategies to achieve and maintain power. While it does acknowledge the king's duty to
uphold order and dharma (righteous conduct), the emphasis is on effective statecraft rather than strictly moral governance. Hence,
statement 3 is correct.

Additional Information
His guidance is credited with playing a pivotal role in Chandragupta's rise to power and the establishment of the Mauryan Empire as a
powerful force in ancient India.
The Arthashastra offers valuable insights into Kautilya's views on governance, economics, diplomacy, and military strategy. The text
covers diverse topics like taxation, public administration, international relations, and warfare.
Some scholars have compared him to figures like Niccolò Machiavelli, the Italian diplomat and political philosopher, for his emphasis
on practical strategies for maintaining power.

Que. 56 What is the primary objective of the Indian Military Heritage Festival?

1. To showcase advancements in modern Indian weaponry


2. To facilitate recruitment into the Indian Armed Forces
3. To celebrate and educate the public about India's military history and culture
4. To foster international cooperation in defense matters

Correct Option - 3
The correct answer is option 3.
Key Points

The Indian Military Heritage Festival is a relatively new event, with the first edition being held in October 2023. It is organized by
the United Service Institution of India (USI), the country's oldest defense think tank.
The Festival is being planned to set a new benchmark in the domain of public engagement through interaction and dialogue while
adhering to the goals for the development of our Armed Forces in the 21st century.
The Festival will focus on India’s rich military heritage and traditions; Security & Strategy; and Atmanirbhar Bharat, in the defence
and military domains.
rich military culture and heritage. It features a variety of events, including panel discussions, exhibitions, art displays, and cultural
performances.

The goal is to educate the public about India's military history and traditions, and to inspire the next generation of soldiers.

United Service Institution of India (USI)

The United Service Institution of India (USI) is a national security and defence services think tank based in New Delhi, India. It
describes its aim as the "furtherance of interest and knowledge in the art, science and literature of the defence services".

32/262
The USI was established in 1870 by Major General Sir Charles MacGregor in Shimla. It was founded as a society in the furtherance of
interest and knowledge in the art, science and literature of the Defence Services.

Que. 57 The Aligarh Movement, associated with Sir Syed Ahmed Khan, is known for promoting:

1. Rejection of Western education and emphasis on traditional Islamic teachings.


2. A blend of Western scientific education with Islamic religious studies.
3. Active participation in the Indian independence movement against the British.
4. The idea of Pan-Islamism and solidarity with the Ottoman Caliphate.

Correct Option - 2
The correct answer is option 2.
Key Points
Sir Syed Ahmed Khan : Sir Syed Ahmed Khan was a prominent Muslim reformer and educator in 19th century India. Believed modern
education was crucial for Muslim advancement. Advocated for studying Western science and technology alongside Islamic tradition.

Sir Syed Ahmed Khan, the driving force behind the Aligarh Movement, believed that the educational backwardness of Muslims in
19th century India was a significant factor contributing to their social and political decline. He saw a solution in acquiring modern
scientific knowledge and skills from the West, while simultaneously preserving Islamic values and traditions. Hence, statement
2 is correct.

Aligarh movement:

The Aligarh Movement, spearheaded by Sir Syed Ahmed Khan, emphasized modernization by advocating for Muslims to embrace
Western scientific and technological education. This was a departure from the traditional focus solely on religious studies. Khan
believed acquiring these skills was vital for Muslims to compete in the job market and contribute meaningfully to society.
The movement didn't propose discarding Islamic teachings altogether. Instead, it championed a blend of Western scientific education
with Islamic religious studies. This aimed to create well-rounded individuals who could excel in the modern world while staying
rooted in their Islamic faith and traditions.
A significant contribution of the Aligarh Movement was the establishment of the Mohammedan Anglo-Oriental College in 1875,
which later became Aligarh Muslim University (AMU). AMU became a prominent center of Western education for Muslims in
India, providing opportunities for them to acquire modern knowledge and skills.
Initially, the Aligarh Movement promoted loyalty to the British Raj. Sir Syed believed that cooperation with the British would create a
favorable environment for Muslim education and social reform. However, the movement's focus on education and social empowerment
eventually laid the groundwork for future generations to participate in the Indian independence movement.
The Aligarh Movement's emphasis on education and social reform had a lasting impact. It empowered Muslims in British India by
equipping them with the necessary skills and knowledge to gain social mobility and participate more effectively in public life. AMU
continues to be a symbol of educational empowerment for Muslims in India

Que. 58 Consider the following statements


1. Katalin Karikó and Drew Weissman were awarded the 2023 Nobel Prize in Medicine, whose role played a critical role in the
fight against COVID-19.
2. Their research played a crucial role in the development of mRNA vaccines for COVID-19.
Choose the correct answer

1. Both statements are correct, and statement 2 explains statement 1.


2. Both statements are correct, but statement 2 is not an explanation of statement 1.
3. Statement 1 is correct, but statement 2 is incorrect.
4. Statement 1 is incorrect, but statement 2 is correct

Correct Option - 2
The correct option is option 2.
Key Points
Katalin Karikó and Drew Weissman were jointly awarded the 2023 Nobel Prize in Physiology or Medicine for their discoveries
concerning nucleoside base modifications that enabled the development of effective messenger RNA (mRNA) vaccines.

Katalin Karikó and Drew Weissman were indeed awarded the 2023 Nobel Prize in Physiology or Medicine. Hence , statement 1 is
correct
This statement is also correct. Their research on modified nucleosides in mRNA was critical for overcoming challenges and enabling
the development of effective mRNA vaccines, including the ones used against COVID-19. Hence , statement 2 is correct

However, statement 2 is not an explanation of statement 1. While their work on mRNA contributed to the development of COVID-
19 vaccines, the Nobel committee specifically recognized them for their broader discoveries concerning nucleoside modifications,
which have applications beyond just COVID-19 vaccines.

33/262
Additional Information
Nobel prize:

The Nobel Prize is widely regarded as the most prestigious award given for intellectual achievement in the world. It honors
outstanding work in six categories: Physics, Chemistry, Physiology or Medicine, Literature, Peace, and Economics. Winning a
Nobel Prize signifies a researcher or contributor has made a groundbreaking discovery or contribution that has significantly impacted
their field.

The prize was established by Swedish industrialist and inventor Alfred Nobel in his will. Nobel, known for inventing dynamite,
paradoxically funded a prize to recognize work promoting peace and advancements that benefit humanity.

The Nobel Prize encompasses six distinct categories. Each year, a laureate (an individual winner or a winning organization for the
Peace Prize) is chosen for each category. In some cases, the prize can be shared by up to three individuals who made significant
contributions together.

The Nobel Prize winners and their work often have a global impact. Their discoveries, advancements, or contributions in various
fields can significantly influence scientific progress, social change, or literary movements, leaving a lasting legacy on the world.

Que. 59 Consider the following statements about the Pahari schools of painting:
1. They were primarily influenced by the Mughal miniature style and share a similar focus on historical events.
2. While some Pahari styles used muted colors, most were known for their vibrant palettes and intricate details in clothing and jewelry.
3. The Kangra school, known for its romantic themes, is considered a later development of the Basohli style, known for its bold, dramatic
compositions.
How many of the following given statements is/are correct?

1. Only one
2. Only two
3. All three
4. None

Correct Option - 1
The correct answer is option 1.
Key Points
Pahari schools of paintings: The Pahari schools of painting refer to a collection of artistic styles that emerged in the Himalayan foothills
of North India, flourishing from the 17th to 19th centuries. These paintings are known for their vibrant colors, intricate details, and
focus on mythological and religious themes.

Mughal paintings did influence Pahari styles to some extent, but Pahari art has a stronger emphasis on religious themes and a distinct
focus on the Himalayan landscapes .Hence , statement 1 is incorrect:
Vibrant colors and intricate details are a hallmark of most Pahari schools, with some exceptions like Chamba using a more muted
palette.Hence , statement 2 is correct:
The chronological order is reversed. Basohli, with its bold style, emerged earlier, while the Kangra school, known for its lyrical
romanticism, developed later. Hence , statement 3 is incorrect.

Additional Information
The term "Pahari" itself means "of the hills" in Hindi, reflecting the geographical origin of these styles. Unlike the more centralized
Mughal style, Pahari paintings developed in various princely states, each with its own distinct characteristics.
Some of the major Pahari schools include:

Basohli: Characterized by bold colors, expressive faces, and elongated figures. This early Pahari style is known for its dramatic scenes
and narrative energy.
Kangra: Renowned for their delicate brushwork, lyrical style, and romantic themes, especially love stories of Radha and Krishna. The
Kangra school is considered the pinnacle of Pahari painting, known for its gracefulness and emotional depth.
Chamba: Developed a unique style with a preference for muted colors and a focus on Krishna's life and legends.
Guler: Often considered a precursor to the Kangra style, Guler paintings are known for their simplicity and use of a limited color
palette.
Mandi: Emphasized dramatic poses, strong facial expressions, and a rich use of color.

Que. 60 Which of the following statements correctly describes Mirabai and the Bhakti movement?

1. Mirabai was a South Indian queen who promoted the worship of Shiva through intricate temple rituals.
2. Mirabai's devotional poems in Tamil celebrated the grandeur of Vishnu's cosmic form.

34/262
3. Mirabai, a Rajput princess, defied social norms through her Bhakti poetry dedicated to Krishna.
4. Mirabai's Sanskrit hymns focused on achieving moksha through strict adherence to Vedic scriptures.

Correct Option - 3
The correct answer is option 3.
Key Points
Saint Meera bai: Mirabai, also referred to as Sant Meerabai, was a 16th-century Hindu mystic poet and devotee of Krishna. She is
considered a central figure in the Bhakti movement in medieval India.

She was a Rajput princess who defied societal expectations by expressing her deep devotion to Krishna through her poetry. This
challenged the dominance of rituals and pristliness often associated with upper castes. Hence , statement 3 is correct.

Additional Information
Mirabai was born into a Rajput royal family in Rajasthan. Her exact birthdate is unknown, but it is estimated to be around 1498.
Raised in a religious household, Mirabai developed a deep devotion to Krishna from a young age. There are many stories about her
childhood piety, including one where as a young girl, she declared Krishna as her husband.

Mirabai was married to a prince of another Rajput kingdom, but she continued to focus on her devotion to Krishna. This reportedly
caused friction with her in-laws, who did not approve of her unconventional religious practices. There are many legendary stories
about attempts to poison or harm Mirabai, but she is said to have remained miraculously unharmed.

Mirabai is famous for her devotional poems and songs, which are still sung and cherished today. Her poems express her love for
Krishna in a passionate and personal way. They use everyday language and imagery, making them relatable to people from all walks of
life. Mirabai's poems helped to popularize the Bhakti movement and spread the message of devotion to Krishna.

Mirabai's life story is shrouded in legend, and there are many different accounts of her life and death. However, there is no doubt that
she was a remarkable woman who left a lasting legacy. Her devotional poems continue to inspire people today, and she is considered
one of the greatest saints of the Bhakti movement.

Que. 61 Consider the following statements about Sant Tukaram:


I. Tukaram's devotional poems, called Abhangas, were written in Sanskrit and praised a variety of Hindu deities.
II. Tukaram challenged the social hierarchy through his writings, advocating for equality among devotees of Vithoba.

1. Statement 1 is true and statement 2 is false


2. Statement 1 is false and statement 2 is true.
3. Both statements are True
4. None of the statements are true.

Correct Option - 2
T he correct answer is option 2.
Key Points
Sant Tukaram:
Tukaram, also known as Sant Tukaram Maharaj, was a 17th-century Hindu saint and poet from Maharashtra, India.
Known for: Devotional poetry called Abhangas, written in Marathi, advocating for social equality and personal devotion to God
(Vithoba, a form of Vishnu).
Part of: Varkari sampradaya, a Bhakti movement tradition worshipping Vithoba of Pandharpur.

Tukaram's devotional poems, called Abhangas , were written in Marathi, not Sanskrit. Sanskrit was a more classical language used
by scholars and not as widely understood by the common people. Tukaram aimed to reach a wider audience through his devotional
message. Abhangas are known for their simplicity and use of everyday language. Hence , statement 1 is false.
Tukaram's writings challenged the social hierarchy of his time, particularly the caste system. He advocated for the equality of all
devotees before Vithoba (a form of Vishnu).His message resonated with people from all walks of life, promoting social inclusion.
Hence, statement 2 is true.

Additional Information

Significance:
Literary genius: Considered one of the greatest Marathi poets, his Abhangas are still cherished for their emotional depth, simple
language, and use of metaphors.
Social reformer: His poems challenged the caste system and emphasized the equality of all devotees before God.
Spiritual influence: His works promote a path of bhakti (devotion) as a means to connect with the divine, accessible to people from all
walks of life.

Interesting Facts:

35/262
Tukaram's life had its share of hardships, including business failures, family loss, and social ostracization. These experiences are reflected
in his poetry, adding authenticity and depth to his devotional expression.
Tukaram is revered by the Varkari community, who undertake annual pilgrimages to Pandharpur. His Abhangas are sung during these
pilgrimages and other religious gatherings.

Que. 62 Which of the following statements is not correct about Sharada Peeth?

1. It was a prominent center of learning in the ancient Indian subcontinent.


2. It was a ruined Hindu temple located in Pakistan-administered Kashmir.
3. It played a role in the development of the Grantha script.
4. It is one of the holiest pilgrimage sites for Kashmiri Pandits.

Correct Option - 3
The correct answer is option 3.
Key Points
Sharada peeth: The Sharada Peeth, also referred to as the Sharada Temple, is a ruined Hindu temple and ancient center of learning
located in the Neelum Valley of Pakistan-administered Azad Kashmir .

Sharada Peeth is known for the Sharada script, not the Grantha script. Grantha script is primarily associated with South India.
Hence , statement 3 is not correct.

Additional Information
Considered one of the most prominent temple universities in the Indian subcontinent between the 6th and 12th centuries CE, Sharada
Peeth was renowned for its vast collection of texts and scholars who traveled great distances to access them.
It played a significant role in the development and spread of the Sharada script in North India, with the script itself being named
after the Sharada Peeth.
Kashmir also earned the moniker "Sharada Desh" because of this temple's influence, translating to "country of Sharada."
Holding immense religious and historical significance, Sharada Peeth is one of the three holiest pilgrimage sites for Kashmiri Pandits,
along with the Martand Sun Temple and the Amarnath Temple.
The temple is situated at an elevation of 6,499 feet above sea level, offering breathtaking views of the surrounding landscapes. While it
is roughly 150 kilometers from Muzaffarabad, the capital of Azad Kashmir, and 130 kilometers from Srinagar, the capital of Kashmir,
it is only 10 kilometers away from the Line of Control (LoC), the dividing line between the Pakistani and Indian-controlled areas of
Kashmir.

Que. 63 Consider the following statements about the Indian Olympic Association (IOA):
1.The IOA is directly responsible for training athletes participating in major sporting events like the Asian Games.
2. The IOA selects athletes based solely on their performance in national-level qualifiers.
3. The IOA plays a crucial role in representing India's interests at the International Olympic Committee (IOC).
How many of the statements given above is/are correct?

1. Only one
2. Only two
3. All three
4. None

Correct Option - 1
The correct answer is Option 1.
Key Points
Indian Olympic Association: The Indian Olympic Association (IOA) is the governing body for the Olympic Movement and the
Commonwealth Games in India.

While the IOA plays a role in facilitating training for athletes, it often works in conjunction with national sports federations or
government bodies that manage specific sports and have their own training programs. Hence ,Statement 1 is Incorrect.
The IOA selection process can involve qualifiers, but it might also consider factors like past performance, international rankings, and
potential in specific events. There can be a degree of discretion involved. Hence, Statement 2 is Incorrect.
The IOA is India's sole representative at the IOC, the governing body of the Olympic Movement. It acts as a liaison between India and
the IOC on matters related to the Olympics and other international sporting events. Hence , Statement 3 is Correct.

Additional Information
IOA:
Governing Body: The Indian Olympic Association (IOA) is the apex organization that governs the Olympic Movement and the
Commonwealth Games in India. It's recognized by the Ministry of Youth Affairs and Sports.
International Representation: The IOA acts as the sole representative of India at the International Olympic Committee (IOC) and
other international sports bodies like the Olympic Council of Asia (OCA) and the Commonwealth Games Federation (CGF).

36/262
Athlete Selection and Management: The IOA is responsible for selecting Indian athletes to compete at major sporting events like the
Olympics, Asian Games, and Commonwealth Games. It also manages the Indian teams by coordinating logistics, accreditation, and
other requirements for participation.
Promoting Sports Development: Beyond athlete selection, the IOA plays a role in promoting the development of sports across India.
This might involve collaborating with national sports federations, establishing training programs, or advocating for increased
government support for athletics.
Upholding Olympic Ideals: The IOA is entrusted with upholding the Olympic ideals of fair play, sportsmanship, and international
cooperation within the Indian sporting landscape.

Que. 64 Consider the following statements about the National Games of India:
I. The National Games of India are held annually to select athletes for the upcoming Olympic Games.
II. The Games exclusively feature sports that are also included in the Olympic program.
III. The Games provide a platform for athletes from various Indian states and union territories to compete.
How many of the statements given above is/are correct?

1. Only one
2. Only two
3. All three
4. None

Correct Option - 1
The correct answer is option 1.

Key Points
National games of India: The National Games of India is a prestigious multi-sport event modeled after the Olympics. It's main objective is
to identify and nurture talented athletes from different Indian states and UTs for potential representation at international sporting events like
the Olympics and Asian Games.

The National Games are not held annually and the frequency can vary. They are not the sole selection criteria for the Olympics. Hence,
Statement I - is Incorrect.
The Games include both Olympic and non-Olympic sports, promoting a wider range of athletic disciplines. Hence, Statement 2 is
Incorrect.
The core purpose of the National Games is to provide a platform for athletes from different Indian states and UTs to showcase their
talent and compete for medals .Hence, Statement 3 is correct.

Additional Information
The National Games of India, held periodically, offer a prestigious platform for athletes from all Indian states and union territories
(UTs) to compete. This multi-sport event, modeled after the Olympics, identifies and nurtures promising athletes who could potentially
represent India at international sporting events like the Asian Games and Olympics.
While the National Games feature some Olympic sports like athletics, archery, and wrestling, it goes beyond the Olympic program.
This allows Indian athletes to excel in and gain recognition for indigenous games like kabaddi, kho kho, and mallakhamb.
The National Games serve as a vital platform for athletes outside major sporting hubs to showcase their talent and potentially gain
selection for national teams.

In News

In the 37th National Games held in Goa from October 26 to November 9, 2023, Maharashtra topped with 228 medals including 80
gold, 69 silver and 79 bronze, while Services stood second with 126 medals including 66 gold, 27 silver and 33 bronze.

Que. 65 Consider the following statements about Sea Buckthorn (Hippophae L):
I. It is a plant found only in the cold deserts of Ladakh and Spiti in India.
II. It is a rich source of Vitamin C and is known for its orange-yellow coloured berries.
III. Due to its soil-binding properties, it helps prevent soil erosion and promotes biodiversity.
How many of the statements given above is/are correct?

1. Only one
2. Only two
3. All three
4. None

Correct Option - 2
The correct answer is option 2.
Key Points
37/262
Sea buckthorn: Sea buckthorn (Hippophae rhamnoides), also known as sandthorn, sallowthorn, or seaberry, is a deciduous shrub native to
Europe and Asia. Sea buckthorn oil is extracted from the berries and seeds and is used in cosmetics, dietary supplements, and traditional
medicine. Thrives in dry, cool climates

While Ladakh and Spiti are major producers of Sea Buckthorn in India, it's also found in other dry areas of the Himalayan region.
Hence Statement I is Incorrect.
Sea Buckthorn (Hippophae L) a shrub which produces an orange-yellow colored edible berry is sour in taste but rich in vitamins,
especially Vitamin C. Hence, Statement II is correct
The soil-binding properties and its contribution to preventing erosion and preserving biodiversity. Hence, Statement III is Correct.

Additional Information
Recently, the Geographical Indication (GI) Registry, operating under the Ministry of Commerce & Industry, Government of India,
has awarded the GI tag to the Department of Industries & Commerce, Ladakh, for 'Ladakh Sea Buckthorn'.
Ladakh Sea Buckthorn is a vital crop in Union Territory Ladakh, with 90% of its production originating from the region.It is also
popularly known as 'Wonder Plant', “Ladakh Gold”, “Golden Bush” or “Gold Mine”
Uses:
Food: Berries can be eaten fresh, dried, or juiced. Sea buckthorn oil is used in various food products.
Medicine: Sea buckthorn oil is used in traditional medicine for a variety of purposes, including skin conditions, digestive problems,
and wound healing. However, there is limited scientific evidence to support many of these uses.
Cosmetics: Sea buckthorn oil is a popular ingredient in skincare products due to its moisturizing and antioxidant properties.

Que. 66 Bali Jatra, a major festival and trade fair in India, is known for commemorating:

1. The victory of a historical battle in Odisha.


2. The harvest season in Odisha.
3. The maritime heritage and trade links of ancient Odisha with Southeast Asia.
4. The birth anniversary of a revered Odia saint.

Correct Option - 3
The correct answer is option 3.
Key Points
Bali Jatra : The Bali Jatra (literally meaning "Voyage to Bali") is a major festival and trade fair held annually in Cuttack, Odisha, India. It
is considered one of Asia's largest open trade fairs. ​Significance:

The festival commemorates the glorious maritime tradition of Odisha and the ancient seafaring voyages undertaken by Odia
merchants, known as Sadhabas, to Southeast Asian countries like Bali, Java, Sumatra, and Borneo for trade and cultural
exchange. Hence , option 3 is correct.

It is celebrated on Kartika Purnima (full moon day in the Hindu month of Kartika), which typically falls in November, and lasts for
seven days or more. During this time, Cuttack transforms into a vibrant hub of commerce and festivity.

Elements of Bali Jatra


Boita Bandana: The festival commences with the Boita Bandana ritual, where miniature replicas of boats (boitas) are decorated and
offered to the river Mahanadi. This symbolizes the commencement of the ancient voyages.
Trade Fair: A sprawling trade fair is set up featuring a diverse range of products from across India, including handicrafts, handlooms,
textiles, agricultural produce, and culinary delights.
Cultural Programs: The festival is a vibrant celebration of Odia culture. There are traditional dance and music performances, folk arts
displays, and theatrical enactments.

Recognition
Bali Jatra holds immense cultural and historical significance for Odisha.
In 2022, the festival achieved a Guinness World Record when over 2,100 students crafted more than 22,000 paper boats in just 35
minutes, solidifying its status as a remarkable event.

The Bali Jatra is a unique blend of commerce, culture, and tradition, offering a glimpse into Odisha's rich maritime heritage and vibrant
cultural tapestry.

Que. 67 Consider the following statements


1. The Khelo India Para Games (KIPG) 2023 was the first edition of a multi-sport event aimed at providing a national platform for
para-athletes in India.
2. KIPG 2023 featured over 20 sporting disciplines for para-athletes.

38/262
Choose the correct answer

1. Both Statement 1 and Statement 2 are correct.


2. Both Statement 1 and Statement 2 are incorrect.
3. Statement 1 is correct; Statement 2 is incorrect.
4. Statement 1 is incorrect; Statement 2 is correct.

Correct Option - 3
The correct answer is option 3.
Key Points
Khelo India para games : The Khelo India Para Games (KIPG) 2023 marked a significant milestone in Indian athletics. Held from
December 10th to 17th, 2023, it was the inaugural Khelo India Para Games. This multi-sport event aimed to provide a national platform
for para-athletes in India, similar to the existing Khelo India Youth Games and University Games.

KIPG 2023 was indeed the inaugural edition. Hence, Statement 1 is correct.
. KIPG 2023 featured only seven sporting disciplines, not over 20. Hence, Statement 2 is incorrect

Additional Information
Significance:
Promoting Para-Athletes: KIPG 2023 provided a much-needed national platform for para-athletes, bringing recognition to their talent
and dedication.
Improved Standards: The event aimed to raise the standards of para-sports in India, with a focus on athlete experience, technical
conduct, and overall execution.
Future Aspirations: KIPG 2023 serves as a stepping stone for the development of para-sports in India, potentially inspiring future
generations of para-athletes and preparing them for international competitions like the Paralympics.

Location and Participation: New Delhi served as the host city, with competitions held across three venues: Indira Gandhi Stadium,
Shooting Range in Tughlakabad, and Jawaharlal Nehru Stadium. The event witnessed participation from around 1,450 para-athletes
representing 32 states and union territories.

Sports Disciplines: Seven sporting disciplines were featured in KIPG 2023:


Para Athletics
Para Shooting
Para Archery
Para Badminton
Para Table Tennis
Para Powerlifting
CP Football (Cerebral Palsy Football)

Que. 68 Which of the following statements best describes Srinivasa Ramanujan's contribution to the field of mathematics?

1. He developed a new set of mathematical axioms.


2. He provided an alternate approach to Euclidean geometry.
3. He made significant advancements in number theory and infinite series despite having minimal formal training.
4. He challenged the fundamental principles of calculus

Correct Option - 3
The correct answer is option 3.
Key Points
Srinivas Ramanujan : Srinivasa Ramanujan was an Indian mathematician who made substantial contributions to mathematical analysis,
number theory, infinite series, and continued fractions, despite having almost no formal training in pure mathematics.

Srinivasa Ramanujan is known for his groundbreaking work in number theory, particularly around properties of partitions, modular
forms, and prime numbers. He also made significant contributions to infinite series, developing novel formulas for calculating pi and
other mathematical constants. Hence , option 3 is correct.

key areas where his work made a lasting impact:


1. Number Theory: Ramanujan's genius shone brightest in number theory, a branch of mathematics focused on properties of integers. He
discovered remarkable relationships between partitions of numbers, modular forms (mathematical objects with specific properties), and
prime numbers (whole numbers divisible only by 1 and themselves). His work on these topics opened new avenues of research and
continues to inspire mathematicians today.
2. Infinite Series: Infinite series are sums of infinitely many numbers. Ramanujan displayed an exceptional ability to manipulate them,
deriving innovative formulas for calculating the value of pi (π) and other mathematical constants. His highly convergent series (series
that converge quickly to a specific value) are particularly noteworthy for their efficiency and accuracy.
3. Theoretical Mathematics: While Ramanujan's background was primarily in applied mathematics, his discoveries had profound
implications for theoretical mathematics as well. His work on mock theta functions (a special class of functions related to theta
functions) and hypergeometric series (a type of infinite series) provided valuable tools for further theoretical explorations.
https://lms.testbook.com/genpdf/fromques.php?qids=65f93cf52fe08b6c70dc4d11,65f93e6eb5431ddfb31dfe15,65f96fd18acd632ab8e39282,65f9712de8d50… 39/262
4. Fast and Efficient Calculations: Despite the complexity of his findings, Ramanujan often devised ingenious methods for calculating
them quickly. His inventive algorithms (step-by-step procedures for solving problems) offered alternative and sometimes more efficient
approaches to solving mathematical problems.

Que. 69 Consider the following statements


1. Under Mitakshara law, a daughter has an equal coparcenary right in ancestral property from birth, similar to sons.
2. The Hindu Succession Act, 2005, brought daughters under the ambit of coparceners in Mitakshara coparcenary.

1. Both Statement I and Statement II are correct.


2. Both Statement I and Statement II are incorrect.
3. Statement I is correct; Statement II is incorrect.
4. Statement I is incorrect; Statement II is correct.

Correct Option - 4
The correct answer is option 4.
Key Points
Mitakshara Law: Mitakshara law is a school of Hindu law that primarily governs inheritance and property rights within a Hindu
Undivided Family (HUF).One of the two major schools of Hindu law, alongside Dayabhaga. Applies to a significant portion of India,
excluding West Bengal and Assam.

Traditionally, under Mitakshara law, daughters did not have coparcenary rights from birth. Only sons acquired these rights. Hence
, statement 1 is incorrect.
The Hindu Succession Act, 2005, amended the Mitakshara coparcenary system, granting daughters coparcenary rights equal to
sons with effect from December 2005. Hence, Statement 2 is correct.

Additional Information

1.The Mitakshara Law applies to the whole of India except Bengal and Assam. The Dayabhaga Law applies to the communities living in
the states of Bengal and Assam and other parts of the world.
2.The difference between Dayabhaga and Mitakshara is in the basic idea of them. Dayabhaga does not give anyone the right to property
before the death of their forefathers whereas Mitakshara gives anyone the right to property just after their birth.

3 .Dayabhaga system prevails in West Bengal and allows both the male and female members of the family to be coparceners. Mitakshara
system, on the other hand, prevails all over India except West Bengal and allows only the male members to be coparceners.

Que. 70 Consider the following statements about the Veerabhadra Temple:


1.It is a Chola dynasty temple dedicated to Lord shiva located in Karnataka.
2.The temple complex is known for its monolithic Nandi sculpture and intricate Vijayanagara style architecture.
3. It is dedicated to Vishnu and is considered one of the five important 'Vaishnav Kshetras'.
How many of the statements given above is/are correct?

1. Only one
2. Only two
3. All three
4. None

Correct Option - 1
The correct answer is option 1.
Key Points
Veerabhadra Temple : The Veerabhadra Temple, Lepakshi, is an ancient Hindu temple situated in Andhra Pradesh, around 15 kilometers
from Anantapur. It is widely considered one of the most spectacular Vijayanagara temples, renowned for its architectural features,
sculptures, and paintings.

The Veerabhadra Temple is dedicated to Veerabhadra, a fierce form of Shiva. Constructed during vijayanagar rule. Hence , statement 1
is incorrect.
The temple is known for its monolithic Nandi sculpture and Vijayanagara style architecture. Hence , statement 2 is correct.
. The temple is dedicated to Veerabhadra, a form of Shiva, not Vishnu. It's also not associated with 'Vaishnav Kshetras' (important
Vishnu pilgrimage sites). Hence ,Statement 3 is Incorrect.

Additional Information

https://lms.testbook.com/genpdf/fromques.php?qids=65f93cf52fe08b6c70dc4d11,65f93e6eb5431ddfb31dfe15,65f96fd18acd632ab8e39282,65f9712de8d50… 40/262
Built in the 16th century, the temple complex houses shrines dedicated to various deities including Shiva, Vishnu, and Durga. The
architecture showcases the Vijayanagara style with intricate carvings and vibrant paintings adorning the walls and ceilings.
Significance:
Dedicated to Veerabhadra, a fierce form of the Lord Shiva created to destroy Daksha, Sati's father, for disrespecting Shiva.
Considered an important pilgrimage site, especially for devotees of Lord Shiva.
Believed to be built on the spot where Jatayu, a mythological bird from the Ramayana, fell mortally wounded after trying to
protect Sita from Ravana.
Architectural Marvels:
Monolithic Nandi: A massive monolithic sculpture of Nandi, the sacred bull vehicle of Lord Shiva, greets visitors at the entrance.
Exquisite Pillars: The temple complex features intricately carved pillars with various mythological scenes and motifs.
Hanging Pillar: A fascinating engineering feat, this pillar appears to be touching the roof but is actually suspended with a gap in
between.
Paintings: The temple walls and ceilings boast vibrantly colored frescoes depicting scenes from Hindu mythology, including the
Ramayana, Mahabharata, and Puranas.

Que. 71 Which of the following statements is not true about Guruvayur Temple?

1. It is a Hindu temple dedicated to Lord Vishnu, also worshipped as Krishna.


2. It is located in the town of Guruvayur in Thrissur district, Kerala.
3. The temple is believed to have been built by gods Indra and Agni.
4. The temple allows entry to all religions and adheres to a relaxed dress code.

Correct Option - 3
The correct answer is option 3.
Key Points
Guruvayur Temple: The Guruvayur Temple is a famous Hindu temple dedicated to Lord Vishnu, but popularly known as Krishna .The
temple is located in the town of Guruvayur in Thrissur district, Kerala.
Additional Information
It is one of the most important places of worship for Hindus in Kerala and Tamil Nadu, and is often referred to as "Bhuloka
Vaikuntha" (Vaikuntha in the earthly realm).

The temple is believed to have been built by the Guru (Jupiter) and Vayu (Wind) themselves. Hence, statement 3 is not true.

The temple has a very strict dress code. Men are required to wear dhoti (mundu) and women are required to wear sarees or
salwar kameez. Non-Hindus are not allowed inside the temple.

The temple is famous for its daily pujas (worship rituals) and its annual festival, which is celebrated in the Malayalam month of
Meenam (March-April). The temple is also known for its elephants, which are used in the temple processions.

Que. 72 Consider the following statements


1. Shree Kalaram Mandir is an ancient Hindu temple dedicated to Lord Ram in the city of Nashik, Maharashtra.
2. Sardar Rangarao Odhekar dreamt of a black statue of Lord Rama in the river Godavari, leading to the construction of the Shree Kalaram
Mandir.
Which of the following is the correct option?

1. Only statement 1 is correct.


2. Only statement 2 is correct.
3. Both statements 1 and 2 are correct.
4. Neither statement 1 nor statement 2 is correct.

Correct Option - 3
The correct answer is option 3.
Key Points
Shree Kalaram Mandir: Shree Kalaram Mandir is an old Hindu temple dedicated to Lord Rama located in the Panchavati area of
Nashik City in Maharashtra, India. It is one of the most important Hindu shrines in the city.
Significance:

The temple derives its name from the statue of Lord Rama that is black in color. The literal translation of Kalaram means "black
Rama". The sanctum sanctorum also houses the statues of goddess Sita and Lakshmana. Thousands of devotees visit the temple every
day.

41/262
The temple was funded by Sardar Rangarao Odhekar and was built around 1792. It is said that Sardar Odhekar had a dream that the
statue of Rama in black colour was there in the river Godavari. He retrieved the statue and built the temple around it. The temple is
one of the finest modern temples of Rama in western India.

Additional Information
Kalaram Satyagraha:
Demand for Entry: The Kalaram Satyagraha was a protest demanding the right to enter the Shree Kalaram Mandir in Nashik for Dalits,
who were traditionally denied entry due to the caste system.
Leader: The movement was led by Dr. B.R. Ambedkar, a prominent social reformer and champion for Dalit rights.
Method: The satyagraha followed the principles of non-violent resistance ( सत्याग्रह, Satyagraha) popularized by Mahatma Gandhi.
Protesters peacefully assembled and asserted their right to enter the temple for prayer.
Significance: Though ultimately unsuccessful in gaining immediate entry rights, the Kalaram Satyagraha was a significant event in
India's fight against caste-based discrimination. It highlighted the sin of untouchability and the need for social reform.

Que. 73 Consider the following statements about the Sri Ranganathaswamy Temple, Srirangam:
1. t is the largest functioning Hindu temple in the world.
2. The temple complex is divided into seven concentric enclosures, each representing a major continent.
3. The Rajagopuram, the southernmost entrance tower, is the tallest temple gopuram in Asia.
How many of the statements given above is/are correct?

1. Only one
2. Only two
3. All three
4. None

Correct Option - 2
The correct answer is Option 2.
Key Points
Sri Ranganathaswamy Temple: The Sri Ranganathaswamy Temple, also known as Thiruvaranga Tirupati, is one of the most illustrious
Vaishnavite temples in India. Dedicated to Ranganatha, a reclining form of the Hindu deity Vishnu, the temple is situated in Srirangam, an
island town formed by the rivers Cauvery and Kollidam in Tamil Nadu, India.

The Sri Ranganathaswamy Temple is considered the largest functioning Hindu temple in the world. Hence, Statement 1 is correct.
The seven concentric walls represent planetary systems in Vedic cosmology, not continents. Hence, Statement 2 is incorrect.
The Rajagopuram is the tallest temple gopuram in Asia. Hence, Statement 3 is correct.

Additional Information
The temple is a sprawling complex that is considered the largest functioning Hindu temple in the world, as Angkor Wat, which is
larger, is no longer a functioning temple .
Here are some of the key features of the temple:
Seven prakarams (enclosures): The temple complex is encapsulated by seven concentric walls, each representing a planetary system
in Vedic cosmology.
21 gopurams (towers): There are 21 ornately decorated towers (gopurams) within the complex, with the Rajagopuram, the
southernmost entrance tower, being the tallest at 239.5 feet, making it the tallest temple gopuram in Asia .
Shrines: The temple complex has over 81 shrines, including the main sanctum dedicated to Ranganatha, shrines dedicated to other
deities, and pavilions.
Water Tanks: There are nine sacred pools within the complex, including the Swami Pushkarani tank, which is considered holy.

Que. 74 Consider the following statements


1. Jhum, a form of shifting cultivation, is a traditional practice of the Bru community.
2. They are one of the Particularly Vulnerable Tribal Groups (PVTGs) in Mizoram.
3. Luira is a popular Bru dance performed during festivals, accompanied by instruments like the mader (flute) and the chang (drum)
How many of the statements given above is/are correct ?

1. Only one
2. Only two
3. All three
4.
Correct Option - 2
The correct answer is option 2.
Key Points
Bru tribe :The Bru community, also known as the Reang community, holds a significant place among the indigenous tribes of Northeast
India. While information varies, a majority of the Bru community likely practices Hinduism, with some following Christianity. They

42/262
celebrate major Hindu festivals like Diwali and Holi. They also have their own unique festivals like the "Hangsohorai" which involves
seeking blessings for a good harvest.

Jhum cultivation is highlighted as a traditional practice of the Bru community. Hence , statement 1 is correct.
Tripura, not Mizoram, is the state where Bru are recognized as a PVTG. Hence , statement 2 is incorrect.
Luira as a popular Bru dance with instruments like the mader (flute) and the cheng (gong), not chang (drum). Hence , statement 3 is
correct.

Additional Information
Living on the Land:
Homesteads: Bru people build practical houses using bamboo walls and thatched roofs, often elevated for protection
against floods and wildlife.
Nature-Inspired Attire: Their clothing reflects their connection to nature. Men wear lungis or dhotis made from colorful handloom
cotton, while women adorn themselves in phanek (skirts) and riha (blouses) embellished with beadwork and embroidery.

Food and Work:


Shifting Cultivation: Traditionally, jhum, a form of hillside farming, was their primary livelihood. However, they are
seeking sustainable alternatives due to population growth and environmental concerns.
Dietary Diversity: Rice is their staple, complemented by vegetables, lentils, and meat (chicken, pork, fish). They also enjoy fermented
foods and bamboo shoots.

Artistic Expression:
Skilled Craftsmanship: The Bru community is known for their exceptional basket weaving, intricate wood carvings, and
beautiful bead and shell jewelry.
Music and Dance: Their vibrant folk music and dance forms celebrate their heritage. Traditional instruments like the mader (flute)
and cheng (gong) accompany the popular Luira dance performed during festivals.

Social Structure and Struggles:


Patriarchal System: The Bru community follows a patriarchal structure, with men holding positions of authority within families and
villages.
Displacement and Reintegration: The Bru community in Mizoram has faced historical tensions and displacement. After decades
as refugees, a significant number have recently been resettled in Tripura with government support.

Que. 75 The Paruveta Utsavam is significant because:

1. It marks the beginning of the monsoon season.


2. It strengthens the bond between the Ahobilam temple and the Chenchu tribe.
3. It is a recently established festival to promote tourism.
4. It involves elaborate fire rituals performed by priests.

Correct Option - 2
The correct answer is option 2.
Key Points
The Paruveta Utsavam, literally translating to "hunting festival," is an annual mock hunt ritual celebrated at the Sri Narasimha Swamy
Temple in Ahobilam, Andhra Pradesh, India. Here's a breakdown of its key aspects:

Uniqueness:
This festival is unique to Ahobilam and not observed in any other temple in the region.
It stands out for its extended duration, lasting for about 40 days (a mandala) compared to most temple festivals.

Significance:
Harmony: It fosters communal harmony as people from all sections of society, irrespective of caste or religion, participate in the
festivities.
Devotion: It symbolizes the Lord's (Narasimha) protection and his role in safeguarding devotees from evil forces.
Tribal Connection: It strengthens the bond between the temple and the Chenchu tribe, who consider Lord Narasimha as their brother-in-
law.

Celebration:
Mock Hunt: The main deity is taken in a procession to around 32 Chenchu tribal villages spread around Ahobilam.
Tribal Welcome: As the procession reaches a village, Chenchu individuals symbolically shoot two arrows at the palanquin, signifying
reverence and a protective gesture.
Sankranti: The festival reaches its peak when the deity arrives at a particular village on the day of Makar Sankranti, a harvest festival.
Offerings: The Chenchu people present honey, linen, and sacred mango leaves to the deity during the procession.
Devotional Chants: Devotees from the Mala community recite vachanas (devotional hymns) during the procession.

Overall, the Paruveta Utsavam is a vibrant cultural and religious event that transcends religious boundaries and strengthens the
connection between the temple, the deity, and the local communities.

43/262
Que. 76 Which of the following statements about Hori Habba sport in Karnataka is correct?

1. It is a traditional bull-taming sport celebrated during the harvest festival of sankranti.


2. It is a form of traditional wrestling practiced in coastal Karnataka.
3. It is an annual kite flying competition held in the city of Mysuru.
4. It is a traditional archery competition conducted during the Dasara festival in Karnataka.

Correct Option - 1
The correct answer is option 1.
Key Points
Hori Habba, also known as Hatti Habba or Kobbari Hori Competition, is a rural sport practiced mainly in the Shivamogga, Haveri, and
Uttara Kannada districts of the Indian state of Karnataka. It typically takes place during the harvest festival.

Here's a breakdown of the key aspects of Hori Habba:

The Spectacle:
Draught Cattle and Bulls: Hundreds of trained and decorated draught cattle and bulls are the main participants in this energetic event.
Thrilling Race: These animals are made to run through a large open ground amidst a huge and enthusiastic crowd.

The Challenge:
Catch and Grab: As the animals race through the crowd, participants (called "catchers") try to subdue them and snatch away prizes
like copra (dried coconut kernel), cash, or gift items tied to their bodies.

The Spirit:
Traditional Sport: Hori Habba is a cherished tradition and a test of courage and skill for both the animals and the catchers.
Cultural Significance: It adds a unique flavor to the Deepavali celebrations in the region.

Possible Concerns:
Animal Welfare: There have been concerns raised about the potential for animal cruelty during the event.

Overall, Hori Habba is a vibrant and exciting rural sport in Karnataka, but it's important to acknowledge and address any animal
welfare concerns that may exist.

Que. 77 Consider the following statements about Grantham inscriptions:


1. They are written in the Tamil script and primarily contain religious texts.
2. They are found in South India, particularly Tamil Nadu, and use a script derived from Brahmi.
3. They provide insights into the development of writing systems, religious practices, and economic activities of the region.
How many of the following statements given above is/are correct?

1. Only one
2. Only two
3. All three
4. None

Correct Option - 2
The correct answer is option 2.
Key Points
Grantham Inscriptions: Grantham inscriptions refer to writings in the Grantha script, used primarily for writing Sanskrit in Tamil Nadu,
South India. This script is derived from the Brahmi script, similar to other South Indian scripts like Tamil and Malayalam. Grantha
inscriptions are predominantly found in Tamil Nadu, particularly in temple complexes and on historical monuments.

Grantham is a distinct script used for writing Sanskrit, not Tamil. Hence, statement 1 is incorrect.
Grantham inscriptions are found in South India, particularly Tamil Nadu, and the script is derived from Brahmi. Hence, statement 2 is
correct.
Grantham inscriptions offer valuable information about the historical development of writing systems, religious practices, and
economic activities of the region. Hence, statement 3 is correct.

Additional Information

The word "Grantha" itself translates to "literary work" in Sanskrit, reflecting its purpose.

Evolution and Use:


Development: The Grantha script's development can be broadly divided into four periods: archaic, transitional, medieval, and modern.
Sanskrit Focus: While Tamil has its own script, Grantha was primarily used to write Sanskrit, a classical Indo-Aryan language. This
reflects the historical influence of Sanskrit on South Indian languages and culture.

Examples and Significance:

44/262
Temple Inscriptions: Grantha inscriptions are commonly found on the walls and pillars of ancient temples in Tamil Nadu, recording
historical events, religious narratives, or donations made to the temple.
Literary Works: Grantha was also used to write some important literary works in Sanskrit, particularly those with a focus on religious or
philosophical themes.
In News
Recent Discovery:
Location: A team from the Virarajendran Archaeological and Historical Research Centre discovered these inscriptions in temples located
in Pazhanchervazhi village, near Kangayam in Tamil Nadu.
Age: The inscriptions are believed to date back to the 11th and 16th centuries, respectively.
Script and Language: One inscription is in the Grantha script, traditionally used for writing Sanskrit, while the other inscription is in
Tamil script.

Que. 78 With reference to recent world's first bamboo crash barrier, which was installed recently, Consider the following statements:
1. It has been installed on a highway connecting Chandrapur and Yavatmal districts in Maharashtra.
2. The recycling value of bamboo barriers is 50-70%, while that of steel barriers is 30-50%.
3. It has been constructed from Bambusa Balcooa , a type of bamboo, treated with creosote oil, and coated with recycled High-Density
Polyethylene (HDPE).
How many of the above statements are correct?

1. Only one
2. Only two
3. All three
4. None

Correct Option - 3
The correct answer is Option 3.
Key Points ​
Bamboo Crash barrier:

Recently, 200-metre-long bamboo crash barrier has been installed on a highway connecting Chandrapur and Yavatmal districts
in Maharashtra, which has been termed as the “world’s first” such exercise. Hence option 1 is correct.
The crash barrier will be an alternative to steel and will be environmentally friendly as bamboo crash barriers have a recycling
value of 50-70% compared to 30–50% for steel barriers. Hence option 2 is correct.
The bamboo species used in the making of this barrier is Bambusa Balcoa, which has been treated with creosote oil and coated with
recycled High-Density Poly Ethylene [HDPE]. Hence option 3 is correct.

Additional Information
The crash barrier has successfully passed rigorous testing and was given a Class 1 rating during the Fire Rating Test conducted at the
Central Building Research Institute (CBRI) in Roorkee.
Furthermore, it has been accredited by the Indian Road Congress, adding to its credibility.
An Eco-Friendly and Economical Alternative to Steel:
The bamboo barrier boasts a recycling value of 50-70% , which is significantly higher than the 30-50% recycling value for steel
barriers, as per the statement released by the Ministry of Road and Highways.
This innovative barrier presents a cost-effective and environmentally friendly substitute for steel, opening up new avenues for
the Indian bamboo industry.This significant accomplishment is a step towards realizing the vision of Aatmanirbhar Bharat , or
self-reliant India.

Que. 79 Consider the following statements regarding the Cyclone Freddy:


1. It Freddy, roaring at more than 240 km/hr was an ‘intense tropical cyclone’ in the Southern Hemisphere.
2. It was the longest-lived tropical cyclone on record, beating the previous record of Hurricane John in 1994.
45/262
3. It also broke the record for the highest accumulated cyclone energy (ACE).
How many of the above statements are correct?

1. Only one
2. Only two
3. All three
4. None

Correct Option - 3
The correct answer is All three.
Key Points
Cyclone Freddy:

Freddy, roaring its away at more than 240 km/hr currently, was an ‘intense tropical cyclone’ in the Southern Hemisphere. It
matches the scale and proportion of a super cyclone in the Northern Hemisphere. Hence, option 1 is correct.
The storm broke at least six meteorological world records and 10 records for the south Indian Ocean. At 37 days, Freddy is the
longest lived tropical cyclone ever recorded in any of Earth’s oceans and has broken the record of 31 days set by Hurricane John in
1994. Hence, option 2 is correct.
Freddy also broke the record for the highest accumulated cyclone energy (ACE) at 87.01 units, breaking the record of 85.27 units by
Hurricane Ioke in 2006. Hence, option 3 is correct.

ACE is the total wind energy generated by a tropical cyclone throughout its lifetime and is calculated as the square of peak wind
speed, observed, calculated and added every six hours.
Cyclone Freddy’s ACE is as much as the total ACE of all hurricanes in an average North Atlantic hurricane season, according to
a press release by the World Meteorological Organization (WMO).

Que. 80 With reference to Green Tug Transition programme consider the following statements:
1) A tugboat is a boat that aids in the mooring or berthing operation of a ship by towing or pushing the vessel towards the port.
2) Under the Green Tug Transition Programme, all the tugboats used in ports would be converted to ‘green tugs’ that run on non-fossil fuels
such as methanol, hydrogen or ammonia by 2030
3) It will also help in achieving the UN’s SDG 14 to sustainably manage and protect marine & coastal ecosystems from pollution,
conservation & sustainable use of ocean based resources.
4) It will help in achieving multiple objectives such as ‘make in India’, greater business opportunities to become carbon neutral by 2070.
How many of the statements given above are correct?

1. only one
2. only two
3. only three
4. all four

Correct Option - 3
The correct solution is Option 3
Key Points
Green Tug Transition Programme :

A tug or a tugboat is a boat that aids in the mooring or berthing operation of a ship by towing or pushing the vessel towards the port.
Hence statement 1 is correct.

46/262
This programme will help in achieving multiple objectives such as ‘make in India’, greater business opportunities to become carbon
neutral by 2070 and a greener environment. Hence statement 2 is correct.
This programme will also help in achieving the UN’s SDG 14 to sustainably manage and protect marine & coastal ecosystems from
pollution, conservation & sustainable use of ocean based resources. Hence statement 3 is correct.
At least 50% of all the tugs are to be converted into Green Tugs by 2030. Hence statement 4 is incorrect.

Additional Information
Union Minister of Ports, Shipping & Waterways (MoPSW) and Ayush Shri Sarbananda SonowaI has said India aims at becoming
‘Global Hub for Green Ship’ building by 2030 with launch of Green Tug Transition Programme(GTTP).
Inaugurating India’s first National Centre of Excellence in Green Port & Shipping (NCoEGPS) in Gurugram,Haryana he said the
programme will start with ‘Green Hybrid Tugs, which will be powered by Green Hybrid Propulsion systems, and subsequently
adopting non-fossil fuel solutions like (Methanol, Ammonia, Hydrogen).
Country’s first National Centre of Excellence in Green Port & Shipping (NCoEGPS) is the result of a collaboration between the
Ministry of Ports, Shipping & Waterways, Government of India and the Energy and Resources Institute (TERI).
With the NCoEGPS acting as the Nodal entity for the industry, the plan is afoot to make India as the ‘Global hub for building Green
Ships’ by 2030.
The Centre will engage in developing the regulatory framework and alternative technology adoption roadmap for green shipping in
India.

Que. 81 Consider the following statements regarding Gandak River:


1. The river flows through only the Indian state of Bihar and joins the Ganges near Patna just downstream of Hajipur.
2. The major tributaries of the Gandak River include the Mayangadi, Bari, Trisuli, Panchand, Sarhad, Budhi Gandak.
3. River Gandak was declared as National Waterway (NW)-37 by National Waterways Act, 2016.
4. It forms the western boundary of Valmiki wildlife sanctuary.
How many of the above statements are correct ?

1. only one
2. only two
3. only three
4. all four

Correct Option - 3
The correct answer is option 3.
Key Points
River Gandak:

The river flows through the Indian states of Bihar and Uttar Pradesh, and joins the Ganges near Patna just downstream of Hajipur. ​
Hence statement 1 is incorrect
The major tributaries of the Gandak River include the Mayangadi, Bari, Trisuli, Panchand, Sarhad, Budhi Gandak. ​Hence
statement 2 is correct.
River Gandak was declared as National Waterway (NW)-37 from Bhaisalotan Barrage to Gandak and Ganga River confluence at
Hajipur, Bihar along with 111 NWs in the country via National Waterways Act, 2016.Hence statement 3 is correct.
River Gandak forms the western boundary of Valmiki wildlife sanctuary. Hence statement 4 is correct.

Additional Information
The river Gandak, is also known as the Gandaki and Narayani River in Nepal.
Valmiki National Park is a tiger reserve in the West Champaran District of Bihar, India. It is the only national park in Bihar.
The important towns in the Indian part of the Gandak river are Valmikinagar (Bhainsalotan) - location of Gandak Barrage, Bagaha,
Bettiah (district headquarters & field directorate of Valmiki Tiger Project), Harinagar (Ramnagar), Hajipur (across the Ganges 10
km from Patna) and Sonepur (also Known as Harihar Kshetra), near Patna.
Under Namami Gange Program, the development of river front on Gandak River in the district of Gopalganj, Bihar has been
undertaken and two Ghats have been constructed.
River Gandak was declared as National Waterway (NW)-37 from Bhaisalotan Barrage to Gandak and Ganga River confluence at
Hajipur, Bihar along with 111 NWs in the country by National Waterways Act, 2016.
The act aims to promote the development of inland water transport and provide an alternative mode of transportation for goods and
passengers.
The Gandak Project at Valmikinagar (Bhainsaloton) intercepts water of a catchment area of 37,410 km which is mostly in Nepal and
partly in India. An agreement was signed on 4 December 1959 between the governments of Nepal and of India on the Gandak
Irrigation and Power Project.
It encompassed the construction of a barrage, canal head regulators and other appurtenant works about 33 m (108 ft) below the existing
Triveni Canal Head Regulator. The agreement was modified in 1964 for the protection of Nepal's riparian rights.
As a part of this bilateral agreement, the Gandak Barrage, a part of Gandak Project, was built in 1968–69 over the Gandak river for
providing irrigation to Nepal, Uttar Pradesh and Bihar.

47/262
Que. 82 Consider the following statements:
1.Vembanad lake is the largest lake in Kerala and the longest in India.
2. Ashtamudi lake is located in Tamil Nadu and is an important Ramsar site of India.
3. Recently The National Green Tribunal (NGT) has imposed a penalty of Rs 10 crore on the State government for failing to protect the
Vembanad and Ashtamudi lakes.
How many of the statements given above is/are correct?

1. Only one
2. Only two
3. All three
4. None

Correct Option - 2
The correct answer is Only two.
Key Points
Vembanad Lake:

Vembanad stands as the lengthiest lake in India and the largest within Kerala. It spans an expanse of 230 square kilometers and
stretches up to 96.5 km at its maximum length.
Kuttanad, renowned as The Rice Bowl of Kerala, holds the distinction of being the lowest altitude region in India and one of the rare
places globally where agriculture occurs below sea level. Positioned on the southern edge of Vembanad,Kuttanad boasts this unique
feature.
A segment of the lake hosts the Nehru Trophy Boat Race. Hence Statement 1 is correct.

Ashtamudi Lake:

Ashtamudi Lake, situated in the Kollam District of Kerala, India, boasts a distinctive wetland ecosystem and a vast palm-shaped (also
likened to an octopus shape) expanse of water, ranking second only to the Vembanad estuary ecosystem in the state.
The name "Ashtamudi" derives from the local Malayalam language, signifying 'eight hills or peaks' (Ashta meaning 'eight' and mudi
meaning 'peaks'), which aptly describes the lake's terrain characterized by multiple branches.
Referred to as the gateway to Kerala's backwaters, Ashtamudi Lake is renowned for its houseboat cruises and backwater resorts.
Notably, the Ashtamudi Wetland has gained recognition as a site of international importance, listed under the Ramsar Convention for
the conservation and sustainable utilization of wetlands.Hence Statement 2 is incorrect.

48/262
The National Green Tribunal (NGT),statutory body formed in 2010 under NGT Act,2010 has fined the Kerala government Rs 10
crore for its failure to safeguard the Vembanad and Ashtamudi lakes, both of which are listed in the Ramsar list of wetlands.
In accordance with the 'polluter pays principle', the imposed penalty is to be deposited into a dedicated account, managed under the
authority of the chief secretary.
These funds are earmarked for conservation or restoration efforts aimed at the affected areas. Hence Statement 3 is correct.

Que. 83 Galathea National Park in news recently is located in:


1. Andaman and Nicobar Islands
2. Lakshwadeep Islands
3. South Pacific Islands
4. Eurasian Region

1. 1
2. 2
3. 3
4. 4

Correct Option - 1
The correct option is option-1 i.e Andaman and Nicobar Islands
Key Points
Galathea National Park, situated within the Union Territory of Andaman and Nicobar Islands, India, encompasses the island of
Great Nicobar in the eastern Indian Ocean, specifically the Nicobar Islands.
Covering an area of approximately 110 square kilometers, it gained recognition as a National Park of India in 1992.
Galathea is an integral part of the Great Nicobar Biosphere Reserve, which also includes the larger Campbell Bay National Park.
These two parks are separated by a 12-kilometer forest buffer zone.
The park harbors numerous unique and rare species of plants and animals, many of which are endemic to the islands due to their
relative geographical isolation.

Que. 84 With reference to ICOMOS , Consider the following statements:


1. ISC20C focuses its efforts on conserving and celebrating mid to late twentieth-century places that are most at risk through lack
of recognition and protection.
2. ICOMOS was founded in 1965 at Warsaw and offers advice to UNESCO on World Heritage Sites.
3. Santiniketan, West Bengal, was declared a world heritage site by UNESCO.
Which of the above statements is/are correct?

1. 1 only
2. 1 and 2 only
3. 3 only
4. 1, 2 and 3

Correct Option - 4
The correct answer is Option 4
Key Points
The International Council on Monuments and Sites:

ICOMOS ( The International Council on Monuments and Sites) works for the conservation and protection of cultural heritage places
around the world.
It is the only global non-government organisation of this kind, which is dedicated to promoting the application of theory,
methodology, and scientific techniques to the conservation of the architectural and archaeological heritage.
Now headquartered in Charenton-le-Pont, France, ICOMOS was founded in 1965 in Warsaw as a result of the Venice Charter of
1964.The International Union for Conservation of Nature (IUCN),The International Council on Monuments and Sites (ICOMOS)
and The International Centre for the Study of the Preservation and Restoration of Cultural Property (ICCROM) are three advisory
committee of UNESCO World Heritage Committee. Hence statement 2 is correct.
International Scientific Committee on 20th Century Heritage (ISC20C) focuses its efforts on conserving and celebrating mid to late
twentieth-century places that are most at risk through lack of recognition and protection. Hence statement 1 is correct
Santiniketan in West Bengal has been declared a World Heritage Site by UNESCO. This is India's 41st World Heritage site. Hence
statement 3 is correct.

Additional Information
ICOMOS is a partner and founding member of the Blue Shield, formerly the International Committee of the Blue Shield, is an
international organization founded in 1996 which works to protect the world's cultural heritage threatened by war and natural
disasters.​
Blue Shield is a non-governmental, non-profit, international organisation committed to the protection of heritage across the
world.
49/262
Article 6 of the 1954 Hague Convention states that cultural property may bear a distinctive emblem so as to facilitate its
recognition.
Article 27.3 of the 1954 Hague Convention Second Protocol (1999) explicitly mentions the International Committee of the
Blue Shield as an advisory body to the Committee for the Protection of Cultural Property in the Event of Armed Conflict.
The Hague convention aims to protect cultural property, such as monuments of architecture, art or history, archaeological sites,
works of art, manuscripts, books and other objects of artistic, historical or archaeological interest, as well as scientific collections
of any kind regardless of their origin or ownership.

Que. 85 'Urea Gold', recently seen in news is related to which of the following?

1. It is Neem Coated Urea


2. It is Sulphur Coated Urea
3. It is Potassium coated Urea
4. It is liquid Nano Urea

Correct Option - 2
The correct answer is option 2
Key Points
Urea Gold is Urea coated with Sulphur. Hence option 1 is correct.
Developed by Rashtriya Chemicals and Fertilizers Ltd (RCF), a leading fertilizer and chemical manufacturing Public Sector company.
Urea Gold is created by infusing urea with sulfur, creating a composite fertilizer with 37% nitrogen (N) and 17% sulfur (S).
Neem-coated urea is a specialized form of urea fertilizer that has been coated with neem oil. It reduces the leaching and volatilization
losses of nitrogen, has insecticidal and nematicidal properties, and improves the soil texture and water holding capacity
Liquid Nano Urea is a nanotechnology-based fertilizer that is sprayed on leaves and is assimilated by the plant cells. It enhances the
nutritional quality and productivity of the crop, reduces the fertilizer consumption, improves the nitrogen use efficiency, and saves the
input costs.

Que. 86 Consider the following statements about Salar de Uyuni:


1. It is the largest playa in the world.
2. It is located near the crest of Andes in Chile.
3. It is part of the lithium triangle.
4. Geometric patterns such as pentagons and hexagon observed on the surface of salt flats worldwide is not found in Salar de Uyuni.
Which of the above statements are correct?

1. 1 and 3 only
2. 1,2 and 3 only
3. 1,3 and 4 only
4. 1,2,3 and 4

Correct Option - 1
The correct answer is option 1
Salar de Uyuni

Salar de Uyuni (or "Salar de Tunupa") is the world's largest salt flat (playa) in the world. Hence statement 1 is correct.​
It is located in the Daniel Campos Province in Potosí, southwest Bolivia, near the crest of the Andes. Hence statement 2 is incorrect.
Lithium triangle: Argentina, Chile, and Bolivia—contain roughly half the world's known lithium. Hence Statement 3 is correct.
In salt flats across the world, the salt on the surface forms ridges that join together in a patchwork of pentagons and hexagons. These
captivating patterns have been photographed as far apart as Bolivia, Chile, China, India (in the Rann of Kutch), Iran, Tunisia, and the
U.S. Hence statement 4 is incorrect.​

Additional Information
Lithium
50/262
It is an alkali mineral, also called ‘white gold’. It is soft, silvery-white metal, the lightest metal of the periodic table.
Lithium metal has the properties of large heat capacity, wide liquid temperature range, high thermal conductivity, low viscosity and
low density.
Lithium is widely used in batteries, ceramics, glass, lubricants, medicine, refrigeration, nuclear, fertilizer and photovoltaic industries.
The top producers of lithium are Australia, Chile, China, and Argentina.
In India, it was first discovered in Jammu & Kashmir in 1999.
Lithium reserves were discovered in Jammu & Kashmir, Rajasthan, Jharkhand and Karnataka.
Geological Survey of India (GSI) had carried out 14 projects on lithium and associated elements in Bihar, Chhattisgarh, Himachal
Pradesh, Jammu & Kashmir, Jharkhand, Madhya Pradesh, Meghalaya, Karnataka and Rajasthan during the approved annual Field
Season Programme (FSP) between 2016-17 and 2020-21. And during the FSP 2021-22, GSI has taken up five projects on lithium and
associated minerals in Arunachal Pradesh, Andhra Pradesh, Chhattisgarh, Jammu & Kashmir and Rajasthan.

Que. 87 The term 'Plastiglomerate' is related to which of the following ?

1. A plastiglomerate is a "rock" made of a mixture of beach sediment or other natural debris (like shells or wood) and plastic.
2. It refers to the substance formed by electro-accumulation of minerals dissolved in seawater.
3. It is floating plastic pebbles with a geogenic appearance.
4. Type of plastic pollution in the form of plastic debris, covering rocks in intertidal shorelines which vary in thickness and colour.

Correct Option - 1
The correct answer is option 1
Key Points
A plastiglomerate is a "rock" made of a mixture of beach sediment or other natural debris (like shells or wood) and plastic.
Hence statement 1 is correct.
Biorock, also known as Seacrete or Seament, refers to the substance formed by electro-accumulation of minerals dissolved in
seawater.
Mineral Accretion Technology: Also called ‘Biorock Technology’, is a method that applies safe, low voltage electrical currents
through seawater, causing dissolved minerals to crystallize on structures, growing into a white limestone (CaCo3) similar to that
which naturally makes up coral reefs and tropical white sand beaches.
Pyroplastics are floating plastic pebbles with a geogenic appearance. They are also remnants of melted plastic derived from the
burning of plastic, but are not mixed with other material.
Plasticrusts are a type of plastic pollution in the form of plastic debris, covering rocks in intertidal shorelines which vary in thickness
and in color.

Additional Information
Plastistone
“Plastistone” serves as an umbrella term for novel plastic forms lithified with natural rocks, including “plastiglomerate”,
“plasticrust”, “plastitar”, “plastisandstone”, and “anthropoquinas”.
It does not encompass purely molten plastic forms like “pyroplastic”.

Que. 88 Peaty soil, often found in wetlands, is characterized by:

1. High clay content and good drainage


2. Low organic matter content and rapid drying
3. High organic matter content and good water retention
4. Sandy texture and coarse particles

Correct Option - 3
The correct answer is option 3.
Key Points
Peaty soil: Peaty soil is a type of soil that is formed from the partial decomposition of organic matter, such as mosses, sedges, and
reeds, in waterlogged conditions. These conditions slow down the decomposition process, allowing the organic matter to accumulate over
time. Peaty soil is typically found in wetlands, such as bogs, fens, and marshes.

Rich in organic matter : Peaty soil is high in organic matter, which can hold a lot of water. This makes peaty soil a good choice for
plants that need a lot of moisture, such as blueberries, cranberries, and azaleas.
Acidity: Peaty soil is typically acidic due to the accumulation of organic acids from decomposing plant matter. This can be a
challenge for some plants that prefer more neutral or alkaline soils.

51/262
Nutrient content: While peaty soil holds a lot of water, it can be deficient in essential plant nutrients like nitrogen, phosphorus,
and potassium. This is because these nutrients are often leached away by the saturated conditions in wetlands.
Sustainability: Peat bogs are important carbon sinks, storing vast amounts of carbon that would otherwise be released into the
atmosphere. However, unsustainable harvesting of peat moss for gardening and other uses can damage these ecosystems.

Que. 89 Consider the following statements

1: October heat in India is characterized by a rise in temperature and a decrease in humidity compared to the monsoon season.

2: October heat is a result of the southward movement of the sun and the weakening of the monsoon trough over the northern plains.

Choose the correct answer

1. Both Statement 1 and Statement 2 are individually true and together explain the phenomenon
2. Both Statement 1 and Statement 2 are individually true but together do not explain the phenomenon
3. Statement 1 is true; Statement 2 is false.
4. Statement 1 is false; Statement 2 is true.

Correct Option - 1
The correct answer is option 1.
Key Points
October heat : The weather in the month of October in the Indian sub-continent is called ‘October heat’. During October and November,
with the apparent movement of the sun towards the south, the monsoon trough or the low-pressure trough over the northern plains
becomes weaker. This is gradually replaced by a high-pressure system. The southwest monsoon winds weaken and start withdrawing
gradually.

October heat refers to the rise in temperatures across the country. Because of precipitation, increased moisture, cloud cover, and
sea waves in coastal regions, many portions of India cool down during the southwest monsoon.
The monsoon had left the northern plains by the beginning of October. The months of October and November are the transitional
months between the hot, rainy season and the dry winter months. Clear skies and a rise in temperature signal the end of the
monsoon season. While the days are hot, the nights are chilly and lovely.
Days are usually warmer, and nights are a little cold and nice during the October heat. The humidity level rises as a result of the high
temperature, making the days uncomfortable in nature. ‘October Heat’ is the name of the entire system.

The monsoons are two of India’s four major seasons, namely:

Rainfall normally occurs from June to September during the southwest monsoon season.

The monsoon season is winding down, and the months of October and November are hot.

The moving ITCZ is caused by the sun’s migration to the north or south.

Que. 90 Fringing reefs differ from barrier reefs in that:

1. Fringing reefs are isolated structures in deep waters.


2. Fringing reefs grow directly offshore and are relatively narrow.
3. Barrier reefs encircle a lagoon and are found in tropical waters.
4. Fringing reefs provide no habitat for marine life.

Correct Option - 2
The correct answer is option 2.
Key Points
Fringing reefs :
Location: Fringing reefs are the most common type of coral reef and grow directly offshore, attached to the coastline. This means you
can swim or wade out from the beach and encounter the reef.
Structure: They are typically narrow compared to other reef formations, with a shallow reef flat closest to shore and a steeper reef
slope facing the open ocean. The reef flat may be exposed during low tide.
Habitat: Fringing reefs, despite their size, are teeming with life. They provide crucial habitat for a wide variety of marine creatures,
including fish, corals, invertebrates, and algae.
Vulnerability: Due to their proximity to the shore, fringing reefs are more susceptible to threats like pollution from land
runoff, sedimentation, and damage from coastal development.
Protection: Fringing reefs play a vital role in coastal protection by absorbing wave energy and reducing erosion. They act as a natural
barrier, safeguarding shorelines from storms and strong currents.
Barrier reefs :

Formation: Barrier reefs are massive underwater ecosystems formed by tiny living organisms called coral polyps. These polyps
secrete calcium carbonate to build hard skeletons, which over time form the reef structure.
52/262
Location: Barrier reefs are found in shallow tropical and subtropical waters, typically around 30 degrees north and south of the
equator. The ideal conditions for coral reef growth include warm, clear, and shallow waters with good sunlight penetration. Some of
the most extensive barrier reefs are found in the Indo-Pacific region, including the Great Barrier Reef off the coast of Australia,
the Belize Barrier Reef, and the Mesoamerican Barrier Reef System.
Structure: Barrier reefs are large elongated structures that run parallel to a coastline, with a deeper body of water, called a lagoon,
separating them from the shore. The lagoon can be rich in marine life due to the nutrients and protection offered by the reef.
Threats: Barrier reefs are under threat from several human activities, including climate change, pollution, and overfishing. Rising
water temperatures due to climate change can cause coral bleaching, where the coral polyps expel the algae living within them, which
can ultimately kill the coral. Pollution from agriculture, sewage, and industrial waste can also harm coral reefs. Overfishing can disrupt
the delicate food web of the reef ecosystem.

Que. 91 Consider the following statements

1. Western disturbances are extratropical storms that originate in the Mediterranean region and bring winter rain and snowfall to the northern
parts of India.

2. They are high-pressure systems embedded in easterly winds that flow from east to west.

3. Western disturbances are a major source of water supply for the northwestern parts of India.

How many of the statements given above is/are correct ?

1. Only one
2. Only two
3. All three
4. None

Correct Option - 2
The correct answer is option 2.
Key Points
Western disturbances : Western disturbances are extratropical storms that originate in the Mediterranean region and bring winter rain
and snowfall to the northwestern parts of the Indian subcontinent, extending eastward to northern Bangladesh and southeastern Nepal.

Western disturbances are indeed extratropical storms that originate in the Mediterranean region and bring winter precipitation to the
northwestern parts of India. Hence ,Statement 1 is correct.
Western disturbances are low-pressure systems embedded in westerly winds, not easterly winds. Hence, Statement 2 is incorrect.
Western disturbances are a crucial source of water supply for the northwestern parts of India during the winter months. Hence
,Statement 3 is correct.

Additional Information

Wind and Moisture: They are low-pressure systems embedded in westerly winds that flow from west to east. The moisture in these
storms usually originates over the Mediterranean Sea, the Caspian Sea, and the Black Sea.
Importance for Agriculture: Western disturbances are crucial for the region's water supply, bringing much-needed precipitation
during the winter months, which is critical for rabi crops like wheat.
Temperature Regulation: They also play a role in regulating temperatures and preventing extreme cold spells.
Potential Hazards: However, western disturbances can also cause extreme weather events such as floods, avalanches, and landslides
if they bring excessive precipitation.

Que. 92 Storm Daniel, a catastrophic event in 2023, is an example of:

1. A powerful El Nino inducing heavy rainfall


2. A landfalling hurricane in the Caribbean
3. A destructive Medicane in the Mediterranean
4. A severe dust storm impacting North Africa

Correct Option - 3
The correct answer is option 3.
Key Points
Storm Daniel was a devastating tropical cyclone that struck the Mediterranean and parts of the Middle East in September 2023. Here's a
quick summary of key points:
Type: Tropical Storm (also referred to as a Medicane - Mediterranean Hurricane)
Dates Active: September 5th, 2023 - September 12th, 2023
Impacts:
Deadliest Mediterranean Cyclone: Caused the most fatalities ever recorded for a Mediterranean cyclone, with confirmed deaths
exceeding 4,361 and estimates reaching as high as 20,000.

53/262
Widespread Damage: Particularly destructive in Libya, where it triggered catastrophic flooding after major dams collapsed due to
heavy rainfall.
Other Affected Areas: Greece, Türkiye, and Bulgaria also experienced impacts from the storm.

Que. 93 Which of the following statements is not true about a lunar eclipse?

1. It occurs when the Moon moves into the Earth's shadow.


2. During a total eclipse, the Moon completely disappears from view.
3. The reddish color of the Moon during totality is caused by sunlight scattered by Earth's atmosphere.
4. A penumbral lunar eclipse is the easiest type to observe with the naked eye.

Correct Option - 2
The correct answer is option 2.
Key Points
Lunar eclipse :
A lunar eclipse is an astronomical event that occurs when the Moon moves into the Earth's shadow, causing the Moon to be darkened or
turn a reddish color.

There are three main types of lunar eclipses:


Total lunar eclipse: The Moon passes completely through Earth's umbra (darkest shadow). During totality, the Moon usually takes on a
reddish hue, which is caused by sunlight filtering through Earth's atmosphere.
Partial lunar eclipse: Only part of the Moon passes through Earth's umbra. The rest of the Moon remains lit by the Sun.
Penumbral lunar eclipse: The Moon passes through the Earth's penumbra (outer, lighter shadow). The darkening of the Moon is slight
and often difficult to notice with the naked eye.

Que. 94 Consider the following statements about Shola Forests:


1. Shola forests are evergreen coniferous forests found in the higher Himalayas.
2. They are stunted evergreen broadleaf forests with a closed canopy, acting as water sponges for the region.
Which of the statements given above are correct?

1. 1 only
2. 2 only
3. Both 1 and 2
4. Neither 1 nor 2.

Correct Option - 2
The correct answer is option 2.
Key Points
Shola forests :The shola forests are patches of stunted tropical montane forests found in valleys amid rolling grasslands in the higher
montane regions of South India, largely in Kerala, Karnataka and Tamilnadu.

These patches of shola forest are found mainly in the valleys and are usually separated from one another by undulating montane
grassland. The shola and grassland together form the shola-grassland complex or mosaic.
Not all such high-elevation grasslands have sholas in their valleys, especially if they are isolated from other such meadows.

key points about shola forests:


Distribution: Shola forests are found in the higher elevation hill regions of the Nilgiris, Anaimalai, around Anamudi, Palni hills,
Meghamalai, Agasthyamalai to the south and the Malnad and associated ranges in parts of Wayanad, Coorg, Baba Budangiri, Kudremukh
up the north, to Goa, Satara district and Sindhudurg district in the states of Karnataka, Kerala, Goa, Maharashtra and Tamil Nadu. Their
altitude can range from 1600 meters to 2000 meters above sea level.
Vegetation: Shola forests are characterized by stunted evergreen broadleaf forests with dense, interlocking branches that form a
closed canopy, blocking out most sunlight from reaching the forest floor. The vegetation is rich in biodiversity, with a variety of plants,
including mosses, ferns, orchids, and rhododendrons.
Importance: Shola forests play a vital role in the ecology of the Western Ghats. They act as water sponges, absorbing and storing
rainwater, which is slowly released throughout the year. This helps to regulate the flow of rivers and streams in the region. Shola forests
also provide habitat for a variety of endemic species of plants and animals.

Que. 95 Consider the following statements about International Day of Zero Waste:
1. It is an initiative of the Zero Waste International Alliance.
2. The resolution for International Day of Zero Waste was forwarded by Türkiye.
3. The UN Environment Programme (UNEP) and UN-Habitat jointly facilitate the observance of International Day of Zero Waste.
4. Promoting zero-waste initiatives through this international day can help advance all the goals and targets in the 2030 Agenda for
Sustainable Development..

54/262
Which of the given statements is/are correct?

1. 1,2 and 3 only


2. 2 only
3. 1 and 2 only
4. 2,3 and 4 only

Correct Option - 4
The correct answer is option 4
Key Points
International Day of Zero Waste

On 14 December 2022, the United Nations General Assembly adopted a resolution at its seventy-seventh session to proclaim 30
March as International Day of Zero Waste, to be observed annually. Hence statement 1 is incorrect.
Türkiye put forward the resolution and 105 other countries joined in sponsoring it. Hence statement 2 is correct.
The United Nations Environment Programme (UNEP) and the United Nations Human Settlements Programme (UN-Habitat)
jointly facilitate the observance of International Day of Zero Waste. Hence statement 3 is correct.
Promoting zero-waste initiatives through this international day can help advance all the goals and targets in the 2030 Agenda for
Sustainable Development, including Sustainable Development Goal 11 and Sustainable Development Goal 12. Hence statement 4 is
correct.​
Swachhotsav 2023- International Day of Zero Waste-
Ministry for Housing and Urban Affairs launched Swachhotsav 2023 campaign to galvanise the participation of women to
realize the goal of ‘Garbage Free Cities’ and help 1,000 cities become 3-star Garbage Free Cities (GFC) by October 2024 under
the Swachh Bharat Mission Urban (SBM-U).

Zero Waste International Alliance

The Zero Waste International Alliance (ZWIA) is a group of environmental professionals dedicated to working towards a world
without waste through public education and practical application of Zero Waste principle.
By disseminating knowledge and providing support to its members ZWIA is promoting the implementation of Zero Waste Principles in
various aspects.

Sustainable Development Goals

Que. 96 Consider the following statements regarding Heatwaves:


1. Heat wave is considered if the maximum temperature of a station reaches at least 30°C or more for Plains
2. An increase of at least 40°C or more for Hilly regions.
3.A health condition where the body overheats due to prolonged exposure to high temperatures where body temperature rises quickly
often above 104°F (40°C)
Which of the above statements are incorrect?

1. 1 and 2 only
2. 1 and 3 only
3. 2 and 3 only
4. all the above

Correct Option - 1
The correct answer is option 1
Key Points
Heat wave
According to Indian Meteorological Department (IMD) Criteria:

55/262
Heat Wave need not be considered till maximum temperature of a station reaches at least 40°C for Plains and at-least 30°C for Hilly
regions.
If the normal maximum temperature of a station is less than or equal to 40°C, then an increase of 5°C to 6°C from the normal
temperature is considered to be heat wave condition.
Further, an increase of 7°C or more from the normal temperature is considered a severe heat wave condition.
If the normal maximum temperature of a station is more than 40°C, then an increase of 4°C to 5°C from the normal temperature is
considered to be heat wave condition. Further, an increase of 6°C or more is considered a severe heat wave condition.
Additionally, if the actual maximum temperature remains 45°C or more irrespective of normal maximum temperature, a heat wave
is declared.
A heat stroke or sunstroke is the result of overheating of the body as a result of exposure to high temperatures and humidity, or due to
prolonged physical exertion at high temperatures.

Additional Information

Heat exhaustion and heatstroke usually happen during a heatwave, or in a hot climate. However, they can also occur when you're doing
very strenuous physical exercise.
Heat exhaustion is where you become very hot and start to lose water or salt from the body.
Heatstroke is where the body is no longer able to cool itself and a person's body temperature becomes dangerously high (sunstroke is
when this is caused by a long amount of time exposed to direct sunlight)
Heatstroke is less common, but more serious. It can put a strain on the brain, heart, lungs, liver and kidneys, and can be life-
threatening.
If heat exhaustion isn't spotted and treated early on, there's a risk it could lead to heatstroke.
Heat exhaustion or heatstroke can develop quickly over a few minutes, or gradually over several hours or days.

Government Initiative Regarding Heat Waves


1.National action Plan for Climate Change (NAPCC):
There are 8 national missions forming the core of the NAPCC which represent multi-pronged, long term and integrated strategies for
achieving key goals in climate change. These are-

National Solar Mission


National Mission for Enhanced Energy Efficiency
National Mission on Sustainable Habitat
National Water Mission
National Mission for Sustaining the Himalayan Ecosystem
National Mission for A Green India
National Mission for Sustainable Agriculture
National Mission on Strategic Knowledge for Climate Change

2.India Cooling Action Plan (ICAP):


ICAP is a long-term vision to address the cooling requirement across sectors. Reducing cooling demand by 20-25% and refrigeration demand
by 25-30% by the year 2037 are the goals of this plan.
3.NDMA Guidelines:
In 2016, the National Disaster Management Authority (NDMA) issued comprehensive guidelines to prepare national level key strategies for
mitigating the impact of heatwaves.

Que. 97 Consider the following statements about Gandhi Sagar Wildlife Sanctuary:
1.The Chambal River passes through the sanctuary dividing it into two parts with the western part is in Nimach district and eastern
part is in Mandsaur district.
2.Gandhi Sagar Wildlife Sanctuary is set to be the second home for cheetahs.
3.A major part of the sanctuary consists of vast open landscapes with sparse vegetation and rocky terrain, with small patches of dense
forests here and there.
Which of the above are incorrect statements ?

1. 1 and 2 only
2. 2 and 3 only
3. all the above
4. none of the above

Correct Option - 4
The correct answer is Option 4.
Key Points
Gandhi Sagar Sanctuary

The sanctuary's western part is in Nimach district and eastern part is in Mandsaur district of Madhya Pradesh.
It was notified in 1974 and more area was added in 1983.
The Chambal River passes through the sanctuary dividing it into two parts.
It is in the Khathiar-Gir dry deciduous forests ecoregion.

56/262
India is preparing to import a fresh batch of cheetahs from South Africa and introduce them into Madhya Pradesh’s Gandhi Sagar
Wildlife Sanctuary.
Herbivores like Chinkara, Nilgai and Spotted Deer, and carnivores like the Indian Leopard, Striped Hyena and Jackal are found in the
region.
The reservoir also has a good population of crocodiles, fish, otters and turtles. Gandhi Sagar Wildlife Sanctuary and the reservoir is
also a designated Important Bird and Biodiversity Area (IBA) as it boasts a large bird diversity with healthy population.

Additional Information
This Sanctuary has many places of historical, archaeological and religious importance.
These are Chaurasigarh, Chaturbhujnath temple, Bhadkaji rock paintings, Narsinghjhar, Hinglajgarh Fort, Taxakeshwar temple .

Que. 98 Consider the following with respect to Census on Water Bodies


It has the highest number of ponds
1. West Bengal
and reservoirs.
Andhra
2. It has the highest number of lakes
Pradesh
3. Tamil Nadu It has the highest number of tanks
It leads in water conservation
4. Maharashtra
schemes.
How many of the above pairs are correctly matched?

1. Only one
2. Only two
3. Only three
4. All four

Correct Option - 2
The correct answer is Option 2
Key Points
First Waterbodies Census

The Ministry of Jal Shakti recently released the report of its first-ever census of water bodies, revealing crucial insights into the
country's water resources.
The census provides an extensive inventory of water sources in India, highlighting disparities between rural and urban areas and
varying levels of encroachment.
The census of waterbodies was conducted in conjunction with the 6th Minor Irrigation Census for 2017-18.
It defines a waterbody as "all-natural or man-made units bounded on all sides with some or no masonry work used for storing water
for irrigation or other purposes."
The census aimed to provide an inventory of India's water resources, including natural and man-made water bodies like ponds, tanks,
lakes, and more, and to collect data on the encroachment of water bodies.

Additional Information

Key Insights from the Census:


1.The census enumerated a total of 24,24,540 water bodies across the country, with West Bengal accounting for the most (7.47 lakh) and
Sikkim the least (134).
2.The report reveals that:

West Bengal has the highest number of ponds and reservoirs. Hence Statement 1 is correct.
The top district in terms of waterbodies is South 24 Parganas in West Bengal.
Andhra Pradesh has the highest number of tanks. Hence statement 2 is incorrect.
Tamil Nadu has the highest number of lakes. Hence statement 3 is incorrect
Maharashtra leads in water conservation schemes. Hence statement 4 is correct.​

3.The report highlights that 97.1% of the waterbodies are in rural areas, with only 2.9% in urban areas.
4. Most of the waterbodies are ponds, followed by tanks, reservoirs, water conservation schemes, percolation tanks, check dams, lakes,
and others.
5. The census also collected data on the encroachment of waterbodies for the first time, revealing that 1.6% of all enumerated waterbodies are
encroached, with 95.4% of encroachments in rural areas and the remaining 4.6% in urban areas.
6.The census provides crucial data for policymakers to make informed decisions regarding water resource management and conservation.
7.It highlights disparities between rural and urban areas and the need for effective measures to prevent encroachment.

57/262
Que. 99 Consider the following statements regarding Seamounts:
1. Distributed across the entire ocean floor, these are numerous submerged volcanoes with pointed peaks..
2. When a volcano emerges above the ocean floor but its peak has been flattened due to erosion and is submerged under water.
3. They form a part of abyssal plains.
Which of the above statements is/are correct?

1. 1 only
2. 1 and 3 only
3. 2 and 3 only
4. 1,2 and 3

Correct Option - 2
The correct answer is option 2
Key Points:
Seamounts

It is a mountain with pointed summits, rising from the seafloor that does not reach the surface of the ocean. Seamounts are volcanic
in origin. These can be 3,000-4,500 m tall.Hence, statement 1 is correct.
The Emperor seamount, an extension of the Hawaiian Islands in the Pacific Ocean, is a good example.

Guyots:

The flat topped mountains (seamounts) are known as Guyots. Hence, Statement 2 is incorrect.
Seamounts and guyots are very common in the Pacific Ocean where they are estimated to number around 10,000.

Abyssal Plains:

Deep sea planes are gently sloping areas of the ocean basins.
These are the flattest and smoothest regions of the world because of terrigenous i.e denoting marine sediment eroded from the
land and shallow water sediments that buries the irregular topography.
It covers nearly 40% of the ocean floor.
The depths vary between 3,000 and 6,000 m.
These plains are covered with fine-grained sediments like clay and silt.​

Additional Information

Que. 100 Consider the following statements about Coastal Aquaculture Authority (Amendment) Bill 2023:
1. It will broaden the definition of Coastal Aquaculture to bring all coastal aquaculture activities like cage culture, seaweed
culture, marine ornamental and fish culture.
2. I will facilitate Aquaculture Units within the No Development Zone (NDZ) by establishments like hatcheries, Broodstock multiplication
centers (BMC), and Nucleus Breeding Centres (NBC) to operate within 300 meters from the High Tide Line (HTL).
3.The amendment aims to address previous uncertainties arising from the interpretation of Section 13(8) of the original CAA Act of 2005,
which had excluded coastal aquaculture from CRZ regulations.
4. The amendments uphold the 'Polluter Pays Principle,' mandating aquaculture unit owners to bear the cost of any environment-related
damage or demolition assessed by the Authority.

58/262
How many of the statements mentioned above are correct?

1. only 1
2. only 2
3. only 3
4. all four

Correct Option - 3
The correct answer is Option 3.
Key Points
Coastal Aquaculture Authority (Amendment) Bill, 2023:

This amendment will broaden the definition of Coastal Aquaculture which aims to bring all coastal aquaculture activities under the
purview of the Act.
This will include emerging aquaculture practices as it acknowledges the evolution of environmentally friendly aquaculture practices
beyond shrimp farming, incorporating methods like cage culture, seaweed culture and marine ornamental fish culture. Hence
statement 1 is correct.
These practices align with the evolving landscape and offer substantial revenue and employment opportunities for coastal
communities.
It will also facilitate aquaculture Units within the No Development Zone (NDZ). Establishments like hatcheries, Broodstock
multiplication centers (BMC), and Nucleus Breeding Centres (NBC) are now permitted to operate within 200 meters from the
High Tide Line (HTL). Hence statement 2 is incorrect.
The amendment aims to address previous uncertainties arising from the interpretation of Section 13(8) of the original CAA Act of
2005, which had excluded coastal aquaculture from CRZ regulations. Hence statement 3 is correct.
The amendments uphold the 'Polluter Pays Principle,' mandating aquaculture unit owners to bear the cost of any environment-
related damage or demolition assessed by the Authority. Hence statement 4 is correct.

Additional Information
India has a coastline of about 7,517 km, and a vast potential for coastal aquaculture development. The major coastal aquaculture
species in India are shrimp, fish, crab, oyster, mussel, seaweed, and pearl.
Over the past 9 years, shrimp production in India surged by 267%.
The nation's seafood exports experienced a doubling effect, surging from Rs 30,213 crore in 2013-14 to Rs 63,969 crore in 2022-23.
Key coastal states such as Andhra Pradesh, Gujarat, Odisha, and Tamil Nadu played a significant role in propelling the expansion
of coastal aquaculture shrimp production and subsequent exports.

Que. 101 Consider the following statements regarding biomass pellets:


1. These are a type of solid fuel made from compressed organic material.
2. These are of two kinds – torrefied and non-torrefied pellets.
3. The Central Pollution Control Board (CPCB) revised the financial grants for biomass pellet manufacturing units.
Which of the above statements are correct?

1. 1 and 2 only
2. 2 and 3 only
3. 1 and 3 only
4. 1,2 and 3

Correct Option - 4
The correct answer is option 4.
Key Points
Biomass Pellets:

These are a type of solid fuel made from compressed organic material (biomass) such as industrial waste and co-products, food waste,
agricultural residues, energy crops, and untreated lumber. Hence, Statement 1 is correct.
These are of two kinds – torrefied (Biomass processed at 250-350°C in the absence of oxygen) and non-torrefied pellets (biomass is
shredded, ground and sent to a pellet reactor, where it is compressed and bonded).Hence, Statement 2 is correct.
Advantage of Biomass Pellets:- Pellets are extremely dense and can be produced with a low moisture content (below 10%) which
allows them to be burned with a very high combustion efficiency.Emissions such as NOx, SOx and volatile organic compounds from
pellet burning equipment are in general very low.
Disadvantage of Biomass Pellets:- A recognized problem is the emission of fine particulate matter into the air.The capital expenditure
for setting up a pellet manufacturing unit entails variables such as land, machinery, air pollution control devices, factory infrastructures
such as sheds and offices, etc.
The Central Pollution Control Board (CPCB) revised the financial grants for biomass pellet manufacturing units. It has earmarked a
maximum allocation of Rs 28 lakhs per tonne per hour (TPH) of production capacity for manufacturing biomass pellets or 40 per
cent of the capital cost of the plant and machinery — whichever is lower. Hence, Statement 3 is correct.

59/262
Additional Information

Que. 102 Consider the following statements about Great Pacific Garbage Patch:
1. It is a vast area in South Pacific Ocean where high concentration of plastic debris is present.
2. It is caused by circular currents of the North Pacific Gyre that trap and concentrate debris in this area.
3. UN Environment Resolution aims to end plastic pollution by the year 2024.
Which of the above statements are correct?

1. 1 and 2 only
2. 2 and 3 only
3. 1 and 3 only
4. 1,2 and 3

Correct Option - 2
The correct answer is option 2.
Key Points
Great Pacific Garbage Patch:

It (also known as Pacific Trash Vortex) is a vast area in North Pacific Ocean where high concentration of plastic debris is present.
Hence Statement 1 is incorrect.
It is roughly 1.6 million square kilometers (3 times the size of France).
It is caused by circular currents of the North Pacific Gyre that trap and concentrate debris in this area.
It is divided into Eastern Garbage Patch (extends from California to Hawaii) and Western Garbage Patch(extends from Hawaii to
Japan).

Que. 103 'Snowpack' refers to which of the following statement?

1. It is snow on the ground, often at high elevation or altitudes, that persists until the arrival of warmer weather.
2. They are large, thick masses of ice that form on land when fallen snow gets compressed into ice over many centuries.
3. It is the boundary between a snow-covered and snow-free surface
4. Ice that broke off from glaciers or shelf ice and is floating in open water.

Correct Option - 1
The correct answer is option 1
Key Points

60/262
Snowpack is snow on the ground in mountainous areas that persists until the arrival of warmer weather. Melting snowpack is an
important source of water for many areas. Hence statement 1 is correct
Glaciers are large, thick masses of ice that form on land when fallen snow gets compressed into ice over many centuries.
An iceberg is ice that broke off from glaciers or shelf ice and is floating in open water.
To be classified as an iceberg, the height of the ice must be greater than 16 feet above sea level and the thickness must be 98-164
feet and the ice must cover an area of at least 5,382 square feet. There are smaller pieces of ice known as “bergy bits” and
“growlers.”
A bergy bit is a medium to large fragment of ice. Its height is generally greater than three feet but less than 16 feet above sea
level and its area is normally about 1,076-3,229 square feet. Growlers are smaller fragments of ice and are roughly the size of a
truck or grand piano.
Icebergs are monitored worldwide by the U.S. National Ice Center (NIC). NIC produces analyses and forecasts of Arctic,
Antarctic, Great Lakes, and Chesapeake Bay ice conditions. NIC is the only organization that names and tracks all Antarctic
Icebergs.
Snow line is the boundary between a snow-covered and snow-free surface. The actual snow line may adjust seasonally, and be either
significantly higher in elevation, or lower. The permanent snow line is the level above which snow will lie all year.

Que. 104 Consider the following statements regarding Zero Shadow Day:
1. It occurs when the Sun will be directly overhead at local noon, and hence any vertical object will cast no shadow. This happens
for places between the Tropic of Capricorn and Arctic Circle.
2. It is clearly different for different places on Earth.
Which of the above statements is/are correct?

1. Only 1
2. Only 2
3. Both 1 and 2
4. Neither 1 nor 2

Correct Option - 2
The correct answer is Option 2.
Key Points
Zero Shadow Day(ZSD):

It is a celestial phenomenon characterized by a temporary absence of shadows cast by vertical objects.


According to the Astronomical Society of India, ZSD typically occurs near the equator, falling between the Tropic of Capricorn and
the Tropic of Cancer. Hence, Statement 1 is incorrect.
Every location between the Tropic of Cancer and the Tropic of Capricorn experiences two Zero Shadow Days annually.
Zero Shadow Day arises when the Sun is directly overhead at local noon, causing vertical objects to cast no shadows. This
phenomenon is observed within the latitudes between the Tropic of Cancer and the Tropic of Capricorn.
This Zero Shadow Day will clearly be different for different places on Earth.Hence, Statement 2 is correct.

Additional Information
Why does a Zero Day happen?

​ he Sun is almost never exactly overhead at noon, but usually transits a bit lower in altitude, a bit to the north or a bit to the south.
T
The Earth's rotation axis is inclined at 23.5 degrees to the plane of its revolution around the Sun, which is why we have seasons. This
also means that the Sun, in its highest point of the day, will move from 23.5 degrees south of the celestial equator to 23.5 degrees north
of the equator (Uttarayan), and back again (Dakshinayan), in a year.
Uttarayan (movement of the Sun from south to north from winter solstice to summer solstice) and Dakshinayan (back from
north to south) happen because Earth’s rotation axis is tilted at an angle of roughly 23.5° to the axis of revolution around the Sun.
The Sun’s location moves from 23.5°N to 23.5°S of Earth’s equator and back.
For people living between +23.5 and -23.5 degrees latitude, the Sun's declination will be equal to their latitude twice - once during
Uttarayan and once during Dakshinayan.
On these two days, the Sun will be exactly overhead at noon and will not cast a shadow of an object on the ground.​

Que. 105 Consider the following statements about blue holes:


1.They are shallow, circular depressions in the ocean floor with vibrant blue water due to abundant algae.
2. Blue holes are formed by the erosion of limestone or coral bedrock by acidic groundwater over millions of years.
Which of the statements given above is/are correct?

1. 1 only
2. 2 only
3. Both 1 and 2
4. Neither 1 nor 2.

Correct Option - 2

61/262
The correct answer is option 2.
Key Points
Blue hole: A blue hole is a large marine cavern or sinkhole that opens to the surface and has formed in limestone or coral reef bedrock.
They are typically circular and have steep walls, and the deep water within the hole appears a much darker blue than the surrounding
shallows.

Blue holes are deep marine caverns, not shallow depressions. The deep water appears dark blue due to its depth, not abundant
algae. Hence , Statement 1 is incorrect.
Blue holes are formed through the erosion of limestone or coral by acidic groundwater. Hence , Statement 2 is Correct.

Additional Information
Formation: Blue holes are formed when acidic groundwater dissolves the underlying limestone or coral rock. This process can
occur over millions of years. Sea levels also play a role - many blue holes were formed during glacial periods when sea levels were
lower, exposing the caves to air and allowing further erosion. When sea levels rose again, the caves were flooded, creating the blue
holes we see today.
Location: Blue holes can be found all over the world, but they are particularly common in the Bahamas, Belize (where the most
famous blue hole, the Great Blue Hole, is located), the Philippines, and the Red Sea.
Depth: Blue holes can vary greatly in depth, from just a few meters to hundreds of meters deep. The Great Blue Hole, for example, is
over 400 feet deep.
Habitat: The deep, dark waters of blue holes are not hospitable to most marine life. However, the shallower areas around the rim of
the hole can be teeming with fish and other marine creatures. Blue holes are also popular spots for scuba diving and free diving

Que. 106 Sludge, a thick residue from Sewage Treatment Plants, is a concern for environmental sustainability because:

1. It is a valuable source of organic chemicals.


2. It contains high concentrations of organic matter, beneficial for plants.
3. Its improper management can lead to re-contamination of water sources.
4. It requires extensive infrastructure for treatment and disposal.

Correct Option - 2
The correct answer is option 2.
In News
The study by IIT Roorkee indeed revealed that Indian sludge has promising potential for use as fertilizer. However, there are crucial
steps to consider before widespread application. The sludge is rich in organic matter, containing essential nutrients like nitrogen and
phosphorus, beneficial for plant growth.

Key Points
Sludge:
Sludge is a thick residue filtered out of Sewage Treatment Plants (STP) that contains a high concentration of organic chemicals and
contaminants. Sludge management is a crucial aspect of ensuring environmental sustainability, particularly in developing countries where
wastewater treatment infrastructure is often inadequate.
Additional Information
The ‘Arth Ganga’ initiative, launched under the National Mission for Clean Ganga, aims to monetize and reuse treated wastewater
and sludge, thereby deriving livelihood opportunities from the river rejuvenation programme.
Sludge can be reused in various ways, including as organic fertilizers, bricks, and even as a potential biofuel. However, the
contaminants in sludge make it imperative to treat it before use to avoid potential hazards to human health and the environment.
Treated sludge is classified as Class A or Class B, with Class A being safe for disposal in the open and useful as organic fertilizer,
while Class B sludge can be used only in “restricted” agricultural applications. However, India does not yet have any standards
classifying sludge as Class A or B.
Under the Namami Ganga Mission, those awarded contracts for developing and maintaining STPs are apportioned land for disposing
of sludge. However, sludge treatment before disposal is rare, leading to the sludge often making its way back into rivers and local
water sources during rains.

Que. 107 The primary objective of the Kalasa-Bhandura project is to:

1. Generate hydropower for electricity production.


2. Improve irrigation facilities in drought-prone regions of Karnataka.
3. Address water scarcity for drinking purposes in northern Karnataka.
4. Enhance water navigation routes within the Mahadayi river basin.

Correct Option - 3
The correct answer is option 3.
In News
62/262
The National Board for Wildlife (NBWL) has deferred a decision on Karnataka’s application for the diversion of forest land for the
construction of the Kalasa- Bhandura nala irrigation projects in the Mahadayi basin.

Key Points
The Kalasa- Bhandura Nala project is a water diversion initiative undertaken by the Government of Karnataka to address water scarcity
in its northern districts. Here's a breakdown of the key points:
Purpose: Aims to improve drinking water supply in Belagavi, Bagalkot, Dharwad and Gadag districts, facing water scarcity.
Functioning: The project involves building diversion dams across Kalasa and Bandura, tributaries of the Mahadayi river, to divert
water to the Malaprabha river, which supplies drinking water to the target districts.
History: The project has been in the pipeline for decades and faced opposition from neighboring states, Goa and Maharashtra, which
share the Mahadayi river basin.
Current Status:
Central Water Commission approved Karnataka's revised project proposal in December 2022.
Karnataka received permission to divert a specific amount of water from the Mahadayi river at the proposed Kalasa and Bandura dams.
Concerns:
Potential impact on downstream water flow in Goa and Maharashtra.
Environmental impact on the Mahadayi basin ecosystem due to dam construction.

Overall, the Kalasa-Bhandura project is a contentious issue with economic and ecological considerations. While it aims to address water
scarcity in North Karnataka, concerns from neighboring states and potential environmental impacts require careful management.

Que. 108 Consider the following statements on River kolak:

1: The Kolak River originates in Gujarat and flows into the Arabian Sea.

2: The Kolak River is known for its pristine water quality and supports a thriving fish population.

Which of the following statements is/are correct?

1. 1 only
2. 2 only
3. Both 1 and 2
4. Neither 1 nor 2

Correct Option - 1
The correct answer is Option 1.
Key Points
The Kolak River is a river in Gujarat, India. It originates in the Kaprada taluka and flows into the Arabian Sea near Udwada. The
river basin has a maximum length of 50 kilometers and a catchment area of 584 square kilometers.
Unfortunately, the Kolak River is heavily polluted. The river becomes polluted from Morai village of Vapi where it meets with
Bhilkhadi river which has heavy chemical effluents from the chemical industries of Vapi GIDC and textile industries. Hence ,
statement 2 is incorrect.
This pollution has devastated the fish populations in the river and is a threat to the health of people who live along its banks.

In News
Fishermen and other villagers on the banks of the River Kolak have attributed the death of aquatic life and the reduced volume of catch to
the chemical pollutants disposed into the river through the sewage network.
Almost one person in every household in the region has suffered from cancer in the past with skin, mouth, blood, and spine being
common forms of cancer among the population.
The Government of Gujarat is planning to construct a pipeline to divert the treated or partially treated effluent into the Arabian Sea.

Que. 109 Consider the following statements


1: The Palghat Gap is a low mountain pass located between kerala and Tamilnadu..
2: Due to the Palghat Gap, the climatic conditions on both sides of the Western Ghats in this region are similar.
3: The Palghat Gap acts as a major passage for transportation and wind power generation in South India.
How many of the statements given above is/are correct?

1. Only one
2. Only two
3. All three
4. None

63/262
Correct Option - 2
The correct answer is option 2.
Key Points
Palghat gap: The Palghat Gap, also known as the Palakkad Gap, is a low mountain pass in the Western Ghats mountain range of
southwestern India. Situated between the Nilgiri Hills to the north and the Anaimalai Hills to the south, the Palghat Gap acts as a natural
gateway between the states of Kerala and Tamil Nadu.

The Palghat Gap is a low mountain pass located between the states of Kerala and Tamilnadu in India. Hence , statement 1 is correct.

The Palghat Gap creates a distinct ecological zone due to the break in the mountain range, leading to different flora and fauna on either
side. Hence , statement 2 is incorrect.

The Palghat Gap serves as a crucial passage for transportation between Kerala and Tamil Nadu and channels winds ideal for wind
power generation in the surrounding areas. Hence , statement 3 is correct.

Additional Information
Historical Significance: Throughout history, the Palghat Gap has served as a vital passage for trade and travel between Kerala and
Tamil Nadu. It facilitated movement for both people and goods, playing a role in the cultural exchange between the regions.
Ecological Biodiversity: Interestingly, the Palghat Gap creates a distinct ecological zone. Due to the break in the mountain range, the
flora and fauna on either side of the gap differ. This variation contributes to the overall biodiversity of the Western Ghats. The gap
itself features dry evergreen forest, contrasting with the rainforests on the western slopes of the Western Ghats.
Climatic Influence: The Palghat Gap plays a role in shaping the climate of southern India. It allows the southwest monsoon winds
and storms from the Bay of Bengal to pass through the Western Ghats, influencing the rainfall patterns in the region.
Wind Power Generation: The Palghat Gap funnels winds from the west, making the surrounding areas, including Palakkad District
in Kerala and Coimbatore and Tirupur districts in Tamil Nadu, a major hub for wind power generation.

Que. 110 Which of the following statements accurately describes the Port of Haifa?

1. The Port of Haifa is located in Cyprus and primarily serves as a hub for European trade.
2. It is one of the smallest seaports in the Mediterranean region, mainly handling fishing vessels
3. The Port of Haifa is situated in Israel and is one of the largest and busiest ports in the Eastern Mediterranean.
4. It primarily focuses on tourism, offering cruise ship services along the Mediterranean coast.

Correct Option - 3
The correct answer is option 3.
In News
Port of Haifa is the major transit point in recently proposed India-Middle East-Europe Economic Corridor(IMEC).
IMEC aims to create a vast trade network connecting India, Europe, and the Middle East. This will involve a combination of sea
routes, railways, and potentially even pipelines for clean hydrogen. The goal is to improve trade efficiency and reduce costs for
businesses across these regions.

Key Points
The Port of Haifa is the largest of Israel's three major international seaports, the others being the Port of Ashdod and the Port of Eilat.
Located on the Mediterranean Sea, it's a natural deep-water harbor that operates year-round. The port serves both passenger and
merchant ships, making it one of the largest ports in the eastern Mediterranean in terms of freight volume.

Que. 111 Which of the following statements best describes the concept of Land Degradation Neutrality (LDN)?

1. LDN aims to accelerate land degradation to make room for sustainable land management practices.
2. LDN seeks to maintain or improve the condition of land resources by achieving a balance between land degradation and restoration.
3. LDN primarily focuses on exploiting land resources without considering the consequences of degradation.
4. LDN targets the complete cessation of all land use activities to prevent any form of degradation.

Correct Option - 2
The correct answer is option 2.
Key Points
Land degradation neutrality (LDN): Land degradation neutrality (LDN) is a global goal set to achieve a balance between the amount of
healthy and productive land resources lost through degradation and the amount of degraded land restored. The concept of LDN emerged
from the UN Conference on Sustainable Development (Rio+20) in 2012. It recognizes the importance of our finite land resources and aims
to achieve a world where human activity has a neutral, or even positive, impact on the land.

64/262
14/05/2024, 13:51 lms.testbook.com/genpdf/fromques.php?qids=65f93cf52fe08b6c70dc4d11,65f93e6eb5431ddfb31dfe15,65f96fd18acd632ab8e39282,65f97…
Three main ways to achieve LDN:
1. Avoiding new degradation: This involves protecting existing healthy land through sustainable land management practices. This can
include things like reducing deforestation, preventing overgrazing, and using cover crops to protect soil health.
2. Reducing existing degradation: This involves restoring degraded land to a more productive state. This can be done through techniques
like reforestation, soil conservation, and rainwater harvesting.
3. Restoring degraded lands: This involves returning degraded land to a natural or more productive state. This can be a long-term
process, but it is essential for ensuring the long-term sustainability of our land resources.

There are many benefits to achieving LDN. These include:


Improved food security: Healthy land is more productive land, which means it can produce more food to feed a growing population.
Enhanced biodiversity: LDN helps to protect and restore ecosystems, which benefits a wide range of plant and animal life.
Increased climate change resilience: Healthy land is better able to store carbon and withstand the impacts of climate change, such as
drought and floods.

Que. 112 In the context of India's electric vehicle (EV) policy, e-retrofitment of existing vehicles refers to:

1. Replacing two-wheelers with electric alternatives provided by ride-sharing companies.


2. Setting up new manufacturing facilities for electric vehicles.
3. Converting gasoline or diesel powered vehicles into electric vehicles.
4. Building a network of charging stations for electric vehicles across the country.

Correct Option - 3
The correct answer is option 3.
Key Points
E-retrofitment: E-retrofitment refers to the process of converting a conventional vehicle powered by an internal combustion engine
(ICE) into an electric vehicle (EV). This involves replacing the original gasoline or diesel engine and related components with an electric
powertrain, including a battery, electric motor, and inverter.
Benefits:
Reduced emissions: E-retrofitted vehicles produce zero tailpipe emissions, contributing to cleaner air and combating climate change.
Extended lifespan: Retrofitting can breathe new life into older vehicles, allowing them to avoid scrappage and remain operational for
several more years.
Cost-effective alternative: Compared to buying a new EV, retrofitting can be a more affordable way to experience the benefits of electric
mobility.

Challenges:
Technical complexity: E-retrofitment is a complex process requiring specialized knowledge and expertise to ensure compatibility and
safety.
Battery limitations: Replacing the ICE with an electric powertrain might limit cargo space due to the battery placement. Additionally,
range anxiety can be a concern depending on battery capacity.
Regulatory hurdles: Regulations around e-retrofitment are still evolving in many countries. Obtaining necessary approvals and ensuring
compliance can be challenging.

Que. 113 Consider the following statements


1. Debt for climate swaps involve creditor nations forgiving a portion of a debtor nation's debt in exchange for the debtor nation
implementing renewable energy policies.
2. The concept of debt for climate swaps is entirely new and has not been attempted before.
3. Debt for climate swaps offer a potential solution to both climate change and the problem of high debt levels in developing countries.
How many of the statements give above is/are correct ?

1. Only one
2. Only two
3. All three
4. None

Correct Option - 2
The correct answer is option 2.
Key Points
Debt for Climate swaps: Debt for climate swaps are financial agreements between creditor and debtor countries. Here's how they work:
Debt Relief: A creditor (lender) nation or organization agrees to forgive a portion of a debtor nation's debt.
Climate Action: In return, the debtor nation commits to taking specific actions to address climate change. This could involve:
Investing in renewable energy sources
Implementing policies to reduce greenhouse gas emissions

65/262
Protecting forests or other natural resources
Debt for climate swaps do involve debt forgiveness in exchange for climate action. Hence ,Statement 1 is correct.
Debt-for-climate swaps emerged from the concept of debt-for-nature swaps, which began in the 1980s to conserve rainforests in Latin
America.
. Hence ,Statement 2 is incorrect.
Debt for climate swaps can help developing nations free up resources for climate action while also reducing their debt burden.
Hence, Statement 3 is correct.
Additional Information
Benefits:
For debtor nations:
Lighter debt burden frees up resources for climate action and development.
For creditor nations and organizations:
Promotes environmental sustainability and combats climate change.

Current Landscape:
Several developing countries, particularly those vulnerable to climate change, are exploring debt-for-climate swaps.
International organizations like the United Nations Development Programme (UNDP) are advocating for these swaps.

Que. 114 Consider the following statements regarding the Carbon Offsetting and Reduction Scheme for International Aviation (CORSIA):
1. CORSIA is a mandatory scheme for all International Civil Aviation Organization (ICAO) member states from its inception.
2. The primary objective of CORSIA is to achieve carbon-negative growth in the international aviation sector.
3. Airlines participating in CORSIA can offset their unavoidable CO2 emissions by purchasing credits from certified programs.
How many of the statements given above is/are correct?

1. Only one
2. Only two
3. All three
4. None

Correct Option - 1
The correct answer is option 1.
Key Points
CORSIA: Carbon Offsetting and Reduction Scheme for International Aviation (CORSIA) is a global program implemented by the
International Civil Aviation Organization (ICAO) to address the growing impact of aviation on climate change.

CORSIA has a phased approach with a voluntary component initially. Hence , Statement 1 is Incorrect.
CORSIA aims to stabilize emissions at 2020 levels, not achieve negative growth. Hence, Statement 2 is Incorrect..
Offsetting emissions through certified programs is a core element of CORSIA. Hence , Statement 3 is Correct.

Additional Information
Goal:
Reduce and stabilize net CO2 emissions from international aviation at 2020 levels.
Achieve carbon neutral growth from 2020 onwards.

Approach:

CORSIA is a three-pronged strategy:


1. Reduce Emissions: Encourage technological advancements, operational improvements, and the use of sustainable aviation fuels to
directly minimize CO2 emissions from airplanes.
2. Offset Emissions: For emissions that cannot be reduced through the above methods, airlines are required to purchase carbon offsets
from certified programs. These offsets represent projects that remove CO2 from the atmosphere, such as tree planting or renewable
energy initiatives.
3. Monitoring, Reporting, and Verification (MRV): A robust system is in place to track airline emissions, ensure the validity of carbon
offsets, and verify compliance with the scheme.

Benefits:
Global Action: Provides a unified approach to tackling aviation emissions on a worldwide scale.
Market-Based Solution: Leverages the carbon market to incentivize emission reduction projects.
Sustainable Growth: Aims to balance aviation growth with environmental responsibility.

Challenges:
Effectiveness of Offsets: Concerns exist about the long-term viability and environmental impact of some carbon offset projects.
Enforcement: Ensuring consistent application and compliance across all member states.

Que. 115 The HI-WISE report focuses on the interconnections between:

1. Water resources and agriculture in the Himalayas.


66/262
2. Glaciers, water availability, ecosystems, and human societies in the Hindu Kush Himalaya.
3. Melting snowpack and tourism industry in the region.
4. Deforestation and soil erosion in the mountains.

Correct Option - 2
The correct answer is option 2.
Key Points
HI - WISE report : The HI-WISE report, also known as the "Water, Ice, Society, and Ecosystems in the Hindu Kush Himalaya" report,
is a significant study on the impact of climate change on the Hindu Kush Himalayan region.
Focus: The report examines the connections between the cryosphere (glaciers, snowpack), water resources, ecosystems, and human
societies in the Hindu Kush Himalaya.
Context: The International Centre for Integrated Mountain Development (ICIMOD) released the Water, Ice, Society, and Ecosystems in
the Hindu Kush Himalaya (HI-WISE) report.
Key Findings:
Melting glaciers are causing increased water flow in the short term, but this will decrease significantly by 2100.
This will have cascading effects on water availability, ecosystems, and people relying on these resources.
Recommendations: The report urges policymakers to prepare for these changes through regional cooperation and adaptation strategies.
Additional Information

Hindu Kush Himalaya (HKH) region:

It is a vast area, encompassing mountain ranges stretching from


the Hindu Kush range in northern Afghanistan to the Arakan range in Myanmar,

with the Himalayan range as its spine, and also includes the Tibetan Plateau.

The region harbours the highest mountain ranges in the world and contains the largest volume of ice on earth outside of the
polar areas and is called “Asia’s water tower”.

Ice and snow in the HKH are important sources of water for 12 rivers that flow through 16 countries in Asia.

These rivers provide freshwater and other vital ecosystem services to 240 million people in the mountains and a further 1.65
billion downstream.

The region is undergoing “unprecedented and largely irreversible” changes triggered by global warming.

ICIMOD:

It is a regional intergovernmental learning and knowledge-sharing centre serving the 8 regional member countries (RMCs)
of the HKH region – Afghanistan, Bangladesh, Bhutan, China, India, Myanmar, Nepal, and Pakistan.

It was established in 1983 and is headquartered at Lalitpur, which is located in the Kathmandu valley of Nepal.

ICIMOD promotes and fosters partnerships amongst the RMCs to secure a better future for the people and environment of
the HKH region

Que. 116 The primary cause of ionization in the ionosphere is:

1. Visible light from the Sun


2. Extreme ultraviolet (EUV) and X-ray radiation from the Sun
3. Reflection of radio waves
4. Collisions between large molecules in the atmosphere

Correct Option - 2
The correct answer is option 2.
Key Points
Ionosphere : The ionosphere is a fascinating layer of Earth's atmosphere, located from about 48 kilometers (30 mi) to 965 kilometers (600
mi) above sea level. It's constantly changing and plays a vital role in many things, including radio communication and the aurora borealis.
Features:

The ionosphere gets its name from the fact that it's ionized, meaning that it contains a high concentration of charged particles,
both ions (atoms that have lost or gained electrons) and free electrons.

This ionization is caused by solar radiation, particularly extreme ultraviolet (EUV) and X-ray radiation. When these high-
energy photons collide with atoms and molecules in the upper atmosphere, they knock off electrons, creating ions and free electrons.

67/262
The ionosphere is not a uniform layer; it has several distinct regions with different properties. The main layers are the D layer, the E
layer, and the F layer. These layers vary in thickness and height depending on the time of day, season, and solar activity.

The ionosphere plays an important role in radio communication. Radio waves can be reflected or refracted by the ionosphere, which
allows them to travel long distances over the Earth's surface.

Que. 117 Consider the following statements above Singhbhum craton


1. The Singhbhum Craton is located entirely within the state of Jharkhand.
2. The presence of greenstone belts in the Singhbhum Craton can aid in understanding the evolution of early life on Earth.
Choose the correct answer.

1. Both Statement 1 and Statement 2 are correct.


2. Both Statement 1 and Statement 2 are incorrect.
3. Statement 1 is correct, Statement 2 is incorrect.
4. Statement 1 is incorrect, Statement 2 is correct.

Correct Option - 4
The correct answer is option 4.
Key Points
Singhbhum Craton: The Singhbhum Craton is a large area of ancient, stable continental crust located in eastern India, spanning parts of
Jharkhand and Odisha states. It's one of the oldest cratons in the world, estimated to be around 3.5 billion years old . Hence, statement 1 is
incorrect.

Greenstone belts contain elements like iron and magnesium, crucial for the formation of early life. Studying their composition sheds
light on the conditions during the Archean era when life emerged. Hence , statement 2 is correct.

Additional Information
Key features of the Singhbhum Craton:
Composition: Primarily composed of Archean rocks, including volcanic and sedimentary rocks known as greenstone belts. These
greenstones are rich in iron and magnesium.
Location: Singhbhum Craton is bordered by the Chhotanagpur plateau to the north, the Bastar craton to the south, the Eastern
Ghats to the southeast, and alluvial plains to the east.
Significance: Provides valuable insights into the Earth's early geological history and tectonic processes. The presence of greenstone
belts helps scientists understand the formation of continents and the emergence of life on Earth.
Minerals: The Singhbhum Craton is rich in mineral resources, including iron ore, copper, uranium, and limestone. These minerals have
played a significant role in the industrial development of eastern India.

Que. 118 Consider the following statements about the Blue Ocean Event (BOE):
I. It refers to a situation where the Arctic Ocean becomes nearly ice-free during summers.
II. It is expected to have minimal impact on global weather patterns.
III. It can disrupt ocean currents and potentially lead to more extreme weather events.
How many of the given above statements is/are correct?

1. Only one
2. Only two
3. All three
4. None

Correct Option - 2
The correct answer is option 2.
Key Points
Blue Ocean Event (BOE) : A Blue Ocean Event (BOE) refers to a scenario where the Arctic Ocean becomes largely ice-free during the
summer months. This means that the sea ice cover in the Arctic shrinks to less than 1 million square kilometers, exposing a vast expanse of
open water.

A Blue Ocean Event (BOE) refers to a situation where the Arctic Ocean loses most of its sea ice cover during summer, exposing a vast
area of open water. Hence , Statement I Correct .
Melting sea ice has a significant impact on global weather patterns. Sea ice acts like a giant reflector, sending sunlight back into space.
When it melts, the darker ocean absorbs more heat, accelerating warming and disrupting ocean currents that influence global climate.
Hence, Statement II is Incorrect.
Disruption of ocean currents due to reduced sea ice cover is a major concern associated with BOE. This disruption can lead to more
extreme weather events worldwide. Hence, Statement III Correct.

The Blue Ocean Event is considered a critical tipping point in Arctic climate change. Here's why it's a concern:
Sea ice reflects sunlight back into space, helping to regulate Earth's temperature. When sea ice melts, the ocean absorbs more solar
radiation, accelerating warming. This creates a vicious cycle that can lead to even faster ice loss.

68/262
Sea ice is crucial for the health of Arctic ecosystems. It provides habitat for polar bears, seals, walruses, and other animals. A melting
Arctic could lead to population declines and even extinction for some species.
The Blue Ocean Event could disrupt weather patterns around the world. Sea ice plays a role in regulating ocean currents, which
influence global climate. A melting Arctic could disrupt these currents, leading to more extreme weather events.

The possibility of a Blue Ocean Event has become a major concern for scientists and policymakers. Some studies suggest that it could
happen as early as the 2030s, even with aggressive action to reduce greenhouse gas emissions.

Que. 119 Consider the following statements about cotton cultivation in India:
1.India is the world's largest producer of cotton by area under cultivation.
2 .India has the highest yield of cotton per hectare globally.
Which of the statements given above is/are correct?

1. 1 only
2. 2 only
3. Both 1 and 2
4. Neither 1 nor 2

Correct Option - 1
The correct answer is option 1.
Key Points
Cotton: Cotton is a versatile crop that provides food, feed, and fiber for various uses, including textiles, cooking oil, and livestock
feed. It is also a major source of income and employment for millions of farmers in India.
India is a major player in the world of cotton. Here's some information on cotton crops in India:
Importance: Cotton is a crucial commercial crop in India, accounting for roughly 25% of global cotton production. It's nicknamed
"White Gold" due to its economic significance [PDF from Ministry of Textiles, Govt. of India]. The textile industry in India is heavily
reliant on cotton, and millions of people depend on it for their livelihood.
Area and Yield: India leads the world in terms of area dedicated to cotton cultivation, with around 36% of the global cotton acreage.
However, India's yield per hectare is lower compared to other major producers.
Growing regions: Cotton cultivation primarily happens in ten major states across three zones:
North zone: Punjab, Haryana, and Rajasthan
Central zone: Madhya Pradesh, Maharashtra, and Gujarat
South zone: Telangana, Andhra Pradesh, Karnataka, and Tamil Nadu.
Planting Season: The sowing season for cotton varies depending on the region. In most parts of India, it's a Kharif crop ( खरीफ), planted
between March and May for irrigated fields and June-July for rain-fed crops, coinciding with the monsoon season. The southernmost
regions have a different planting window.

Que. 120 What is the main reason why tropical cyclones form over warm ocean waters?

1. The warm water creates low pressure.


2. The warm water provides energy for the storm to develop.
3. The warm water disrupts the Coriolis effect.
4. The warm water creates strong wind shear.

Correct Option - 2
The correct answer is option 2.
Key Points
Tropical cyclones : Tropical cyclones are intense rotating storm systems that form over warm tropical waters. They are known by different
names depending on their location:
Hurricane: Atlantic Ocean and northeastern Pacific Ocean
Typhoon: Northwestern Pacific Ocean
Cyclonic storm: Indian Ocean and South Pacific Ocean

Tropical cyclones are powerful weather phenomena that can cause significant damage with strong winds, heavy rain, and storm surge.

Key characteristics of tropical cyclones:


Low pressure center: Tropical cyclones have a central low-pressure region, which is why winds spiral inward towards the center.
Closed circulation: The winds in a tropical cyclone rotate in a closed circulation, counter-clockwise in the Northern Hemisphere and
clockwise in the Southern Hemisphere.
Warm core: Tropical cyclones are warm-core storms, meaning the air at their center is warmer than the surrounding air.
Spiral thunderstorms: Tropical cyclones are characterized by heavy rain and thunderstorms that are arranged in a spiral pattern around
the center.
Tropical cyclones form over warm ocean waters (typically at least 80°F or 27°C) where there is little wind shear (change in wind
speed or direction with height). Tropical cyclones are essentially giant heat engines. The warm water provides the energy for the storm
to develop and intensify.
The Coriolis effect, caused by the Earth's rotation, deflects the winds in a tropical cyclone, giving it its characteristic spiral shape

69/262
Que. 121 The Indian Ocean Geoid Low (IOGL) is also known as a:

1. Magnetic Anomaly
2. Gravity Hole
3. Seismic Zone
4. Oceanic Trench

Correct Option - 2
The correct answer is option 2.
Key Points
IOGL :The Indian Ocean has a fascinating anomaly called the Indian Ocean Geoid Low (IOGL), often referred to as a "gravity hole.
Here's what we know about it:

It's Earth's Largest Gravity Anomaly: Gravity in this region is weaker than anywhere else on Earth.
It's a Depression in Sea Level: If not for counteracting forces, the sea level here would be up to 106 meters lower than the global
average.
It's Caused by Uneven Mass Distribution: Variations in Earth's density create this anomaly.
Recent Explanation Points to Ancient Origins: A team from the Indian Institute of Science (IISc) suggests it formed from remnants
of an ancient seafloor.

The IOGL is a relatively recent scientific discovery (1948) and only recently (May 2023) have scientists explained its formation
through computer simulations and seismic data.

Que. 122 Consider the following statements about shelf clouds:


1.Shelf clouds are high-altitude formations that indicate fair weather.
2. They appear as a leading edge of a thunderstorm, often with a rolling motion.
Which of the statements given above is/are correct?

1. 1 only
2. 2 only
3. Both 1 and 2
4. Neither 1 nor 2

Correct Option - 2
The correct answer is option 2.
Key Points
Shelf cloud : A shelf cloud, also known as an arcus cloud, is a low-lying, horizontal cloud formation that appears ahead of a
thunderstorm. Hence, statement 1 is incorrect.

Shelf clouds are a leading edge of a thunderstorm and often exhibit a rolling motion along their horizontal form. This rolling
motion is caused by the cold air downdraft pushing warm, moist air upwards. Hence, statement 2 is correct.

Additional information about shelf clouds:

Formation: A shelf cloud forms when a cold downdraft from a thunderstorm rushes towards the ground. The cold air spreads rapidly
along the ground, pushing existing warm moist air upwards. As the cold air cools the warm air, condensation occurs, creating the shelf
cloud.
Indication of Possible Severe Weather: Shelf clouds are often a sign that a strong downdraft, or gust front, is approaching. This can
bring strong winds, heavy rain, and even hail.
Distinguishing from Other Clouds: Shelf clouds can be distinguished from wall clouds and tornadoes by their shape and movement.
Shelf clouds are horizontal and flat, while wall clouds are lower, more vertical, and rotate. Tornadoes are funnel-shaped clouds that
rotate violently and touch the ground.

Que. 123 Consider the following statements about E20 fuel:


1. It is a blend of gasoline and ethanol, with ethanol content being 20%.
2. Due to its biofuel nature, it contributes to a reduction in overall greenhouse gas emissions.
3. E20 fuel offers superior fuel efficiency compared to regular gasoline.
How many of the statements given above is/are correct?

70/262
1. Only one
2. Only two
3. All three
4. None

Correct Option - 2
The correct answer is option 2.
Key Points
E-20 Fuel : E20 fuel is a blend of gasoline and ethanol, where ethanol makes up 20% of the mixture and gasoline makes up the remaining
80%. Ethanol is a biofuel that is produced from plant material such as corn or sugarcane.

E20 fuel indeed refers to a mixture where 20% is ethanol and 80% is gasoline. Hence , statement 1 is correct.
Ethanol is considered a biofuel because it's derived from renewable sources like crops. While burning E20 does release emissions, the
plants used to produce the ethanol absorb carbon dioxide during their growth, potentially offsetting some of those emissions. Hence ,
statement 2 is correct.
E20 fuel actually has a slightly lower fuel efficiency compared to regular gasoline. Ethanol contains less energy per unit volume
compared to gasoline. While the difference might be minimal, you might see a slight decrease in gas mileage using E20. Hence ,
statement 3 is incorrect.

Additional Information
Benefits of E20 fuel:
Reduced greenhouse gas emissions: Since ethanol is a biofuel, it is considered to be carbon neutral. This means that the amount of
carbon dioxide released when the fuel is burned is offset by the amount of carbon dioxide absorbed by the plants used to produce the
ethanol.
Reduced dependence on fossil fuels: By using ethanol in gasoline, we can reduce our reliance on oil.
Renewable resource: Ethanol can be produced from a variety of renewable resources, such as corn, sugarcane, and other biomass.

Things to consider with E20 fuel:


Not all vehicles are compatible with E20 fuel: Check your owner's manual to see if your vehicle is approved for E20 fuel.
Slightly lower fuel efficiency: E20 fuel contains less energy than gasoline, so you may see a slight decrease in fuel economy.
Availability: E20 fuel is not available at all gas stations.

Que. 124 Consider the following statements about lightning:


I. Lightning strikes are a completely unpredictable natural phenomenon.
II. Deaths caused by lightning can be significantly reduced through awareness campaigns.
III. The Indian government classifies lightning strikes as natural disasters for purposes of relief funds.
How many of the above given statements is/are correct?

1. Only one
2. Only two
3. All three
4. None

Correct Option - 1
The correct answer is option 1.
Key Points
Lightning : Lightning is a powerful and visible electrical phenomenon that takes place when there is a buildup of electrical charges
within clouds and between clouds and the ground. This electrical energy is released, producing the distinctive thunder that goes along with
lightning along with a dazzling flash of light and a quick expansion of air.

While the exact path of a lightning strike might be difficult to pinpoint, lightning typically occurs during thunderstorms, which are
predictable weather events. Hence ,Statement I is Incorrect.
Public education campaigns on lightning safety can significantly reduce casualties by teaching people to avoid open areas, tall objects,
and electrical appliances during thunderstorms. Hence ,Statement II is correct.
The Indian government does not currently classify lightning strikes as natural disasters [1]. This is because they argue deaths are
preventable with proper awareness. Hence ,Statement III is correct.

Additional Information
Current Status:

In India, for instance, the government disagrees with classifying lightning as a natural disaster. Their reasoning lies in the belief that focusing
on education and preventive measures can significantly reduce lightning-related fatalities. This avoids the need for disaster relief funds,
which are typically used for large-scale events with widespread damage.

So, lightning occupies a grey area. It's a powerful natural phenomenon, but unlike traditional disasters, human behavior plays a significant
role in mitigating its impact.

71/262
Que. 125 The period of least solar activity in the Sun's 11-year cycle is characterized by:

1. Increased sunspots and frequent solar flares


2. A decrease in sunspots and less frequent solar flares
3. Continuous auroras at high latitudes
4. o change in the Sun's overall brightness

Correct Option - 2
The correct answer is option 2.
Key Points
The sun goes through a cycle of activity that waxes and wanes roughly every 11 years. This is known as the solar cycle. The two extremes
of this cycle are called solar minimum and solar maximum.

Solar Minimum
The solar minimum is the period of least solar activity in the Sun's 11-year cycle.
During solar minimum, sunspot and solar flare activity diminishes, and often does not occur for days at a time.
Sunspots are cooler, darker areas on the Sun's surface caused by intense magnetic activity.
Solar flares are sudden, intense bursts of radiation from the Sun's surface.
Coronal mass ejections (CMEs) are large releases of matter and magnetism from the Sun's corona.

Solar Maximum
Solar maximum is the peak of the solar cycle, when solar activity is at its highest.
This is marked by a large increase in sunspots, solar flares, and CMEs.
During solar maximum, the Sun can be spotted with hundreds of sunspots at a time.
Solar flares and CMEs can be much more powerful during solar maximum than at other times in the cycle.
The effects of solar activity can be felt on Earth. Solar flares and CMEs can disrupt radio communications, damage satellites, and even cause
power outages. They can also create beautiful auroras at high latitudes.

Que. 126 Which of the following statements is not true about waterspouts?

1. They are rotating columns of air that form over water.


2. They can be weaker or stronger depending on their formation.
3. All waterspouts are associated with severe thunderstorms.
4. They are most common in cold waters near the poles.

Correct Option - 3
The correct answer is option 3.
Key Points
Waterspout : A waterspout is an intense, rotating column of air that forms over a body of water. They are often referred to as
"tornadoes over water" because they share many similarities with tornadoes that form over land. However, waterspouts are typically
weaker and shorter-lived than tornadoes.
There are two main types of waterspouts:
Fair weather waterspouts: These are the weaker and more common type of waterspout. They form over warm water on sunny days,
and are not associated with thunderstorms. Hence, option 3 is correct.
Tornadic waterspouts: These are stronger and more dangerous than fair weather waterspouts. They form in association with severe
thunderstorms, and can be accompanied by high winds, hail, and lightning.

Waterspouts can form anywhere in the world, but they are most common in tropical and subtropical waters. They are also relatively
common in the Great Lakes and off the coast of the United States.

Que. 127 Which of the following statements regarding plasticulture in agriculture is not correct?

1. Plasticulture involves the use of plastic materials for purposes such as mulching, irrigation, and crop protection.
2. Plastic mulches are primarily used to enhance soil fertility and promote beneficial microbial activity.
3. Drip irrigation systems, often employing plastic tubing, are utilized in plasticulture to deliver water directly to plant roots, thus
conserving water.
4. Plasticulture can contribute to increased crop productivity, efficient water usage, and extended growing seasons in controlled
environments.

Correct Option - 2
The correct answer is option 2.
Key Points

72/262
Plasticulture : Plasticulture refers to the use of plastic materials in agriculture for various purposes such as mulching, irrigation, and
covering. It's a modern agricultural practice that involves the use of plastic films, sheets, and containers to enhance crop production,
conserve water, control weeds, and protect crops from pests and diseases.

While plastic mulches in plasticulture are indeed used to suppress weeds, conserve moisture, regulate soil temperature, and
prevent soil erosion, they are not primarily intended to enhance soil fertility or promote beneficial microbial activity. Instead,
plastic mulches create a physical barrier between the soil and the atmosphere, altering the microclimate around plants. Hence , Option
2 is correct answer.

Additional Information
Some common applications of plasticulture include:

Mulching: Plastic mulches are used to cover the soil around plants to suppress weeds, conserve moisture, regulate soil temperature, and
prevent soil erosion.

Irrigation: Drip irrigation systems, often using plastic tubing, are employed to deliver water directly to the plant roots, minimizing water
wastage and maximizing efficiency.

Greenhouse Covering: Greenhouses utilize plastic films to create a controlled environment for plant growth, allowing farmers to extend the
growing season and protect crops from adverse weather conditions.

Crop Protection: Row covers made of plastic materials can be used to shield crops from pests, birds, and harsh weather, promoting
healthier plant growth and higher yields.

Soil Fumigation: Plastic films can be used to trap fumigants in the soil, effectively sterilizing it and controlling soil-borne pests and diseases
before planting.

Plasticulture has gained popularity in many regions due to its ability to increase productivity, reduce labor costs, and improve resource
efficiency. However, there are also concerns associated with its environmental impact, such as plastic waste accumulation and potential soil
and water contamination from plastic residues.

Que. 128 Consider the following statements about cloudbursts:


1. Cloudbursts are intense downpours of rain occurring over a large geographical area.
2. They are more common in areas with high altitude variations like mountains.
3. Cloudbursts are infrequent and require specific atmospheric conditions to form.
How many of the statements given above is/are correct?

1. Only one
2. Only two
3. All three
4. None

Correct Option - 2
The correct answer is option 2.
Key Points
Cloudburst : A cloudburst is an intense downpour of rain occurring over a limited geographical area for a short duration. It's
basically a sudden, very heavy rainfall, sometimes accompanied by hail and thunder, which is capable of creating flash floods and
landslides.

Cloudbursts occur over a limited geographical area, not a large one. Hence ,Statement 1 is incorrect.
Cloudbursts are more common in mountainous areas due to orographic lift. Hence ,Statement 2 is correct.
Cloudbursts are infrequent events requiring specific atmospheric conditions. Hence , Statement 3 is correct.

Additional Information
Cloudbursts can happen anywhere but are more common in mountainous areas. This is because warm air currents associated with
thunderstorms tend to follow the upward slope of a mountain.
The rising air cools and condenses, forming massive rain clouds that can unload large amounts of water in a short period. The
effects of heavy rain are often worse on mountain slopes because the falling water is concentrated in valleys and streams, which can
quickly overflow their banks and cause flash floods.

73/262
Cloudbursts are infrequent as they require specific atmospheric conditions to form. They
typically occur via:

Orographic lift: When warm, moist air is forced upwards by a mountain range, it cools and
condenses, forming clouds that can produce heavy precipitation.
Convective uplift: When warm air near the ground rises due to buoyancy, it cools and
condenses, forming clouds that can produce heavy rain.

sha mor

Que. 129 Palaeo proxies are used to study:

1. Economic trends in ancient civilizations


2. Past climate and environmental conditions
3. Movements of tectonic plates
4. Composition of stars and galaxies

Correct Option - 2
The correct answer is option 2.
Key Points
Palaeo proxies : Paleo proxies, also known as paleoclimate or paleoenvironmental proxies, are indications that scientists use to recreate
historical climate and environmental conditions.

These proxies are usually produced from physical, biological, or chemical processes that react to variations in temperature or other
climatic conditions. Because direct measurements of temperature in the distant past are impossible, scientists rely on proxy records to
understand historical climate fluctuations and long-term trends.

Examples:
Ice Cores: Ice cores dug from glaciers and polar ice sheets include trapped air bubbles and isotopic compositions that reveal historical
climatic conditions such as temperature and greenhouse gas concentrations.
Tree Rings: The width, density, and isotopic composition of tree rings can disclose past climatic variations and tree growth conditions,
making them a useful proxy for temperature and precipitation changes.

74/262
Limitations: The "paleo proxy" technique relies on the premise that the processes used to develop the proxies were similar to those used now.
However, proxies buried in ocean and lake sediments can only detect temperature anomalies over centuries or thousands of years, making daily
temperature estimates difficult.

Que. 130 Canalisation of a river involves:

1. Building dams upstream to regulate water flow.


2. Keeping floodplains clear of development.
3. Physically altering the river channel to control water flow.
4. Encouraging growth of vegetation along the river banks.

Correct Option - 3
The correct answer is option 3.
Key Points
Canalisation :
Canalisation is a river management technique that involves physically altering a river channel to control and regulate the flow of water.
This can be done in a number of ways, such as:
Straightening the river channel
Deepening the river channel
Widening the river channel
Lining the banks of the river with concrete or other materials

The goal of canalisation is to increase the capacity of the river to carry water, thereby reducing the risk of flooding.

Effectiveness of Canalisation for Flood Management

Canalisation can be an effective way to reduce flooding in some cases. However, it is important to be aware of the potential drawbacks of this
technique. These include:
Increased erosion: When a river is straightened, deepened, or widened, it can increase the speed of the water flow. This can lead to
increased erosion of the river banks and bed.
Reduced habitat diversity: Canalisation can destroy the natural habitat of fish and other aquatic life.
Increased flood risk downstream: While canalisation may reduce flooding in the immediate area, it can actually increase the risk of
flooding downstream. This is because the faster-flowing water can reach downstream areas more quickly, and with more force.

Que. 131 Consider the following statements


1: El Niño events are associated with strengthened monsoon winds and increased rainfall in South Asia.
2: El Niño disrupts atmospheric circulation patterns, leading to weaker trade winds in the Pacific and impacting monsoon activity.
3: La Niña, the cooling phase of the El Niño-Southern Oscillation (ENSO), typically brings drier monsoons to India.
How many of the statements given above is/are correct?

1. Only one
2. Only two
3. All three
4. None

Correct Option - 2
The correct answer is option 2.
Key Points
El Niño: This is a warming of the sea surface temperatures in the equatorial Pacific Ocean. It disrupts typical weather patterns worldwide.
Monsoons: These are seasonal winds that bring significant rainfall to regions like India and Southeast Asia.

El Niño weakens monsoon winds, leading to reduced rainfall in South Asia. Hence, Statement 1 is incorrect.
El Niño disrupts atmospheric circulation and weakens trade winds, impacting monsoon activity. Hence ,Statement 2 is correct.
. La Niña is associated with stronger monsoons and above-average rainfall. Hence ,Statement 3 is incorrect

Additional Information
El Niño and monsoons:
El Niño's Impact: El Niño events weaken monsoon winds, leading to reduced rainfall across regions like India. This can cause droughts
and impact agriculture.
The Mechanism: El Niño disrupts atmospheric circulation patterns. Weaker trade winds in the Pacific affect the movement of
moisture-laden air masses that contribute to monsoons.
Strength of Connection: The El Niño-monsoon link is strong, but it varies by region and over time.
Not Guaranteed: El Niño often leads to drier monsoons, but it's not a sure prediction. Other factors also influence monsoon rainfall.

75/262
La Niña, El Niño's counterpart with cooler Pacific waters, is generally associated with stronger monsoons and above-average rainfall. They
are both part of a larger climate pattern called the El Niño-Southern Oscillation (ENSO).

Que. 132 The First Global Symposium on Farmers' Rights was held in 2023 with a focus on:

1. Promoting organic farming practices worldwide


2. Discussing the rights of farmers under the International Treaty on Plant Genetic Resources for Food and Agriculture
3. Addressing the issue of farmer suicides in India
4. Negotiating international trade agreements for agricultural products

Correct Option - 2
The correct answer is option 2.
Key Points

The First Global Symposium on Farmers' Rights

The First Global Symposium on Farmers' Rights, held in New Delhi, India, from September 12th to 15th, 2023, was a significant event for
global agriculture and food security. Here's a deeper dive into the symposium:

Background:
The International Treaty on Plant Genetic Resources for Food and Agriculture (International Treaty) recognizes Farmers' Rights in
Article 9.
These rights encompass farmers' saved seeds, participation in benefit-sharing, and their role in conserving plant biodiversity.

Symposium Goals:
Promote Farmers' Rights: The primary objective was to discuss and advance the implementation of Farmers' Rights as outlined in the
International Treaty.
Share Experiences: Participants exchanged best practices and challenges faced in upholding Farmers' Rights across different
countries.
Identify Improvements: The event aimed to identify ways to strengthen the implementation of Farmers' Rights frameworks and improve
farmers' situations.
Future Directions: Discussions explored future strategies and policies to ensure continued recognition and protection of Farmers' Rights
in the evolving agricultural landscape.

Que. 133 The 2023 Climate Ambition Summit aimed to:

1. Secure new financial commitments from developed nations for developing countries
2. Showcase countries with the highest historical greenhouse gas emissions.
3. Highlight concrete actions and plans to tackle climate change.
4. Focus on theoretical discussions about the future of climate policy

Correct Option - 3
The correct answer is option 3
Key Points
The Climate Ambition Summit 2023 took place on September 20th, 2023, at the United Nations Headquarters in New York City. It aimed to
jumpstart significant climate action ahead of the COP28 conference.

Here are some key details about the summit:


Goal: Showcase concrete actions from governments, businesses, and other organizations to accelerate reducing greenhouse gas
emissions and address climate justice.
Focus: "First movers and doers" who presented real plans for a faster transition to a clean energy economy.
Outcomes:
Leaders acknowledged the urgency of the climate crisis and the need to move faster on solutions.
Importance of a just and equitable transition to a low-carbon future was emphasized.
Highlighted specific actions and partnerships to achieve these goals.

While some saw it as a "Summit of Hope," some major emitters like China, India, and the US were absent.

Que. 134 Which of the following statements is not an advantage of using weed harvesters for maintaining lakes and ponds?

1. They remove weeds that can interfere with recreational activities like swimming and boating.
2. They help control the spread of invasive aquatic plants.
3. They improve water quality by reducing nutrients for algae growth.
4. They till the bottom sediment, promoting healthier plant life.

76/262
Correct Option - 4
The correct answer is option 4.
Key Points
Weed harvester : A weed harvester, also sometimes called an aquatic weed harvester, is a mechanized device used to remove unwanted
weeds and other vegetation from bodies of water. These harvesters are typically large, self-propelled machines that resemble boats.

Statements (1), (2), and (3) are all advantages of weed harvesters. They effectively remove weeds, control invasive species, and
improve water quality. However, weed harvesters typically don't till the bottom sediment. In fact, tilling the bottom can be disruptive to
the ecosystem. Hence, option (4) is the incorrect statement.

Types of Weed Harvesters

There are two main types of weed harvesters:


Mechanical Cutters: These are the most common type of weed harvester. They use rotating blades to cut the weeds, which are then
collected by a conveyor system.
Uprooters: These harvesters use a mechanism to pull weeds up from their roots. This can be a more effective way to control weeds, as it
prevents them from growing back quickly. However, uprooters are not as common as mechanical cutters because they can be more
difficult to operate and may damage the bottom of the waterway.

Applications of Weed Harvesters

Weed harvesters are used in a variety of applications, including:


Maintaining lakes and ponds: Weed harvesters are used to remove weeds that can interfere with swimming, boating, and fishing.
Controlling invasive species: Weed harvesters can be used to control the spread of invasive aquatic plants.
Improving water quality: By removing weeds, weed harvesters can help to improve water quality by reducing the amount of nutrients
available for algae growth.
Restoring wetlands: Weed harvesters can be used to remove unwanted vegetation from wetlands in order to restore their ecological
balance.

Que. 135
Consider the followings statements

1. Tapioca plants, also known as cassava, are a widely cultivated crop in India, particularly in Tamil Nadu and Kerala.

2. Consumption of tapioca products by humans carries no significant side effects.

Choose the best answer

1. Both Statement 1 and Statement 2 are individually true and Statement 2 is the CORRECT explanation of Statement 1.
2. Both Statement 1 and Statement 2 are individually true but Statement 2 is NOT the CORRECT explanation of Statement 1.
3. Statement 1 is true but Statement 2 is false.
4. Statement 1 is false but Statement 2 is true.

Correct Option - 3
The correct option is option 3.
In News

Recently, the Central Tuber Crops Research Institute issued an advisory on using parts of tapioca plants to feed cattle.The ICAR-
Central Tuber Crops Research Institute (CTCRI) here has issued an advisory on feeding animals with parts of cassava (tapioca) in
view of the incident in Idukki where 13 cows died in a farm recently.

Key Points
Tapioca :

Tapioca plants belong to the family Euphorbiaceae and thrive best in tropical, warm humid climates with temperatures conducive to
growth.
Cultivation of tapioca requires well-drained soil, preferably red lateritic loam, with a pH range of 5.5 - 7.0. It can tolerate altitudes of
up to 1000 meters and requires an annual rainfall of over 100 cm for optimal growth.
It is cultivated throughout the tropical world for its tuberous roots, from which cassava flour, bread, tapioca, laundry starch, and an
alcoholic beverage are derived. Cassava, also known as Tapioca, production in India is estimated to be 4.98 million tonnes.

Tamil Nadu, followed by Kerala accounts for a major amount of production. Its cultivation is also extending towards non-traditional areas
like Maharashtra.

Uses:- Cassava has a high starch content and ability to grow under low management conditions due to which it has been globally recognized as a
potential candidate for bioethanol production. (National Bioenergy Programme)

77/262
Side Effects:- There are no significant side effects associated with tapioca consumption. However, any cassava-based product can pose the risk
of cyanide poisoning.

Que. 136 Which of the following statements is/are most accurate about PB knot technology?

1. It utilizes synthetic hormones to kill pink bollworms directly


2. It disrupts the mating cycle of pink bollworms by releasing pheromones.
3. It is a type of genetically modified cotton plant resistant to pink bollworms.
4. It is a highly toxic pesticide that requires extensive safety precautions.

Correct Option - 2
The correct answer is option 2.
Key Points
PB knot technology: PB knot technology, also referred to as PB Rope L, is a pest control method specifically designed to target the pink
bollworm (PBW) in cotton crops. Here's a breakdown of how it works:
Mating Disruption: PB knot disrupts the mating cycle of the pink bollworm. By releasing synthetic sex pheromones, it confuses male
moths and makes it difficult for them to locate female moths for mating.
Reduced Pest Population: With mating disrupted, the pink bollworm population growth is significantly reduced. This translates to less
crop damage for farmers.
Potential Eradication: Studies have shown that PB knot technology can be highly effective. In the United States, for instance,
implementing this technology in 2008 led to the complete eradication of the pink bollworm.
Pink bollworm:
The pink bollworm (Pectinophora gossypiella) is a destructive insect pest that targets cotton crops.
The culprit for damage is the larval stage. This pinkish caterpillar chews through cotton bolls, harming both the valuable fibers
(lint) and the seeds.
Originally from Asia, the pink bollworm has become invasive in many cotton-growing regions worldwide.

Que. 137 Consider the following statements

1: Enteric fermentation in cows is the single largest source of methane emissions globally.

2: Capturing methane from landfills and converting it to electricity can be a viable strategy for mitigating climate change.

3: Leak detection and repair programs in the oil and gas industry have limited effectiveness in reducing methane emissions.

How many of the statements give above is/are correct?

1. Only one
2. Only two
3. All three
4. None

Correct Option - 2
The correct answer is option 2.
Key Points
Methane mitigation: Methane mitigation refers to the strategies that can be taken to reduce the amount of methane released into the
atmosphere. Methane is a potent greenhouse gas, and reducing its emissions is crucial for slowing climate change.

While enteric fermentation is a major source of methane, it is not the single largest. Other sources like agriculture waste and the oil and
gas industry also contribute significantly. Hence, Statement 1 is correct.
Capturing landfill gas for energy generation is a recognized strategy for methane mitigation. Hence, Statement 2 is correct.
Leak detection and repair programs are effective in reducing methane emissions from pipelines and infrastructure. Hence, Statement
3 is incorrect.

There are a number of ways to mitigate methane emissions, from capturing it at its source to improving the efficiency of how we use natural
gas. Here are some of the key strategies:
Oil and Gas Industry: Methane is released from all stages of oil and gas production, from drilling and extraction to processing and
transportation. Leak detection and repair programs, along with upgrading equipment and pipelines, can significantly reduce these
emissions.
Agriculture: Enteric fermentation in the digestive systems of ruminant animals like cows, sheep, and goats is a major source of
methane emissions. Dietary changes, such as adding seaweed or probiotics to feed, can help to reduce these emissions.
Waste Management: Landfills are another major source of methane emissions. Capturing landfill gas and using it to generate
electricity or heat homes and businesses can help to reduce these emissions.
New technologies are also being developed to help mitigate methane emissions. These include satellites and drones that can be used to
identify methane leaks, and new methods for capturing and utilizing methane from landfills and agricultural waste

78/262
Que. 138 The World Meteorological Organization's 2022 State of Global Water Resources report indicates that:

1. Climate change has no significant impact on the hydrological cycle.


2. Droughts and floods are becoming less frequent due to improved water management practices.
3. Melting glaciers pose a threat to long-term water security in some regions.
4. There is an abundance of data available for effective global water resource management.

Correct Option - 3
The correct answer is option 3.
Key Points
State of Global Water Resources report :The 2022 State of Global Water Resources report by the World Meteorological Organization
(WMO) offers a grim assessment of our planet's freshwater. Here's a detailed breakdown:

The Big Picture:


Hydrological Imbalance: Climate change and human activities are throwing the natural water cycle (evaporation, precipitation,
runoff) out of whack.
Water Extremes: Droughts and intense rainfall events are becoming more frequent, causing havoc on lives and economies.
Melting Glaciers: The accelerating melt of glaciers, snowpack, and ice sheets threatens long-term water security for millions. These
sources feed rivers and provide freshwater.
Data Deficiencies: Significant gaps exist in our knowledge about the true state of global freshwater resources, hindering effective
management.

Report Highlights:
River Discharge: Over half of the world's catchment areas (areas that drain into a common body of water) experienced abnormal river
discharge in 2022. Most faced drier than usual conditions, mirroring trends from 2021.
Soil Moisture and Evapotranspiration: Similar to river discharge patterns, soil moisture and evapotranspiration (transfer of water to
the atmosphere) showed anomalies throughout 2022. For instance, Europe saw increased evapotranspiration and decreased soil moisture
during the summer drought.
High-Impact Events: The report provides a global overview of major floods and droughts that occurred in 2022.
Improved Data Collection: There's positive progress in data collection, with a notable increase in observed river discharge data from
member countries

Que. 139 Consider the following statements about the Interconnected Disaster Risks Report 2023:
1.It identifies six tipping points that could have cascading effects on global security.
2. It emphasizes the need for technological advancements to address these tipping points.
3. It categorizes solutions based on a framework focusing on avoidance and adaptation strategies.
How many of the statements given above is/are correct?

1. Only one
2. Only two
3. All three
4. None

Correct Option - 2
The correct answer is option 2.
Key Points
Interconnected disaster risks reports : The Interconnected Disaster Risks Report 2023, published by the United Nations University
Institute for Environment and Human Security (UNU-EHS), examines six critical tipping points that pose escalating risks around the
world.

The report identifies six critical tipping points, emphasizing their interconnected nature. Hence, Statement 1 is correct.
The report focuses on existing technologies and system changes rather than solely relying on advancements. Hence, Statement 2 is
incorrect.
The report outlines a framework with categories like Avoid-Delay, Adapt-Transform etc. Hence, Statement 3 is correct.

Interconnected issues are:


Accelerating extinction: This refers to the rapid loss of species, potentially leading to cascading ecosystem collapse.
Groundwater depletion: Overuse of groundwater resources threatens drinking water supplies and agricultural production.
Mountain glacier melting: Melting glaciers reduce freshwater availability and increase risks of flooding.
Space debris: Growing amounts of debris in orbit threaten satellites critical for communication and navigation.
Unbearable heat: Extreme heat events are becoming more frequent and severe, making some regions uninhabitable.
Uninsurable future: Rising disaster risks may make it impossible or unaffordable to obtain insurance.

The report also proposes a new framework for classifying and prioritizing solutions to address these tipping points. This framework
categorizes solutions into four types:
Avoid-Delay: These are measures that aim to slow the progression towards disaster using existing technologies.
79/262
Avoid-Transform: These involve significant changes in practices and systems to prevent disasters altogether.
Adapt-Delay: These actions focus on preparing for disasters to buy time for response and recovery.
Adapt-Transform: These involve large-scale transformations in how we live to adapt to a changing environment.

Que. 140 Which of the following statements is/are correct regarding MD-15?

1. It is a mixture of 100% methanol and special additives.


2. It is a blend of methanol, mineral diesel, and indigenously developed additives.
3. It is designed to increase diesel consumption and emissions.
4. It is solely developed by the Indian Oil Corporation Ltd (IOCL).

Correct Option - 2
The correct answer is option 2.

Key Points
Methanol Diesel-15 (MD-15) :
Methanol Diesel-15 (MD-15) is a special fuel blend that is being developed in India as a way to reduce emissions and dependence on
imported oil. It is a mixture of:
15% methanol
71% mineral diesel
14% indigenously developed additives

The additives are important because methanol can be slightly corrosive to engine parts, and the additives help to mitigate this effect.

MD-15 is being developed by the Indian Railways' Research Design and Standards Organisation (RDSO) in collaboration with the
Indian Oil Corporation Ltd (IOCL). There are a number of potential benefits to using MD-15, including:
Reduced diesel consumption by up to 15%
Lower emissions of harmful pollutants, such as NOx
Increased energy security by reducing reliance on imported oil

MD-15 is still under development, but it has the potential to be a significant step forward in terms of clean and sustainable transportation.

Que. 141 White hydrogen is a recently discovered form of hydrogen with potential as a clean energy source.
Which of the following statements is/are correct regarding white hydrogen?

1. It is produced from water using electrolysis.


2. It is a byproduct of natural gas processing.
3. It occurs naturally underground and produces no harmful emissions during combustion.
4. Its extraction methods are well established and environmentally friendly.

Correct Option - 3
The correct answer is option 3.
Key Points
White hydrogen: White hydrogen, also known as natural hydrogen, geologic hydrogen, or gold hydrogen, is hydrogen gas that occurs
naturally underground. It is distinct from other forms of hydrogen production:
Green hydrogen: Produced from water using renewable energy sources like solar or wind power through electrolysis.
Grey hydrogen: Produced from natural gas through steam reforming, releasing greenhouse gases.
Blue hydrogen: Similar to grey hydrogen, but with technology to capture the released carbon dioxide (CO2).

The discovery of naturally occurring white hydrogen is relatively recent, with significant deposits found in France in 2023. While research is
ongoing, here are some of the potential benefits of white hydrogen:
Clean energy source: When used in fuel cells, white hydrogen produces only water vapor as a byproduct, making it a clean burning fuel.
Reduced reliance on fossil fuels: If readily available, white hydrogen could lessen dependence on traditional fossil fuels for energy
production.
Potentially lower cost: Extracting white hydrogen from natural deposits may be more economical than other production methods.

However, there are also some challenges to consider:


Limited research: White hydrogen is a new area of exploration, and more research is needed to understand its full potential and
environmental impact.
Extraction challenges: Extracting hydrogen from underground deposits may require techniques similar to fracking, which has its own
environmental concerns.
Leakage risks: Hydrogen is a light gas that can easily leak from storage facilities. Leaks could counteract the environmental benefits if
the gas reaches the upper atmosphere.

Que. 142 Which of the following statements is not true about Pantoea Tagorei?

80/262
1. It is a newly discovered bacteria named after Rabindranath Tagore.
2. It promotes plant growth by improving the availability of potassium and phosphorus.
3. It can fix nitrogen from the atmosphere, a crucial plant nutrient.
4. It is a type of pesticide that can be used to control agricultural pests.

Correct Option - 4
The correct answer is option 4.

Que. 143 Consider the following statements


1.Coal gasification can be a cleaner alternative to traditional coal-fired power plants by capturing carbon dioxide emissions during
the process.
2. Coal gasification can efficiently utilize low-grade coal that is not suitable for direct combustion in power plants.
3. Syngas, the product of coal gasification, has no use in electricity generation.
How many of the statements given above is/are correct?

1. Only one
2. Only two
3. All three
4. None

Correct Option - 2
The correct answer is option 2.
Key Points
Coal gasification : Coal gasification is a process that transforms coal into a synthetic gas called syngas. This syngas is a mixture of gases,
primarily carbon monoxide, hydrogen, carbon dioxide, methane, and water vapor.

Coal gasification can capture carbon dioxide emissions during the process, making it a cleaner alternative to traditional coal-fired
power plants. Hence ,Statement 1 is correct.
Coal gasification can efficiently utilize low-grade coal that is not suitable for direct combustion in power plants. Hence .Statement 2 is
correct.
Syngas, the product of coal gasification, can be burned in a turbine to generate electricity. Hence ,Statement 3 is incorrect.

Syngas can be used for a variety of purposes, including:


Electricity generation: Syngas can be burned in a turbine to generate electricity.
Production of fuels: Syngas can be converted into cleaner-burning fuels, such as methanol or synthetic natural gas (LNG).
Production of chemicals: Syngas can be used as a feedstock for the production of a variety of chemicals, such as hydrogen, ammonia,
and urea.

Coal gasification has several potential advantages over traditional coal-fired power plants. These advantages include:
Reduced emissions: Coal gasification plants can be designed to capture carbon dioxide emissions, which can help to mitigate climate
change.
Increased efficiency: Coal gasification plants can be more efficient than traditional coal-fired power plants, which means that they can
produce more electricity from the same amount of coal.
Use of low-quality coal: Coal gasification plants can use low-quality coal that cannot be used in traditional coal-fired power plants.

However, coal gasification also has some disadvantages. These disadvantages include:
High cost: Coal gasification plants are expensive to build and operate.
Water use: Coal gasification plants can use a large amount of water.
Air pollution: Coal gasification plants can still produce some air pollutants, such as nitrogen oxides and sulfur oxides.

Overall, coal gasification is a complex technology with both advantages and disadvantages. It is an important option to consider as we look
for ways to clean up our coal-fired power plants and produce cleaner-burning fuels.

Que. 144 Rejupave technology is helpful in constructing roads in:

1. Coastal areas prone to frequent cyclones


2. High-altitude regions with freezing temperatures
3. Densely populated urban centers with heavy traffic
4. Earthquake-prone zones requiring special reinforcement

Correct Option - 2
The correct answer is option 2.
Key Points
Rejupave : Rejupave is an innovative road construction technology developed by India's CSIR-Central Road Research Institute
(CSIR-CRRI) specifically for building asphalt roads in harsh, low and sub-zero temperature conditions often encountered in high-altitude
areas.
Features and benefits:

81/262
Lower production and rolling temperatures: Rejupave reduces the required temperature for asphalt mix production and
compaction by 30-40 degrees Celsius. This significantly benefits construction in cold climates where maintaining high temperatures for
traditional asphalt is challenging.
Extended workability: A special hydrocarbon-based additive in Rejupave enhances the heat retention properties of the asphalt mix. This
allows the mix to stay workable for longer durations during transportation from the manufacturing plant to the construction site, even in
areas with snowfall.
Unique construction approach: Rejupave allows for a unique "pre-fab" paving method. Here, a base layer is laid and cured completely
before overlaying it with the top hot asphalt layer. This two-stage approach is particularly advantageous in cold weather conditions.
Eco-friendly: Rejupave's asphalt modifier is bio-oil based, making it an eco-friendly alternative to traditional methods. This is crucial for
preserving the fragile ecosystems of high-altitude regions.

Successful Application:

The Border Roads Organisation (BRO) has successfully deployed Rejupave technology in constructing critical high-altitude road sections in
Arunachal Pradesh, near the India-China border. This includes sections of the world's highest Sela Tunnel and the LGG-Damteng-
Yangste (LDY) road.

Que. 145 Consider the following statements regarding Seaweed Cultivation in India:
1. India has a rich diversity of seaweed species but currently faces a shortage of raw materials for seaweed-based industries.
2. Seaweed cultivation offers potential for income generation in coastal communities and can be used as a bio-indicator for marine pollution.
3. Agar, a thickening agent used in food products, is primarily extracted from brown seaweed.
How many of the statements given above is/are correct?

1. Only one
2. Only two
3. All three
4. None

Correct Option - 2
The correct answer is option 2.
In News
Recently, the National Conference on Promotion of Seaweed Cultivation was held in Koteshwar (Kori Creek), Kutch, Gujarat.It
aimed at implementing seaweed cultivation on a pan-India basis, emphasizing the promotion of seaweed cultivation to diversify
marine production and enhance fish farmer income.

Key Points
Sea weed cultivation: Seaweeds are macroscopic, multicellular, marine algae. They come in a variety of coluors, including red, green,
and brown.They are referred to as the 'Medical Food of the 21st Century.

India has a high number of seaweed species but current production is low, leading to a shortage for industries that rely on seaweed
extracts like agar and alginate. Hence , Statement 1 is correct
Seaweed cultivation requires minimal investment and can be a source of income for coastal communities. Additionally, some seaweed
species act as bio-indicators for pollution. Hence ,Statement 2 is correct.
Agar is extracted from red seaweed, not brown seaweed. Brown seaweed is a source of alginate, another thickening agent. Hence
,Statement 3 is incorrect.

Distribution: Seaweeds are found mostly in the intertidal region, in shallow and deep waters of the sea and also in estuaries and
backwaters.

Large seaweeds form dense underwater forests known as kelp forests, which act as underwater nurseries for fish, snails and sea
urchins.

Seaweed Species in India: India boasts approximately 844 reported seaweed species in its seas.

Some specific species, such as Gelidiella acerosa, Gracilaria spp., Sargassum spp., Turbinaria spp., and Cystoseira
trinodis are cultivated for the production of agar, alginates, and liquid seaweed fertilizer.

Major Seaweed Beds in India: Abundant seaweed resources are found along the Tamil Nadu and Gujarat coasts, as well as
around Lakshadweep and the Andaman & Nicobar Islands.

Notable seaweed beds exist around Mumbai, Ratnagiri, Goa, Karwar, Varkala, Vizhinjam, and Pulicat in Tamil Nadu, Andhra
Pradesh, and Chilka in Orissa.

82/262
Significance:

Bio-indicator: They act as bio-indicator by absorbing excess nutrients and signalling marine chemical damage caused by waste
from agriculture, industries, and households, often leading to algal blooming.

They play a vital role in restoring ecosystem balance.

Food Source: Seaweed is a nutritional powerhouse, rich in vitamins, minerals, and dietary fibre.

It Is used in various food products, from sushi and salads to snacks and thickeners.

Many seaweeds contain anti-inflammatory and anti-microbial agents. Seaweed is the best source of iodine.

Sea weed park : The foundation stone for the establishment of the first Multi-Purpose Seaweed Park was laid in
September 2023 in Ramanathapuram district, Tamil Nadu .This initiative aims to promote seaweed cultivation in 136
coastal fishing villages across six coastal districts of Tamil Nadu.

Que. 146 Consider the following statements

1. Atmospheric rivers are long, narrow corridors in the atmosphere that transport significant amounts of water vapor from the tropics towards
higher latitudes.

2. They are associated with dry and arid conditions upon reaching land.

Which of the statements given above is/are correct?

1. Statement 1 only
2. Statement 2 only
3. Both Statement 1 and 2
4. Neither Statement 1 nor 2

Correct Option - 1
The correct answer is option 1.
Key Points
Atmospheric rivers: Atmospheric rivers are long, narrow bands in the atmosphere that transport massive amounts of moisture, like rivers
in the sky. They are responsible for transporting about 90% of the moisture from the tropics towards the poles.
These narrow corridors of concentrated moisture can carry an immense amount of water vapor – sometimes as much as the Amazon River
discharges in a day.

When atmospheric rivers make landfall, they can unleash tremendous amounts of rain and snow. They play a crucial role in delivering water
to many parts of the world, but they can also cause flooding and landslides.

Here are some interesting facts about atmospheric rivers:


They can be thousands of miles long but are only a few hundred miles wide.
They are most common in the winter months.
The strongest atmospheric rivers can cause extreme precipitation events, leading to flooding and landslides.
Climate change is expected to increase the intensity of atmospheric rivers.

Que. 147 Which of the following statements is not true about brumation in reptiles?

1. It is a period of reduced activity similar to hibernation in mammals.


2. Body temperature remains close to ambient temperature during brumation.
3. Brumation is triggered primarily by changes in day length
4. Brumation is a survival strategy to conserve energy during cold weather.

Correct Option - 3
The correct answer is option 3.
Key Points
Brumation :Brumation is a period of inactivity that some reptiles undergo during cold weather, similar to hibernation in mammals.
However, there are some key differences between the two:

Hibernating mammals enter a state of deep sleep, their body temperature drops significantly, and their heart rate and
breathing slow down. Brumating reptiles, on the other hand, experience a period of reduced activity, but their body temperature
remains closer to the ambient temperature.

83/262
They may also become less responsive to stimuli and eat less frequently, but they do not enter a state of deep sleep.

Brumation is a survival strategy that allows reptiles to conserve energy during times when food is scarce and temperatures are low.
By slowing down their metabolism, reptiles can survive on their stored fat reserves until spring arrives and they can become active
again.

Brumation is not triggered by the same factors as hibernation. Hibernation is primarily triggered by changes in day length, while
brumation is triggered by changes in temperature. Some reptiles may also brumate in response to changes in food availability.

Not all reptiles brumate. Some species, such as those that live in tropical climates, do not experience cold temperatures and
therefore do not need to brumate. Other species may brumate only if they are exposed to cold temperatures for an extended period.

Que. 148 Consider the following statements about ‘Greening and Restoration of Wasteland with Agroforestry (GROW)’ initiative:
1. It promotes planting trees on degraded land to improve soil quality and prevent erosion.
2. It aims to generate additional income for farmers by incorporating trees into their agricultural practices.
3. It is a sustainable land management system launched by the World Bank.
How many of the statements given above is/are correct?

1. Only one
2. Only two
3. All three
4. None

Correct Option - 2
The correct answer is option 2.
Key Points
GROW : India's NITI Aayog launched the Greening and Restoration of Wasteland with Agroforestry (GROW) initiative in 2020. This
initiative aims to restore 26 million hectares of degraded land in India by 2030 using agroforestry practices. The GROW initiative is a
positive step towards addressing land degradation in India and improving the livelihoods of farmers.

Agroforestry involves planting trees on degraded land to improve soil health and prevent erosion. Hence, Statement 1 is correct.
Farmers can earn income from fruits, nuts, timber, etc., produced by the trees integrated into their farms. Hence, Statement 2 is
correct.
The GROW initiative is launched by NITI Aayog, a government of India think tank. Hence, Statement 3 is incorrect.

Agroforestry is a land-use management system that integrates trees or shrubs on farms and rangelands. It is a promising approach to
reversing land degradation and restoring wastelands. Here's how it works:
Planting trees and shrubs on degraded land: Trees and shrubs help to improve soil fertility, prevent soil erosion, and increase water
infiltration. Their roots help to break up compacted soil and create channels for water to move down into the ground. The leaves and other
organic matter that fall from trees and shrubs also help to improve soil quality.
Providing habitat for wildlife: Trees and shrubs can provide food and shelter for a variety of wildlife species. This can help to
improve biodiversity and ecosystem health.
Increasing crop yields: Trees and shrubs can provide shade and shelter for crops, which can help to improve crop yields. They can also
help to suppress weeds and pests.
Generating income for farmers: Trees and shrubs can be a source of income for farmers. Farmers can sell the fruits, nuts, timber, or
other products that they produce.

Agroforestry is a sustainable land-use management system that can help to address a variety of environmental challenges. It is a promising
approach to greening and restoring wastelands, and it can provide a number of benefits for farmers and the environment.

Que. 149 The Terai Arc Landscape (TAL) is a transboundary conservation area known for its rich biodiversity. Consider the following
statements about TAL:
1. It encompasses protected areas in the foothills of the Himalayas, spanning India and Bhutan.
2. It includes Corbett National Park and provides critical habitat for the endangered Bengal tiger.
3. The landscape primarily consists of high altitude mountainous terrain.
How many of the statements given above is/are correct?

1. Only one
2. Only two
3. All three
4. None

Correct Option - 2
The correct answer is option 2.

84/262
Key Points
Terai Arc landscape: The Terai Arc Landscape (TAL) is a vast stretch of land encompassing around 5 million hectares
across the foothills of the Himalayas in India and Nepal. It's a crucial wildlife corridor and conservation area known for its
rich biodiversity.

TAL covers the foothills of the Himalayas in India and Nepal, not Bhutan. Hence , Statement 1 is incorrect.
Corbett National Park is a famous protected area within TAL, and the landscape is crucial for Bengal tiger conservation. Hence
,Statement 2 is correct.
The Terai Arc Landscape primarily consists of foothills, floodplains, and Shivalik hills, not high-altitude mountains. Hence, Statement 3
is incorrect.

Features of the Terai Arc Landscape


Geographic Scope: It stretches for approximately 810 kilometers between the Yamuna River in the west and the Bagmati River in
the east. It covers the states of Uttarakhand, Uttar Pradesh, and Bihar in India and the southern plains of Nepal.
Protected Areas: The landscape is an amalgamation of 14 protected areas, including famous national parks and wildlife sanctuaries like
Corbett National Park, Dudhwa Tiger Reserve, Valmiki National Park in India, and Chitwan National Park, Bardia National Park,
and Shukla Phanta Wildlife Sanctuary in Nepal.
Biodiversity: The Terai Arc Landscape is a haven for diverse wildlife species. It's one of the most critical habitats for the endangered
Bengal tiger, with an estimated 22% of India's tiger population residing here. Other significant mammals found in the region include the
one-horned rhinoceros, Asian elephant, gangetic dolphin, several species of deer and antelopes, and a rich variety of birdlife.

The Terai Arc Landscape faces various challenges, including habitat loss due to human encroachment, poaching, and human-wildlife conflict.
However, several conservation initiatives are underway to address these threats and protect this vital ecological region.

Que. 150 Broom grass (Thysanolaena maxima) is an important crop cultivated in India with a variety of uses.
Which of the following statements is/are correct regarding Broom grass in India?

1. It is primarily cultivated in the arid regions of Rajasthan and Gujarat.


2. The stalks are used for making brooms and the leaves have no commercial significance.
3. It is a low-maintenance crop suitable for marginal lands and fallow fields.
4. It is a major source of biofuel in India.

Correct Option - 3
The correct answer is option 3.
Key Points
Broom grass: Broom grass is a perennial, high-value, non-perishable, nontimber forest product (NTFP) that grows abundantly in
degraded, steep or marginal land. Broom grass, also known scientifically as Thysanolaena maxima, is an important crop grown in India.
Cultivation:
It is particularly prevalent in the hilly areas of northeastern India, with Assam being the leading producer.
The crop thrives in a variety of soil conditions, from sandy loam to clay loam, and can even be grown on marginal lands and fallow
fields.
Cultivation requires minimal investment and is a source of income for many rural communities.
85/262
Uses:
The key purpose of broom grass is, of course, the production of brooms. The long, sturdy stalks are ideal for sweeping and widely
used across India.
Broom grass is a versatile plant with additional applications. The leaves serve as fodder for livestock, and the stalks can be used as a
biofuel. The plant also helps prevent soil erosion.

Que. 151 Thrips parvispinus, an invasive thrips species, is a growing threat to Indian agriculture.
Which of the following statements are correct regarding Thrips parvispinus in India?

1. It is a native pest that primarily affects mango and tamarind crops.


2. It was first detected in India in 2015 and has spread rapidly across various states.
3. It primarily targets wheat and pulses during the Kharif season (summer cropping season in India).
4. It is easily controlled using conventional insecticides.

Correct Option - 2
The correct answer is option 2.
In News
Thrips Parvispinus has been making headlines as an invasive pest affecting chilli crops in Andhra Pradesh and Telangana. This tiny
insect, belonging to the Thripidae family, has been causing significant damage to chilli plants, leading to economic losses for farmers.
Key Points
Thrips parvispinus : Thrips parvispinus is a tiny insect belonging to the thrips group. They are considered a major agricultural pest due to
the damage they cause to various crops and ornamentals.

Status in India:

Invasion and Spread:


First reported in India on papaya in 2015 around Bengaluru .
Since then, it has spread rapidly across various states, including Andhra Pradesh, Assam, Chhattisgarh, Gujarat, Karnataka, Kerala,
Maharashtra, Odisha, and Tamil Nadu .
This thrips thrives in diverse climates and has been found attacking a wide range of crops, becoming a significant threat.

Impact on Crops:
T. parvispinus primarily targets chilies, causing severe damage, especially during the Rabi season (winter cropping season in India) .
Other susceptible crops include papaya, cotton, dahlia, mango, tamarind, and several vegetables .
Feeding by thrips damages leaves, flowers, and fruits, leading to:
Heavy flower drop
Deformed and stunted fruit growth
Reduced fruit yield
Significant economic losses for farmers .

Management Strategies:
The Indian government has issued advisories for thrips management in chili cultivation . These strategies include:
Monitoring crops regularly for thrips presence.
Using insecticides specifically effective against thrips species.
Implementing cultural practices like weed removal and mulching to create a less favorable environment for thrips.

Que. 152 Consider the following statements about the Goods and Services Tax Appellate Tribunal (GSTAT):
1. It was established soon after the implementation of the Goods and Services Tax (GST) in 2017.
2. The Supreme Court recently expressed concern about the delay in setting up the GSTAT.
3. The GSTAT will function with a central bench in Delhi and regional benches in every state.
How many of the statements given above is/are correct?

1. Only one
2. Only two
3. All three
4. None

Correct Option - 2
The correct answer option 2.
Key Points
GST Appellate Tribunal: The GST Appellate Tribunal is a quasi-judicial body proposed to be established to resolve disputes related to
the Goods and Services Tax (GST) in India. It will function as an independent body to hear appeals against orders passed by the GST
authorities or the Appellate Authority.

86/262
The GSTAT was not established soon after the implementation of GST and faced delays. Hence , Statement 1 is incorrect.
The Supreme Court highlighted the need for a functioning GSTAT to address GST disputes. Hence, Statement 2 is correct.
The GSTAT will have a central bench and state benches. Hence, Statement 3 is correct.

The GST Appellate Tribunal (GSTAT) has been in the news recently due to its delayed establishment and ongoing appointment process.
Here's a breakdown:

Delay and Challenges:


The Central Goods and Services Tax (CGST) Act was introduced four years ago, but the GSTAT, a crucial body for resolving disputes
related to GST, wasn't set up.
The Supreme Court pulled up the Central Government for this delay in August 2021, highlighting the need for a functioning tribunal.
Various challenges arose, including a writ petition filed by the Sales Tax Bar Association and a stay order by the Allahabad High Court.

Recent Developments:
A positive step was taken in June 2022 when the GST Council formed a Group of Ministers (GoM) to address the GSTAT's
establishment.
The government finally announced the constitution of state benches for the tribunal in September 2023.
As of December 2023, Finance Minister Nirmala Sitharaman informed the Lok Sabha about anomalies flagged by Chief Justice of India
D.Y. Chandrachud in GSTAT provisions.

Current Status:
The appointment process for GSTAT members has begun. The tribunal will have a principal bench in Delhi and benches in each state.
As of March 26, 2024, expectations are that the GSTAT will be functional by July or August 2024.

Importance of GSTAT:
The tribunal's establishment is significant for businesses facing GST-related disputes.
It will provide a platform for faster and more efficient dispute resolution, reducing reliance on over-burdened courts.

Overall, the GSTAT's delayed formation has caused inconvenience for businesses. However, the recent progress in the appointment process
signifies a move towards a more streamlined GST dispute resolution mechanism.

Que. 153 The Bar Council of India is responsible for:

1. Setting eligibility criteria for judges in High Courts and Supreme Court.
2. Conducting entrance exams for admission to law schools in India
3. Regulating and setting standards for legal practice and education.
4. Representing the interests of all lawyers in India in court cases.

Correct Option - 3
The correct answer is option 3.
Key Points
Bar Council of India (BCI):

It is a statutory body established under the Advocates Act 1961 to regulate and represent the Indian bar.

Functions:

It performs the regulatory function by prescribing standards of professional conduct and etiquette and by exercising disciplinary
jurisdiction over the bar.
It also sets standards for legal education and grants recognition to Universities whose degrees in law will serve as qualification
for enrolment as an advocate.
It conducts the All India Bar Examination (AIBE) to grant a 'Certificate of Practice' to advocates practising law in India.
Legal Aid: The Bar Council of India plays a role in organizing legal aid to ensure access to justice for the poor and marginalized.
International Recognition: The BCI has the authority to grant reciprocal recognition to foreign law qualifications, allowing
lawyers with qualifications from other countries to potentially practice in India under certain conditions.
Financial Management: They are responsible for managing and investing the funds of the Bar Council, ensuring proper financial
health and resource allocation for its activities.
Elections and Governance: The BCI conducts elections for its members who form the governing body, ensuring democratic
representation within the organization.
Supervision and Oversight: The BCI exercises general supervision over State Bar Councils, providing guidance and ensuring
uniformity in the regulation of the legal profession across different states in India.

Que. 154 The Government of India proposes to allow entities other than government departments to perform Aadhaar authentication. What
is the primary purpose behind this proposal?

87/262
1. To improve tax collection efficiency
2. To enhance national security
3. To restrict access to social welfare benefits
4. To facilitate ease of living and better access to services

Correct Option - 4
The correct answer is option 4.
In News
As part of its commitment to make Aadhaar people-friendly and enable ease of living and better access to services for citizens, Ministry
of Electronics and Information Technology (MeitY) has proposed rules to enable Aadhaar authentication by entities other than
Government Ministries and Departments.
Key Points
Through an amendment enacted in 2019 to the Aadhaar (Targeted Delivery of Financial and Other Subsidies, Benefits and
Services) Act, 2016; entities were allowed to perform authentication if the Unique Identification Authority of India (UIDAI) is
satisfied about their compliance with regard to standards of privacy and security specified by regulations and is either permitted by
law to offer authentication services or seeks authentication for a prescribed purpose.
At present, Government Ministries and Departments are allowed to undertake Aadhaar Authentication under the Aadhaar
Authentication for Good Governance (Social Welfare, Innovation, Knowledge) Rules, 2020 in the interest of good governance,
preventing leakage of public funds and enablement of innovation and the spread of knowledge.
Now, it is proposed that any entity other than a Government Ministry or Department that desires to use Aadhaar authentication for the
purpose of promoting ease of living and enabling better access to services, or usage of digital platforms to ensure good governance, or
preventing dissipation of social welfare benefits, or enabling innovation and spread of knowledge, it shall prepare a proposal giving
justification as to how the authentication sought is for one of the said purposes and in the interest of State and submit the same to the
concerned Ministry or Department of the Central Government in respect of Central subjects and of the State Government in respect of
State subjects.
If the Ministry/Department is of the opinion that the proposal submitted fulfils such a purpose and is in the interest of the State, it will
forward the proposal along with its recommendation to MeitY.

Que. 155 Consider the following statements about the Ethics Committee of the Parliament in India:
1. The Ethics Committee can investigate allegations of unethical conduct against any citizen.
2. The Ethics Committee is composed of members from both the ruling party and the opposition.
Which of the statements given above is/are correct?

1. 1 only
2. 2 only
3. All three
4. None

Correct Option - 2
The correct answer is option 2.
Key Points
Ethics Committee : The Ethics Committee of the Parliament is an important body that plays a crucial role in upholding the integrity and
ethical conduct of Members of Parliament (MPs) in India.
key points about the Ethics Committee:
Function: The committee's primary function is to oversee the moral and ethical conduct of MPs and examine cases of alleged
misconduct referred to it by the Speaker or Chairman of the respective house (Lok Sabha or Rajya Sabha).
Composition: The Ethics Committee is a group of MPs appointed by the Speaker or Chairman for a specific period. The committee is
usually composed of a mix of members from the ruling party and the opposition.
Powers: The committee has the authority to investigate allegations of unethical behavior against MPs, which can include things like
corruption, abuse of power, or conflicts of interest. The committee can also recommend appropriate action to be taken against the erring
MP, which could range from a reprimand to expulsion from the house.
Importance: The Ethics Committee plays a vital role in maintaining public trust in the Parliament and ensuring that MPs are held
accountable for their actions. It also helps to promote ethical conduct within the legislature.

Examples of Cases Handled by the Ethics Committee:


The Ethics Committee has investigated cases of MPs being involved in financial irregularities, criminal offenses, and violation of the code
of conduct.
In some instances, the committee has recommended expulsion of MPs from the house for serious misconduct.

Que. 156 The Codex Alimentarius Commission (CAC) is an international organization that deals with:

1. Intellectual Property Rights for new agricultural varieties

88/262
2. Setting international food safety standards and practices
3. Space exploration and research
4. Regulating global maritime trade

Correct Option - 2
The correct answer is option 2.
Key Points
Codex Alimentarius Commission (CAC) : The Codex Alimentarius Commission (CAC) is an international organization working to ensure
food safety and fair practices in the global food trade.
Key points about the CAC:

Joint Effort:
Established in 1963 by the Food and Agriculture Organization (FAO) of the United Nations and the World Health Organization
(WHO).

Mission:
Protects consumer health by setting international food standards, guidelines, and codes of practice.
Ensures fair practices in the food trade by promoting harmonization of food standards.
Facilitates international trade in food by providing a common platform for countries to agree on food safety and quality requirements.

Membership:
Open to all member nations and associate members of FAO and WHO. Currently, there are 189 members, including 188 countries and the
European Union.

Structure:
Governed by the Codex Alimentarius Commission (CAC) which meets annually.
Supported by subsidiary bodies like General Subject Committees, Commodity Committees, Task Forces, and the Executive
Committee.

Work Process:
Develops food standards based on scientific risk assessment and consensus among member countries.
Standards cover a wide range of food products, including contaminants, additives, labeling, hygiene, and residues.

Impact:
Codex standards are recognized by the World Trade Organization (WTO) as the benchmark for food safety and quality in international
trade.
Play a vital role in protecting consumers from unsafe food and promoting a level playing field for food producers and exporters.

Que. 157 With reference to Section 6A of the Citizenship Act, 1955, consider the following statements:
1. It grants automatic citizenship to all migrants who arrived in Assam before 1971.
2. It was enacted to address illegal migration from Bangladesh.
3. It applies uniformly throughout India.
How many of the statements given above is/are correct?

1. Only one
2. Only two
3. All three
4. None

Correct Option - 1
The correct answer is option 3.
Key Points
Section 6A of the Citizenship Act, 1955 (Assam Accord) : Section 6A was enacted in 1985 to implement the Assam Accord. This
agreement, signed between the Indian government and Assamese representatives, aimed to address illegal migration from Bangladesh
into Assam.

Section 6A categorizes migrants based on arrival dates. Only those arriving before 1966 are automatically considered citizens. Hence,
Statement 1 is incorrect.
Section 6A is a direct outcome of the Assam Accord to manage migration from Bangladesh. Hence ,Statement 2 is correct.
Section 6A is a special provision specific to Assam. The rest of India follows the main provisions of the Citizenship Act. Hence
,Statement 3 is incorrect.

Classification of Migrants: The section classifies migrants based on their arrival date in Assam:
Before January 1, 1966: Individuals who entered Assam before this date are automatically considered Indian citizens.
Between January 1, 1966, and March 25, 1971: Migrants who arrived during this period can register for Indian citizenship if they meet
specific requirements and have resided in Assam for 10 years.
After March 25, 1971: People who entered Assam after this cut-off date are considered illegal immigrants and are subject to
deportation procedures as per law.

89/262
Purpose: The primary purpose of Section 6A is to identify and regularize the citizenship status of migrants in Assam, particularly those
who arrived before the cut-off date of March 25, 1971. This aims to protect the rights of indigenous Assamese people who felt their
culture and identity were threatened by large-scale migration.

Controversy: Section 6A has been criticized for potentially violating the Indian Constitution's principle of equality. Critics argue it
discriminates against migrants based solely on their arrival date. The Supreme Court of India is currently examining the validity of
Section 6A in light of these concerns.

Special Provision: It's crucial to remember that Section 6A is a special provision applicable only to Assam. The citizenship process for
the rest of India follows the main provisions of the Citizenship Act, 1955.

Ongoing Debate: The implementation of Section 6A has been a complex and contentious issue. The ongoing legal case and discussions
surrounding the section highlight the challenges of balancing national security concerns with the rights of migrants and indigenous
communities.

Que. 158 Which of the following statements is/are not true about the Partnership for Global Infrastructure and Investment (PGII)?

1. It is a collaborative effort by the G20 to develop infrastructure projects.


2. It aims to mobilize private sector investment for infrastructure projects in developing countries.
3. It is seen as a counter to China's Belt and Road Initiative (BRI).
4. It focuses on sectors like climate change, health, and digital technology.

Correct Option - 1
The correct answer is option 2.
Key Points
Partnership for Global Infrastructure and Investment (PGII) : The Partnership for Global Infrastructure and Investment (PGII) is a
collaborative effort by the Group of Seven (G7) to fund infrastructure projects in developing nations. It is seen as a counter to China's Belt
and Road Initiative (BRI).

The PGII aims to catalyze funding for quality infrastructure for low- and middle-income countries from the private sector and will
encourage private-sector investments that support "climate, health and health security, digital technology, and gender equity and
equality."
The initiative was relaunched and renamed as the Partnership for Global Infrastructure and Investment (PGII) at the 48th G7 summit
in Germany in June 2022. The G7 has pledged to mobilize $600 billion by 2027 for the PGII.
Governance: The PGII is guided by a steering committee with representatives from G7 member states. Partner countries will also
have a voice in project selection and implementation.
Synergy with other initiatives: The PGII is envisioned to work alongside existing development finance institutions like the World
Bank and regional development banks.

Que. 159 Consider the following statements


1: UNCITRAL primarily focuses on developing binding international agreements for member states.
2: UNCITRAL aims to promote uniformity and fairness in the legal framework for international trade.
Which if the following is the correct answer?

1. Both Statement 1 and Statement 2 are correct.


2. Both Statement 1 and Statement 2 are incorrect.
3. Statement 1 is correct, Statement 2 is incorrect.
4. Statement 1 is incorrect, Statement 2 is correct.

Correct Option - 4
The correct answer is option 4.
Key Points
United Nations Commission on International Trade Law (UNCITRAL) : The United Nations Commission on International Trade Law
(UNCITRAL) is a key player in facilitating international trade and investment. Established in 1966, it's the core legal body within the UN
system for international trade law.

UNCITRAL's key role is to harmonize and modernize international trade law, not solely create binding agreements. They offer various
tools like model laws for countries to adapt. Hence ,Statement 1 Incorrect.
UNCITRAL aims to achieve fair and consistent legal frameworks for international transactions, promoting uniformity and fairness.
Hence ,Statement 2 Correct.

UNCITRAL's role:
Mission: UNCITRAL works to modernize and harmonize the rules of international business transactions. They aim to create fair and
consistent legal frameworks for different countries.
Activities: UNCITRAL achieves this by developing various instruments like:
Conventions: International agreements that countries can ratify.
Model Laws: Legal frameworks that countries can adapt into their national laws.
Legislative guides and recommendations on various trade law topics.
90/262
Areas of Focus: Dispute resolution, international sales, commercial arbitration, insolvency, electronic commerce, and more.

Que. 160 Which of the following statements are correct regarding the UNESCO Creative Cities Network (UCCN) and India?

1. It was launched in 2010 and focuses solely on promoting traditional crafts.


2. Chennai is the only Indian city included in the UCCN for its musical heritage.
3. Mumbai and Hyderabad were the first Indian cities to join the network in 2017.
4. Srinagar and Kozhikode were recently added to the UCCN in recognition of their contributions to literature and crafts, respectively.

Correct Option - 4
The correct answer is option 4.
Key Points
UNESCO Creative Cities Network (UCCN) :The UNESCO Creative Cities Network (UCCN) was established in 2004 to promote
cooperation among cities that recognize creativity as a driving force for sustainable urban development. There are nearly 300 cities in the
network worldwide.
Objective: Placing creativity and cultural industries at the heart of their development plans at the local level and cooperating actively at the
international level.

UNESCO designates the creative cities in seven fields: craft, folk art, media arts, film design, gastronomy, literature, and music.
The applications in India are routed through the Ministry of Culture.

Indian Cities in the UNESCO Creative Cities Network:


Jaipur (2015): Recognized for its rich heritage and vibrant crafts and folk arts scene.
Varanasi (2015): Designated as a Creative City of Music due to its longstanding traditions and significance in Indian music.
Chennai (2017): Another Indian city on the Creative Cities of Music list, known for its Carnatic music and rich musical heritage.
Mumbai (2019): Selected for its world-renowned film industry, Bollywood.
Hyderabad (2019): Credited for its unique and diverse Hyderabadi cuisine.
Srinagar (2021): Added to the network for its exquisite crafts and folk arts traditions.
Kozhikode (2023): Recently included for its contributions to literature.
Gwalior (2023): Another recent addition recognized for its rich musical heritage.

Que. 161 Consider the following statements about the African Union (AU):
1. The AU was established in 2002, succeeding the Organization of African Unity (OAU).
2. The primary goal of the AU is to promote economic and political cooperation among member states.
3. India recently advocated for the AU's permanent membership in the BRICS grouping.
How many of the statements given above is/are correct?

1. Only one
2. Only two
3. All three
4. None

Correct Option - 2
The correct answer is option 2.
Key Points
African Union : The African Union (AU) is a continental union consisting of all 55 member states located in Africa. Established in 2002, it
succeeded the Organization of African Unity (OAU).

The AU was formed in 2002, taking over from the OAU. Hence ,Statement 1 is correct.
Promoting cooperation among member states is a core objective of the AU. Hence ,Statement 2 is correct.
The BRICS grouping consists of Brazil, Russia, India, China, and South Africa. The AU is a continental union, not a specific economic
bloc. Therefore, India would advocate for AU membership in the G20 (Group of 20), not BRICS. Hence ,Statement 3 is incorrect.

Goals:
Promote greater unity and solidarity among African states and its people.
Defend the sovereignty and independence of member states.
Accelerate political and socio-economic development across Africa.

Structure:
The Assembly of the African Union: The highest decision-making body, consisting of all heads of state or government of member states.
The Pan-African Parliament: A representative body with 265 members elected by national legislatures.
Other political and administrative bodies dealing with specific issues.

Headquarters: Addis Ababa, Ethiopia

Recent News (India Connection):

91/262
The AU was in the news recently due to India's advocacy for granting it permanent membership in the Group of 20 (G20) during the
2023 G20 Summit in New Delhi. This inclusion signifies Africa's growing importance and strengthens India's position as a champion for a
more inclusive global order.

Que. 162 Which of the following organizations is tasked with facilitating a peaceful resolution to the Nagorno-Karabakh conflict?

1. Shanghai Cooperation Organisation (SCO)


2. Organisation for Security and Co-operation in Europe (OSCE)
3. North Atlantic Treaty Organization (NATO)
4. European Union (EU)

Correct Option - 2
The correct answer is option 2.
Key Points
OSCE Minsk Group : The OSCE Minsk Group is a body created in 1992 by the Organization for Security and Co-operation in
Europe (OSCE) to facilitate a peaceful resolution to the Nagorno-Karabakh conflict.
Additional Information
Leadership Structure: The group is co-chaired by three countries: France, Russia, and the United States. These three nations form
the core leadership body, also known as the Minsk Troika.
Participating States: In addition to the co-chairs, the Minsk Group includes representatives from Armenia, Azerbaijan (the main
parties to the conflict), Belarus, Finland, Germany, Italy, Sweden, and Turkey. The OSCE Troika also participates on a rotational
basis.
Stalemated Progress: Despite ongoing efforts, the Minsk Group has not been able to achieve a lasting ceasefire or a final peace
agreement between Armenia and Azerbaijan. The most recent major outbreak of violence in the region occurred in late 2020.
Continued Relevance: Despite the lack of recent breakthroughs, the Minsk Group remains the only internationally recognized forum for
negotiations on the Nagorno-Karabakh conflict.

Que. 163 With reference to the Government of India, the Indian Councils Act of 1892 introduced for the first time:

1. Universal adult franchise for elections to the legislative councils.


2. A majority of elected members in the central legislative council.
3. The concept of separate electorates based on religion.
4. A platform for Indian voices to be heard within the administration.

Correct Option - 4
The correct answer is option 4.
Key Points
Lord cross's act : Lord Cross's Act, also formally known as the Indian Councils Act of 1892, was a significant piece of legislation passed
by the British Parliament.
Background and Purpose:
Introduced in 1892 by Richard Assheton Cross, 1st Viscount Cross.
Aimed to address the growing demands for political participation by Indian nationalists.

Impact on Legislative Councils:


Increased the number of members in both the central and provincial legislative councils.
Introduced a system for nominating or electing some council members, though the British maintained an overall majority.
Expanded the functions of the councils:
Allowed for discussions on the budget.
Enabled members to ask questions about the government's actions (with limitations).

Significance:
Considered a first step towards a representative form of government in India.
Provided a platform for Indian voices to be heard within the British administration.
Though limited, it helped pave the way for future reforms and the eventual independence movement.

The Indian Councils Act of 1892, while modest in its reforms, marked a turning point in British India. It planted the seeds for a more
inclusive political system that would eventually blossom into Indian democracy.

Que. 164 The Paris Principles are associated with:

1. Standards for effective international trade agreements.


2. Guidelines for establishing credible national human rights institutions.
3. Principles for sustainable development goals.
4. Regulations for international environmental protection agencies.

92/262
Correct Option - 2
The correct answer is option 2.
Key Points
Paris Principles : The Paris Principles refer to a set of guidelines established to ensure effective national human rights institutions
(NHRIs).
Key points:
Origin and Purpose:

Defined in 1991 at the first International Workshop on National Institutions for Human Rights Promotion and Protection.
Adopted by the United Nations to set minimum standards for NHRIs.

Key Principles:

Broad Mandate: NHRIs should address all human rights issues.


Independence: Freedom from government influence is crucial for impartiality.
Pluralism: The NHRI's composition should reflect society's diversity.
Adequate Resources: Sufficient funding and staffing are necessary for effective operation.
Adequate Powers: NHRIs need investigative and reporting abilities to fulfill their roles.

Significance:

The Paris Principles are a cornerstone for establishing credible and impactful NHRIs.
Compliance with these principles is a requirement for NHRI accreditation, allowing participation in UN human rights mechanisms.
They promote effective human rights protection within countries.

In News
For the second time in a decade, Global Alliance of National Human Rights Institutions (GANHRI) deferred the accreditation of
National Human Rights Commission (NHRC), citing objections like political interference in appointments among others.

The GANHRI had granted ‘A’ status of accreditation to NHRC in 2017, after deferring it the year before — the first such instance
since NHRC was established (1993).
Without the accreditation, NHRC will be unable to represent India at the UN Human Rights Council.

Que. 165 What does the term "expunging" refer to in the context of the Indian Parliament?

1. The process of recording parliamentary proceedings


2. The removal of unparliamentary remarks from the official record
3. The allocation of parliamentary time for debates
4. The appointment of parliamentary committees

Correct Option - 2
The correct answer is option 2.
Key Points
Expunging : In the context of the Indian Parliament, "expunging" refers to the removal or deletion of certain remarks made by members
of Parliament (MPs) during the proceedings. These remarks might be deemed unparliamentary, offensive, or against the rules of conduct
within the Parliament.
Process :

Unparliamentary Remarks: If a member makes a statement that is considered unparliamentary, offensive, or violates the rules of
conduct, any other member or the Speaker of the House can raise an objection.
Objection Raised: When an objection is raised, the Speaker of the House reviews the statement in question to determine whether it
violates parliamentary decorum.
Decision by the Speaker: If the Speaker finds that the remark is indeed unparliamentary, offensive, or in violation of the rules, they
may expunge or strike out the remark from the official record of the proceedings.
Notification: The Speaker typically informs the member who made the objectionable statement about the decision to expunge it.
Impact: Once expunged, the offending remark is considered to be removed from the official record. It's as though it was never
uttered during the proceedings.

Que. 166 Consider the following statements


1. The content of the oath taken by High Court and Supreme Court judges in India differs.
2. Judges in both High Courts and the Supreme Court pledge to uphold the Constitution and laws of India.
3. The oath is administered by the Chief Justice of India for all judges.
How many of the statements given above is/are correct?

1. Only one
2. Only two
93/262
3. All three
4. None

Correct Option - 2
The correct answer is option 2.
Key Points
Oaths of Judges : Judges in both the High Court and Supreme Court of India take an oath (or affirmation) before assuming office. The
oath emphasizes their commitment to upholding the Constitution and performing their duties impartially.

The wording of the oath is identical for both High Court and Supreme Court judges, as specified in the Third Schedule of the Indian
Constitution. Hence ,Statement 1 is Incorrect.
Upholding the Constitution and laws is a central pledge in the oath. Hence ,Statement 2 is Correct.
The oath is administered by the President/designee for Supreme Court judges and the Governor/designee for High Court
judges. Hence ,Statement 3 is Incorrect.

The Oath : It states that the judge will:

Bear true faith and allegiance to the Constitution of India.


Uphold the sovereignty and integrity of India.
Duly and faithfully perform their duties to the best of their ability, knowledge, and judgment.
Discharge their duties without fear or favor, affection, or ill-will.
Uphold the Constitution and the laws of India.

Who Administers the Oath


High Court Judges: The oath is administered by the Governor of the State where the High Court is located, or someone designated by the
Governor [Article 219, Constitution of India].
Supreme Court Judges: The oath is administered by the President of India, or someone designated by the President [Article 124(6),
Constitution of India].
This oath serves as a solemn pledge by the judges to uphold the rule of law and deliver justice fairly in India's judicial system.

Que. 167 Consider the following statements


1. The Central Council of Local Government is a body mandated to implement policies for urban local bodies.
2 . The Union Minister for Rural Development heads the Central Council of Local Government.
3. The Council advises the central government on matters concerning urban local governance.
How many of the statements given above is/are correct?

1. Only one
2. Only two
3. All three
4. None

Correct Option - 1
The correct answer is option 1.
Key Points
Central Council of Local Government (CCLG) : The Central Council of Local Government (CCLG) is an advisory body constituted
under Article 263 of the Indian Constitution . It was established in 1954 and was originally known as the Central Council of Local Self-
Government.

The CCLG is an advisory body, not a policy-making or implementing one. Hence , Statement 1 is Incorrect.
The Union Minister for Urban Development, not Rural Development, heads the CCLG. Hence ,Statement 2 is Incorrect.
This is a key function of the Council - to provide advice on improving urban local governance. Hence ,Statement 3 is Correct.​

Composition:

The CCLG comprises the Union Minister for Urban Development as the Chairperson.
Other members include ministers in charge of local government from each state and union territory.

Function:
The CCLG is not a policy-making body but an advisory one. It deliberates on issues related to urban local governance and makes
recommendations to the central government and state governments for improvement. Here are some of its functions:

Discussing and making recommendations on legislation related to urban local bodies.


Reviewing the financial position of urban local bodies and suggesting measures for their improvement.
Advising the Central Government on matters concerning urban local government.
Disseminating information on urban local government among states.

Historical Background:

Initially, the CCLG dealt with both urban and rural local governments. However, after the Panchayati Raj institutions were established
in the 1950s, the CCLG's focus shifted solely to urban local government.
94/262
The 1980s saw a name change from the Central Council of Local Self-Government to the Central Council of Local Government.

Significance:
The CCLG provides a platform for consultation and exchange of ideas between the central government and state governments on matters
related to urban local governance. Its recommendations can play a vital role in shaping policies and improving the functioning of urban local
bodies in India.

Que. 168 The Cabinet Committee on Parliamentary Affairs (CCPA) is primarily concerned with:

1. Formulating foreign policy strategies


2. Managing the national budget
3. Ensuring smooth functioning of government business in Parliament
4. Overseeing the functioning of public sector undertakings.

Correct Option - 3
The correct answer is option 3.
Key Points
Cabinet Committee on Parliamentary Affairs (CCPA):
The Cabinet Committee on Parliamentary Affairs (CCPA) is an important committee within the Indian government. Here's a breakdown of its
key aspects:
Function:

The CCPA acts as a bridge between the government and the Parliament.
Its primary role is to ensure the smooth and efficient functioning of government business in both houses of Parliament (Lok Sabha
and Rajya Sabha).

This includes tasks like:

Monitoring the progress of government bills and legislations.


Deciding the government's stance on non-official bills and resolutions presented in Parliament.
Reviewing state-level legislations from an all-India perspective.
Recommending when to summon or prorogue (suspend) sessions of Parliament.

Composition:

The CCPA is chaired by a senior cabinet minister, typically the Minister of Parliamentary Affairs.
Other members can include important cabinet ministers involved in key legislative matters.

Decision-Making:

The CCPA usually makes decisions by circulating papers for consideration among its members.
However, the chairperson can also call formal meetings to discuss complex issues.

Significance:

The CCPA plays a crucial role in maintaining a smooth relationship between the executive (government) and the legislature
(Parliament).
By strategically managing government business in Parliament, the CCPA helps ensure timely passage of important bills and promotes
effective governance.

Que. 169 With reference to default bail in India, consider the following statements:
1. It is a legal right that allows an accused person to be released on bail if the investigation is not completed within a specific time
frame.
2. The time limit for filing a chargesheet after investigation is always 90 days.
3. The concept of default bail discourages holding people in detention without charges.
How many of the statements given above is/are correct?

1. Only one
2. Only two
3. All three
4. None

Correct Option - 2
The correct answer is option 2.
Key Points
Default bail : Default bail, also known as statutory bail, is a legal right in India that allows an accused person to be released on bail if
the investigation against them isn't completed within a specific time frame. It essentially functions as a safeguard against unreasonable
detention.

95/262
Default bail is a statutory right enshrined in Section 167(2) of the CrPC. Hence, statement 1 is correct.
The time limit for filing a chargesheet varies depending on the offense. It's generally 60 days for most offences and 90 days for some
serious crimes. Hence, statement 2 is incorrect.
Default bail acts as a safeguard against unreasonable detention and incentivizes law enforcement to conduct investigations
efficiently. Hence , statement 3 is correct.

How it works:

Time Limits: The Code of Criminal Procedure (CrPC) dictates the time limits for investigations. Generally, police have 60 or 90 days
to complete their investigation and file a chargesheet, depending on the offense.
Accruing the Right: If the investigation isn't finished within the stipulated timeframe, the accused gains the right to apply for
default bail.
Application Process: While it's a right, the accused needs to formally request bail through an application to the court. The court
doesn't automatically grant bail upon expiry of the time period.

Purpose:
The concept of default bail serves two main purposes:

Ensure Speedy Investigation: It incentivizes law enforcement to conduct investigations efficiently and discourages holding people
unnecessarily.
Protect Individual Liberty: It safeguards against the accused being detained for an extended period without charges being filed,
upholding their right to liberty.

Que. 170 The Enforcement Directorate (ED) in India is primarily concerned with enforcing laws related to:

1. Cybercrime and online fraud


2. Narcotics and drug trafficking
3. Financial crimes and money laundering
4. Environmental pollution and wildlife protection

Correct Option - 3
The correct answer is option 3.
Key Points
Enforcement Directorate (ED) :
The Enforcement Directorate (ED) is primarily responsible for enforcing financial laws in India. Here are the main Acts under which the
ED can take action:
Prevention of Money Laundering Act, 2002 (PMLA): This Act empowers the ED to investigate and prosecute cases of money laundering.
Money laundering involves converting illegally obtained money into legitimate-looking funds. The ED can attach assets, conduct searches
and seizures, and arrest individuals suspected of money laundering offenses.
Foreign Exchange Management Act, 1999 (FEMA): This Act regulates foreign exchange transactions in India. The ED can investigate
violations of FEMA, such as unauthorized foreign exchange dealings or holding foreign assets beyond permissible limits. They can
impose penalties and seize assets acquired through illegal foreign exchange activities.
Fugitive Economic Offenders Act, 2018 (FEOA): This Act allows the ED to take action against economic offenders who flee the country
to avoid facing legal proceedings in India. The ED can attach and confiscate the properties of such offenders to recover dues and deter
economic offenses.
Additionally:
The ED might also play a role in investigations related to the Conservation of Foreign Exchange and Prevention of Smuggling Activities
Act, 1974 (COFEPOSA), though enforcement of penalties under this Act usually falls under designated authorities.

Que. 171 Which of the following statements regarding electoral bonds is true?

1. The identity of the donor is publicly disclosed.


2. Electoral bonds can be encashed by any individual or organization.
3. Only political parties registered under Section 29A of the Representation of the People Act, 1951 are eligible to receive electoral
bonds.
4. Electoral bonds are valid for an indefinite period.

Correct Option - 3
The correct answer is option 3.
Key Points
Electoral bonds : Electoral bonds are a financial instrument introduced by the Government of India in 2018 as an alternative to cash
donations made to political parties. They were proposed as a means to bring transparency to political funding. The scheme allows
individuals and corporate entities to purchase bonds from authorized banks using white money and then donate these bonds to their
chosen political parties.
The key aspects of electoral bonds include:

96/262
Anonymity: The identity of the donor is not disclosed to the public or to the receiving political party. However, the government
and the bank have records of the donors.
Validity: Electoral bonds are valid for fifteen days from the date of issuance, and they can be encashed only by registered political
parties.
Authorized Banks: Electoral bonds can only be purchased from specified branches of authorized banks.
Eligibility of Recipients: Only political parties that are registered under Section 29A of the Representation of the People Act, 1951
and have secured at least 1% of the votes polled in the most recent general election or assembly election are eligible to receive
electoral bonds.
Transparency: The government claims that the electoral bond system enhances transparency in political funding by reducing the
use of cash donations, which are often unaccounted for and can potentially involve black money.

Que. 172 How does the Electronically Transmitted Ballot Polling System (ETBPS) facilitate voting?

1. It allows voters to cast their votes using electronic voting machines.


2. It enables eligible voters to receive and cast their ballots electronically from remote locations.
3. It conducts polls through social media platforms.
4. It involves physical ballot paper voting but with electronic counting.

Correct Option - 2
The correct answer is option 2.
Key Points
Electronically Transmitted Ballot Polling System (ETBPS) : The Electronically Transmitted Ballot Polling System (ETBPS) facilitates
voting by leveraging electronic means to enable eligible voters to receive and cast their ballots remotely, without the need to physically
visit a polling station. This system is particularly beneficial for individuals who are unable to be present at their designated polling
stations on election day due to various reasons such as mobility issues, illness, travel constraints, or being stationed in remote locations.
Process :

ETBPS typically operates through secure online platforms or electronic voting mechanisms where voters can access their ballots,
mark their preferences, and submit their votes electronically.
The system ensures the integrity and security of the voting process through robust encryption, authentication measures, and
verification protocols to prevent tampering or fraud.

Significance :

By allowing voters to participate in the electoral process from remote locations using electronic devices such as computers, tablets,
or smartphones, ETBPS significantly enhances the accessibility and inclusivity of the voting process.
It enables greater participation in elections, especially among individuals who might face barriers to physical voting, thereby
promoting democratic principles of representation and engagement.

Que. 173 Which of the following statements is true regarding statutory bail and regular bail in the context of criminal proceedings?

1. Statutory bail is granted by a court after a formal bail application is made, while regular bail is granted automatically by operation of
law.
2. Statutory bail is granted automatically by operation of law when certain conditions are not met within a specified time frame, while
regular bail is granted by a court after a formal application and judicial review process.
3. Statutory bail is granted only to individuals with no prior criminal record, while regular bail is granted to repeat offenders.
4. Statutory bail is granted for minor offenses, while regular bail is reserved for serious offenses.

Correct Option - 2
The correct answer is option 2.
Key Points
Statutory Bail:

Statutory bail is granted to an accused person by operation of law, typically when certain conditions are not met within a specified
time frame.
It is granted automatically by virtue of specific provisions in the law, such as a statute or code, without the need for a court order or
hearing.
Statutory bail is often granted when the investigating agency or law enforcement fails to complete their investigation or file
charges within the time limit prescribed by law.
The conditions for statutory bail and the time frame within which it must be granted are usually outlined in the relevant legal
statutes.

Regular Bail:

Regular bail is granted by a court after a formal bail application is made by the accused or their legal representative.
It involves a judicial process where the court evaluates various factors, such as the nature of the offense, the likelihood of the
accused fleeing, the strength of the evidence against the accused, and the accused's ties to the community, before deciding whether to
grant bail.

97/262
Regular bail can be granted at any stage of the criminal proceedings, including before or after charges are filed, during trial, or even
after conviction pending appeal.
The court may impose certain conditions on the grant of regular bail, such as surrendering the passport, reporting to a police station at
regular intervals, or refraining from contacting certain individuals.

Que. 174 1. The NCRWC was established to recommend changes to the Indian Constitution to address the challenges faced by the country
after 50 years of independence.
2. The Commission's recommendations have been entirely implemented by the government.
How many of the statements given above is/are correct?

1. 1 only
2. 2 only
3. Both 1 and 2
4. Neither 1 nor 2.

Correct Option - 1
The correct answer is option 1.
Key Points
National Commission to Review the Working of the Constitution (NCRWC):
The National Commission to Review the Working of the Constitution (NCRWC), also known as the JMNR Venkatachaliah
Commission, was established by the Indian government in 2000.
Purpose :

The commission's purpose was to examine the Indian Constitution's effectiveness after fifty years of independence and suggest
potential changes.
The commission's recommendations were intended to keep up with contemporary India's evolving needs for effective governance
and socioeconomic development while upholding the parliamentary democracy framework.

The commission submitted its report in 2002, which included recommendations on a variety of topics, including:

Fundamental rights, directive principles, and fundamental duties


Electoral processes and political parties
Parliament and state legislatures
The executive and public administration
The judiciary
Union-state relations
Decentralization and devolution
The speed of socioeconomic change and development

The commission's recommendations have been debated, but none have been adopted in their entirety. The notion of the Constitution's
"basic structure," which the Supreme Court of India has established as unchangeable, has served as a barrier to substantial reform.

Que. 175 The E-Government Development Index (EGDI) is published by:

1. World Bank
2. International Telecommunication Union (ITU)
3. United Nations Department of Economic and Social Affairs (UNDESA)
4. World Economic Forum (WEF)

Correct Option - 3
The correct answer is option 3.
Key Points
E-Government Development Index (EGDI) : The E-Government Development Index (EGDI) is a valuable tool for understanding a
nation's progress in establishing online government services .The United Nations Department of Economic and Social Affairs (UNDESA)
publishes the EGDI along with the UN E-Government Survey.

What it Measures:

The EGDI is a composite index, meaning it combines scores from three sub-indices to provide a single value:
Online Service Index (OSI): This evaluates the breadth and quality of online services offered by the government. It assesses factors
like citizen access to forms, applications, and information through government websites.
Telecommunication Infrastructure Index (TII): This gauges a country's telecommunication infrastructure, including internet
penetration, bandwidth availability, and mobile network coverage. Robust infrastructure is essential for citizens to access e-government
services effectively.
Human Capital Index (HCI): This measures a nation's population's education and literacy levels. A well-educated populace is more
likely to have the digital literacy skills necessary to utilize e-government services.

98/262
Importance of EGDI:

The EGDI serves several crucial purposes:


Benchmarking: Countries can utilize their EGDI scores to compare their performance with others and identify areas needing
improvement.
Development Tool: It functions as a guide for countries to pinpoint strengths and weaknesses in their e-government initiatives. This
allows them to develop targeted strategies for advancement.
Policy Development: The EGDI can inform policymakers in crafting strategies to enhance e-government services. By understanding
citizen needs and infrastructure limitations, they can create more accessible and user-friendly online services.

Que. 176 With reference to the safeguards for linguistic minorities in India, consider the following statements:
1. The office of the Special Officer for Linguistic Minorities is mandated by Article 350.
2. The Commissioner for Linguistic Minorities submits reports directly to the State Governments.
Which of the following statements is/are correct?

1. 1 only
2. 2 only
3. Both 1 and 2
4. Neither 1 nor 2.

Correct Option - 1
The correct answer is option 1.
Key Points
Special Officer for Linguistic Minorities:
The position of Special Officer for Linguistic Minorities is indeed established by the Indian Constitution. Here's a breakdown of the relevant
details:
Article: The provision is included in Article 350B. It's important to note that this article wasn't part of the original constitution but was
introduced later through the Constitution (Seventh Amendment) Act of 1956.
Appointment: The President of India appoints the Special Officer.
Responsibilities: The Special Officer investigates all matters related to the safeguards provided for linguistic minorities under the
Constitution. They then report their findings to the President at designated intervals.
Reporting: The President is required to present these reports before both houses of Parliament and send them to the governments of the
states concerned.

In practice, the Special Officer is referred to as the Commissioner for Linguistic Minorities (CLM). The CLM's office works to ensure that
the rights of linguistic minorities in India are protected as per the Constitution.

Que. 177 The Antiquities and Art Treasures Act, 1972 primarily aims to:

1. Promote international trade in Indian antiquities.


2. Facilitate the easy sale and possession of antiquities within India.
3. Restrict the export and regulate the trade in antiquities to preserve cultural heritage.
4. Provide financial assistance for restoration of private collections of antiquities.

Correct Option - 3
The correct answer is option 3.
Key Points
Antiquities and Art Treasures Act, 1972 :
The Antiquities and Art Treasures Act, 1972 (AATA for short) is a law in India aimed at protecting the country's cultural heritage by
regulating the trade in antiquities and art treasures. Here's a breakdown of its key points:
Main Objectives:

Prevent Permanent Export: The Act restricts the export of antiquities and art treasures to preserve India's cultural wealth. Only
the Central Government or authorized agencies can export them under specific circumstances.
Curb Smuggling and Fraud: It discourages illegal activities like smuggling and fraudulent dealings involving antiquities.
Regulate Sale and Possession: The Act regulates the sale and possession of antiquities within India. Licenses are required for selling
antiquities, and owners may need to register them.
Public Preservation: The Act allows for the compulsory acquisition of valuable antiquities and art treasures for preservation in
public museums and institutions.

Key Features:

Definition of Antiques: The Act provides a broad definition of "antiquity" encompassing objects like sculptures, paintings,
manuscripts, and other items exceeding 100 years old.
Registration: Owners of antiquities may be required to register them with designated authorities.
Licensing: A license is needed for selling antiquities.
Export Permission: Exporting antiquities generally requires government permission.

99/262
Penalties: The Act prescribes penalties for violating its provisions, including imprisonment and fines for offenses like illegal export or
dealing in antiquities.

Que. 178 Consider the following statements regarding Civil Society 20 (C20):
1. C20 is an official forum within the G20 process representing the interests of multinational corporations.
2. C20 provides a platform for civil society organizations to advocate for global issues like climate change and human rights.
3. C20 recommendations are binding on G20 member countries.
How many of the statements given above is/are correct?

1. Only one
2. Only two
3. All three
4. None

Correct Option - 1
The correct answer is option 1.
Key Points
Civil Society 20 (C20) :
Civil Society 20 (C20) refers to the official engagement group representing civil society organizations within the G20 process.

C20 represents civil society, not corporations. Hence , Statement 1 is Incorrect.


C20 is a platform for civil society voices on global issues. Hence ,Statement 2 is Correct.
C20 recommendations are not binding but aim to influence G20 decisions. Hence ,Statement 3 is Incorrect.

Here's a breakdown of key points about C20:


Purpose: C20 provides a platform for non-governmental and non-business voices to be heard on global issues addressed by the G20.
Functioning: It works through consultations and recommendations. Civil society organizations collaborate to develop policy
recommendations on various topics like:
Environment and sustainable development
Gender equality
Human rights
Social, economic, and climate justice
Engagement with G20: C20 presents its policy proposals and demands to G20 leaders for consideration during the annual G20
Summit.
Openness: C20 is an open community, welcoming participation from civil society organizations worldwide.

Here are some additional details to consider:


Formation: Established in 2013.
Leadership: Each year's C20 is led by a coalition of civil society organizations from the G20 presidency country.
Recent Example: In 2023, when India held the G20 presidency, Amrita Vishwa Vidyapeetham led the C20 India chapter.

Que. 179 The doctrine of promissory estoppel applies when:

1. A written contract exists with a clear breach by one party.


2. A vague promise is made with no expectation of reliance.
3. A clear promise leads to reasonable reliance that causes harm if broken.
4. Both parties enter a verbal agreement witnessed by a third party.

Correct Option - 3
The correct answer is option 3.
Key Points
Doctrine of promissory estoppel :
The doctrine of promissory estoppel is a legal principle in contract law that prevents someone from going back on a promise, even if the
promise itself wouldn't be considered a legally binding contract. Here's a breakdown of the key points:
Core Idea: Promissory estoppel essentially says that if you make a clear promise that someone relies on and suffers harm because you
break it, you may be held liable for those damages.
Requirements: For promissory estoppel to apply, there are typically three main requirements:
1. Clear and Unambiguous Promise: The promisor (the person making the promise) must have made a clear and unambiguous
statement that could be reasonably understood as a promise.
2. Reasonable Reliance: The promisee (the person relying on the promise) must have acted in reasonable reliance on the promise.
This means they took some action or made a decision based on the expectation that the promise would be kept.
3. Detrimental Reliance: The promisee must have suffered some form of detriment (harm or loss) as a result of relying on the broken
promise.
Key Point: Promissory estoppel is not a substitute for a formal contract. It's a way to enforce a promise when there's no formal agreement
but fairness dictates that the promise should be upheld to avoid causing harm.

100/262
Que. 180 Consider the following statements regarding recess in the Parliament of India:
1.Recess refers to a period when both houses of Parliament are adjourned sine die, marking the end of a session.
2. Recess allows Members of Parliament (MPs) to return to their constituencies and Parliamentary committees to conduct in-depth studies on
specific issues.
Which of the statements given above is/are correct?

1. 1 only
2. 2 only
3. Both 1 and 2
4. Neither 1 nor 2

Correct Option - 2
The correct answer is option 2.
Key Points
Recess in the Parliament : In the context of Parliament, a recess refers to a break period during a parliamentary session (year) when
neither house (e.g, Lok Sabha) meets to conduct official business. There are typically several recesses throughout a session, often coinciding
with major holidays or to allow for committee work.

Adjournment sine die signifies the end of a session, while recess is a break within a session. Hence, Statement 1 is Incorrect
Recess provides an opportunity for MPs to connect with constituents and committees to delve into specific issues. Hence, Statement 2
is Correct.

Key points about recess in Parliaments:


Purpose: Recess provides opportunities for:
Members of Parliament (MPs) to return to their constituencies and meet with constituents.
Parliamentary committees to hold meetings and delve deeper into specific issues.
MPs to take a break and recharge.
Frequency and Duration: The exact frequency and duration of recesses can vary depending on the specific Parliament. However,
common recess periods include:
Summer: A longer recess, often coinciding with summer holidays.
Christmas: A recess around Christmas and New Year's.
Easter: A shorter recess around Easter.
Other Holidays: Recesses might also be scheduled for other national holidays.
Formal Terminology: Recess is sometimes referred to as a periodic adjournment.

Here are some additional points to consider:


The decision to declare a recess is usually made by the presiding officer of the Parliament (e.g., Speaker, Chairperson).
The length of the recess can be influenced by the workload of the Parliament and the political climate.
During recess, some essential parliamentary functions, like urgent legislation, might still be addressed through special sittings or
committees.

Que. 181 With reference to the Indian Army, PALM 400 refers to:

1. A new tank with enhanced armor protection.


2. A long-range, high-altitude communication system.
3. A precision attack loitering munition system.
4. A portable air defense radar for short-range threats.

Correct Option - 3
The correct answer is option 3.
Key Points
PALM 400 :
The PALM 400, which stands for Precision Attack Loitering Munition, is a long-range, high-precision loitering munition system
developed by a joint venture between AVision Systems (Israel), UVision Air Ltd, and Aditya Precitech Private Ltd (APPL) for the Indian
Army.
Here's a breakdown of its key features:
Function: It is designed to be a flying weapon system that can loiter over an area for extended periods, identify and track targets, and
then launch a precision attack when needed.
Capabilities:

Long range: Over 100 km (62 miles)


High endurance: Up to 120 minutes loitering time
Day and night operation: Equipped with electro-optical/infrared stabilized cameras for all-weather target acquisition
Multi-purpose weapon system: Can engage static and moving targets with high accuracy
Mid-air abort capability: Allows for re-entry into loitering mode, re-engagement, or return to the recovery area using a parachute.

101/262
Benefits: The PALM 400 offers several advantages to the Indian Army, including:

Increased battlefield effectiveness: Provides long-range precision strike capability against a variety of targets.
Reduced risk to troops: Allows for engagement of targets from a safe distance.
Improved operational flexibility: Can be used for various missions, including reconnaissance, surveillance, and target acquisition.

The PALM 400 is a significant development for the Indian Army, enhancing its offensive capabilities and providing a tactical advantage in
modern warfare scenarios.

Que. 182 With reference to international military cooperation, Operation Interflex is a program focused on:

1. Joint naval exercises between the US and India in the Pacific Ocean.
2. Training Ukrainian Armed Forces recruits by a group of nations led by the UK.
3. Cybersecurity collaboration between European nations and NATO.
4. Peacekeeping operations conducted by the African Union in conflict zones.

Correct Option - 2
The correct answer is option 2.
Key Points
Operation Inter flex : Operation Interflex is a British-led multinational military training operation established in July 2022. Its primary
objective is to train and support Ukrainian Armed Forces recruits in their fight against the Russian invasion.
Key details about Operation Interflex:

Location: Training takes place within the United Kingdom at various designated sites.
Successor to: Operation Orbital, which previously provided training to Ukrainian forces from 2015 to 2022.
International Collaboration: Several countries contribute personnel and expertise to Operation Interflex, including Canada,
Denmark, and New Zealand.

Focus of Training:

Basic soldiering skills like weapons handling, battlefield first aid, and combat tactics.
Law of Armed Conflict awareness to ensure adherence to international law during wartime.
Training is tailored for new recruits with little to no prior military experience.

Importance: Operation Interflex plays a crucial role in equipping Ukrainian recruits with the necessary skills and knowledge to defend their
homeland effectively.

Que. 183 Consider the following statements regarding Department-Related Standing Committees (DRSCs) in India:
1. DRSCs are permanent committees established under the Rules of Procedure and Conduct of Business in Lok Sabha.
2. Each DRSC has an equal number of members nominated by the Rajya Sabha and Lok Sabha.
3. DRSCs primarily focus on reviewing the budgetary allocations of government ministries.
How many of the statements given above is/are correct?

1. Only one
2. Only two
3. All three
4. None

Correct Option - 4
The correct answer is option 4.
Key Points
Department-Related Standing Committees (DRSCs): Department-Related Standing Committees (DRSCs) are parliamentary
committees in India established under the Constitution (Article 268). They play a crucial role in ensuring parliamentary oversight over
the functioning of various government ministries and departments.

DRSCs are established under the Constitution (Article 268) and are temporary, reconstituted annually. Hence ,Statement 1
is Incorrect.
DRSCs have more members nominated by the Lok Sabha compared to the Rajya Sabha . Hence ,Statement 2 is Incorrect.
While examining budgetary demands is a function, DRSCs also scrutinize bills and review departmental work. Hence ,Statement 3 is
Incorrect.

Structure and Composition:

There are 24 DRSCs, each corresponding to a group of related ministries/departments.


Each committee has a maximum of 31 members, with:
10 members nominated by the Chairman of the Rajya Sabha (Upper House)
21 members nominated by the Speaker of the Lok Sabha (Lower House)
Ministers are not included in these committees.

102/262
Each DRSC has a Chairperson chosen from its members.

Functions:
DRSCs perform several important functions, including:

Examining the Demands for Grants for the ministries/departments under their purview. They analyze the budgetary allocations and
submit reports with recommendations for improvement.
Scrutinizing Bills introduced by the government related to the assigned ministries/departments. They may suggest amendments or
recommend rejection.
Reviewing the working of the ministries/departments to ensure efficient administration, accountability, and adherence to policies.

Benefits:
DRSCs contribute significantly to effective parliamentary democracy by:
Strengthening Parliamentary Oversight: They hold the government accountable for its actions and policies related to specific departments.
Improving Policy Formulation: Committees provide a platform for expert analysis and recommendations, leading to better policy
decisions.
Enhancing Public Grievance Redressal: Citizens' concerns regarding departmental functioning can be raised before these committees.

Que. 184 Which of the following statements is not a potential benefit of the CPTPP?

1. Increased trade and economic growth for member countries.


2. Creation of new jobs in the manufacturing sector.
3. Fostering a more integrated and competitive regional economy.
4. Upholding high standards for intellectual property.

Correct Option - 2
The correct answer is option 2.
Key Points
Comprehensive and Progressive Agreement for Trans-Pacific Partnership (CPTPP): The Comprehensive and Progressive Agreement
for Trans-Pacific Partnership (CPTPP), also known as TPP11, is a free trade agreement between 11 countries in the Asia-Pacific region.

Origin and Current Status

The CPTPP emerged from the Trans-Pacific Partnership (TPP), which the United States withdrew from in 2017. The remaining
members negotiated modifications and brought the CPTPP into effect in December 2018. In July 2023, the United Kingdom formally
joined the agreement.

Key Objectives

The CPTPP aims to:


Eliminate or reduce tariffs and other trade barriers between member countries.
Promote smoother customs procedures and trade facilitation.
Establish common standards for intellectual property, competition policy, and other areas.
Encourage foreign investment among member states.

Potential Benefits

Supporters of the CPTPP believe it can:


Increase trade and economic growth for member countries.
Create new jobs and export opportunities for businesses.
Foster a more integrated and competitive regional economy.
Uphold high standards for labor, environment, and intellectual property.

Potential Concerns

Critics of the CPTPP raise concerns about:


The impact on domestic jobs, particularly in manufacturing sectors.
The potential for lower environmental or labor standards.
The benefits primarily accruing to large corporations.
The complexity of the agreement for small and medium-sized businesses.
While the CPTPP aims to create jobs, some critics argue it might lead to job losses in certain sectors like manufacturing due to increased
competition.

Que. 185 ​Under Article 252 of the Indian Constitution, the Parliament can enact a law for two or more states if:

1. The subject matter falls exclusively in the Union List.


2. The President recommends such legislation.
3. The Legislatures of the concerned states pass resolutions requesting such a law.
4. The Rajya Sabha approves the legislation by a two-thirds majority.

103/262
Correct Option - 3
The correct answer is option 3.
Key Points
Article 252 : Article 252 of the Indian Constitution deals with the power of the Parliament to legislate for two or more states with their
consent and subsequent adoption by other states
Key points:
Power to Legislate for Specific Situations
Parliament can make laws on matters that are normally outside its jurisdiction in states (outside the Union List) under specific
circumstances.
Conditions for Legislation:

Legislative Initiative: Legislatures of two or more states must pass resolutions requesting Parliament to make the law.
Subject Matter: The subject of the law cannot be part of the Union List (matters under Central Government's control) or those
already provided for in Articles 249 and 250 (concurrent powers of Centre and States).

Applicability of the Law :

The law passed by Parliament applies to the states that requested it initially.
Other states can adopt the law later by passing a resolution in their respective legislatures.

Amending or Repealing the Law :

Parliament can amend or repeal the law through another Act passed in the same way (with resolutions from requesting states).
Individual states cannot amend or repeal the law on their own.

Example :
Imagine two or more states want a uniform law on a specific environmental issue. They can request Parliament to make such a law under
Article 252. The law would then apply to those states initially and can be adopted by other states if they choose.

Que. 186 Which article of the Indian Constitution establishes the position of the Solicitor General of India?

1. Article 74
2. Article 75
3. Article 76
4. Article 77

Correct Option - 3
The correct answer is option 3.
Key Points
Solicitor general of India : The Solicitor General of India is one of the top law officers of the Government of India. The role is
established under Article 76(1) of the Constitution of India.
Article 76(1):

"There shall be an Attorney General for India who shall be appointed by the President and shall hold office during the pleasure of the
President. The President shall appoint a person who is qualified to be appointed a Judge of the Supreme Court to be Attorney General
for India."
The Solicitor General of India is the second-highest law officer of the government and assists the Attorney General of India in his
duties.

Duties :

Legal Advisor to the Government: The Solicitor General advises the government on legal matters and provides legal opinions on
various issues.
Representation in Court: One of the primary duties of the Solicitor General is to represent the Government of India in the
Supreme Court of India. They may also represent the government in other courts and tribunals when necessary.
Assisting in Litigation: The Solicitor General assists the Attorney General of India in handling important cases and representing the
government's interests effectively.
Constitutional Advice: Providing advice to the government on matters related to constitutional law, including interpretation and
application of the Indian Constitution.
Legal Research and Drafting: Conducting legal research on complex legal issues and drafting legal documents such as pleadings,
affidavits, and opinions.
Coordination with Other Legal Authorities: Collaborating with other legal authorities within the government, such as the Attorney
General, Additional Solicitors General, and state legal officers, to ensure cohesive legal representation and strategy.
Defending Government Policies and Actions: Defending the legality and constitutionality of government policies, actions, and
legislations in courts when challenged.
Public Interest Litigation (PIL): Responding to public interest litigations filed in courts and representing the government's stance
on such matters.
Advising Government Departments: Providing legal guidance to various ministries and departments of the government on
legislative matters, contracts, and administrative decisions.

104/262
Assisting Law Officers: Assisting the Attorney General and other law officers in the discharge of their duties as required.

Que. 187 1. The Prime Minister's Office (PMO) of India is responsible for formulating national policies.
2. The PMO coordinates between various ministries and government departments to ensure smooth functioning of the
government.
Which of the following statements is/are correct?

1. 1 only
2. 2 only
3. All three
4. None

Correct Option - 2
The correct answer is option 2.
Key Points
PMO :The Prime Minister's Office (PMO) of India is the central office that assists the Prime Minister in carrying out his duties.

The PMO does not directly formulate national policies. That's the responsibility of the Cabinet and individual ministries.
The PMO plays a crucial role in coordinating between ministries and departments, ensuring smooth execution of policies and
government programs.

Here's a breakdown of its key features:


Function: Provides secretarial and administrative support to the Prime Minister.
Leadership: Headed by the Principal Secretary to the Prime Minister, currently Pramod Kumar Mishra.
Activities:

Coordinates with various ministries and government departments.


Oversees the functioning of the Anti-Corruption Unit.
Manages public grievances through its Public Wing.
Provides infrastructure and manpower support to the Prime Minister.

Location: South Block of the Secretariat Building, New Delhi.

Que. 188 The Rules of Procedure and Conduct of Business in Lok Sabha are essential for:

1. Defining the salary and benefits of Members of Parliament


2. Determining the eligibility criteria for contesting Lok Sabha elections
3. Ensuring orderly and productive functioning of the Lok Sabha
4. Specifying the powers and limitations of the judiciary

Correct Option - 3
The correct answer is option 3.
Key Points
Rules of Procedure and Conduct of Business in Lok Sabha :
The Lok Sabha, India's lower house of parliament, functions according to a set of rules called the "Rules of Procedure and Conduct of
Business in Lok Sabha". These rules govern various aspects including:
Functioning of the House: They establish procedures for things like oath-taking by new members, seating arrangements, and presiding
officers.
Conduct of Business: The rules lay out the process for introducing and debating legislation, asking questions of ministers, and raising
matters of public importance.
Member's Conduct: They establish guidelines for respectful behavior and participation in debates and proceedings.
Constitutional Basis:
Article 118 of the Indian Constitution empowers each house of Parliament (Lok Sabha and Rajya Sabha) to make rules for regulating its
procedure and conduct of business.
Amending the Rules:

The Speaker of the Lok Sabha has the authority to amend the rules with recommendations from a dedicated Rules Committee.
Amendments require the approval of the House.

Importance of the Rules:

These rules are crucial for ensuring the orderly and productive functioning of the Lok Sabha.
They establish a framework for fair debate, effective legislation, and holding the government accountable.
The rules help maintain decorum and prevent disruptions during proceedings.

105/262
Que. 189 The Doha Programme of Action focuses on supporting development in which of the following countries?

1. Landlocked Developing Countries


2. Lower-Middle-Income Countries
3. Least Developed Countries
4. Small Island Developing States

Correct Option - 3
The correct answer is option 3.
Key Points
Doha Programme : The Doha Programme is the abbreviation for the Doha Programme of Action for the Least Developed Countries for
the Decade 2022-2031 (DPoA). It's a UN initiative that sets out a framework for international cooperation to support development in the
world's least developed countries (LDCs).

The DPoA was adopted in March 2022 at the Fifth United Nations Conference on Least Developed Countries (LDC5) held in New
York. It represents a renewed commitment by developed countries to assist LDCs in achieving sustainable development.

Key areas of focus for the next decade:

Investing in people in LDCs: This includes eradicating poverty, improving health and education outcomes, and empowering women
and girls.
Leveraging science, technology, and innovation: The DPoA recognizes the importance of science and technology in driving
development progress in LDCs.
Supporting structural transformation: This refers to helping LDCs diversify their economies and move away from a reliance on
primary commodities.
Enhancing international trade of LDCs and regional integration: The DPoA calls for increased market access for LDC products
and support for regional economic integration initiatives.
Addressing climate change, environmental degradation, and building resilience: The Programme acknowledges the particular
vulnerabilities of LDCs to climate change and environmental shocks.
Fostering peaceful and inclusive societies: The DPoA emphasizes the importance of good governance, human rights, and the rule of
law for sustainable development.

Que. 190 During Zero Hour in the Lok Sabha, Members of Parliament can raise issues of public importance:

1. Through pre-submitted written questions.


2. By submitting a motion one day in advance.
3. Without any prior notice, subject to Speaker's approval.
4. Only if the issue is related to the day's listed agenda.

Correct Option - 3
The correct answer is option 3.
Key Points
Zero Hour in Indian Parliament : Zero Hour is a unique feature of the Indian Parliament, specifically the Lok Sabha (lower house). It
provides a platform for Members of Parliament (MPs) to raise issues of urgent public importance.
Key points about Zero Hour:

Timing: It follows immediately after the Question Hour and before the day's planned agenda begins. Since Question Hour has a
variable duration, Zero Hour doesn't have a fixed time slot.
Raising Issues: Unlike Question Hour, which requires pre-submitted written questions, Zero Hour allows MPs to raise issues
without prior notice. However, they must inform the Speaker by 10 am on the day of the sitting.
Focus: The focus of Zero Hour is on urgent matters that require immediate attention from the government. These can be recent
events, pressing problems, or unforeseen situations demanding a response.
Duration: The duration of Zero Hour is limited. In the Lok Sabha, it's capped at 30 minutes. The Speaker allocates time to each MP,
typically around 3 minutes, to raise their concern.
Discretion of the Speaker: The Speaker has the discretion to decide which issues can be raised during Zero Hour. This helps
maintain order and ensures that only truly urgent matters are addressed.

Significance of Zero Hour:

Provides a crucial tool for MPs to highlight pressing issues and hold the government accountable for immediate action.
Allows for raising spontaneous concerns that may not have been anticipated beforehand.
Brings public attention to critical problems and sparks discussions within the Parliament.​

Que. 191 Consider the following statements


1. District Planning Committees are mandated by the Constitution of India.

106/262
2. DPCs consolidate plans prepared by urban and rural local bodies within the district.
3. DPCs have the final authority to approve development plans for the district.
How many of the above statements given above is/are correct?

1. Only one
2. Only two
3. All three
4. None

Correct Option - 2
The correct answer is option 2.
Key Points
District planning committee :
A district planning committee (DPC) is a committee created in India at the district level for planning at the district and below level.

Article 243ZD of the Constitution mandates the composition, functions, and other details of the DPC. Hence ,Statement 1 is correct.
DPCs consolidate plans prepared by Panchayats (rural local bodies) and municipalities (urban local bodies) within the district. Hence
,Statement 2 is correct.
DPCs recommend the draft development plan to the state government for approval. The state government has the final
authority. Hence ,Statement 3 is incorrect.

Additional Information
There is a DPC in each district which is responsible for consolidating the plans prepared by the Panchayats and the
municipalities in the district and preparing a draft development plan for the district as a whole .
The composition, functions and other details of the DPC are mandated by Article 243ZD of the Constitution of India.

Here's a breakdown of the DPC's roles and responsibilities:

Consolidates plans prepared by Panchayats and municipalities within the district.


Prepares a draft development plan for the entire district.
Considers priorities and needs of various stakeholders including elected representatives from Panchayats, municipalities, and the state
Legislative Assembly.
Recommends the draft development plan to the state government for approval.
The DPC plays a vital role in ensuring inclusive and balanced development at the district level in India.

Que. 192 The territorial nexus doctrine is primarily concerned with:

1. Determining the eligibility for government jobs.


2. Defining the fundamental rights of citizens.
3. Ascertaining the applicability of state laws beyond the state's borders.
4. Setting guidelines for interstate trade disputes.

Correct Option - 3
The correct answer is option 3.
Key Points
Doctrine of territorial nexus : The doctrine of territorial nexus is a legal principle applied in India to determine the applicability of a
state law beyond its geographical borders.
What it means:

Laws made by a state legislature are generally not applicable outside that state.
However, the doctrine allows a state law to have an extraterritorial operation if there's a sufficient connection (nexus) between the
state and the subject matter of the law.

Origin and Authority:

The doctrine derives its power from Article 245 of the Indian Constitution. This article empowers state legislatures to make laws
for the whole or any part of the state.

Applications:

This doctrine is particularly relevant in taxation matters. It helps determine whether a state can levy taxes on someone or something
located outside its territory.
Courts use the doctrine to assess if there's a strong enough link between the state and the taxable activity for the state law to be valid.

Examples:

A state can tax income earned within its borders even if the person resides outside the state. (Nexus: income generation within the
state)

107/262
A state might struggle to tax a property located in another state owned by a resident within its borders. (Weaker nexus)

Que. 193 Consider the following statements


1. Sessions Judges are the highest judicial authority at the district level in India.
2. Sessions Courts try all criminal offences, irrespective of the severity.
Which of the following statements are correct?

1. 1 only
2. 2 only
3. Both 1 and 2
4. None

Correct Option - 1
The correct answer is option 1.
Key Points
Sessions Judge : A Sessions Judge presides over a Sessions Court, which is the highest criminal court at the district level in India.

Sessions Courts are the highest court for criminal cases at the district level. Hence, Statement 1 is Correct.
Sessions Courts handle serious offences, not all criminal cases. Lower courts like Magistrates' Courts handle less serious
offences. Hence ,Statement 2 is Incorrect.

Sessions Court:
It's the court of first instance for trying serious offenses, meaning it conducts the initial trial.

These offenses typically carry punishments of:


Imprisonment for more than seven years
Life imprisonment
Death penalty

Sessions Judge:

Appointed by the High Court of the state.


Conducts trials for these serious offenses.
Hears appeals from lower courts within their district (in some cases).
May also handle additional responsibilities as assigned by the High Court.

Que. 194 The primary focus of the W20 is to:

1. Discuss social issues faced by women globally.


2. Promote policies that empower women economically.
3. Organize international conferences on gender equality.
4. Award recognition to individuals working for women's rights.

Correct Option - 2
The correct answer is option 2.
Key Points
W 20 : W20 likely refers to the Women 20 (W20), an official engagement group connected to the G20. The W20's mission is to advocate
for gender equality and women's economic empowerment.
Breakdown of W20:

Function: Works to ensure gender equality is a key focus of G20 discussions and recommendations.
Goal: Promotes policies that advance gender equality and empower women economically.
Established: 2015 during the Turkish G20 presidency.

India and W20:

India's Role: India actively participates in the W20 process, contributing to discussions and recommendations.
Focus During India's G20 Presidency (2023): If India held the G20 presidency in 2023 (as of today, March 27, 2024, it hasn't been
confirmed), the W20 under India's leadership would likely prioritize issues relevant to the Indian context, such as:
Bridging the gender digital divide.
Enhancing women's participation in the workforce, particularly in STEM fields.
Promoting financial inclusion for women.
Addressing challenges faced by women entrepreneurs.

Que. 195 The motto of the Munich Security Conference is:

108/262
1. Security through Strength
2. Peace through Dialogue
3. Cooperation for Prosperity
4. Diplomacy for Development

Correct Option - 2
The correct answer is option 2.
Key Points
Munich security conference : The Munich Security Conference (MSC) is the world's leading forum for discussing international security
issues. It's an annual event that brings together world leaders to address pressing security concerns.
Purpose: It provides a platform for diplomatic initiatives to address global security challenges.
Who attends: Heads of state and government, ministers, leaders of international organizations, industry representatives, media figures,
academics, and civil society.
Motto: "Peace through Dialogue"
Location: Munich, Bavaria, Germany
Chairman: Ambassador Christoph Heusgen (as of February 2024)

Que. 196 1. The International Health Regulations (2005) is a legally binding agreement that emphasizes measures proportionate to health
risks to prevent unnecessary disruptions to travel and trade.
2. IHR 2005 requires countries to report only confirmed outbreaks of serious infectious diseases to the WHO.
Which of the following is correct in the context of the above statements?

1. Only statement 1 is correct.


2. Only statement 2 is correct.
3. Both statements 1 and 2 are correct.
4. Neither statement 1 nor 2 is correct.

Correct Option - 1
The correct answer is option 1.
Key Points
International Health Regulations (2005 ) : The International Health Regulations (2005), also known as IHR 2005, is a significant
agreement adopted by the World Health Assembly in 2005. It's a legally binding international framework for 196 countries, including
WHO member states.

IHR 2005 is a binding agreement focusing on proportionate measures to minimize travel and trade disruptions. Hence ,Statement 1 is
correct
IHR 2005 mandates reporting of public health events with potential international concern, even if not confirmed outbreaks. Hence
,Statement 2 is incorrect.

Summary of the IHR 2005:


Goal: The main purpose is to prevent, protect against, control, and respond to the international spread of diseases. It emphasizes measures
that are proportionate to the health risks and avoids unnecessary disruptions to travel and trade.
Core functions for countries: IHR 2005 outlines specific actions for countries, including:

Developing and maintaining core capacities for surveillance and response at all levels (national, intermediate, and primary healthcare)
Establishing capacities at designated international airports, ports, and ground crossings
Reporting public health events of potential international concern to the WHO

Measures at points of entry: The IHR allows countries to take specific measures at ports, airports, and ground crossings to limit the spread
of health risks across borders.
The IHR 2005 has been crucial in recent disease outbreaks, including COVID-19, demonstrating the importance of international cooperation
in managing global health threats.

Que. 197 1. The Leader of the Opposition is a constitutional post in India.


2. The Leader of the Opposition is the leader of the party with the greatest numerical strength in the opposition.
Which of the following statements is correct in context of above statements ?

1. Both Statement 1 and Statement 2 are correct


2. Both Statement 1 and Statement 2 are incorrect
3. Statement 1 is correct; Statement 2 is incorrect
4. Statement 1 is incorrect; Statement 2 is correct

Correct Option - 4
The correct answer is option 4.
Key Points
109/262
Leader of opposition : The Leader of the Opposition in India plays a crucial role in the parliamentary system. They represent the largest
political party or coalition in the Lok Sabha (the lower house of the Indian Parliament) that is not in power.

The Leader of the Opposition is not a constitutional post in India. It is a statutory position established by the Salary and Allowances of
Leaders of Opposition in Parliament Act, 1977. Hence statement 1 is incorrect.
The Leader of the Opposition is indeed the leader of the party with the greatest numerical strength in the opposition, provided the party
has at least 10% of the members of the House (as established by convention, not by the Act). The Speaker of the Lok Sabha or the
Chairman of the Rajya Sabha recognizes the leader based on this criterion. Hence , statement 2 is correct.

Significance :

Critical Oversight: Serving as a check on the ruling government, the Leader of the Opposition ensures accountability by scrutinizing
government policies, actions, and decisions, thus fostering transparency in governance.
Participation in Committees: They and their party members have the right to participate in parliamentary committees, contributing to
the examination of legislative proposals, government department activities, and policy implementation.
Legislative Role: The Leader of the Opposition plays a pivotal role in the legislative process, leading debates, proposing amendments,
and ensuring thorough scrutiny of bills introduced by the government.
Alternative Governance: Providing an alternative voice, they present opposing policies, critique governmental actions, and offer
solutions to address various national issues, thus enriching the democratic discourse.
Symbol of Democracy: Their presence exemplifies the democratic ethos of India's parliamentary system, showcasing the importance
of diverse viewpoints and ensuring that all voices, including those critical of the government, are heard and respected.

Que. 198 In the context of Mutual Legal Assistance Treaties (MLATs), what role do designated central authorities play?

1. They oversee economic sanctions imposed on member states


2. They facilitate communication and coordination between signatory countries for legal assistance requests
3. They manage international trade agreements between participating nations
4. They provide financial aid to countries in need of legal assistance

Correct Option - 2
The correct answer is option 2.
Key Points
MLAT : MLAT is a mechanism whereby countries cooperate with one another to provide and obtain formal assistance in the
prevention, suppression, investigation and prosecution of crime to ensure that the criminals do not escape or sabotage the due process of
law for want of evidence available in different countries.

These treaties typically involve cooperation in areas such as obtaining evidence, serving legal documents, locating and extraditing
individuals, and freezing or forfeiting assets.

MLATs are essential tools in combating transnational crime, including terrorism, drug trafficking, money laundering, cybercrime,
and other forms of organized crime.

They provide a formal framework for countries to request and provide assistance in gathering evidence or taking other legal actions
that cross international borders.

The terms and conditions of MLATs can vary significantly between countries, and the process for making and responding to requests
typically involves designated central authorities within each signatory country.

Requests made under MLATs must generally comply with the laws and procedures of both the requesting and requested
countries.

Que. 199 Consider the following Supreme Court cases related to Basic Structure Doctrine:
1. Keshavanand Bharti Case
2. Golaknath Case
3. Shankari Prasad Case
4. Sajjan Singh Case
Arrange the cases in chronological order :

1. 1-2-4-3
2. 3-4-2-1
3. 4-3-2-1
4. 3-2-4-1

Correct Option - 2
The correct answer is option 2:- 3-4-2-1
Key Points

110/262
In the early years of Independence, the Supreme Court conceded absolute power to Parliament in amending the Constitution, as was
seen in the verdicts in Shankari Prasad case (1951) and Sajjan Singh case (1965).
In both the cases the court had ruled that the term “law” in Article 13 must be taken to mean rules or regulations made in exercise of
ordinary legislative power and not amendments to the Constitution made in exercise of constituent power under Article 368.
This means Parliament had the power to amend any part of the constitution including Fundamental rights.
Article 13(2) reads, "The State shall not make any law which takes away or abridges the right conferred by this Part (Part-III) and any
law made in contravention of this clause shall, to the extent of contravention, be void."
However, in the Golaknath case (1967), the Supreme Court held that Parliament could not amend Fundamental Rights, and this
power would be only with a Constituent Assembly.
The Court held that an amendment under Article 368 is "law" within the meaning of Article 13 of the Constitution and therefore, if
an amendment "takes away or abridges" a Fundamental Right conferred by Part III, it is void.

Keshavanand Bharti Case (1973):-


The Constitutional Bench in Kesavananda Bharati case ruled by a 7-6 verdict that Parliament could amend any part of the Constitution so
long as it did not alter or amend the basic structure or essential features of the Constitution. Hence, option 2 is correct.

Que. 200 Consider the following statements regarding Digital India Land Records Modernization Programme (DILRMP)
1. DILRMP is a centrally sponsered scheme with the aim to modernize the land records system in the country.
2. A Unique Land Parcel Identification Number (ULPIN) or Bhu-Aadhaar number has been assigned for a land parcel, which is a 14 digit
alphanumeric unique ID.
3. The Records of Rights have been transliterated in all the 22 scheduled languages mentioned in the Constitution.
Which of the following statements is/are correct?

1. 1 and 2 only
2. 2 and 3 only
3. 1 and 3 only
4. 1,2 and 3

Correct Option - 2
The correct answer is Option 2.
Key Points
The National Land Record Modernization Programme (NLRMP) was a centrally sponsored scheme that was launched by the
Government of India in 2008 with the aim to modernize the land records system in the country and implement the conclusive land-
titling system with title guarantee.
The NLRMP was later revamped and renamed as the Digital India Land Records Modernization Programme (DILRMP) in 2016
as a central sector scheme with 100% funding by the Centre. Hence Statement 1 is incorrect.

​Salient Features of DILRMP:-

A Unique Land Parcel Identification Number (ULPIN) or Bhu-Aadhaar number has been assigned for a land parcel, which is a 14
digit alphanumeric unique ID based on geo-coordinates which will serve as a pan-India number. Hence Statement 2 is correct.
A uniform system called National Generic Document Registration System (NGDRS) has been developed for addressing the
diversity prevailing across States with regards to registration of deeds/documents.
The Records of Rights have been transliterated in all the 22 scheduled languages mentioned in the Constitution to address the
problem of linguistic barriers in land governance in the country. Hence Statement 3 is incorrect.
The DILRMP scheme will also facilitate various services such as providing caste, income and domicile certificates, and online
information on crop profile, crop insurance and e-linkages to credit facilities/banks.
A comprehensive land record management system will also help resolve long pending arbitration cases and boundary-related
disputes amicably, thus reducing the burden on the judiciary and the administration.​

Que. 201 Consider the following statements regarding declaration of a national party:
1. It is ‘recognised’ in four or more states.
2. If its candidates have secured at least 6% of total valid votes in at least 4 states (in latest Lok Sabha or Assembly elections) and the party
has at least 4 MPs in the last Lok Sabha polls;
3. If it has won at least 2% of the total seats in the Lok Sabha from at least 3 states.
Which of the following statements is/are correct?

1. Only one statement correct.


2. Only two statements correct
3. Three statements correct
4. None correct

Correct Option - 3
111/262
The correct answer is Option 3- Three statements correct.
Key Points
The Election Commission of India (ECI) registers political parties for the purpose of elections and grants them recognition as
national or state parties on the basis of their poll performance.The other parties are simply declared as registered-unrecognised
parties.
As per the Representation of People Act 1951, Registered political parties, in course of time, can get recognition as 'State Party’ or
National Party’.

National Party:

As the name suggests, it has a nationwide presence as opposed to a regional party that is restricted to only a particular state or region.

Conditions for Declaring a Party ‘National’:

It is ‘recognised’ in four or more states; or


If its candidates have secured at least 6% of total valid votes in at least 4 states (in latest Lok Sabha or Assembly elections) and the
party has at least 4 MPs in the last Lok Sabha polls; or
If it has won at least 2% of the total seats in the Lok Sabha from at least 3 states.

State Party
A party is recognised as a state party in a state if any of the following conditions is fulfilled:

If it secures 6% of the valid votes polled in the state at a general election to the respective state legislative assembly (state LA) and
also, it wins 2 seats in the same state LA.
If it secures 6% of the total valid votes in the state at a general election to the LS; and also, it wins 1 seat in the LS from the same
state.
If it wins 3% of seats in the LA at a general election to the legislative assembly of the state concerned or 3 seats in the assembly
(whichever is more).
If it wins 1 seat in the LS for every 25 seats or any fraction thereof allotted to the state at a general election to the LS from the state
concerned.
If it secures 8% of the total valid votes polled in the state at a General Election to the LS from the state or to the State LA.

Que. 202 Consider the following statement about India Justice Report:
1. It is an initiative of Tata Trusts in collaboration with Centre for Social Justice.
2. It was first published in 2018.
3. According the the recent report of 2022,Tamil Nadu has topped the ranking.
Which of the following statements is/are correct?

1. Only one statement correct


2. Two statements are correct
3. Three statements are correct.
4. None

Correct Option - 1
The correct answer is Option 1.
Key Points
India Justice Report (IJR):-

The IJR is an initiative of Tata Trusts in collaboration with Centre for Social Justice, Common Cause and Commonwealth Human
Rights Initiative among others. Hence Statement 1 is correct.
It was first published in 2019.Hence Statement 2 is incorrect.
It assesses the performance of states in terms of justice delivery, by considering several parameters such as police, judiciary,
prisons, and legal aid to assess the overall performance of each state.​
According to the India Justice Report (IJR) 2022, Karnataka has achieved the top rank among the 18 large and mid-sized states in
delivery of justice with populations over one crore.Tamil Nadu has ranked in the second position; Telangana third. Hence Statement
3 is incorrect.​

Additional Information
Key Findings of IJR :-
Ranking of Justice Delivery:

The list of 7 Small States with a population less than one crore each, was topped by Sikkim which was ranked second in 2020.
Sikkim has been followed by Arunachal Pradesh and Tripura. The State of Goa is at rank Seven which is the lowest.

Shortage of Judges:

The Indian judiciary is facing a severe shortage of judges and infrastructure, leading to rising pendency, increasing caseloads, and
declining case clearance rates (CCR) in lower courts.
112/262
As of December 2022, the High Courts were functioning with only 778 judges against a sanctioned strength of 1,108 judges.

Pendency:

The number of cases pending per judge is rising in most states over the last five years, while the sanctioned strength has remained the
same.
The average pendency in High Courts is highest in Uttar Pradesh (11.34 years) and West Bengal (9.9 years), while it is lowest in
Tripura (1 year), Sikkim (1.9 years), and Meghalaya (2.1 years).

Increased Caseload:

The caseload per judge has steadily increased, with the caseload per judge increasing in 22 states and Union Territories between 2018
and 2022.

Case Clearance Rate:

The CCR in High Courts improved by six percentage points (88.5% to 94.6%) between 2018-19 and 2022 but declined by 3.6 points in
lower courts (93% to 89.4%).
High Courts are increasingly clearing more cases annually than subordinate courts.
In 2018-19, only four High Courts had a CCR of 100% or more. In 2022, this more than double to 12 High Courts.

Court Halls:

Nationally, the number of court halls appears sufficient for the number of actual judges, but space will become a problem if all the
sanctioned posts are filled.
In August 2022, there were 21,014 court halls for the 24,631 sanctioned judges' posts, a shortfall of 14.7%.

Que. 203 Consider the following statement about Subordinate Courts in India :
1. The District Courts of India are established by the State governments in India.
2. Appointments of district judges in any State shall be made by the the Chief Justice of the concerned State High Court.
Which of the following statements is/are correct?

1. 1 only
2. 2 only
3. Both 1 and 2
4. Neither 1 nor 2

Correct Option - 1
The correct answer is option 1
Key Points
District Courts

The District Courts of India are established by the State governments in India for every district or for one or more districts together
taking into account the number of cases, population distribution in the district. Hence statement 1 is correct.
These courts are under administrative control of the High Court of the State to which the district concerned belongs.

Article 233 in Constitution of India

(1) Appointments of persons to be, and the posting and promotion of, district judges in any State shall be made by the
Governor of the State in consultation with the High Court exercising jurisdiction in relation to such State, Hence statement 2 is
incorrect.
(2) A person not already in the service of the Union or of the State shall only be eligible to be appointed a district judge if he has been
for not less than seven years an advocate or a pleader and is recommended by the High Court for appointment.

Additional Information
As per Article 234 Appointment of persons other than district judges to the judicial service of a State shall be made by the
Governor of the State in accordance with rules made by him in that behalf after consultation with the Slate Public Service
Commission and with the High Court exercising jurisdiction in relation to such State.
Recruitment to the cadre of district judge can be made from two sources, viz.,
promotion from the subordinate judiciary and
direct recruitment from the Bar.
In the matter of promotion from the subordinate judiciary, power is conferred on the Governor to give promotion in consultation with
the High Court exercising jurisdiction in relation to it.
In the matter of recruitment from the Bar, the appointment can be made on the recommendation of the High Court. Thus, the High
Court has a preponderating voice in the matter of recruitment to the cadre of district judge, whereas power of making appointments
vests in the Government.

113/262
Que. 204 Consider the following statements about Narco Analysis Test:
1.This test involves the intravenous administration of a drug (such as sodium pentothal, scopolamine and sodium amytal) that
causes the subject to enter into various stages of anaesthesia.
2. India stop the use of Narco Test since enactment of the Indian Constitution as it violates Article 20(3).
3.Any information or material that is subsequently discovered with the help of voluntarily administered narco test can be admitted, in
accordance with Section 27 of the Evidence Act, 1872 in the court.
Which of the following statements is/are correct?

1. 1 and 2 only
2. 3 only
3. 1 and 3 only
4. 1,2 and 3

Correct Option - 3
The correct option is 3
Key Points
Narco Analysis Test

The Narco analysis test is also known as Truth Serum Test. It involves the intravenous administration of a drug (such as sodium
pentothal, scopolamine and sodium amytal) that causes the subject to enter into various stages of anaesthesia. Hence statement 1 is
correct.
In India, Narco analysis tests was used in the 2002 Gujarat riots case, and the 26/11 Mumbai terror attack case. Hence statement
2 is incorrect.
Narco-analysis tests do not hold any constitutional legal validity because the person undergoing such tests is in the semi-conscious or
unconscious state of mind and is not in the position to understand the true nature of questions so posed by the authorities because of
which the results do not hold any evidentiary value and hence are not admissible in the court of law. But it does act an aid for further
investigation.

Any information or material that is subsequently discovered with the help of voluntarily administered narco test can be admitted, in
accordance with Section 27 of the Evidence Act, 1872 in the court. Hence statement 3 is correct.
Additional Information
Selvi vs State of Karnataka & Anr Case 2010:

The SC ruled on the legality and admissibility of narco tests establishing that the involuntary administration of narco or lie detector
tests constitutes an intrusion into an individual's "mental privacy."
The apex court held that narco tests violate the fundamental right against self-incrimination under Article 20(3) of the Constitution,
which states that no person accused of any offence shall be compelled to be a witness against himself.

D.K. Basu vs. State of West Bengal case, 1997:

The SC ruled that involuntary administration of the polygraph and narcos test will amount to cruel, inhuman, and degrading treatment
in the context of Article 21 or the Right to Life and Liberty.

Deception detection tests or DDTs

Deception detection tests or DDTs are scientific and psychiatric tests that are being used to detect lies or to gather some important
information.
DDTs include the use of Narco-Analysis, Polygraph Test, and Brain-mapping.

Que. 205 Consider the following statements about Article 142 of the Constitution of India:
1. Article 142 empowers the Supreme Court to pass any decree or order necessary for doing complete justice in any case or matter
pending before it.
2. Any decree so passed or order so made shall be enforceable throughout the territory of India in such manner as may be prescribed by or
under any law made by Parliament.
3. Article 142, was invoked by Supreme Court to issue Vishakha Guidlines, 1997.
Which of the following statements is/are correct?

1. Only One
2. Only Two
3. All of the Above
4. None of the Above

Correct Option - 3
The correct answer is Option 3
Key Points
Article 142​
114/262
“142. Enforcement of decrees and orders of Supreme Court and unless as to discovery, etc.-
(1) The Supreme Court in the exercise of its jurisdiction may pass such decree or make such order as is necessary for doing complete
justice in any cause or matter pending before it, and any decree so passed or orders so made shall be enforceable throughout the
territory of India in such manner as may be prescribed by or under any law made by Parliament and, until provision in that
behalf is so made, in such manner as the President may by order prescribe.
(2) Subject to the provisions of any law made in this behalf by Parliament, the Supreme Court shall, as respects the whole of the
territory of India, have all and every power to make any order for the purpose of securing the attendance of any person, the discovery
or production of any documents, or the investigation or punishment of any contempt of itself.” Hence statement 1 and 2 are correct.​
In Vishaka and Others v. State of Rajasthan, Court used Article 142 to lay down guidelines for the prevention of sexual harassment at
the workplace. Hence statement 3 is correct.

Additional Information
Supreme Court’s Power Under Article 142

These are SC’s plenary powers for ensuring complete justice and complement its other constitutional and statutory powers.
These powers are of wide amplitude and are supplementary in nature.
No independent proceeding or procedure is required before the court seeking the application of such powers.
The application of such powers depends upon the discretion of the apex court.
The application of these powers cannot be done while ignoring or overriding the existing statutory or constitutional provisions.
These constitutional powers cannot, in any way, be controlled by any statutory provisions.

Que. 206 Consider the following statements about Broadcasting Services Regulation Bill 2023 :
1.It also provides comprehensive accessibility guidelines for the Divyangjan community.
2.The Bill introduces a self-regulation regime, requiring broadcasters to establish Content Evaluation Committees (CECs) comprising
members from diverse social groups.
3.It proposes a three-Tier Regulatory Structure involving operators self-regulation, self-regulatory organizations for compliance and
grievance resolution and a Broadcast Advisory Council overseeing regulation implementation and making recommendations to the
government.
How many of the above statements are incorrect?

1. Only 1 and 2
2. Only 2 and 3
3. Only 1 and 3
4. None of the above

Correct Option - 4
The correct answer is Option 4.
Key Points
The Broadcasting Services (Regulation) Bill 2023:

The Bill mandates registration for all broadcasting entities, including cable, satellite, terrestrial, and internet-based services,
excluding certain authorized bodies.
It enforces adherence to Programme and Advertisement Codes for all forms of broadcasting services. These regulations extend to
online news platforms operating as a systematic business. Accessibility guidelines for persons with disabilities are introduced,
mandating measures such as audio descriptions, subtitles, and accessible applications. Hence Statement 1 is correct
The Bill introduces a self-regulation regime, requiring broadcasters to establish Content Evaluation Committees (CECs)
comprising members from diverse social groups. CECs will certify programs for airing, except for those specifically exempted by the
government. Hence Statement 2 is correct.
It proposes a three-Tier Regulatory Structure involving operators' self-regulation, self-regulatory organizations for compliance
and grievance resolution, and a Broadcast Advisory Council overseeing regulation implementation and making recommendations
to the government. Hence Statement 3 is correct.
The government can conduct inspections without prior notice and seize equipment if violations are suspected. Penalties for non-
compliance range from show removal to cancellation of registration, depending on the seriousness of the violation.


Additional Information
In the realm of traditional media, cinema releases are regulated by the Central Board of Film Certification (CBFC) and the
Cinematographic Act 1952. Television content is overseen by the Broadcasting Content Complaints Council (BCCC). However,
OTT platforms, despite their widespread influence, do not have a dedicated regulatory body.
Telecom Disputes Settlement Appellate Tribunal (TDSAT) has ruled that Over The Top (OTT) platforms like Hotstar are not in the
jurisdiction of the Telecom Regulatory Authority of India (TRAI) and are governed by the Information Technology Rules, 2021,
notified by the Ministry of Electronics and Information Technology (MeitY).

Que. 207 Consider the following statements regarding Right to remain silent:
1.The Constitution grants the right to remain silent: The court emphasised that the Constitution of India gives individuals the right
against self-incrimination, stating that no one can be forced to be a witness against themselves.

115/262
2.Article 20(3): It states that “no person accused of any offence shall be compelled to be a witness against himself.” The protection of this
clause is limited only to criminal proceedings. Therefore, under civil proceedings, a person cannot refuse to answer a question using the
defence of Article 20(3).
3.In Selvi vs State of Karnataka (2010) The Supreme Court held that a narcoanalysis test without the consent of the accused would amount to
violation of the right against self-incrimination.
Which of following statements are correct?

1. Only 1 and 2
2. Only 1 and 3
3. Only 2 and 3
4. All of the above

Correct Option - 4
The correct answer is Option 4
Key Points
Right to Silence:

The Constitution grants the right to remain silent: The court emphasized that the Constitution of India gives individuals the right
against self-incrimination, stating that no one can be forced to be a witness against themselves. Hence Statement 1 is correct
Cooperation does not mean confession: The court clarified that cooperation with an investigation should not be seen as an admission of
guilt.
Remaining silent cannot be considered non-cooperation, as individuals have the right to choose not to speak.
Prosecution’s burden of proof: It is the responsibility of the prosecution to prove the accused’s guilt beyond reasonable doubt.
Refusing confession doesn’t limit freedom: The court stated that an accused person cannot be deprived of their freedom solely
because they have not admitted to the alleged crimes.

Constitutional Provisions:

Article 20: It grants protection against arbitrary and excessive punishment to an accused person, whether citizen or foreigner or legal
person like a company or a corporation. It contains three provisions in that direction:

1.No ex-post-facto law, No double jeopardy, No self-incrimination.


2.No self-incrimination: No person accused of any offense shall be compelled to be a witness against himself.

Article 20(3): It states that “no person accused of any offence shall be compelled to be a witness against himself.”

The protection of this clause is limited only to criminal proceedings. Therefore, under civil proceedings, a person cannot refuse to
answer a question using the defence of Article 20(3). Hence Statement 2 is correct.
Supreme Court Rulings:

The State of Bombay versus Kathi Kalu Oghad (1961): The Supreme Court ruled that obtaining photographs, fingerprints,
signatures, and thumb impressions would not violate the right against self-incrimination of an accused.
Ritesh Sinha versus State of Uttar Pradesh (2019): The Supreme Court in its ruling broadened the parameters of handwriting
samples to include voice samples, adding that this would not violate the right against self-incrimination.
Selvi v State of Karnataka (2010): The Supreme Court held that a narcoanalysis test without the consent of the accused would
amount to violation of the right against self-incrimination. Hence Statement 3 is correct.

Que. 208 The Centre has clarified about the jurisdiction of Ministry of Corporate Affairs (MCA) and Ministry of Electronics and
information Technology (MeitY). Consider the following statements.
1. Ministry of Corporate Affairs (MCA) would be the nodal Ministry to look into all the sector-specific & technical issues.
2. Ministry of Information Technology (MeitY) will look into competition issues in the digital market.
3. MCA appointed an inter ministerial panel Committee on Digital Competition law (CDCL) to look into matters involving both the
ministries.
How many of the above statements are correct?

1. Only 1 and 2
2. Only 3
3. Only 2 and 3
4. All of the above

Correct Option - 2
The correct answer is Option 2.
Key Points
Background

There was frequent turf war between the Ministry of Corporate Affairs (MCA) and the Ministry of Electronics and Information
Technology (Meity) over regulation of the digital markets, with both having jurisdiction because the sector involves both technology

116/262
matters as well as competition issues.
Centre has now made it clear that Ministry of Corporate Affairs (MCA) would be the nodal Ministry to look into all the
“competition issues in the digital market” and the Ministry of Information Technology (MeitY) will look into sector-specific &
technical issues. Statement 1 and Statement 2 are incorrect.
Clarity of roles and oversight areas is critical as it comes at a time when the Government is working towards introducing several pieces
of legislations including Digital India Act and Digital Competition law for regulating various facets of the booming digital market in
India.

Additional Information
Significance:

Digital India Act by MeitY will not have provisions around competition issues of technology or gatekeeper platforms.
Digital Competition law would as planned look to bring digital gatekeepers including AI platforms under an ex-ante framework to be
introduced through the legislation.
MCA appointed an inter ministerial panel Committee on Digital Competition law (CDCL) to look into matters involving both the
ministries. Hence Statement 3 is correct.
CDCL was already tasked to examine the need for a separate legal framework for ex-ante regulation of Systemically Important
Digital Intermediaries (SIDIs)
The CDCL is in the last lap of firming up a draft of the Digital Competition Law.

Que. 209 Consider the following statements:


1.As per the Seventh Schedule of the Constitution, local elections are a state subject.
2. Some State Election commission uses the Election Commission voters list as the basis for the preparation and revision of rolls for
municipality and panchayat elections.
3. Some states have their own electoral rolls and do not adopt Election Commission roll for local body polls.
4. India has Common Electoral Roll for Lok Sabha, Vidhan Sabha and Local Bodies like Panchayat and Municipalities.
Which of the above statements is/are correct?

1. 1 only
2. 1 and 2 only
3. 2,3 and 4only
4. 1,2 and 3 only

Correct Option - 4
The correct answer is option 4
Key Points
Electoral Roll​

An electoral roll is a list of people who have enrolled to vote for particular elections in a particular jurisdiction.
As per the Seventh Schedule of the Constitution, local elections are a state subject. Hence statement 1 is correct.
The superintendence, direction and control of the preparation of electoral rolls for, and the conduct of, all elections to the
Municipalities shall be vested in the State Election Commission.
Some State Election commission uses the Election Commission voters list as the basis for the preparation and revision of rolls
for municipality and panchayat elections. Hence statement 2 is correct.
Some states have their own electoral rolls and do not adopt Election Commission roll for local body polls. Hence statement 3 is
correct.
Article 234 (1) The superintendence, direction and control of the preparation of the electoral rolls for, and the conduct of, all
elections to Parliament and to the Legislature of every State and of elections to the offices of President and Vice-President held
under this Constitution shall be vested in a Commission (referred to in this Constitution as the Election Commission). Hence
Statement 4 is incorrect.

Additional Information
Common Electoral Roll

Under the Common Electoral Roll, only one voter list will be used for Lok Sabha, Vidhan Sabha and other elections

Que. 210 Consider the following statements regarding Nari Shakti Vandan Adhiniyam,2023:
1. It provides for half of reservation for women in legislature.
2. It provides for reservation in the seats of Lok Sabha and Rajya Sabha.
3. There is a provision for rotation of seats after each delimitation.
How many of the above statements is/are correct?

1. Only one
2. Only two
3. All three
4. None
117/262
Correct Option - 1
The correct option is Option 1.
Key Points
Nari Shakti Vandan Adhiniyam (108th Constitutional Amendment) Bill:-

The bill reserves one-third of the seats in Lok Sabha,State legislative assemblies and the Delhi assembly. Hence, Statement 1 is
incorrect.
This will also apply to the seats reserved for SCs (Scheduled Castes) and STs (Scheduled Tribes) in Lok Sabha and State Legislatures.
Hence, Statement 2 is incorrect.
The Bill provided for inserting Article 330A to the constitution, which borrows from the provisions of Article 330, which provides for
reservation of seats to SCs/STs in the Lok Sabha.
The reservation will be effective after the census conducted after the commencement of this Bill has been published. Based on the
census, delimitation will be undertaken to reserve seats for women.
The reservation will be provided for a period of 15 years. However, it shall continue till such date as determined by a law made by
Parliament.
Seats reserved for women will be rotated after each delimitation, as determined by a law made by Parliament.Hence, Statement 3 is
correct.

Que. 211 Consider the following statements regarding Simultaneous Elections:-


1. It refers to the idea of holding elections for the Lok Sabha, State legislative assemblies, and local bodies like across the country
at the same time.
2. It has never happened before in Indian elections.
3. Recently a committee has been formed to study the viability of simultaneous elections.
Which of the above statements is/are correct?

1. Only 1 and 2
2. Only 2 and 3
3. Only 1 and 3
4. 1,2 and 3

Correct Option - 3
The correct answer is Option 3.
Key Points
Simultaneous Elections:-

It refers to the idea of holding elections for the Lok Sabha (the lower house of Parliament), State legislative assemblies, and local
bodies like municipalities and panchayats across the country at the same time.
This concept proposes to synchronize the electoral cycles of these different tiers of governance, aiming to conduct all elections
together, ideally once every five years.
The initial four general elections in India synchronized Lok Sabha and State Assembly elections till 1967.Presently, Lok Sabha
elections align with Assembly polls in Andhra Pradesh, Odisha, Arunachal Pradesh, and Sikkim.
In late October 2023, the panel under Former President Ramnath Kovind tasked to check the feasibility of simultaneous elections
had an interaction with the Law Commission to discuss the roadmap to synchronize Parliamentary and Assembly elections by
2029.

Additional Information
Benefits of Simultaneous Elections:

Resource Efficiency: Conducting elections at various levels requires significant financial resources. Synchronizing elections would
consolidate these expenses, leading to substantial cost savings for the government.
Optimized Administration: Simultaneous elections would streamline the deployment of security forces and administrative staff,
minimizing disruptions caused by election-related duties and allowing officials to focus more consistently on governance and
development.
Continuity in Policies: With elections occurring simultaneously, there would be fewer interruptions in policy implementation due to
the Model Code of Conduct, ensuring more sustained and consistent governance.
Enhanced Voter Turnout: Reducing the frequency of elections might combat voter fatigue and increase voter participation, leading to
more representative outcomes and increased legitimacy for elected representatives.
Increased Accountability: When voters cast their ballots for various levels of governance together, politicians are held accountable
for their actions across different tiers, fostering a more comprehensive accountability structure.
Reduced Polarization: Simultaneous elections could potentially diminish the influence of regional, caste-based, or communal politics
by bringing national issues to the forefront, thus encouraging more inclusive campaigning and policy-making.

Associated Challenges:

Constitutional Amendments: Synchronizing elections requires amendments to various constitutional articles.

118/262
Changes in tenure provisions, dissolution of legislative bodies, and aligning different election cycles pose substantial legal challenges.
For example, Articles such as 83(2), 85(2), 172(1), and 174(2) govern the duration and dissolution of the Lok Sabha and State
Assemblies, allowing premature dissolution under certain circumstances, which would need repeal for simultaneous elections.

Federalism Concerns: India's federal structure involves multiple states with varying political landscapes.

Any move towards simultaneous elections needs widespread consensus among states, which may have differing political agendas.
Also, linking general and local body elections faces hurdles due to local governance being a state subject, requiring changes in diverse
state laws (56 legal provisions from 28 states' Panchayati Raj Acts and Municipal Acts).

Technology and Infrastructure: Upgrading technological infrastructure, such as electronic voting machines (EVMs) and Voter Verifiable
Paper Audit Trail Machines (VVPATs), on a large scale poses challenges in procurement, maintenance, and ensuring reliability.
By-elections and Legislative Councils: Synchronizing all elections might exclude by-elections and elections to legislative councils, leading
to potential gaps in representation and governance.
Diverse Political Landscapes: India's multi-party system involves diverse political ideologies and regional priorities.Simultaneous elections
may overlook regional issues and dilute the representation of smaller or regional parties.

Que. 212 Which of the following can be possible impacts of Artificial Intelligence in elections?
1. Increased and Effective Voter Engagement
2. Influence through Deceptive Content
3. Promoting inclusiveness
4. Microtargeting and privacy concerns

1. 1 and 2 only
2. 2 and 3 only
3. 2 and 4 only
4. All of the above

Correct Option - 4
The correct answer is Option 4.
Key Points
Artificial Intelligence:

It refers to the simulation of human intelligence in machines that are programmed to think like humans and mimic their actions.
It includes technologies like machine learning, pattern recognition, big data, self algorithm etc.

Positive impact of AI in elections:-

Increased and Effective Voter Engagement: By spreading awareness through social platforms in education campaigns, AI can help
the voters to understand the issues and candidates, which can consequently result in increased engagement and inform voters more
effectively.Hence, Statement 1 is correct.
The ECI can use social media to disseminate relevant information like important dates for the new voter registration, date and time
of voting, etc. It can also encourage first-time voters to participate in the democratic process.
Promoting Inclusiveness: With the help of AI-based Apps like Bhashini, the information can be made available in multiple Indian
languages. This will be helpful for the deprived sections of the society.AI technologies can help voters with disabilities.Hence,
Statement 3 is correct.
Election Transparency and Security: AI can help in the implementation of transparent advertising policies, rolling out content labels,
and restricting election-related queries to combat misinformation.
To Streamline Electoral Processes: AI can automate various associated tasks, from voter registration to tallying and can lead to more
efficient electoral processes, reduce human error and provide faster, more reliable results.
Strengthened Democracy: The ECI is also making optimum use of the technology with options like online voter registration and
releasing the voter’s list on the website. They can use AI and Social Media to create awareness and broader reach and address
grievances.
Cost-Effective: Generative AI can create campaign materials at a significantly lower cost and with greater efficiency.AI allows
marketers and advertisers to automate and optimize advertising campaigns, saving time and resources.

Challenges Associated with AI in elections:-

Misinformation & Disinformation: Deepfakes and other AI-generated content can create hyper-realistic digital falsification and can
potentially be used to damage reputations, fabricate evidence, and undermine trust in democratic institutions.
Supportive Studies: A new study published in PNAS Nexus predicts that disinformation campaigns will increasingly use generative
AI to propagate election falsehoods and will help spread toxic content across social media platforms almost daily in 2024.
The World Economic Forum’s Global Risks Perception Survey ranks misinformation and disinformation among the top 10 risks,
with easy-to-use interfaces of large-scale AI models enabling a boom in false information and “synthetic” content.

119/262
Influence through Deceptive Content: The hyper-realistic deep fakes of pictures, audio, or video could influence voters powerfully
before being fact-checked. Hence, Statement 2 is correct.
Disinformation: It is false information deliberately spread to deceive people.
Misinformation: It is incorrect or misleading information. It differs from disinformation, which is deliberately deceptive and
propagated information.
Visual Tools: Generative AI companies with the most popular visual tools prohibit users from creating “misleading” images. However,
researchers with the British nonprofit Centre for Countering Digital Hate (CCDH) succeeded in making deceptive election-related
images more than 40% of the time.
Social Media Amplification: Social media companies such as Facebook and Twitter compound the influence and misinformation
risks, which significantly cut their fact-checking and election integrity teams.
Microtargeting: through micro-targeting techniques, AI algorithms can be used to manipulate voter preferences and undermine the
fairness of elections by influencing voters.Hence, Statement 4 is correct.
Biasness: AI systems could exhibit bias due to the data they were trained on.Example: This could make the Cambridge Analytica
scandal appear microscopic.
Privacy Concerns: The main privacy concerns surrounding AI is the potential for data breaches and unauthorized access to personal
information. With so much data being collected and processed, there is a risk that it could fall into the wrong hands, either through
hacking or other security breaches.
Erosion of Trust: The mere existence of AI-generated content can foster a general atmosphere of distrust, where people question the
authenticity of all information. This phenomenon is known as the liar’s dividend.
No Specific Law: India lacks specific laws to address deepfakes and AI-related crimes, but provisions under many legislations could
offer civil and criminal relief. Example: Section 66E of the Information Technology Act, 2000 (IT Act) is applicable in cases of
deepfake crimes.

Que. 213 The Cable Television Networks (Regulation) Act, 1995 AIMS to:

1. Promote exclusive rights for broadcasting companies to telecast specific content


2. Facilitate the unfettered operation of cable television networks without any regulations.
3. Ensure adherence to a code for program content and advertisements on cable television.
4. Grant complete autonomy to cable operators in determining subscription fees.

Correct Option - 3
The correct answer is option 3.
Explanation :
Cable Television Networks (Regulation) Act, 1995 : The Cable Television Networks (Regulation) Act, 1995, is a key piece of legislation in
India that governs the operation of cable television networks in the country.
The Cable Television Networks (Regulation) Act, 1995, is a key piece of legislation in India that governs the operation of cable television
networks in the country. Here's a breakdown of its key aspects:

Purpose:
To regulate the operation of cable television networks.
To ensure the content broadcasted adheres to certain standards.
To protect the interests of consumers.

Key Provisions:
Registration: Cable operators need to register with the government before starting operations.
Programme and Advertisement Code: The Act prescribes a code for program content and advertising to ensure responsible
broadcasting. This might include restrictions on violence, obscenity, and misleading advertisements.

Que. 214 1. Special and Local Laws (SLLs) are a category of criminal laws enacted by the central government of India to address specific
regional issues.
2. A significant portion of cognizable offenses in India are registered under SLLs, highlighting their importance in the criminal justice
system.
Which of the statements given above are correct?

1. 1 only
2. 2 only
3. All three
4. None

Correct Option - 1
The correct answer is option 1.
Explanation:
Special and Local Laws (SLLs) : In India, Special and Local Laws (SLLs) are a category of laws distinct from the Indian Penal Code (IPC).
What are SLLs?

SLLs are laws enacted by individual state governments to address specific issues or concerns relevant to their particular region or
community.
They are distinct from the IPC, which applies uniformly throughout India and covers a broader range of criminal offenses.
120/262
Examples of SLLs:

The Excise Act (regulates sale and consumption of alcohol)


The Gambling Act (prohibits or regulates gambling activities)
The Cattle Trespass Act (deals with offenses related to stray cattle)

Significance of SLLs:

SLLs play a crucial role in the Indian criminal justice system, encompassing a significant portion of cognizable offenses (crimes for
which police can arrest without a warrant).
They allow states to address unique cultural, social, or environmental concerns within their jurisdiction.

Criticisms of SLLs:

Some argue that the existence of numerous SLLs creates a complex legal landscape and can lead to confusion or inconsistencies.
There are concerns that certain SLLs might be outdated or infringe upon individual liberties.

Que. 215 Consider the following statements


1. The National Cadet Corps (NCC) was established in India by a British Act in the early 20th century.
2. The NCC curriculum presently emphasizes social service activities alongside military training.
3. NCC participation is mandatory for all students enrolled in higher secondary schools and colleges in India.
How many of the statements given above is/are correct?

1. Only one
2. Only two
3. All three
4. None

Correct Option - 2
The correct answer is option 2.
Key Points
National Cadet Corps (NCC): The National Cadet Corps (NCC) is the youth wing of the Indian Armed Forces, playing a crucial role in
shaping the lives of young Indians.

The NCC has roots in the University Corps formed under the Indian Defence Act of 1917. Hence ,Statement 1 is Correct.
Community development and social service are part of the NCC syllabus since 1952. Hence ,Statement 2 is Correct.
NCC participation is voluntary for students. Hence Statement 3 is Incorrect.

History:
Origin: The NCC can trace its roots back to the 'University Corps' established in 1917 under the Indian Defence Act. Its aim was to address
the shortage of personnel in the British Indian Army.
Formation: Post-independence, the National Cadet Corps Act of 1948 officially established the NCC on July 15, 1948.
Evolution: The NCC has undergone several changes since its inception.
1949: The Girls Division was formed to provide equal opportunities for girls in schools and colleges.
1950: The NCC gained an inter-service image with the addition of the Air Wing.
1952: The Naval Wing was established.
Focus Shift: Following the 1965 and 1971 wars, the NCC curriculum's emphasis shifted from being a second line of defense to developing
leadership and officer-like qualities.
Social Service: At the behest of Pandit Jawaharlal Nehru, community development and social service activities were incorporated into the
NCC syllabus in 1952.
Compulsory Service: In 1963, NCC training became compulsory due to the requirement for national defense personnel. This was later
revoked.

Present Day NCC:

Voluntary: Today, NCC participation is voluntary for school and college students.
Tri-Services: It comprises three divisions - Army, Navy, and Air Force. Each division focuses on specific skills and knowledge
relevant to their respective branches.
Motto: The NCC motto, "Unity and Discipline" (Ekta aur Anushasan), reflects its core values.

Que. 216 1. The Fast Track Special Courts (FTSCs) Scheme is aimed at expediting trials for all criminal offenses in India.
2. The FTSCs are funded entirely by the central government.
Which of the following is correct in the context of the above statements?

1. Both Statement I and Statement II are correct.


2. Both Statement I and Statement II are incorrect.

121/262
3. Statement I is correct; Statement II is incorrect.
4. Statement I is incorrect; Statement II is correct.

Correct Option - 3
The correct answer is option 3.
Key Points
Fast Track Special Courts (FTSCs) Scheme : The Fast Track Special Courts (FTSCs) Scheme is a program by the Indian government
to expedite trials for cases related to sexual offenses, particularly rape and violations under the Protection of Children from Sexual
Offences Act (POCSO Act).

FTSCs are specifically designed for cases related to sexual offenses, particularly rape and POCSO Act violations, not all criminal
offenses. Hence, Statement 1 is correct
FTSCs are a Centrally Sponsored Scheme, meaning the central government provides funds but the implementation is by state
governments. Hence, Statement 2 is correct .​

Objectives
Ensure speedy justice for victims of sexual assault
Reduce pendency of cases
Discourage crimes against women and children

Implementation
Launched in October 2019 by the Department of Justice
Centrally Sponsored Scheme - funded by the central government and implemented by state governments
As of January 2024, 755 FTSCs including 410 exclusive POCSO Courts are functional across India

Funding
₹200 crore allocated for the Financial Year 2023-24

Monitoring
Department of Justice has created an online monitoring framework
Regular review meetings with state functionaries

Achievements
Over 221,000 cases disposed of since inception

Recent Developments
The scheme was initially launched for one year and has been extended subsequently
In August 2023, the cabinet approved the continuation of FTSCs for three more years till 2026

Que. 217 Gram Manchitra is a:

1. Social media platform for rural communities


2. Financial inclusion scheme for Gram Panchayats
3. Geo-spatial information system for rural development planning
4. Skill development program for rural youth

Correct Option - 3
The correct answer is option 3.
Key Points
Gram Manchitra : Gram Manchitra is a Geographic Information System (GIS) application launched by the Ministry of Panchayati
Raj in India.Gram Manchitra's primary goal is to empower Gram Panchayats (village councils) and enhance spatial planning efficiency at
the grassroots level. It accomplishes this by leveraging the capabilities of geo-spatial technology.
Key Features of Gram Manchitra
Unified Geo-Spatial Platform: Gram Manchitra provides a single platform for Gram Panchayats to visualize developmental projects and
activities. This fosters transparency and informed decision-making.
Sector-Wise Planning: It enables Gram Panchayats to plan and execute developmental works across different sectors, like agriculture,
education, healthcare, etc. This ensures a holistic approach to rural development.
Development Plan Tools: Gram Manchitra offers various tools that utilize GIS technology to aid Gram Panchayat officials in formulating
realistic and achievable development plans. These tools include:
Identifying potential sites for development projects
Tracking assets
Estimating project costs
Assessing the impact of projects

Overall, Gram Manchitra is a valuable tool for Gram Panchayats to improve rural development planning and implementation in India.

Que. 218 Consider the following statements

122/262
1. Tribunals in India are established solely under Article 323 of the Indian Constitution.
2. Tribunals are meant to provide a faster and more specialized alternative to the regular court system for specific disputes.
3. All tribunals in India are presided over by retired judges.
How many of the statements given above is /are correct ?

1. Only one
2. Only two
3. All three
4. None

Correct Option - 1
The correct answer is option 1.
Key Points
India's tribunal system : India's tribunal system consists of quasi-judicial bodies established to provide an alternative to the traditional
court system for resolving disputes related to specific matters like administration, taxation, environment, and securities. These tribunals aim
to deliver faster, more cost-effective, and specialized dispute resolution mechanisms compared to regular courts.

While Article 323 empowers Parliament to establish tribunals, there are two categories: those under Article 323A (Centre-State
disputes) and those under Article 323B (other specific matters) established by Parliament or State Legislatures. Hence ,Statement 1
is Incorrect.
Speedier and more specialized dispute resolution is a key objective of the tribunal system. Hence ,Statement 2 is Correct.
While presiding officers may be retired judges, they can also be experts in the relevant field. Hence ,Statement 3 is Incorrect.

key aspects of the tribunal system in India:


Establishment: Tribunals are instituted by Acts of Parliament or State Legislatures under Article 323 of the Indian Constitution. There are
two main categories:
Tribunals under Article 323A: Established by Parliament for service matters and disputes related to Centre-State relations.
Tribunals under Article 323B: Established by Parliament or State Legislatures for other specific matters.
Functions: Tribunals perform various functions, including:
Adjudicating disputes between parties.
Determining legal rights and obligations.
Reviewing and revising administrative decisions.
Imposing penalties for violations.
Composition: Tribunals typically consist of a presiding officer, who may be a retired judge or an expert in the relevant field, along with
technical members.
Advantages: The tribunal system offers several advantages over the regular court system:
Speedier resolution: Tribunals are designed to handle cases faster due to simpler procedures and fewer formalities.
Subject-matter expertise: Tribunal members possess specialized knowledge in the areas they handle, leading to more informed
decisions.
Reduced costs: Tribunal proceedings are generally less expensive than regular court cases.

Que. 219 Consider the following statements about dual citizenship and India:
1. India allows its citizens to hold dual citizenship with other countries.
2. The Overseas Citizenship of India (OCI) grants full citizenship rights to people of Indian origin.
Which of the following statements is/are correct?

1. 1 only
2. 2 only
3. Both 1 and 2
4. Neither 1 and 2

Correct Option - 4
The correct answer is option 4.
Key Points
Dual citizenship : Dual citizenship, also known as multiple citizenship, allows a person to be a legal citizen of two countries at the same
time. This means they have the rights and responsibilities of both citizenships.

India currently does not allow dual citizenship. Hence ,Statement 1 is Incorrect
OCI is not full citizenship and does not grant all rights and responsibilities of an Indian citizen. Hence, Statement 2 is Incorrect.

Dual citizenship in India:


India currently does not allow dual citizenship. This means you cannot hold Indian citizenship and citizenship of another country at the
same time. This is according to the Indian Constitution.

However, there is an alternative program called the Overseas Citizenship of India (OCI). This is not technically dual citizenship, but it
offers some benefits to people of Indian origin who are citizens of other countries.

Here's a quick breakdown of OCI:

123/262
Eligibility: You may be eligible for OCI if you are a person of Indian origin, and your country allows dual citizenship. People with
heritage from Pakistan or Bangladesh are not eligible.
Benefits: OCI grants you a lifelong visa for India, along with other privileges like easier business and property acquisition in India.
Not Dual Citizenship: It's important to remember that OCI is not the same as dual citizenship. You won't have the same rights and
responsibilities as an Indian citizen.

Que. 220 The Supreme Court Legal Services Committee (SLSC) is entrusted to offer free legal aid to ensure

1. Faster resolution of commercial disputes


2. Easier access to justice for underprivileged sections
3. Enhanced judicial review of government policies
4. Streamlined appointment of judges in High Courts

Correct Option - 2
The correct answer is option 2.
Key Points
Supreme Court Legal Services Committee (SLSC) :The Supreme Court Legal Services Committee (SLSC) is an important committee that
works to provide free and competent legal services to the weaker sections of society in cases that fall under the jurisdiction of the Supreme
Court of India.
Function of SCLSC

The SCLSC is constituted under Section 3A of the Legal Services Authorities Act, 1987. It works to ensure that everyone has access to
justice, regardless of their financial background. The committee provides a variety of services, including:
Legal advice and counseling
Representation in court
Drafting of legal documents
Assistance in filing legal claims

Who can avail the services of SCLSC?

The services of the SCLSC are available to people who meet the following criteria:
People who belong to Scheduled Castes and Scheduled Tribes
Women
Children
Victims of trafficking
Persons with disabilities
People living below the poverty line
Members of Parliament and State Legislatures in custody
Others as notified by the Central Government

Que. 221 The Social Audit Advisory Body (SAAB) is primarily concerned with:

1. Financial audits of government departments


2. Environmental impact assessment of development projects
3. Transparency and effectiveness of social welfare schemes
4. Standardization of technical education courses

Correct Option - 3
The correct answer is option 3.
Key Points
Social Audit Advisory Body (SAAB) : The Social Audit Advisory Body (SAAB) is a recent initiative in India established in 2024 under the
Ministry of Social Justice & Empowerment. It functions as an advisory body to guide the Ministry in institutionalizing social audits for
its various schemes.
Breakdown of SAAB:
Purpose: To improve the effectiveness and transparency of social welfare programs by ensuring proper implementation and utilization of
allocated funds.
Focus: Primarily on schemes managed by the Ministry of Social Justice & Empowerment.
Function: Advises the Ministry on methodologies, capacity building, and best practices for social audits.
Significance: Enhances accountability and empowers beneficiaries to track the progress of schemes meant to uplift them.

It's important to note that SAAB is a new initiative, and its long-term impact on social welfare programs in India remains to be seen.

Que. 222 Consider the following statements about Doha Political Declaration:
1. It focuses on implementing the Doha Programme of Action (DPoA), the 10-year plan to put the world’s 26 most vulnerable
countries back on track to achieving the Sustainable Development Goals (SDG).
2. DPoA (2022-2031) consisted of Six Key Focus Areas.

124/262
3. Doha Programme of Action (DPoA) consists of key deliverables which include food stockholding, an online university, an international
investment support centre and a sustainable graduation support facility.
4. Several participants also ensured financial support to the tune of $470 million to protect biodiversity and to counter malnutrition prevailing
in LDCs.
How many of the above statements are correct?

1. Only one
2. Only two
3. Only three
4. All four

Correct Option - 3
The correct answer is Option 3.
Key Points
Doha Programme of Action:

It focuses on implementing the Doha Programme of Action (DPoA), the 10-year plan to put the world’s 46 most vulnerable countries
back on track to achieving the Sustainable Development Goals (SDG). Hence statement 1 is incorrect.
DPoA for the Decade (2022-2031) was agreed upon during the first part of the LDC5 conference in March 2022 in New York, the US.
DPoA (2022-2031) consisted of Six Key Focus Areas. Hence statement 2 is correct.

1.Eradicating Poverty
2.Leveraging the potential of science and technology to fight against multidimensional vulnerabilities and to achieve the SDGs
3.Addressing Climate Change
4.Environmental Degradation
5.Recovering from Covid-19
6.Building resilience against future shocks for risk-informed sustainable development.

Doha Programme of Action (DPoA) consists of key deliverables which include food stockholding, an online university, an
international investment support centre and a sustainable graduation support facility. Hence statement 3 is correct.
Several participants also ensured financial support to the tune of $470 million to protect biodiversity and to counter malnutrition
prevailing in LDCs.Hence statement 4 is correct.

Que. 223 Which of the statement given below is related to UN Security Council Resolution 2396?

1. It calls governments to meet their responsibility to keep citizens secure while travelling by air.
2. It global money laundering and terrorist financing watchdog, sets international standards that aim to prevent these illegal activities
and the harm they cause to society.
3. It calls for global cooperation in trade.
4. It requires Member States to develop and implement systems to collect biometric data in order to identify terrorists responsibly and
properly.

Correct Option - 4
The correct answer is Option 4.
Key Points
Global Efforts to counter Terrorism:

The United Nations Office of Counter-Terrorism (UNOCT) leads and coordinates an all-of-UN approach to prevent and counter-
terrorism and violent extremism.
UN Counter-Terrorism Centre (UNCCT) under UNOCT, promotes international cooperation in the fight against terrorism and
supports the Member States in implementing the Global Counter-Terrorism Strategy.
The Terrorism Prevention Branch (TPB) of the United Nations Office on Drugs and Crime (UNODC) plays a significant role in
international efforts.
It works to assist the Member States, upon request, with the ratification, legislative incorporation and implementation of the universal
legal framework against terrorism.
The Financial Action Task Force (FATF) which is a global money laundering and terrorist financing watchdog, sets international
standards that aim to prevent these illegal activities and the harm they cause to society.
United Nations Security Council Resolution 2396, which requires Member States to develop and implement systems to collect
biometric data in order to identify terrorists responsibly and properly. Hence Statement 4 is correct.

Que. 224 Consider the following statements about Windsor Framework:


1.The Northern Ireland Protocol will be replaced by the “Windsor Framework”.
2.The Protocol entered into force in January 2021, and since then, the EU and the UK have been in negotiations for two years.

125/262
3.Northern Ireland can continue to be a part of the EU’s single market and customs union according to the Windsor Framework.
4.It will remove many of the onerous procedures that were put in place on products coming from the British mainland as part of the first
Brexit agreement.
Commodities moving from Great Britain to Northern Ireland and those continuing into the EU will be divided into “green” and “red” lanes.
How many of the above statements are incorrect?

1. Only one
2. Only two
3. Only three
4. None of the above.

Correct Option - 4
The correct answer is Option 4
Key Points
Windsor Framework:

It will replace the Northern Ireland Protocol, which had proved to be among the thorniest of Brexit fall-outs, creating problems both
economic and political.Hence Statement 1 is correct.
The Protocol entered into force in January 2021, and since then, the EU and the UK have been in negotiations for two years. Hence
statement 2 is correct.
Northern Ireland can continue to be a part of the EU’s single market and customs union according to the Windsor Framework. Hence
statement 3 is correct
It will remove many of the onerous procedures that were put in place on products coming from the British mainland as part of the first
Brexit agreement.

With the new arrangement, commodities moving from Great Britain to Northern Ireland and those continuing into the EU will be
divided into “green” and "red” lanes. Hence Statement 4 is correct

The framework has two crucial aspects:

1.Introduction of a green lane and red lane system - For goods that will stay in Northern Ireland and those that will go to the EU
respectively.
2.The Stormont Brake - Allows Northern Ireland lawmakers and London to veto any EU regulation they believe affects the region
adversely.

The two lane system– British goods meant for Northern Ireland will use the green lane at the ports, and will be allowed to pass with
minimal paperwork and checks.
Physical checks will be conducted if the goods are deemed suspicious, in place of the routine checks now.

Additional Information
With the Windsor Framework, UK hopes to improve trade and other ties with the EU.
The deal has allowed Sunak to do away with the Northern Ireland Protocol Bill introduced by his predecessor Boris Johnson.
The bill involved the UK government reneging on the promise it made to the EU to follow the Protocol.
windsor framework

Que. 225 Consider the following statements about Operation Interflex:


1.Operation Interflex is part of the United Kingdom's £2.3 billion commitment of military aid and support to Ukraine.
2.The operation, launched by U.K. and nine other partner countries aims to train the Ukrainian military in weapons handling, range activity,
field tactics, combat casualty drills, law of armed conflict, first aid and cyber security awareness.
3.Other nations involved in this initiative are Canada, New Zealand, Australia, Norway, Denmark, Finland, Sweden, Lithuania and the
Netherlands.
4.Presently The Ukrainian military is being trained in in Canada, New Zealand, Australia, Norway, Denmark, and Finland.
How many of the above statements are correct?

1. Only one
2. Only two
3. Only three
126/262
4. All four

Correct Option - 3
The correct answer is Option 3.
Key Points
Operation Interflex:

Operation Interflex is part of the United Kingdom's £2.3 billion commitment of military aid and support to Ukraine. Hence statement
1 is correct
The operation, launched by U.K. and nine other partner countries aims to train the Ukrainian military in weapons handling, range
activity, field tactics, combat casualty drills, law of armed conflict, first aid and cyber security awareness. Hence Statement 2 is
correct
Other nations involved in this initiative are Canada, New Zealand, Australia, Norway, Denmark, Finland, Sweden, Lithuania and
the Netherlands. Hence Statement 3 is correct.
Presently, the Ukrainian military is being trained in UK’s Yorkshire, Kent, Wiltshire and Northumberland. Hence Statement 4 is
incorrect.

Additional Information
Operation Interflex began, which involves the United Kingdom (U.K.) and several other countries training nearly 2,000 Ukraine military
recruits for five weeks. This initiative is part of the U.K.’s £2.3 billion commitment towards military aid and support to Ukraine in:
1. Areas of Focus in the Training
The training provided to the recruits under Operation Interflex focuses on several areas, including weapon handling, range activity,
marksmanship, fieldcraft basics, field tactics, battle casualty drills, counter explosives, the laws of armed conflict, first aid, and cyber security
awareness. The aim is to give them the key skills and experiences to operate once they go back to Ukraine.
2. The Training Team
The Canadian-U.K. training wing for Operation Interflex is headed by Major Jürgen Miranda. The training is being delivered by more than
200 personnel, including 60 from Canada and 150 from the U.K.
3. Equipment Given to Trainees
Each trainee is provided with personal protective equipment, including helmets, body armor, eye, ear, and pelvic protectors, individual first-
aid kits, field uniforms and boots, cold and wet weather clothing, bergens, day sacks, and webbing and additional equipment required for field
conditions, including ponchos, sleeping bags, and entrenching tools.
4. High-Tech Military Equipment Supplied by the U.K.
Apart from providing training, the U.K. has also supplied high-tech military equipment to Ukraine, including 14 Challenger 2 tanks, AS90
and 28 M109 155mm self-propelled guns, hundreds of armored and protected vehicles, more than 10,000 anti-tank missiles (including
NLAW, Javelin, Brimstone, and other anti-tank weapons), and multiple launch rocket systems.
5. Expected Number of Recruits Trained by Year-End
Around 20,000 recruits are expected to be trained by year-end under Operation Interflex. The training is aimed at building the necessary skills
and knowledge base in the recruits to enhance Ukraine’s defence against Russia.

Que. 226 Which of the following countries shares border with Armenia?
1. Turkey
2. Iran
3. Georgia
4. Azerbaizan
5. Russia

1. 1 and 4 only
2. 2,3 and 4only
3. 1,2,3 and 4 only
4. 1,2,3,4 and 5

Correct Option - 3
The correct answer is option 3
Key Points

127/262
Que. 227 Consider the following statements about International Fund for Agriculture Development (IFAD):
1. It is an international financial institution and a specialised agency of the United Nations.
2. Enhanced Linkages between Private Sector and Small-scale Producers initiative (ELPS) is an initiative of IFAD.
3. IFAD is the only multilateral development institution that focuses exclusively on transforming rural economies and food systems.
4. India is not a member of IFAD.
Which of the following statements are correct?

1. 1 and 2 only
2. 2 and 4 only
3. 1 and 3 only
4. 1,2 and 3 only

Correct Option - 4
The correct answer is option 4
Key Points
International Fund for Agriculture Development (IFAD)​

It is an international financial institution and a specialised agency of the United Nations. Hence statement 1 is correct.
It is an initiative By IFAD in cooperation with the Ministry of Agriculture, Forestry and Fisheries of Japan . Hence statement 2
is correct.
IFAD is the only multilateral development institution that focuses exclusively on transforming rural economies and food
systems. Hence statement 3 is correct.
India is a founding member of IFAD and the partnership spans more than 40 years. Hence statement 4 is incorrect.

Additional Information
International Fund for Agricultural Development (IFAD):​

It was established as an international financial institution in 1977 through United Nations General Assembly Resolution 32/107 (15
December 1977) as one of the major outcomes of the 1974 World Food Conference.
It is dedicated to eradicating poverty and hunger in rural areas of developing countries.
It seeks to empower rural people to increase their food security, improve the nutrition of their families and increase their incomes
through funding of grants and low-interest loans to several projects.
IFAD is the only multilateral development institution that focuses exclusively on transforming rural economies and food systems.
It is a member of the United Nations Development Group,
IFAD headquarters is in Rome, Italy.

Enhanced Linkages between Private Sector and Small-Scale Producers (ELPS) Initiative:

The objective of the ELPS initiative is to transform small-scale producers’ farming in a sustainable manner, and to improve
productivity and market access by catalysing private sector engagement and investment in food systems.
It has a specific focus on connecting small-scale producers and private companies.
It enables the producers to benefit from the expertise of the private sector in enhancing their productivity.

Other Initiatives of IFAD:

Adaptation for Smallholder Agriculture Programme (ASAP).


Agri-Business Capital (ABC) Fund.
Pacific Islands Rural and Agriculture Stimulus Facility.
Platform for Agricultural Risk Management.

128/262
Que. 228 Which of the following pair is/are correctly matched?
Good Friday Between factions of Northern Ireland and the
1
Agreement governments of Britain and Ireland.
Oslo Peace
2 Between Israel and Egypt.
Accords
3 Abraham Accords Agreements between Israel and the Palestinians.
4 Shimla Agreement Between India and Pakistan.

1. Only One
2. Only Two
3. Only Three
4. All of the above

Correct Option - 2
The correct answer is option 2
Key Points
Good Friday Agreement​

Also known as the Belfast Agreement, signed on April 10, 1998, between factions of Northern Ireland and the governments of
Britain and Ireland, to end 30 years of the violence known as ‘The Troubles’ in Northern Ireland. Hence statement 1 is correct.
John HUME and David Trimble were awarded the Nobel Peace Prize for the Agreement.

Oslo Peace Accords ​

Oslo Accords are a series of agreements between Israel and the Palestinians signed in the 1990s. Hence statement 2 is incorrect.
Oslo I (1993) is formally known as the Declaration of Principles (DOP). The pact established a timetable for the Middle East
peace process. It planned for an interim Palestinian government in Gaza and Jericho in the West Bank
Oslo II is officially called the Israeli-Palestinian Interim Agreement on the West Bank and Gaza (1995), expanded on Oslo I.
The question of Jerusalem was left undecided under the Oslo Accords.

Abraham Accords

The Abraham Accords are a series of agreements signed in 2020 between Israel and several Arab states, marking a historic shift in
diplomatic relations in the Middle East. Hence statement 3 is incorrect.
The primary countries involved in the Abraham Accords include Israel, United Arab Emirates, Bahrain, Sudan and Morocco.

Shimla Agreement

The Shimla Agreement (also called the Shimla Accord or Shimla Treaty), signed between the Prime Minister of India, Indira
Gandhi and Zulfiqar Ali Bhutto of Pakistan, after Indo-Pak war of 1971.
The treaty laid out the principles which would govern the future bilateral relations between the two countries.
They made commitments to have a direct bilateral approach to resolve issues peacefully.

Que. 229 Which of the following statement is correct about Operation Kaveri?

1. It is a codename for India's evacuation effort to bring back its citizens stranded in Sudan .
2. It is an evacuation mission to bring back all the Indian nationals who are currently stranded in Ukraine.
3. It is an operation to bring back Indians from Israel as the conflict escalated with Israel retaliating against Hamas's attack.
4. It is an operation of the Indian Armed Forces to evacuate Indian citizens and foreign nationals from Afghanistan

Correct Option - 1
The correct answer is option 1
Key Points
Operation Kaveri

It is a codename for India's evacuation effort to bring back its citizens stranded in Sudan. Hence statement 1 is correct.
The operation involves the deployment of Indian Navy's INS Sumedha, a stealth offshore patrol vessel, and two Indian Air Force
C-130J special operations aircraft on standby in Jeddah.

Additional Information
Operation Ajay (2023): launched to facilitate the return from Israel of those Indians who wish to come back home as a series of
brazen attacks on Israeli towns by Hamas militants over the weekend triggered fresh tensions in the region.
Operation Ganga (2022): an evacuation mission carried out by the Indian government to rescue its citizens stranded in Ukraine and
neighboring countries during the 2022 Russia-Ukraine war. The citizens were transported from Romania, Hungary, Poland, Moldova,
129/262
and Slovakia to India with assistance from these countries.
Operation Devi Shakti (2021): an operation of the Indian Armed Forces to evacuate Indian citizens and foreign nationals from
Afghanistan after the collapse of the Islamic Republic of Afghanistan and the fall of Kabul.
Operation Dost (2023) : search and rescue operation initiated by the government to aid Syria and Turkey after the 2023 Turkey–
Syria earthquake devastated both countries in February.
Vande Bharat Mission (2020): launched during covid pandemic to bring back Indian citizens stranded in foreign countries.
Operation Samudra Setu (2020): It was a naval operation as part of the national effort to bring home Indian citizens from overseas
during the Covid-19 pandemic. Indian Naval ships Jalashwa (Landing Platform Dock), and Airavat, Shardul and Magar
(Landing Ship Tanks) participated in this operation which lasted over 55 days and involved traversing more than 23,000 km by sea.
Operation Sankat Mochan (2016): an operation of the Indian Air Force to evacuate Indian citizens and other foreign nationals
during the South Sudanese Civil War.
Operation Maitri (2015): rescue and relief operation in Nepal by the government of India and Indian armed forces in the aftermath
of the April 2015 Nepal earthquake.
Operation Raahat (2015): was an operation of the Indian Armed Forces to evacuate Indian citizens and foreign nationals from
Yemen during the 2015 military intervention by Saudi Arabia and its allies in that country during the Yemeni Crisis.

Que. 230 Consider the following statements about Pokhran-II nuclear tests:-
1. The Pokhran-II tests were a series of five nuclear bomb test explosions conducted by India at the Indian Army’s Pokhran Test
Range.
2. These nuclear tests were codenamed as Operation Shakti.
3. India joined the Nuclear Proliferation Treaty after Pokhran-II.
Which of the above statements is/are correct?

1. 1 and 2 only
2. 1 and 3 only
3. 2 and 3 only
4. 1,2 and 3

Correct Option - 1
The correct answer is option 1.
Key Points
Pokhran -II nuclear tests:-

The Pokhran-II tests were a series of five nuclear bomb test explosions conducted by India at the Indian Army’s Pokhran Test Range.
Hence, Statement 1 is correct.
These nuclear tests were codenamed as Operation Shakti (literally, “strength”). Hence, Statement 2 is correct.
These tests displayed India’s capability to build fission and thermonuclear weapons with yields up to 200 kilotons.
It helped India enter the highly guarded club of countries with capability to deploy nuclear weapons.
Pokhran-II was also the culmination of a long journey that began back in the 1940s-50s.
India did not join Non Proliferation Treaty(NPT) as it was considered discriminatory by India. Hence, Statement 3 is incorrect.

Additional Information

Foundation and evolution of India’s nuclear programme


Foundational phase:

India’s nuclear programme can be traced to the work of physicist Homi J Bhabha- ‘the father of India’s nuclear programme’.
In 1945, after Bhabha’s successful lobbying of India’s biggest industrial family, the Tata Institute of Fundamental Research was
opened in Bombay. TIFR was India’s first research institution dedicated to the study of nuclear physics.
In 1954, the Department of Atomic Energy (DAE) was founded, with Bhabha as director.

Pokhran-I:

By the 1970s, India was capable of conducting a nuclear bomb test. Bhaba’s successor at the DAE, Vikram Sarabhai, had worked to
significantly broaden India’s nuclear technology
Indira Gandhi sanctioned the first nuclear test in May 1974.
Pokhran-I, codenamed Operation Smiling Buddha, would be billed as a “peaceful nuclear explosion”, with “few military
implications”.
Atomic Energy Commission: In 1988-89, Rajiv Gandhi gave the go-ahead to the Atomic Energy Commission, and the Defence
Research and Development Organisation, to begin creating an Indian nuclear deterrent.

Pokhran-II- Last phase of nuclear weapons programme:

By 1990, India had a fully developed nuclear weapons programme, which every subsequent Prime Minister had approved of.
In 1995, then Prime Minister PV Narasimha Rao permitted the preparations for carrying out a nuclear test in 1995.

130/262
The Vajpayee government decided to go for the historic Shakti tests and declared India as a state possessing nuclear weapons
following Pokhran-II.

Que. 231 The term 'advance notice' in news is related to :-

1. Policing
2. Judiciary
3. International trade
4. Information Technology

Correct Option - 3
The correct answer is option 3.
Key Points
The US has proposed that advance notices of tariff changes and export restrictions be considered by the 14 member-nations of the
Indo-Pacific Economic Framework for Prosperity (IPEF).Hence, international trade is the correct answer.

Additional Information
Pros of advance notice

Advance information eliminates surprises, gives time for adjustment and minimises losses of trading partners.For entities with goods in
transit or that are holding stock.As it is seen with sudden change in tariffs and restrictions, entities with goods in transit or that are
holding stock either make huge losses or profit.
Sometimes export restrictions can go as far as disturbing the macro-economic indicators of a country or even the entire world.
Few countries have the resilience and economic depth to bear such shocks.

Cons of advance notice :-

Advanced knowledge about tariffs and export controls carries with it the capacity to create problems of a different order. For instance,
disruptions and opportunities for extracting undue gains will emerge.
It might also expose the country putting tariffs and restrictions to pressure from fellow members of the IPEF.
This will put limits on the powers of the member-countries to pursue an independent tariff policy to manage local shortage or glut and
also their ability to formulate incentives for the different sectors of their economy.

Que. 232 Washington Declaration in news is an agreement between which two countries?

1. USA and Russia


2. USA and China
3. USA and India
4. USA and South Korea

Correct Option - 4
The correct answer is option 4.
Key Points
Washington Declaration:-

It is an agreement signed between the United States and South Korea, outlining a joint nuclear deterrence strategy.Hence option 4 is
correct.
The declaration reaffirms that South Korea would not make its own nuclear capabilities and would instead focus on deterrence
measures through an alliance-based approach.
The declaration makes it possible for the US and South Korea to establish a Nuclear Consultative Group (NCG) similar to the one
that exists between the US and NATO.
Through this group, South Korea can have more control over nuclear response planning and coordination, although the nuclear
weapons will be under the exclusive control of the US.
The declaration mandates the U.S. President as the only ‘sole authority’ to use the nuclear arsenal of the U.S. in the event of a nuclear
confrontation.​

Que. 233 'Silkworm Eating Strategy' in news is related to which of the following:-

1. It is a kind of eating prevalent in East Asia.


2. It is a Chinese Salami Slicing Strategy against other countries.
3. It is a strategy employed by people towards environmental protection.
4. It is a misnomer.

Correct Option - 2
131/262
The correct answer is option 2.
Key Points
Silkworm Eating also known as ‘Salami Slicing’ is a strategy where a nation executes minor, steady actions to extend control in
contested territories. Over time, these actions collectively lead to a major shift in control, often without immediate conflict.
It is associated with China and its military strategy especially related to Taiwan and its activites in South China Sea.

Additional Information
In military terms, salami slicing is described as a strategy that involves divide and conquer process of threats and alliances to
overcome opposition and acquire new territories.
It is series of many small actions, often performed by clandestine means, that as an accumulated whole produces a much larger action
or result that would be difficult or unlawful to perform all at once.
In the context of China, salami slicing denotes its strategy of territorial expansion in the South China Sea and the Himalayan regions.

Que. 234 Consider the following statements :

1. The growing digital market in India presents a lucrative opportunity for increased exports of digital services from the USA.

2. Data localization requirements in India and stricter intellectual property rules in the USA are major hurdles in strengthening digital trade
relations between the two countries.

3. The signing of a limited agreement on digital services taxes in 2021 signifies a significant step towards a more open and transparent digital
trade environment between India and the USA.

How many of the statements given above is/are correct ?

1. Only one
2. Only two
3. All three
4. None

Correct Option - 3
The correct answer is option 3.
Key Points
India- USA digital trade relations : India and the USA have a complex relationship when it comes to digital trade. The US is a major
exporter of digital services, while India has a rapidly growing digital market. This presents a significant opportunity for increased trade
between the two countries. Collaboration on areas like cybersecurity, digital infrastructure development, and innovation could be
mutually beneficial.
India's growing digital market offers a vast potential for US tech companies. Hence ,Statement 1 is correct
Data localization and intellectual property rules create friction between the two countries. Hence ,Statement 2 is correct
The signing of agreement on digital services taxes in 2021 is seen as a significant step. Hence ,Statement 3 is correct.

Challenges:
Policy Disagreements: India's data localization requirements and digital taxes create friction for US tech companies. The US, on the
other hand, may have stricter intellectual property rules that might not favor Indian firms.
Market Access: US companies argue for a more open Indian market with fewer restrictions on foreign investment and data movement.
Regulatory Uncertainty: Frequent policy changes in India can make it difficult for US companies to plan for the long term.

Initiatives:
Despite these challenges, there are ongoing efforts to improve digital trade relations.
The two countries have held dialogues on digital trade issues and signed a limited agreement on digital services taxes in 2021.
The Defence Technology and Trade Initiative (DTTI) aims to facilitate collaboration in the tech sector, including co-development and co-
production of technologies.

Que. 235 The Asia-Pacific Plant Protection Commission (APPPC) is primarily concerned with:

1. Controlling soil erosion in the region.


2. Promoting sustainable fishing practices.
3. Protecting plant health and facilitating trade in the Asia-Pacific.
4. Managing water resources in the region

Correct Option - 3
The correct answer is option 3.
Key Points
Asia-Pacific Plant Protection Commission (APPPC) : The Asia-Pacific Plant Protection Commission (APPPC) is an intergovernmental
organization established in 1956 . It works to protect plant, human and animal health, the environment, and facilitate trade in the Asia-
132/262
Pacific region.

The APPPC's main activities include:


Developing regional standards for phytosanitary measures (RSPMs)
Promoting integrated pest management (IPM)
Implementing the Code of Conduct for the Distribution and Use of Pesticides
Facilitating information sharing among member countries

The Commission is headquartered in Bangkok, Thailand, and has 38 member countries. The current Chairperson is Ms. Xueliang Chen,
from China.

Here are some of the benefits of APPPC membership:


Access to a forum for discussing regional plant protection issues
Participation in the development of RSPMs
Access to training and technical assistance
Improved market access for plant products

Que. 236 The Global Crisis Response Group (GCRG) was established by the UN Secretary-General to address issues related to:

1. Climate Change and Environmental Degradation


2. Peacekeeping and Conflict Resolution
3. Food Security, Energy Security, and Financial Stability
4. Human Rights and Sustainable Development

Correct Option - 3
The correct answer is option 3.
GCRG:

The GCRG is a group established by the UN Secretary-General in March 2022. Its purpose is to address urgent global issues related to:
Food Security: Ensuring people have access to enough nutritious food.
Energy Security: Providing reliable and affordable energy sources.
Financial Stability: Managing financial risks and promoting economic growth.

How does the GCRG work?


The GCRG has 32 members and is overseen by a smaller group called the Champions Group.
The Champions Group consists of Heads of State or Government from various countries.
The GCRG aims to provide high-level political leadership to tackle these interconnected crises.
It works on developing strategies and mobilizing solutions to help countries navigate these challenges.

Why was the GCRG formed?

The GCRG emerged due to a confluence of factors, including:


The COVID-19 Pandemic's impact: The pandemic disrupted global supply chains and economies, leading to food insecurity and
financial difficulties for many countries.
The War in Ukraine: The conflict further exacerbated food and energy security concerns, especially for developing nations.
Climate Change: The ongoing threat of climate change poses long-term risks to food and water security.

What are the GCRG's challenges?

The GCRG faces several challenges, including:


Achieving consensus among member countries with diverse interests.
Securing sufficient resources to implement planned solutions.
Addressing the long-term causes of food, energy, and financial crises.

Que. 237 With reference to European security, consider the following statements:
1. The European Sky Shield Initiative (ESSI) is a collaborative effort to develop a unified European air and missile defense
system.
2. All NATO member states are automatically part of the ESSI program.
3. France has opted out of ESSI due to concerns about operational costs.
How many of the statements given above is/are correct?

1. Only one
2. Only two
3. All three
4. None

Correct Option - 1
The correct answer is option 1.

133/262
Key Points
European Sky Shield Initiative (ESSI) : The European Sky Shield Initiative (ESSI) is a project to create a joint European air and missile
defense system.

ESSI aims to create a joint European air defense system. Hence ,Statement 1 is correct.
Participation in ESSI is optional, not automatic for NATO members. Hence ,Statement 2 is incorrect
France is not part of ESSI due to disagreements over equipment procurement, not cost concerns. Hence, Statement 3 is incorrect.

Breakdown of the key points:


Goal: Develop an integrated air defense system for Europe, with anti-ballistic missile capabilities.
Initiated by: Germany in 2022, following concerns about European air defense limitations in the face of the Russia-Ukraine war.
Members: As of February 2024, 21 European countries participate, with Greece and Turkey expected to join soon. Notably, France
has not joined due to disagreements over equipment procurement.
Benefits:
Cost-effectiveness: By jointly procuring systems, members can save money.
Increased capability: A unified system strengthens overall European air defense.
NATO interoperability: ESSI complements NATO's Integrated Air and Missile Defence (IAMD).
Challenges:
French reservations: France argues for including European-developed systems.
Logistical hurdles: Merging different national air defense systems requires complex coordination.

Que. 238 The Orkney Islands have recently been in the news due to their:

1. Economic dependence on fishing industry


2. Dispute over maritime boundaries with neighboring countries
3. Exploration of greater autonomy or even separation from the United Kingdom
4. Discovery of a new natural resource deposit

Correct Option - 3
The correct answer is option 3.
Key Points
Orkney Islands: The Orkney Islands are an archipelago in Scotland, located off the northeastern coast of the mainland. The islands are
known for their rich history, dating back to Neolithic times, and their stunning natural beauty.
In News
The Orkney Islands have been in the news recently due to their consideration of separating from the United Kingdom.
Breakdown of the situation:
Potential Breakaway: The Orkney Islands are exploring more autonomy or even separation from the UK.
Reasons for Consideration:
Feeling neglected by the UK and Scottish governments: The islanders believe they don't receive a fair share of funding despite their
economic contributions.
Desire for greater economic independence: Orkney aims to leverage its renewable energy resources (wind) for economic self-
sufficiency.
Possible Options:
Nordic Connection: Re-establishing ties with Nordic countries like Norway, reflecting their historical links.
Crown Dependency: Similar to Isle of Man or Jersey, with self-government under the British crown.
Increased Autonomy: More control over local affairs within the UK.

Que. 239 Consider the following statements


1. The International Organization of Legal Metrology (OIML) is an intergovernmental body that promotes the standardization of
measurement units globally.
2. OIML recommendations help ensure that measuring instruments certified in one country are recognized as valid in other member states,
facilitating international trade.
Which of the following statements is/are correct ?

1. 1 only
2. 2 only
3. Both 1 and 2
4. Neither 1 nor 2.

Correct Option - 3
The correct answer is option 3.
Key Points
134/262
International Organization of Legal Metrology (OIML) : The International Organization of Legal Metrology (OIML) is an
intergovernmental organisation established in 1955 to promote the global harmonisation of legal metrology procedures that underpin and
facilitate international trade.
Features :

Intergovernmental Organization: Established in 1955, OIML is an international body with member states working together on legal
metrology.
Harmonization of Legal Metrology: OIML's primary objective is to achieve international harmonization in legal metrology
practices. This ensures consistent regulations and technical requirements for measuring instruments across member states.
Development of Recommendations: OIML doesn't set strict laws. Instead, they develop technical recommendations for legal
metrology. These recommendations serve as a foundation for member countries to create national laws and regulations for measuring
instruments used in trade, health, safety, and environmental protection.
Focus on Fair Trade: By harmonizing regulations, OIML recommendations eliminate technical barriers to trade. Measuring
instruments certified in one member state are more likely to be recognized as valid in another, facilitating smoother international trade.
Consumer and Public Protection: OIML's work safeguards consumers and the public by ensuring the accuracy and reliability of
measuring instruments used in areas like gas pumps, weighing scales, and medical devices.
Technical Committees and Collaboration: OIML achieves its goals through a network of Technical Committees (TCs) with
volunteer experts from member states. These TCs address specific aspects of legal metrology and collaborate on developing
recommendations.

Que. 240 Which of the following statements is correct regarding the L.69 Group?

1. It is a group of developed nations advocating for increased power for developing countries within the UN.
2. It is a coalition pushing for reform of the International Court of Justice.
3. It is a regional organization focused on economic development in Africa.
4. It is a group of developing countries advocating for reform of the UN Security Council.

Correct Option - 4
The correct answer is option 4.
Key Points
L.69 Group : The L.69 Group is a coalition of developing countries that advocate for reform of the United Nations Security Council.
L.69 Group: Championing Developing Nations in the UN Security Council
Composition: A coalition of 32 developing countries from Africa, Asia, Latin America and the Caribbean, and the Pacific.
Objective: To achieve comprehensive and lasting reform of the UN Security Council.
Focus: Expansion of the Security Council's permanent and non-permanent categories.
Desired Outcome: Enhance the Council's accountability, representativeness, effectiveness, and transparency in reflecting today's global
landscape.
Motivation: The current structure, established after World War II, does not adequately reflect the power shifts and increased role of
developing nations in global affairs.
Advocacy: The L.69 Group consistently pushes for discussions and action on Security Council reform at the UN General Assembly.
Significance: Their voice strengthens the call for a more inclusive and representative Security Council capable of addressing
contemporary global challenges.

Que. 241 Which of the following statements regarding the Yom Kippur War is/are correct?
1. The Yom Kippur War began on October 6, 1973, coinciding with the Jewish holiday of Yom Kippur.
2. The Yom Kippur War primarily involved Israel fighting against a coalition of Arab states led by Egypt and Syria, with the aim of regaining
territory lost during the Six-Day War of 1967.

1. 1 Only
2. 2 Only
3. Both 1 and 2
4. Neither 1 nor 2.

Correct Option - 3
The correct answer is option 2.
Key Points
Yom Kippur War : The Yom Kippur War, also known as the October War, was a conflict fought between Israel and a coalition of Arab
states led by Egypt and Syria. It began on October 6, 1973, which was Yom Kippur, the holiest day in Judaism. The timing of the attack was

135/262
a surprise to Israel, as they were observing the holiday and thus caught off guard.
CHRONOLOGY :

The war was primarily fought over territory that Israel had occupied since the Six-Day War in 1967, specifically the Sinai Peninsula,
which was captured from Egypt, and the Golan Heights, captured from Syria.
The Arab states sought to regain these territories and to break the political and military stalemate that had persisted since the Six-
Day War.

Egypt launched a surprise attack across the Suez Canal, while Syria attacked the Golan Heights. Initially, the Arab forces made
significant gains, exploiting Israeli weaknesses and failures in intelligence and preparation.

However, Israel managed to mobilize its forces and launched a counteroffensive, ultimately pushing back the Arab armies and even
advancing into Egyptian and Syrian territory.

The war ended on October 25, 1973, with a ceasefire brokered by the United Nations. While the conflict did not result in
significant territorial changes, it had far-reaching consequences.

It led to a shift in the balance of power in the Middle East, prompting efforts towards peace negotiations and ultimately contributing
to the Camp David Accords between Israel and Egypt in 1978.

Que. 242 Which of the following is the primary objective of inter-parliamentary forums?

1. To provide exclusive trade benefits to member nations


2. To establish a global taxation system
3. To foster discussion and cooperation on global issues among parliamentarians
4. To create a unified international parliament

Correct Option - 3
The correct answer is option 3.
Key Points
Inter parliamentary forum : An inter-parliamentary forum is a platform that brings together parliamentarians from different national
parliaments. These forums can be formal international organizations or informal gatherings.

The overall goal of inter-parliamentary forums is to provide a space for parliamentarians to discuss issues of common interest, share
best practices, and promote cooperation on a global scale. Some specific goals of inter-parliamentary forums include:
To promote peace and security
To strengthen democracy
To protect human rights
To address global challenges such as climate change and poverty
To develop common approaches to regional and international issues

Que. 243 Consider the following statements regarding the Voice of Global South Summit (VOGSS):
1. It is a platform for developing countries launched by the United Nations.
2. It aims to address shared challenges faced by the Global South and promote a unified voice.
3. The inaugural summit focused on the impact of technological advancements on developing nations.
How many of the statements given above is/are correct?

1. Only one
2. Only two
3. All three
4. None

Correct Option - 1
The correct answer is option 1.
Key Points
Voice of Global South Summit (VOGSS) : The Voice of Global South Summit (VOGSS) is a recent initiative launched by India to
address the concerns and priorities of developing countries, often referred to as the Global South.

The Voice of Global South Summit (VOGSS) was launched by India, not the United Nations. Hence ,Statement 1 is Incorrect
The VOGSS aims to address shared challenges faced by developing countries and promote a unified voice. Hence ,Statement 2
is Correct
The inaugural summit focused on the theme "Unity of Voice, Unity of Purpose" which is broader than just technological
advancements. Hence ,Statement 3 is Incorrect .

Origin and Objectives:


The first VOGSS was a virtual summit held in January 2023 under the theme "Unity of Voice, Unity of Purpose."
A second edition followed in November 2023 with the theme "Global South: Together for Everyone's Growth, Everyone's Trust."
The core objective is to provide a common platform for developing nations to:

136/262
Discuss shared challenges and interests.
Exchange ideas and solutions.
Develop a unified voice and approach to pressing global issues.

Focus Areas:

The summits have addressed a range of topics critical to the Global South, including:

The COVID-19 pandemic and its impact.


Climate change and ensuring a sustainable future.
Rising food and energy insecurity.
Access to concessional financing for development.
Achieving the Sustainable Development Goals (SDGs).

Outcomes:
The VOGSS summits have fostered solidarity and created a platform for the Global South to present a united front on the international
stage.
Discussions have yielded valuable insights and potential solutions to shared challenges.
India, as the host nation, aims to ensure these voices are heard and considered in global decision-making processes, like the G20.

Que. 244 A key challenge in strengthening India's strategic partnership with the GCC lies in:

1. Balancing its growing ties with Iran, a rival of the GCC.


2. Integrating the large Indian diaspora into the political structures of the GCC nations.
3. Addressing the dominance of oil and gas trade in the bilateral economic relationship.
4. Negotiating a comprehensive security alliance with all GCC members.

Correct Option - 3
The correct answer is option 3.
Key Points
Gulf cooperation council: The Gulf Cooperation Council (GCC) is a regional, intergovernmental political and economic union comprising
six countries around the Persian Gulf. They are Bahrain, Kuwait, Oman, Qatar, Saudi Arabia, and the United Arab Emirates. The
council's main headquarters is located in Riyadh, the capital of Saudi Arabia.
India-GCC Relations: India and the Gulf Cooperation Council (GCC), a group of six nations in the Persian Gulf, share a strong and
complex relationship built on historical ties, economic interdependence, and strategic considerations.
Key aspects of this partnership:
Human Capital Connection:
Indian Diaspora: A significant aspect of the relationship is the large Indian expatriate community residing in GCC countries. There are
roughly 6 million Indians working in the GCC, contributing to their host economies and sending remittances back to India. These
remittances are a vital source of income for many Indian families.

Strategic Partnership:
Shared Security Concerns: Both India and the GCC have a vested interest in maintaining peace and stability in the Gulf region. This
has led to cooperation on security matters like counter-terrorism and maritime security.

Looking Ahead: Broadening the Horizon

While the India-GCC relationship is strong in traditional areas like trade and energy, there's potential for further collaboration in:
Knowledge-based sectors: Information technology and education are promising areas for cooperation.
Infrastructure development: India's expertise can be valuable for the GCC's infrastructure projects.
Renewable energy: Both sides are looking to diversify their energy sources, making collaboration in renewable energy a potential growth
area.

Challenges and Opportunities:


Political Diversification: While economic ties are strong, there's a need for deeper political and strategic engagement to create a more
well-rounded partnership.
Diversifying Trade: Currently, trade heavily relies on oil and gas. Efforts are underway to diversify the trade basket and include more
manufactured goods and services.

Que. 245 Consider the following statements


1. The Secretary-General is mandated to bring any situation that might threaten international peace and security to the attention of
the Security Council under Article 99 of the UN Charter.
2.Article 99 serves as a significant tool for the Secretary-General to prevent potential conflicts by bringing them to the Security Council's
notice.
Which of the above statements is/ are correct ?

137/262
1. 1 only
2. 2 only
3. Both 1 and 2
4. Neither 1 nor 2.

Correct Option - 2
The correct answer is option 2.
Key Points
Article 99 : Article 99 of the United Nations Charter empowers the Secretary-General to bring critical matters to the attention of the
Security Council. Here's a breakdown of its key points:
Function: The Secretary-General acts as a watchdog, monitoring situations that could potentially threaten international peace and
security.
Discretionary Power: The article grants the Secretary-General the authority, but not the obligation, to bring such issues to the Security
Council's notice. This allows them to use their judgment in selecting significant matters.
Security Council Action: When the Secretary-General invokes Article 99, the Security Council is obligated to consider the raised
matter. This draws the Council's attention to potential threats and facilitates discussion or action if necessary.

Examples of Use:
The Secretary-General has used Article 99 sparingly throughout history. Some documented instances include:
The situation in the Republic of the Congo (1960)
Tunisia's complaint against France's military actions (1961)

Significance:

Article 99 serves as a crucial tool for the Secretary-General to proactively address potential threats to international peace and security. It
allows them to raise critical issues and ensure the Security Council is aware of situations that could escalate into conflicts.

Que. 246 The Global Cooperation and Training Framework (GCTF) is an initiative by:

1. The United Nations to promote sustainable development goals.


2. A group of BRICS nations for capacity building in emerging economies.
3. The United States and Taiwan to address global challenges through cooperation.
4. The European Union to enhance training programs for member states.

Correct Option - 3
The correct answer is option 3.
Key Points
Global Cooperation and Training Framework (GCTF) : The Global Cooperation and Training Framework (GCTF) is a platform
established in 2015 to address global challenges through cooperation and capacity building.
Key Features:
Founding Partners: The United States and Taiwan
Objective: Leverage Taiwan's expertise in various fields to assist countries, particularly in the Indo-Pacific region.
Focus Areas: Workshops and training programs on topics like:
Law enforcement cooperation
Women's empowerment
Energy efficiency
E-commerce
Cyber security
Humanitarian assistance and disaster relief
Media literacy
Public health
Current Status (as of March 2024):
Expansion: Additional members include Japan (2019) and Australia (2021).
Reach: Over 24 workshops held in 12 countries worldwide.
Openness: Welcomes participation from other like-minded countries.
Significance:
Shares Taiwan's expertise with the international community.
Addresses global challenges through collaborative efforts.
Provides capacity building for countries in the Indo-Pacific region.

Additional Notes:
Taiwan's participation in international organizations is a complex issue. While the GCTF offers a valuable platform, Taiwan's formal
membership in many international organizations remains a point of contention.

138/262
Que. 247 Consider the following statements on Information Fusion Centre – Indian Ocean Region (IFC-IOR)

1. The primary function of the IFC-IOR is to independently conduct operations to counter maritime threats in the Indian Ocean Region.

2. The IFC-IOR functions under the Indian Navy and promotes information sharing among regional countries for enhanced maritime security.

3. The IFC-IOR focuses solely on monitoring military vessels in the Indian Ocean Region.

How many of the above statements given above is/are correct ?

1. Only one
2. Only two
3. All three
4. None

Correct Option - 1
The correct answer is option 1.
Key Points
Information Fusion Centre – Indian Ocean Region (IFC-IOR) : The Information Fusion Centre – Indian Ocean Region (IFC-IOR) is
a maritime security centre hosted by the Indian Navy. Launched in December 2018, it works towards enhancing maritime security and
safety in the Indian Ocean Region (IOR).

The IFC-IOR is not an operational center. It focuses on information gathering, analysis, and dissemination. Hence ,Statement 1 is
incorrect ..
The IFC-IOR is under the Indian Navy and facilitates information sharing for regional maritime security. Hence ,Statement 2 is
correct .
The IFC-IOR's focus is on all maritime traffic, including commercial shipping (White Shipping) for better Maritime Domain
Awareness (MDA). Hence ,Statement 3 is incorrect .

Objective:
Promote collaborative maritime safety and security in the IOR.
Achieve a peaceful, stable, and prosperous Indian Ocean Region.

Activities:
Fuses, analyzes, and disseminates information on maritime traffic in the Indian Ocean, particularly focusing on "White Shipping"
which refers to commercial shipping information about the movement of cargo ships.
Shares this information with partner nations and agencies to enhance Maritime Domain Awareness (MDA).
Coordinates activities related to maritime security through information sharing and cooperation.

Benefits:
Contributes to combating maritime threats like piracy, terrorism, arms smuggling, human trafficking, and illegal fishing.
Improves overall security and safety in the Indian Ocean region, fostering a more stable environment for economic activity.

Structure:
Located in Gurugram, Haryana, India.
Functions under the Indian Navy's Information Management and Analysis Centre (IMAC).
Collaborates with member nations through International Liaison Officers (ILOs) stationed at the IFC-IOR.
Has working-level linkages with a significant number of countries and multi-national maritime security centers.

Que. 248 Consider the following statements


1. OPEC's primary objective is to ensure a steady supply of oil to consuming nations, even if it means sacrificing some profit for
oil producers.
2. OPEC member countries collectively decide on production quotas to influence global oil prices.
3. Decisions within OPEC are made by a majority vote of its member countries.
How many of the statements given above is/ are correct?

1. Only one
2. Only two
3. All three
4. None

Correct Option - 2
The correct answer is option 2.
Key Points
OPEC : OPEC, which stands for the Organization of the Petroleum Exporting Countries, is a group of 13 countries that control a
significant portion of the world's oil reserves. Their main objective is to coordinate and unify the petroleum policies of its member

139/262
countries.
In News
In December 2023, Angola, Africa's second-largest oil producer, made a surprising move by withdrawing from the Organization of the
Petroleum Exporting Countries (OPEC). This decision came after a period of disagreement with OPEC regarding production quotas.
OPEC aims for stable prices that benefit both producers and consumers, but ensuring a steady supply is also a key objective. Hence,
Statement 1 is correct.
Production quotas are a core mechanism for OPEC to influence oil prices. Hence ,Statement 2 is correct
Decisions within OPEC are generally made by consensus among all members, not by majority vote. Hence ,Statement 3 is incorrect.
Structure:
OPEC headquarters are located in Vienna, Austria.
The organization functions through a ministerial conference, a secretariat, and various committees.
Decisions are generally made by consensus among all members.

Influence:
OPEC plays a significant role in the global oil market by influencing oil prices through production quotas.
It can collectively decide to increase or decrease production to impact supply and consequently, oil prices.

Criticisms:
OPEC has been criticized for manipulating oil prices for its own benefit, potentially harming consumers in oil-importing countries.
Some argue that OPEC's production quotas hinder overall market efficiency.

OPEC+:
A larger group called OPEC+ was formed in late 2016. It includes OPEC members and some non-OPEC oil producers like Russia.
OPEC+ aims to exert even greater control over the global oil market by coordinating production quotas beyond just OPEC members.

Que. 249 Consider the following statements


1. The UN Statistical Commission is a UN body responsible for establishing international trade regulations.
2. The UNSC plays a critical role in setting international standards for data collection and analysis.
Which of the above statements is /are correct.

1. 1 only
2. 2 only
3. Both 1 and 2
4. Neither 1 nor 2

Correct Option - 2
The correct answer is option 2.
Key Points
United Nations Statistical Commission (UNSC) : The United Nations Statistical Commission (UNSC) is the highest decision-making
body for international statistical activities. Established in 1947, it plays a vital role in promoting the development of a global statistical
system.

The UNSC deals with international statistics, not trade regulations. International trade regulations are handled by bodies like the World
Trade Organization (WTO). Hence ,Statement 1 is correct .
Setting standards for data collection and analysis is a core function of the UNSC, ensuring data consistency and comparability across
nations. Hence ,Statement 2 is correct .

UNSC's key functions:

Setting Statistical Standards: The UNSC establishes international standards and methodologies for data collection, analysis, and
dissemination. This ensures consistency and comparability of statistical data across different countries.
Coordinating Statistical Work: The UNSC coordinates the statistical activities of various UN agencies and specialized organizations.
This prevents duplication of efforts and fosters collaboration.
Overseeing the UN Statistics Division: The UNSC provides oversight and guidance to the UN Statistics Division (UNSD), which serves
as the secretariat of the commission and the central body for global statistical activities within the UN system.
Advising UN Organs: The UNSC advises the UN Economic and Social Council (ECOSOC) and other UN bodies on general statistical
matters. This helps in integrating statistical information into broader policy decisions.

Que. 250 Which of the following institutions are included under Market Infrastructure Institutions?
1. Bombay Stock Exchange.
2. National Stock Exchange.
3. Multi Commodity Stock Exchange of India.
4. Central Depository Services Limited.
5. National Securities Depository Limited.

1. 1,2 only
140/262
2. 1,2,3 only
3. 4 and 5 only
4. 1,2,3,4,5

Correct Option - 4
The correct answer is option 4.
Key Points
Market Infrastructure Institutions:-

Stock exchanges, depositories and clearing houses are all Market Infrastructure Institutions and constitute a key part of the
nation’s vital economic infrastructure.Hence, all the options are correct.
A panel set up under the chairmanship of former RBI Governor Bimal Jalan — to examine issues arising from the ownership and
governance of MIIs — in its 2010 report said that the term ‘infrastructure’ would mean the basic, underlying framework or
features of a system
The term ‘market infrastructure’ denotes such fundamental facilities and systems serving this market.
Well-functioning MIIs, constitute the nucleus of capital allocation system, are indispensable for economic growth and have a net
positive effect on society like any other infrastructure institution.
That MIIs are systemically important in India is clear from the phenomenal growth of these institutions in terms of market
capitalization of listed companies, capital raised and the number of investor accounts with brokers and depositories and the value of
assets held in the depositories’ account, as highlighted by the Jalan committee.
Unlike typical financial institutions, the number of stock exchanges, depositories and clearing corporations in an economy is limited
due to the nature of its business, although they cater to the entire marketplace.

Que. 251 Consider the following statements regarding Corporate Debt Market Development Fund (CDMDF):-
1. It is an alternative investment fund established to address the needs of the corporate debt market in India.
2. CDMDF will be launched as a closed-ended scheme with an initial tenure of 15 years.
3. It is regulated by Reserve Bank of India.
How many of the above statements is/are correct?

1. Only one
2. Only two
3. All three
4. None

Correct Option - 2
The correct answer is option 2.
Key Points
Corporate Debt Market Development Fund (CDMDF):-

CDMDF is an Alternative Investment Fund(AIF) established to address the needs of the corporate debt market in India and it
will be launched as a close-ended scheme.Hence Statement 1 is correct.
CDMDF serves as a backstop facility for investment-grade corporate debt securities, providing stability and enhancing investor
confidence in the market.
CDMDF aims to enhance secondary market liquidity by creating a permanent institutional framework that can be activated during
periods of market stress.
The fund acts as a safety net for investors during times of market dislocation, providing support and stability to the corporate debt
market.
During normal market conditions, CDMDF focuses on dealing in low duration government securities (G-sec), treasury bills, and
guaranteed corporate bond repo with a maturity not exceeding seven days.
CDMDF is authorized to purchase only listed corporate debt securities with a residual maturity of up to five years.
The fund refrains from acquiring unlisted, below-investment-grade, or defaulted debt securities.
Securities that present a material possibility of default or adverse credit news or views are also excluded.
CDMDF buys securities at a fair price, factoring in liquidity risk, interest rate risk, and credit risk to ensure transparency and
market stability.
CDMDF will be launched as a closed-ended scheme with an initial tenure of 15 years.Hence,Statement 2 is correct.
The possibility of extension lies at the discretion of the Department of Economic Affairs (DEA) in consultation with SEBI.Hence
Statement 3 is incorrect.

Que. 252 Consider the following statements regarding e-commerce in India:


1. Marketplace model is where the inventory of goods and services is owned by e-commerce entity and is sold to the consumers
directly
2. 100% FDI under automatic route is permitted in the inventory model of e-commerce, while FDI is not permitted in market place based
model of e-commerce.
Which of the above statements is/are correct?

141/262
1. 1 only
2. 2 only
3. Both 1 and 2
4. Neither 1 nor 2

Correct Option - 4
The correct answer is option 4.
Key Points
Marketplace and Inventory-Based Model:-

Marketplace based model of e-commerce means providing an information technology platform by an e-commerce entity on a
digital & electronic network to act as a facilitator between the buyer and seller.Hence Statement 1 is incorrect.
In a marketplace model, the e-commerce firm is not allowed to directly or indirectly influence the sale price of goods or services
and is required to offer a level playing field to all vendors.
Inventory based model of e-commerce means an e-commerce activity where the inventory of goods and services is owned by e-
commerce entity and is sold to the consumers directly.

FDI guidelines for e-Commerce:-DIPP has issued guidelines for FDI in e-commerce:

In India 100% FDI is permitted in B2B e-commerce, however, No FDI is permitted in B2C e-commerce.
100% FDI under automatic route is permitted in the marketplace model of e-commerce, while FDI is not permitted in inventory
based model of e-commerce.Hence Statement 2 is incorrect.​

Que. 253 Consider the statements regarding green shoe option-


1. Under this option, the issuing company has the option to allocate additional equity shares up to a specified amount.
2. It allows the underwriter of a public offer to sell additional shares to the public if the demand is high.
Which of the above statements is/are correct?

1. 1 only
2. 2 only
3. Both 1 and 2
4. Neither 1 nor 2

Correct Option - 3
The correct answer is option 3.
Key Points
Greenshoe Option:-

A greenshoe option is an over-allotment option.


In the context of an initial public offering (IPO), it is a provision in an underwriting agreement that grants the underwriter the right
to sell investors more shares than initially planned by the issuer, if the demand for a security issue proves higher than
expected.Hence,Statement 1 and 2 are correct.
It provides price stability and liquidity.
It provides buying power to cover short positions if prices fall, without the risk of having to buy shares if the price rises.
In risk capital parlance, exercising a green shoe option allows a venture capital or private equity firm to raise capital over and above its
original target corpus, having received greater interest from investors, backing its investment thesis or track record.​

Que. 254 Bedaquiline is a crucial drug for the treatment of which disease?

1. AIDS
2. Malaria
3. Tuberculosis
4. Ebola

Correct Option - 3
The correct answer is option 3.
Key Points
Bedaquiline:-

Bedaquiline is the core drug for the treatment of drug-resistant TB (DR-TB).


Bedaquiline is a crucial drug in the treatment of multi-drug resistant TB (MDR-TB) patients for whom the first-line drug treatment
has stopped working.
It is also an oral medicine with minimal side-effects as compared to other such medicines like Kanamycin.​

https://lms.testbook.com/genpdf/fromques.php?qids=65f93cf52fe08b6c70dc4d11,65f93e6eb5431ddfb31dfe15,65f96fd18acd632ab8e39282,65f9712de8d5… 142/262
Tuberculosis:-

Tuberculosis (TB) is an infectious disease that most often affects the lungs.
It is caused by a type of bacteria called Mycobacterium tuberculosis. TB bacteria spread through the air from one person to another
It spreads through the air when infected people cough, sneeze or spit.It is preventable and curable.

Additional Information

Que. 255 Which of the following statements is correct about TARGET2?

1. TARGET2 is the real-time gross settlement (RTGS) system owned and operated by the Eurosystem.
2. It is target to achieved Worldwide Covid vaccine coverage by 2030.
3. It is real-time gross settlement system recently launched by RBI
4. It is target to reduce road accident related fatalities to less than 10000 in a year.

Correct Option - 1
The correct answer is option 1.
Key Points
TARGET2 (Trans-European Automated Real-time Gross Settlement Express Transfer System)

It is the real-time gross settlement (RTGS) system for the Eurozone, and is available to non-Eurozone countries.
TARGET2 is the real-time gross settlement (RTGS) system owned and operated by the Eurosystem. Hence statement 1 is correct.

Que. 256 Consider the following Statements about Dabba trading:


1. It refers to an illegal practice of trading that takes place outside the purview of stock exchanges.
2. There is real transaction to take physical ownership of a particular stock even thou it took place outside the purview of Stock Exchange.
3. The traders have to registered with the Securities and Exchange Board of India (SEBI) for Dabba Trading.
How many of the statements given above are correct?

1. Only one
2. Only two
3. All three
4. None

Correct Option - 1
The correct answer is option 1
Key Points
Dabba trading

It is also known as box trading or bucket trading. It refers to an illegal practice of trading that takes place outside the purview of
stock exchanges. Hence statement 1 is correct.

143/262
Some traders and brokers bet on stock price movements without incurring a real transaction to take physical ownership of a
particular stock. Simply put, it is gambling centred around stock price movements. Hence statement 2 is incorrect.
The traders are not registered with the Securities and Exchange Board of India (SEBI) either. Hence statement 3 is incorrect.

Additional Information
The biggest lure of this practice is avoiding taxation.
Transactions happen in cash without any record, this practice can lead to the growth of black money that can be used for other criminal
activities.
Dabba trading is recognised as an offence under Section 23(1) of the Securities Contracts (Regulation) Act (SCRA), 1956.
Dabba trading falls within the purview of Sections 406, 420 and 120-B of the Indian Penal Code, 1870.

Que. 257 'Dakar Declaration' recently seen in news is related to which of the following?

1. It is related to road safety.


2. It is related to reduction of tobacco production.
3. It is related to invasive species.
4. It is related to reduction of antibiotic use in animal husbandry.

Correct Option - 1
The correct answer is option 1.
Key Points
Dakar Declaration​

‘Dakar Declaration’ was adopted by 21 African countries committing to strengthening reporting on road crash fatalities, enhancing data
capture, analysis, sharing, and coordination to shape better road safety policies. Hence statement 1 is correct.
The Declaration is in line with the Global Plan for the United Nations Decade of Action for Road Safety, which seeks to halve road
crash deaths by 2030.

Additional Information
The UN General Assembly adopted resolution "Improving global road safety " with the ambitious target of preventing at least
50% of road traffic deaths and injuries by 2030.
World Health Organization (WHO) released a report titled-The Global Status Report on Road Safety 2023

Que. 258 Consider the following statements about Urban Co-operative Banks (UCBs):
1. The term Urban Cooperative Banks (UCBs) are formally defined by the Cooperative Societies Act of the State.
2. UCBs can open new branch only with the prior approval of RBI.
2. UCBs can to do One-Time Settlement at par with commercial banks.
How many of the statements given above are correct?

1. Only one
2. Only two
3. All three
4. None

Correct Option - 1
The correct answer is option 1
Urban Cooperative Banks (UCBs)

The term Urban Cooperative Banks (UCBs) is not formally defined but refers to primary cooperative banks located in urban and
semi-urban areas. Hence statement 1 is incorrect.
RBI notifies 4 key measures to help strengthen 1,514 Urban Co-operative Banks.
UCBs can now open new branches up to 10 per cent (maximum 5 branches) of the number of branches in the previous
financial year without prior approval of RBI in their approved area of operation. Hence statement 2 is incorrect.
UCBs can also do One-Time Settlement at par with commercial banks. Hence Statement 3 is correct.
The RBI has decided to extend the timeline for UCBs to achieve Priority Sector Lending (PSL) targets by two years i.e. up
to March 31, 2026.
In order to meet the long pending demand of the cooperative sector for closer coordination and focused interaction, the RBI has
recently notified a nodal officer as well.​​

Additional Information
Cooperatives Banks in India

144/262
It is an institution established on a cooperative basis to deal with the ordinary banking business.
Cooperative banks are founded by collecting funds through shares, accepting deposits, and granting loans.
They are Cooperative credit societies where members from a community group together to extend loans to each other, at favourable
terms.
They are registered under the Cooperative Societies Act of the State concerned or the Multi-State Cooperative Societies Act,
2002.
​Cooperatives Banks are broadly divided into Urban and Rural cooperative banks.
Dual Control
Banking functions– RBI under Banking Regulation Act 1949.
Management functions- Central Registrar of Cooperative societies.
​For Multistate UCBs
Registered- Multi State Cooperative Societies Act, 2002.
Dual Control
Banking functions– RBI under Banking Regulation Act 1949.
Management functions- Central Registrar of Cooperative societies.

Que. 259 Which of the following statements about Bracket Creep?


1. It is a situation where inflation pushes income into higher tax brackets.
2. Its result is an increase in income taxes but no increase in real purchasing power.
3. It may lead to moderation of inflation, and can be characterized as an automatic stabilizer of the economy.
How many of the above statements are correct?

1. Only one
2. Only two
3. Only three
4. None

Correct Option - 3
The correct answer is option 3
Key Points
Bracket Creep

It is a situation where inflation pushes income into higher tax brackets. Hence statement 1 is correct.
The result is an increase in income taxes but no increase in real purchasing power. Hence statement 2 is correct.
This is a problem during periods of high inflation, as income tax codes typically take longer to change than the rate of inflation.
It may lead to moderation of inflation as there is no increase in real purchasing power and can be characterized as an automatic
stabilizer of the economy. Hence statement 3 is correct.​

Additional Information​
Fiscal Drag

Fiscal drag happens when government’s net fiscal position ( minus taxation) fails to cover the net savings desires of the private
economy, it is also called the private economy’s spending gap.
The resulting lack of aggregate demand leads to deflationary pressure, or drag, in the economy, essentially due to lack of state spending
or to excessive taxation.
One cause of fiscal drag is bracket creep, where progressive taxation increases automatically as taxpayers move into higher tax
brackets due to inflation.
Fiscal drag can also be a result of a hawkish stance towards government finances.

Que. 260 Consider the following statements about initiative on Critical and Emerging Technology (iCET)
1. It is an initiative of India India and Israel.
2.The initiative focuses on addressing regulatory barriers, aligning export controls, and fostering deeper cooperation in critical and emerging
fields.
3. Defence industrial cooperation, defence technological cooperation, and defence start-ups are some of its focus areas.
Which of the given statements are correct?

1. 1 and 2 only
2. 2 and 3 only
3. 1,2 and 3
4. None

Correct Option - 2
145/262
The correct answer is option 2
Key Points
initiative on Critical and Emerging Technology (iCET)

India’s National Security Advisor (NSA) Ajit Doval and the U.S. NSA Jake Sullivan officially launched the United States-India
initiative on Critical and Emerging Technology (iCET). Hence statement 2 is incorrect.
The initiative focuses on addressing regulatory barriers, aligning export controls, and fostering deeper cooperation in critical and
emerging fields. Hence statement 2 is correct.
Defence industrial cooperation, defence technological cooperation, and defence start-ups are some of its focus areas. Hence statement
3 is correct.​​

Additional Information
It is being run by the National Security Council of both countries.​
Under iCET, both countries have identified six areas of cooperation which would include co-development and co-production, that
would gradually be expanded to QUAD, then to NATO, followed by Europe and the rest of the world.
Under iCET, India is ready to share its core technologies with the US and expects Washington to do the same.
AI research agency partnership, Defence industrial cooperation, defence technological cooperation, and defence start-ups, Innovation
Ecosystems, semiconductor ecosystem development, Cooperation on human spaceflight, advancement in 5G and 6G technologies, and
adoption of OpenRAN network technology in India are some of the focus areas of the initiative.

Que. 261 Consider the following statements about Incremental Cash Reserve Ratio:
1. Reserve Bank of India (RBI) announced that it would discontinue the Incremental Cash Reserve Ratio (I-CRR) in a phased
manner.
2. In August 2023, after announcing the monetary policy, the RBI Governor said that banks will have to maintain an I-CRR of 15% on the
increase in their net demand and time liabilities (NDTL).
3. The I-CRR is an additional cash balance which the RBI can ask banks to maintain over and above the CRR - for a specific period.
4. The RBI introduced the I-CRR as a temporary measure to manage excess Liquidity in the banking system.
How many of the above statements are incorrect?

1. Only one
2. Only two
3. Only three
4. All of above.

Correct Option - 1
The correct answer is Option 1.
Key Points
Incremental Cash Reserve Ratio (I-CRR)

The Reserve Bank of India (RBI) announced the phased discontinuation of the Incremental Cash Reserve Ratio (I-CRR) on
September 8, 2023.This measure aimed to absorb surplus liquidity created by factors such as the return of Rs 2,000 notes to the
banking system.
RBI conducted a review and decided to discontinue I-CRR in stages.Hence Statement 1 is correct.
I-CRR was introduced on August 10, 2023, as a temporary measure by RBI to absorb surplus liquidity.
Banks were required to maintain an I-CRR of 10% on the increase in their Net Demand and Time Liabilities (NDTL) between
May 19, 2023, and July 28, 2023.Hence Statement 2 is incorrrect.
It came into effect from the fortnight starting August 12, 2023.The RBI has the authority to implement an additional measure called
Incremental Cash Reserve Ratio (ICRR), in addition to the standard CRR. Hence Statement 3 is correct.
ICRR is employed during periods characterized by excess liquidity in the financial system.Essentially, ICRR mandates that banks park
even more liquid cash with the RBI than what is required under CRR. Hence Statement 4 is correct.
This serves as a means to further manage and control liquidity in the banking system.

Additional Information
Impact of I-CRR on Liquidity Conditions

I-CRR was expected to absorb over Rs 1 lakh crore of excess liquidity from the banking system.It temporarily shifted the banking
system’s liquidity from surplus to deficit on August 21.
Factors like GST outflows and central bank selling of dollars contributed to tight liquidity.
However, liquidity conditions reverted to surplus from August 24.On September 8, RBI absorbed Rs 76,047 crore of surplus
liquidity from the system.

Understanding Cash Reserve Ratio (CRR)

CRR is a fundamental concept before delving into Incremental Cash Reserve Ratio (ICRR).
Banks are mandated to maintain a certain portion of their deposits and specific liabilities in liquid cash with the RBI.

146/262
CRR serves as a crucial tool in the RBI’s arsenal for managing liquidity in the economy and acts as a safety net during times of
banking stress.
Currently, banks are required to uphold 4.5% of their Net Demand and Time Liabilities as CRR with the RBI.

Que. 262 Consider the following statements about Urban Infrastructure Development Fund:
1.UIDF will be established through the use of priority sector lending shortfall
2.Purpose: The fund will be used by public agencies to create urban infrastructure in tier-1 city
3.It will be managed by the Punjab National Bank.
4.It will be established on the lines of the Rural Infrastructure Development Fund (RIDF).
How many of the above statements are correct?

1. Only one
2. Only two
3. Only three
4. All four

Correct Option - 2
The correct answer is Option 2.
Key Points
The Urban Infrastructure Development Fund (UIDF):

The UIDF is set to be created utilizing the deficit in priority sector lending. Hence Statement 1 is correct
Objective: This fund will be deployed by governmental bodies to develop urban infrastructure in tier-2 and tier-3 cities. Hence
Statement 2 is incorrect.
The National Housing Bank will oversee its management. Hence Statement 3 is incorrect.
Its formation will be inspired by the model of the Rural Infrastructure Development Fund (RIDF). Hence Statement 4 is correct.
States will be encouraged to leverage resources from the grants of the 15th Finance Commission, as well as existing schemes, to adopt
appropriate user charges while accessing the UIDF.
The initial corpus for this Fund is ₹10,000 crore.

Additional Information
What are tier-2 and tier-3 cities?

Cities with a population in the range of 50,000 to 100,000 are classified as tier 2 cities, while those with a population of 20,000 to
50,000 are classified as tier 3 cities.

What is Rural Infrastructure Development Fund (RIDF)?

The RIDF was set up by the Government in 1995-96 for financing ongoing rural Infrastructure projects.
The Fund is maintained by the National Bank for Agriculture and Rural Development (NABARD).
Contribution: Domestic commercial banks contribute to the Fund to the extent of their shortfall in stipulated priority sector lending to
agriculture.
Main Objective: To provide loans to State Governments and State-owned corporations to enable them to complete ongoing rural
infrastructure projects.
Repayment period: Loan to be repaid in equal annual instalments within seven years from the date of withdrawal, including a
grace period of two years.

Que. 263 Match the following :


Ports Features
1 Mumbai Tidal Port
2 JNPT(Nava Sheva) Deepest port
3 Vishakhapatnam Largest artficial port
4 Kandla Largest natural port

1. 1 (c), 2 (b), 2 (a), 4 (d)


2. 1 (a), 2 (d), 3 (c), 4 (b)
3. 1 (d), 2 (c), 3 (b), 4 (a)
4. 1 (b), 2 (a), 3(d), 4(c)

Correct Option - 3
The correct answer is Option 3.
Key Points
147/262
Major Ports in India:

India has 12 major seaports and 200 notified minor and intermediate ports that handle a huge volume of traffic.
About 95 per cent by volume and 70 per cent by value of India’s total international trade are carried on through maritime
transportation.
All major ports, except one Ennore Port are government administered. It is the first port in India which is a public company.
The Government of India plans to modernise these ports and has approved a project called Sagarmala.
India is a signatory to Hong Kong International Convention for the safe and environmentally sound recycling of ships.
Largest Port in Arabian Sea –NhavaSheva Port (Maharashtra)
Largest Port in Bay of Bengal- Chennai Port (Tamil Nadu)
Tamil Nadu has the maximum number of major sea ports in India.
Port Blair was notified as a major port in 2010 but was removed of its status recently.

Minor Ports:

As per the Indian Constitution the maritime transport comes under the concurrent list.
The ports are thus either under the management of the Central government or the State government
The Central Shipping Ministry controls and supervises the major ports, whereas the minor ports and intermediate are managed by
the state government’s maritime board.
The increase in private sector participation has resulted in an increase in the number of minor ports.
Cargo traffic handled by the minor ports has exceeded cargo traffic at major ports.
Some of the important minor ports are: Azhikkal Port (Mouth of Valapattanam River Kerala) and Kannur (Kerala)

Zone State Port Features

Eastern Coast Tamil Nadu Chennai Artificial Port

Second busiest port

Western Coast Kerala Kochi Sited in the Vembanad Lake


Exports of spices and salts

Eastern Coast Tamil Nadu Ennore India’s First corporatized port

Eastern Coast West Bengal Kolkata India’s only major Riverine port

Situated on Hugli river


Known as Diamond Harbour

Western Coast Gujarat Kandla Known as Tidal Port


Acknowledged as Trade Free Zone
Largest port by volume of cargo
handled.

Western Coast Karnataka Mangalore Deals with the iron ore exports

Western Coast Goa Mormugao Situated on the estuary of the river


Zuari

Western Coast Maharashtra Mumbai Port Largest Natural Port and harbour In
Trust India

The busiest port in India

Western Coast Maharashtra Jawaharlal Nehru Largest Artificial Port


Port Trust
(JNPT) also
It is the Largest Container Port in India.
known as Nhava
148/262
Sheva, Navi
Mumbai

Eastern Coast Odisha Paradip Natural Harbor


deals with the export of iron and
aluminium

Eastern Coast Tamil Nadu Tuticorin A major port in South India


deals with the fertilizers and
petrochemical products

Eastern Coast Andhra Pradesh Visakhapatnam Deepest port of India


deals with the export of iron ore to
Japan. Amenities for building and
fixing of ships are available

Bay of Bengal Andaman & Port Blair The port connected to the mainland of
Nicobar Islands India through ship and flight. This port
is situated in between two international
shipping lines namely Saudi Arabia &
US Singapore.

​ rom the table we find Mumbai Port Trust Largest Natural Port and harbour In Inda .It is also the busiest port in India.
F
Jawaharlal Nehru Port Trust (JNPT) also known as Nhava Sheva, Navi Mumbai Largest Artificial Port.It is the Largest Container Port in
India.
Visakhapatnam is the deepest port of India .It also deals with the export of iron ore to Japan. Amenities for building and fixing of
ships are available.
Kandla Known as Tidal Port.It is also acknowledged as Trade Free Zone.It is also the largest port by volume of cargo handled.

Que. 264 Consider the following statements about Pharmacy (Amendment) Act, 2023:
1.The bill aims to replace the Pharmacy Act 1998.
2.The Pharmacy (Amendment) Bill, 2023 has been introduced in Lok Sabha to insert a new section 32 C into the Jammu and Kashmir
Pharmacy Act, 2011, which pertains to persons registered or qualified under this Act.
3.National Pharmacy Commission oversees and regulates pharmacy institutions, ensuring competency through evaluations and licensing.
How many of the above statements are incorrect?

1. Only one
2. Only two
3. Only three
4. None

Correct Option - 1
The correct answer is Option 1.
Key Points
The Pharmacy (Amendment) Bill, 2023

The bill will extend the Pharmacy Act, 1948 to the UT of J&K. Hence Statement 1 is incorrect
It has been introduced in Lok Sabha to insert a new section 32 C into the Jammu and Kashmir Pharmacy Act, 2011, which pertains to
persons registered or qualified under this Act. Hence Statement 2 is correct.
The bill proposes constitution of National Pharmacy Commission headquartered in New Delhi.The commission will consist of a
Chairperson, 13 ex-officio members, and 14 part-time members. Hence Statement 3 is correct
The commission will perform the given functions:

1.Setting Standards:
The commission will set education standards, facilities, assessments, training, and tuition fees.
It will establish standards for pharmacy faculty, clinical facilities, and implement a uniform admission mechanism.
2.Regulation and Oversight:
The commission will oversee and regulate pharmacy institutions, research activities, professionals, and associates.
It will ensure the competency of pharmacy professionals through evaluations and licensing procedures.
3.Collaboration and Innovation:
The commission is tasked with collaborating with industry and institutions for the use of technology and hybrid education.
It will focus on driving innovation and research in pharmacy and training professionals with global mobility in mind.

149/262
Additional Information
About Pharmacy Act 1948
The Pharmacy Act of 1948 outlines regulations for the pharmacy profession.
It requires the formation of the Pharmacy Council of India and State Pharmacy Councils. Practising pharmacy in India necessitates
registration under this act
Drugs and Cosmetics Act, 1940, also demand the presence of a registered pharmacist with a license for overseeing drug dispensing.
Breaching these regulations leads to penalties and imprisonment
However, the recent Jan Vishwas Bill 2023 proposes the decriminalisation of sections related to penalties and imprisonment of the Pharmacy
Act, 1948, and Drugs & Cosmetics Act, 1940.

Que. 265 Consider the following statements about Technical Textiles.


1.Technical textiles are functional fabrics that have applications across various industries
2.Based on usage, there are 10 technical textile segments.
3.The biggest players are the China followed by USA, western Europe and Japan.
How many of the above statements are incorrect?

1. Only One
2. Only Two
3. Only Three
4. None of the above

Correct Option - 2
The correct answer is option 2
Key Points
Technical Textiles:

Technical textiles are specialized materials designed for utility rather than aesthetics, serving a wide range of sectors such as
automotive, civil engineering and construction, agriculture, healthcare, safety, and personal protection. Hence Statement 1 is correct.
The market for technical textile products is closely linked to a country's level of development and industrialization.
Technical textiles are categorized into 12 segments according to their application: Agrotech, Meditech, Buildtech, Mobiltech,
Clothtech, Oekotech, Geotech, Packtech, Hometech, Protech, Indutech, and Sportech. For instance, 'mobiltech' includes automotive
textiles like seat belts and airbags, as well as seating for airplanes. Meanwhile, 'geotech', which is notably the most rapidly expanding
sub-segment, encompasses products utilized for soil stabilization and other similar purposes. Hence statement 2 is incorrect.
India is an aspiring player with USD 40 billion in this market (8% Share).The biggest players are the USA, western Europe, China and
Japan (20-40% share). Hence Statement 3 is incorrect.

Additional Information
Initiatives Related to Technical Textile:

Production Linked Incentive (PLI) Scheme for Textiles Sector: It aims to promote the production of high value Man-Made Fiber
(MMF) fabrics, garments and technical textiles.
Harmonized System of Nomenclature (HSN) Codes for Technical Textile: In 2019, Government of India dedicated 207 HSN codes to
technical textiles to help in monitoring the data of import and export, in providing financial support and other incentives to
manufacturers.
100% FDI under Automatic Route: The Government of India allows 100% Foreign Direct Investment (FDI) under automatic route.
International technical textile manufacturers such as Ahlstrom, Johnson & Johnson etc have already initiated operations in India.
Technotex India: It is a flagship event organized by the Ministry of Textiles, in collaboration with Federation of Indian Chambers of
Commerce & Industry (FICCI) and comprises exhibitions, conferences and seminars with participation of stakeholders from across the
global technical textile value chain.
Amended Technology Upgradation Fund Scheme: To improve exports and indirectly promote investments in textile machinery.

Que. 266 The Alkire Foster (AF) Methodology is related to which of the following?

1. Approach for measuring multidimensional poverty.


2. Approach for measuring Crude oil price volatility.
3. Approach for measuring soil fertility.
4. Approach for measuring ground water extraction intensity.

Correct Option - 1
The correct answer is option 1
Key Points
Alkire Foster (AF) Methodology

150/262
It is a comprehensive approach for measuring multidimensional poverty developed by economists Sabina Alkire and James
Foster. Hence statement 1 is correct.
It captures the multiple deprivations that individuals or households experience in different aspects of their lives.
Multidimensional Approach: Unlike traditional poverty measures focusing solely on income, the AF method considers various
dimensions like health, education, and living standards.

Additional Information
MPI’s global methodology is based on the robust Alkire and Foster (AF) method that identifies people as poor based on universally
acknowledged metrics designed to assess acute poverty, providing a complementary perspective to conventional monetary poverty
measures.
The National Multidimensional Poverty measures simultaneous deprivations across three equally weighted dimensions of Health,
Education, and Standard of Living that are represented by 12 Sustainable Development Goals-aligned indicators.
National MPI covers 12 indicators while global MPI covers 10 indicators.

Que. 267 Reverse Flipping in news recently is related to which of the following?

1. It is the process of shifting the domicile of an Indian company back to India.


2. It is the process employed by gymnasts to enhance their performance.
3. It is an emerging technology to speed the performance of computers.
4. It is the technique which is employed to meet the Panchamrit commitments of India.

Correct Option - 1
The correct answer is option 1.
Key Points
Reverse Flipping:-

It has become a trend among Indian startups, especially in the fintech sector, as they plan for Initial Public Offers (IPOs) or seek
long-term benefits in the home market.
It is the process of shifting the domicile of an Indian company back to India after it had moved its headquarters overseas,
usually for tax or regulatory reasons. It is also known as ‘re-domiciling’.Hence, Statement 1 is correct.
This strategic move is fuelled by India's flourishing economy, a huge market, promising venture capital, favorable tax
structures, robust intellectual property protection, young and educated population and supportive government policies.

Que. 268 Consider the following statements about "Payments Infrastructure Development Fund (PIDF) Scheme":
1. It is is a fund set up by the Payments Council of India (PCI).
2. It will include beneficiaries of the PM Vishwakarma scheme.
3. Merchants providing essential services like transport and hospitality services may be included.
How many of the given statements are correct?

1. Only 1
2. Only two
3. All three
4. None

Correct Option - 2
The correct answer is option 2
Key Points
Payments Infrastructure Development Fund (PIDF) Scheme

151/262
It was first launched by the RBI in January 2021 for a period of three years. RBI recently extended the scheme by another two
years.
It is a fund set up by the RBI, in consultation with major authorised card networks, to facilitate the development of payment
acceptance infrastructure in tier-3 to tier-6 cities and the north-eastern states of India. Hence statement 1 is incorrect.
The scheme provides financial assistance to banks and non-bank financial companies (NBFCs) for the deployment of PoS terminals
and other payment acceptance infrastructure in eligible regions.
Merchants providing essential services (transport, hospitality, etc.), government payments, fuel pumps, PDS shops, healthcare, kirana
shops, street vendors, etc., may be covered, especially in the targeted geographies. Hence statement 3 is correct.
Beneficiaries of the PM SVANidhi Scheme in Tier-1 and 2 centres were included in August 2021 and beneficiaries of the PM
Vishwakarma scheme were also included in 2023. Hence statement 2 is correct.

Additional Information
The PIDF will be governed through an Advisory Council and managed and administered by the RBI.
The implementation of targets shall be monitored by the RBI with assistance from card networks, the Indian Banks’ Association (IBA)
and the Payments Council of India (PCI).

PM Vishwakarma scheme​

It is a Central Sector Scheme


This initiative supports small businesses, focusing on traditional artists and craftspeople.
It provides small workers and craftsmen with financial help, training, improved methods, and skill mentoring.
It also aims at improving the quality as well as the reach of the products and services of artisans and craftsmen.
Under this scheme, the artisans and craftspeople will be provided recognition through the PM Vishwakarma certificate and ID card,
credit support upto 1 lakh (first tranche) and Rs.2 lakh (second tranche), with a concessional interest rate of 5%.

Pradhan Mantri Street Vendor’s AtmaNirbhar Nidhi (PM SVANidhi)

To empower street vendors by not only extending loans to them but also for their holistic development and economic upliftment.
The scheme intends to facilitate-collateral free working capital loans of up to Rs.10,000/- of one-year tenure, to approximately 50
lakh street vendors.
Nodal Ministry: Ministry of Housing and Urban Affairs.

Que. 269 Consider the following statements regarding SBI Report on Income Inequality:-
1. According to it, there has been a increase in income inequality in India.
2. The report indicates rising female labour force participation.
3. It also highlighted the decline in Gini Coefficient.
How many of the above statements is/are correct?

1. Only one statement


2. Only two statements
3. All statements
4. None

Correct Option - 2
The correct answer is option 2.
Key Points
Income inequality shrinks, Mobility on the Rise : SBI Research Report” :- key findings

There has been a reduction in income inequality in India, signaling a favorable shift towards upward mobility and the expansion
of the middle class.Hence, Statement 1 is incorrect.
Increasing tax base- The report, citing CBDT data, notes a continuous expansion in the Income Tax base. The number of tax filers
increased to 74 million in AY 2022-23 from 70 million in AY 2021-22.
Decline in Gini Coefficient- Gini Coefficient(a measure of income inequality) has declined from 0.472 during Assessment Year
2014-15 to 0.402 for AY 2022-23.Hence,Statement 3 is correct.
Share of Top earners declined – The share of top earners with incomes exceeding Rs 10 crores and Rs 100 crores has decreased
from 2013-14 to 2020-21.
Improving upward mobility – The drop in income inequality stems from significant upward mobility.More than 36% of individuals
originally in the lowest income bracket in FY14 have transitioned to higher income levels. It led to a notable 21% increase in their
earnings from FY14 to FY21.
The report indicates rising female labour force participation.Hence, Statement 2 is correct.
Evident shifts in income levels for MSMEs and evolving consumption patterns in the aftermath of the Covid-19 pandemic debunk
the myth of K-shaped growth.

Additional Information
Gini Coefficient:-

The Gini coefficient, derived from the Lorenz curve, serves as an indicator of income or wealth inequality.

152/262
The coefficient ranges from 0 (0%) to 1 (100%), 0 representing perfect equality (where every resident in a country has the same
income) and 1 representing perfect inequality (one resident earned all income and the rest earned nothing). Higher the value of Gini
coefficient, higher the inequality.

Que. 270 Consider the following statements about Regulatory Sandbox Scheme:-
1. It refers to live testing of new products or services in a controlled regulatory environment.
2. It acts as a "safe space" for business as the regulators may or may not permit certain relaxations for the limited purpose of testing.
3. It enables more dynamic, evidence-based regulatory environments which learn from, and evolve with, emerging technologies.
How many of the above statements is/are correct?

1. Only one
2. Only two
3. All three
4. None

Correct Option - 3
The correct answer is option 3.
Key Points
Regulatory Sandbox Scheme:-

It refers to live testing of new products or services in a controlled regulatory environment. Hence, Statement 1 is correct.
It acts as a "safe space" for business as the regulators may or may not permit certain relaxations for the limited purpose of testing.
Hence, Statement 2 is correct.
It can provide a structured avenue for the regulator to engage with the ecosystem and to develop innovation-enabling or
innovation-responsive regulations that facilitate delivery of relevant, low-cost financial products.
It is potentially an important tool which enables more dynamic, evidence-based regulatory environments which learn from, and
evolve with, emerging technologies.Hence, Statement 3 is correct.
The RBI had issued the 'Enabling Framework for Regulatory Sandbox' in August 2019, after wide ranging consultations with
stakeholders.

The recently updated framework requires:-

The sandbox entities to ensure compliance with provisions of the Digital Personal Data Protection Act, 2023.
The timelines of the various stages of the Regulatory Sandbox process have been revised from seven months to nine months
The target applicants for entry to the RS are fintech companies, including startups, banks, financial institutions, any other company,
Limited Liability Partnership (LLP) and partnership firms, partnering with or providing support to financial services businesses.

Que. 271 Consider the following statements


1. The "Har Ghar Payment Digital" mission was launched by the World Bank to promote financial inclusion in India.
2. This mission aims to educate people about the benefits and security of digital payment methods.
3. The Reserve Bank of India (RBI) collaborates with banks and payment operators to run campaigns under this mission.
How many of the statements given above is/are correct?

1. Only one
2. Only two
3. All three
4. None

Correct Option - 2
The correct answer is option 2.
Key Points
Har Payment Digital : "Har Payment Digital" is a mission launched by the Reserve Bank of India (RBI) to promote digital payments
across India. It aims to make every citizen a user of digital payment methods by 2025.

The mission was launched by the Reserve Bank of India (RBI), not the World Bank. Hence ,Statement 1 is incorrect.
Educating people about the benefits and security of digital payments is a key objective of the mission. Hence ,Statement 2 is correct.
The RBI collaborates with banks and payment operators to run campaigns promoting digital payments. Hence, Statement 3 is correct.​

Breakdown of the mission:


Launched in 2023 during the Digital Payments Awareness Week.
Aims to bridge the gap between those who already use digital payments and those who don't.
Focuses on educating people about the benefits and ease of using digital payments.
153/262
Encourages existing users to teach others about digital payments.

The RBI collaborated with banks and payment system operators to run campaigns across various media platforms like print, television, radio
and social media.

Here are some of the mission's goals:


Reinforce the convenience and ease of digital payments.
Facilitate onboarding new users into the digital payment system.
Increase awareness about various digital payment channels available.

Que. 272 Under the Export Promotion Capital Goods (EPCG) scheme, a manufacturer-exporter can avail duty-free import of capital goods.
This benefit comes with an obligation to:

1. Pay a fixed export tax within a specified timeframe.


2. Invest the saved duty amount in research and development.
3. Export finished goods worth a specific value within a set period.
4. Source a certain percentage of raw materials from domestic suppliers.

Correct Option - 3
The correct answer is option 3.
Key Points
​ xport Promotion Capital Goods : The Export Promotion Capital Goods (EPCG) Scheme is a program offered by the Indian government
E
to boost exports and make Indian manufacturing more competitive in the global market.
Benefits:
Duty-free import of capital goods: This includes machinery, equipment, and other items needed for pre-production, production, and
post-production activities.
Reduced costs for exporters: By eliminating customs duties, the EPCG scheme helps exporters save money on importing essential
equipment.

Eligibility:
Various exporters can benefit: The scheme applies to manufacturer-exporters, merchant exporters tied to supporting manufacturers, and
service providers.
IEC (Import Export Code) is mandatory: You'll need a valid IEC to apply for the EPCG scheme.

Conditions:
Duty-free imports come with an export obligation: In return for duty-free imports, exporters must fulfill a specific export obligation.
This means they need to export finished goods worth a certain value within a specified timeframe (usually six years). The exact value
depends on the duty saved on the imported capital goods.
There are two options: There's a zero-duty EPCG scheme with a higher export obligation (6 times the duty saved) and a concessional 3%
duty EPCG scheme with a slightly lower obligation (8 times the duty saved).

Overall, the EPCG scheme is a valuable initiative for Indian exporters. It helps them acquire advanced machinery at reduced costs,
ultimately enhancing their production capacity and export competitiveness.

Que. 273 Consider the following statements about the Stand-Up India Scheme:
1. It facilitates bank loans between ₹10 lakh and ₹1 crore specifically for SC/ST and women entrepreneurs.
2. The scheme is targeted towards financing existing businesses for expansion purposes.
3. Loans under the scheme are provided with a credit guarantee mechanism, reducing collateral requirements.
How many of the statements given above is/are correct?

1. Only one
2. Only two
3. All three
4. None

Correct Option - 2
The correct answer is option 2.
Key Points
Stand-Up India scheme: The Stand-Up India scheme is a government program launched in 2016 to promote entrepreneurship among
women and members of Scheduled Caste (SC) and Scheduled Tribe (ST) communities in India.

Stand-Up India aims to provide loans for SC/ST and women entrepreneurs to establish new businesses. Hence ,Statement 1 is correct
The scheme focuses on greenfield projects, which are new ventures, not existing businesses looking to expand. Hence ,Statement 2 is
incorrect.
The scheme offers collateral-free loans through a credit guarantee mechanism, making it easier for these entrepreneurs to access
credit. Hence ,Statement 3 is correct.

154/262
Key features of the Stand-Up India scheme:
Loan(suvidha) (facility): It provides loans between Rs. 10 lakh and Rs. 1 crore to eligible applicants.
Target beneficiaries: The scheme is specifically for SC/ST and women entrepreneurs.
Business focus: Loans are for setting up new businesses (greenfield projects) in manufacturing, services, trading sectors, or activities
allied to agriculture.
Loan guarantee: The scheme offers collateral-free loans through a credit guarantee mechanism.
Margin money: The minimum margin money required from the borrower is 10% of the project cost. There are also provisions for
convergence with other government schemes to meet the margin money requirement.

Overall, Stand-Up India aims to empower these communities by facilitating access to credit and enabling them to participate in the country's
economic growth.

Que. 274
With reference to the SATHI portal, consider the following statements:
1. It is an online platform for managing seed production and distribution in India.
2. The portal facilitates seed traceability from breeder to farmer.
3. SATHI is a joint initiative of the Ministry of Agriculture and State Agriculture Departments.

How many of the statements given above is/are correct?

1. Only one
2. Only two
3. All three
4. None

Correct Option - 3
The correct answer is option 3.
Key Points
SATHI Portal : The SATHI Portal, which stands for Seed Traceability, Authentication and Holistic Inventory, is a government initiative in
India launched by the Ministry of Agriculture and Farmers' Welfare.

The SATHI portal goes beyond just production; it manages the entire seed lifecycle, including distribution. Hence ,Statement 1 is
correct
Seed traceability is a core function of SATHI. It tracks the movement of seeds from the breeder to the farmer, ensuring authenticity
and quality. Hence ,Statement 2 is correct
The portal is a collaborative effort between the Ministry of Agriculture and Farmers' Welfare (MoA&FW) and State Agriculture
Departments. The MoA&FW owns the platform, while state departments contribute content. Hence ,Statement 3 is correct.

SATHI Portal:
Purpose: It's a centralized online system designed to manage the entire seed lifecycle in India.
Focus areas:
Seed traceability: Tracks seeds throughout their journey, from production to sale.
Seed authentication: Ensures the seeds are genuine and of high quality.
Seed inventory management: Provides a holistic view of seed stock availability across the supply chain.
Benefits:
Improved seed quality for farmers.
Reduced risk of fake seeds.
Increased transparency and efficiency in the seed supply chain.
Target users:
Seed producers
Seed certifiers
Seed dealers
Farmers (through information access)
Government department: Owned by the Ministry of Agriculture and Farmers' Welfare, Govt. of India.
Content contribution: Collaborative effort by various State Agriculture Departments.

Que. 275 With the focus on sustainable urban development, which of the following is not a key theme under the Indian government's
CITIIS 2.0 program?

1. Promotion of electric vehicles and improved public transport


2. Creating more parks and recreational areas
3. Utilizing technology for efficient waste management
4. Encouraging private investments in urban infrastructure

Correct Option - 3
The correct answer is option 3.
Key Points
155/262
CITIIS 2.0 : CITIIS 2.0 stands for "City Investments to Innovate, Integrate and Sustain 2.0". It's a program launched by the Ministry
of Housing and Urban Affairs (MoHUA) in India to promote sustainable urban development. It supports Indian Smart Cities in developing
and implementing innovative and sustainable urban infrastructure projects.
CITIIS 2.0 focuses on four main themes:
Sustainable mobility: Encouraging the use of public transport, cycling, and walking.
Public open spaces: Creating and improving parks, squares, and other public spaces.
Urban e-governance and ICT: Using technology to improve government services and citizen engagement.
Social and organisational innovation for low-income settlements: Finding new ways to improve the lives of people living in slums and
other informal settlements.
While waste management is a crucial aspect of sustainable urban development, it's not explicitly mentioned as a key theme of CITIIS 2.0.
The focus here is on using technology for citizen engagement and improved government services.

Que. 276 Consider the following statements


1. The Production Linked Incentive (PLI) scheme offers a fixed financial assistance to companies irrespective of their production
increase.
2. The PLI scheme aims to make India a manufacturing hub by incentivizing both domestic and foreign companies.
Which of the following statements are correct?

1. 1 only
2. 2 only
3. Both 1 and 2
4. Neither 1 nor 2.

Correct Option - 2
The correct answer is option 2.
Key Points
Production Linked Incentive (PLI) Scheme : The PLI scheme is a program launched by the Indian government to boost domestic
manufacturing and attract investments in specific sectors.
The PLI scheme provides incentives based on the incremental sales of manufactured goods in India, not a fixed amount. Hence, Statement
1 is incorrect.
The PLI scheme aims to attract investments from both domestic and foreign companies, ultimately making India a manufacturing
hub. Hence ,Statement 2 is correct.
Objectives:
Increase Production: Encourage companies to set up or expand manufacturing units in India, leading to higher production of goods.
Attract Investments: Incite foreign companies to set up shop in India and encourage domestic companies to invest in growth.
Generate Jobs: Increased manufacturing activity is expected to create new employment opportunities.
Reduce Reliance on Imports: By producing more goods domestically, India aims to reduce its dependence on imports.
Enhance Exports: The PLI scheme incentivizes companies to not only produce but also export goods manufactured in India.

How it Works:

The scheme provides financial incentives to companies based on their incremental sales (increase in sales compared to a base year) of
products manufactured in India. The incentive percentage and duration of the scheme can vary depending on the specific sector.

Benefits:
For Companies: Financial assistance to boost production and potentially gain a competitive edge in the market.
For the Economy: Increased production, creation of jobs, reduced import bills, and potentially higher exports.

Que. 277 With reference to financial inclusion in India, consider the following statements:
1. Antardrishti is a mobile application launched by the Government of India.
2. It provides real-time data on financial inclusion metrics across the country.
3. It is solely focused on monitoring progress at the national level.
How many of the statements given above is/are correct?

1. Only one
2. Only two
3. All three
4. None

Correct Option - 1
The correct answer is option 1.
Key Points
Antardrishti : Antardrishti is a financial inclusion dashboard launched by the Reserve Bank of India (RBI) in 2021. Antardrishti's name
reflects its objective - to provide "inner vision" on financial inclusion in India.

Antardrishti is a dashboard launched by the RBI, not a mobile application by the government. Hence ,statement 1 is incorrect.

156/262
It is designed to monitor financial inclusion and provides data for analysis. Hence ,statement 2 is correct.
While it offers national-level insights, it can also delve deeper into specific districts. Hence, statement 3 is incorrect.

Purpose :
Tracks progress: It monitors various data points and indicators to assess the development of financial inclusion initiatives across the
country.
Identifies gaps: By analyzing this data, it helps pinpoint areas where there's a lack of financial inclusion, allowing policymakers to
focus their efforts on those regions.
Granular view: Antardrishti goes beyond national-level data. It provides insights at a more granular level, enabling identification of
specific districts or communities that require targeted interventions.

Benefits of Antardrishti:
Informed decision-making: By providing data-driven insights, Antardrishti helps policymakers make informed decisions regarding
financial inclusion strategies.
Targeted interventions: Identifying specific areas of exclusion allows for more focused efforts to improve financial access and literacy.
Improved financial inclusion: Ultimately, Antardrishti aims to accelerate the progress of financial inclusion in India, ensuring a larger
portion of the population has access to essential financial services.

Que. 278 Under the Liberalised Remittance Scheme (LRS), which of the following is not a permissible use for an Indian resident to send
money abroad?

1. Investment in overseas stocks


2. Medical expenses for a family member studying abroad
3. Payment for purchasing a property abroad
4. Donation to a charitable organization overseas

Correct Option - 3
The correct answer is option 3.
Key Points
LRS : The Liberalised Remittance Scheme (LRS) is a program introduced by the Reserve Bank of India (RBI) that simplifies sending
money abroad for Indian residents.
Purpose: LRS eases limitations on outward remittance from India, as set by the Foreign Exchange Management Act (FEMA).
Eligibility: Resident individuals, including minors, can utilize LRS. It excludes corporates, partnership firms, and trusts.
Limit: An Indian resident (and their family) can send up to USD 250,000 per financial year (April-March) under LRS.
Permitted Transactions: The scheme allows outward remittances for various purposes, including:
Medical expenses (for self or dependents)
Education expenses (for self or dependents)
Travel expenses
Gifts and maintenance of close relatives abroad
Investment in overseas assets (up to a specific limit)
Restrictions: LRS doesn't allow sending money for prohibited activities like buying lottery tickets or banned magazines, purchasing
property abroad, is typically classified as a real estate transaction and is not allowed under LRS.

Que. 279 The Lakhpati Didi scheme focuses on:

1. Providing financial aid to rural women irrespective of their association with SHGs
2. Encouraging sustainable livelihood practices for women in SHGs
3. Offering skill development programs for urban women entrepreneurs
4. Promoting financial inclusion without any focus on women's empowerment.

Correct Option - 2
The correct answer is option 2.
Key Points
Lakhpati Didi : The Lakhpati Didi is an initiative by the Ministry of Rural Development in India . It aims to empower women in rural
areas associated with Self-Help Groups (SHGs) to achieve an annual household income of over Rs. 1,00,000.
Goal: Empower rural women from SHGs to become "Lakhpati Didis" (women earning Rs. 1 lakh or more annually) through sustainable
livelihood practices.
Target: The initial target was 2 crore Lakhpati Didis across India, but it has been increased to 3 crore for 2024-25.
Strategy:
Livelihood options: Provide support and resources for women to undertake diversified income-generating activities.
Capacity building: Offer training programs to SHG members on livelihood planning, business management, and other relevant skills.
Financial assistance: Some states may offer interest-free loans (this seems to be the case with Rajasthan).

Here are some important things to note:

157/262
The Lakhpati Didi program is a nationwide initiative, but there might be variations in implementation depending on the state.
While some sources mention an interest-free loan component, the official source (National Portal of India) doesn't explicitly mention
it. It's best to check with your local authorities for the specific benefits offered in your state.
The program emphasizes not just financial inclusion but also empowering women to become successful entrepreneurs.

Que. 280 Consider the following statements about the BharatNet project:
1. It aims to connect all schools in rural India with high-speed internet access.
2. It is a government initiative to bridge the digital divide between rural and urban areas by providing broadband internet in Gram
Panchayats.
Choose the correct option:

1. 1 only
2. 2 only
3. Both 1 and 2
4. Neither 1 nor 2.

Correct Option - 2
The correct answer is option 2.
Key Points
BharatNet project : The BharatNet project is a massive Indian government initiative aimed at bringing high-speed internet access to rural
areas.
Goals:
Connect all Gram Panchayats (around 2.5 lakh) in India with broadband internet.
Provide a minimum bandwidth of 100 Mbps at each Gram Panchayat.
Bridge the digital divide between rural and urban India.
Facilitate access to e-governance, e-health, e-education, and other online services in rural areas.

Implementation:
Launched in 2011 as the National Optical Fibre Network (NOFN), renamed BharatNet in 2015.
Implemented in phases:
Phase-I: Focused on connecting around 1 lakh Gram Panchayats using existing optical fiber network infrastructure.
Phase-II: Aims to connect the remaining 1.5 lakh Gram Panchayats with an additional 1 million kilometers of optical fiber.

Benefits:
Improved access to information and communication technology (ICT) in rural areas.
Increased opportunities for education, healthcare, and economic development.
Enhanced delivery of government services online.
Potential for boosting rural entrepreneurship and innovation.

Challenges:
Project faced delays in implementation due to various factors like land acquisition issues and right-of-way permissions.
Ensuring last-mile connectivity and service adoption in remote villages remains a challenge.
Digital literacy and affordability of internet access need to be addressed for wider user base.

Que. 281 Consider the following statements about the eSanjeevani platform:
1. It is a doctor-to-doctor telemedicine platform only, facilitating consultations between specialists in different hospitals.
2. It offers two service models, allowing patients to directly consult doctors from home or with assistance at healthcare centers.
3. It is a private initiative aimed at promoting online consultations in urban areas.
How many of the statements given above is/are correct ?

1. Only one
2. Only two
3. All three
4. None

Correct Option - 1
The correct answer is option 1.
eSanjeevani platform : eSanjeevani is a revolutionary telemedicine platform launched by the Ministry of Health and Family Welfare
(MoHFW) in India. It aims to bridge the gap in healthcare access, particularly for people residing in remote and underserved areas.
eSanjeevani caters to both doctor-to-doctor consultations (eSanjeevaniAB-HWC) and direct patient-to-doctor consultations
(eSanjeevaniOPD). Hence ,Statement 1 is Incorrect.
This statement accurately describes the two service models offered by eSanjeevani:
eSanjeevaniOPD: Facilitates direct consultations from home or nearby health centers.
eSanjeevaniAB-HWC: Provides assisted consultations for patients in health centers with a healthcare worker. Hence ,Statement 2
is Correct.
eSanjeevani is a government initiative specifically designed to address healthcare access issues in underserved areas, not a private
platform focused on urban areas .Hence ,Statement 3 is incorrect.
158/262
Key features:
What it Offers:
Patient-to-Doctor Consultations: eSanjeevani facilitates online consultations between patients and doctors/specialists through video
conferencing.
Two Service Models:
eSanjeevaniOPD: Enables direct patient-to-doctor consultations from the comfort of their homes or nearby Ayushman Bharat - Health
and Wellness Centres (AB-HWCs).
eSanjeevaniAB-HWC: Provides assisted consultations for patients in AB-HWCs with the help of a healthcare worker who facilitates
the connection with a doctor/specialist at a secondary or tertiary healthcare facility.

Benefits:
Increased Accessibility: Makes quality healthcare consultations available in remote locations, reducing travel time and costs for patients.
Improved Efficiency: Optimizes healthcare workforce utilization by enabling doctors to reach a wider patient base.
Enhanced Convenience: Allows consultations to occur remotely, saving time and effort for both patients and doctors.
Reduced Risk: Minimizes the risk of infection, especially beneficial during pandemics or in areas with limited medical facilities.

Que. 282 The DigiReady Certification portal is an initiative by the Government of India to:

1. Help large corporations improve their digital marketing strategies.


2. Offer cybersecurity training programs for government agencies.
3. Assist Micro, Small and Medium Enterprises (MSMEs) in becoming sellers on the Open Network for Digital Commerce (ONDC)
platform.
4. Provide certification for IT professionals seeking jobs in the e-commerce sector.

Correct Option - 3
The correct answer is option 3.
Key Points
DigiReady Certification Portal : The DigiReady Certification Portal is an online platform launched by the Quality Council of India
(QCI) in collaboration with the Open Network for Digital Commerce (ONDC).It aims to assess and certify the digital readiness of Micro,
Small and Medium Enterprises (MSMEs) in India.
Target Audience:
MSMEs and small retailers: The portal is designed specifically for these businesses to assess their digital preparedness for e-
commerce.

Assessment Method:
Self-assessment tool: The portal utilizes an online platform where businesses can evaluate their digital readiness through a series of
questions and criteria.

Focus Areas:

The self-assessment process analyzes various aspects of an MSME's digital capabilities, including:
Documentation: This assesses whether the business has necessary documents like licenses, permits, and tax registrations for online
operations.
Software and Technology Proficiency: The portal evaluates the business's familiarity and comfort level with using digital tools and
technologies relevant to e-commerce.
Workflow Integration: This aspect looks at how well the business can integrate online selling activities with their existing business
workflows (inventory management, order processing, etc.).
Order and Inventory Management: The assessment gauges the business's efficiency in handling online orders and managing their
product catalogue for online marketplaces.

Benefits of DigiReady Certification:


Increased Business Opportunities: By successfully going through the assessment and potentially achieving certification, MSMEs
demonstrate their readiness to sell on the Open Network for Digital Commerce (ONDC) platform. This opens doors to a wider customer
base and increased sales potential.
Enhanced Digital Capabilities: The self-assessment process itself helps identify areas for improvement in digital skills and technology
adoption. This empowers MSMEs to strengthen their overall digital presence.
Streamlined Seller Journey: The DigiReady certification can simplify the onboarding process for MSMEs to become sellers on the
ONDC platform.

Que. 283 With reference to the Payments Infrastructure Development Fund (PIDF)PIDF Scheme, consider the following statements:
1. It was launched by the Government of India.
2. It aims to increase the acceptance of digital payments in underserved regions.

Which of the statements given above is/are correct?

1. 1 only
2. 2 only

159/262
3. Both 1 and 2
4. Neither 1 nor 2.

Correct Option - 2
The correct answer is option 2.
Key Points
Payments Infrastructure Development Fund (PIDF) : The Payments Infrastructure Development Fund (PIDF) Scheme is an initiative by
the Reserve Bank of India (RBI) to promote digital payments in India. The objective of the scheme is to increase the acceptance of digital
payments across the country, with a focus on underserved regions.

The PIDF Scheme was launched by the Reserve Bank of India (RBI), not the Government of India. Hence ,Statement 1 is Incorrect.
The scheme's objective is to increase the acceptance of digital payments in underserved regions like Tier-3 to Tier-6 centers, North-
Eastern states, etc. Hence ,Statement 2 is Correct.

Target Areas:
Tier-3 to Tier-6 centers (smaller towns and rural areas)
North-Eastern states
Jammu & Kashmir
Ladakh

Beneficiaries:
Banks
Non-banking financial companies (NBFCs)
Merchants under the PM Vishwakarma Scheme (as of September 2023)

How it Works:
The PIDF provides financial assistance to banks and NBFCs to deploy payment acceptance infrastructure like:
Point-of-sale (PoS) terminals
QR codes
Soundbox devices (newly added)
Aadhaar-enabled biometric devices (newly added)

Benefits:
Subsidies for deploying payment infrastructure in eligible regions.
Increased access to digital payment options for merchants and citizens in these areas.
Expected addition of 3 million new payment acceptance touchpoints every year.

Que. 284
Consider the following statements:
1. UDGAM is an online portal launched by the Securities and Exchange Board of India (SEBI) to track unclaimed investments.
2. It allows users to search for unclaimed deposits across various banks in India using their PAN or account number.
3. The portal provides the name and branch details of the bank holding the unclaimed deposit.

How many of the statements given above is/are correct?

1. Only one
2. Only two
3. All three
4. None

Correct Option - 1
The correct answer is option 1.
Key Points
UDGAM PORTAL : UDGAM, standing for "Unclaimed Deposits-Gateway to Access Information," is an online portal launched by the
Reserve Bank of India (RBI) to empower a person to find unclaimed deposits he/she might have across various banks in India. It acts as a
one-stop shop, simplifying the search process significantly.

UDGAM stands for "Unclaimed Deposits-Gateway to Access Information" and is launched by the Reserve Bank of India (RBI), not
SEBI. Hence , statement 1 is incorrect.
The statement that UDGAM allows searching for unclaimed deposits using PAN or account number is correct. Hence ,statement 2 is
correct.
UDGAM protects user privacy by assigning a Unique Reference Number (UDRN) to each unclaimed account. It doesn't reveal the
specific bank branch or account holder details. Hence , statement 3 is correct.

Unclaimed Deposits

Unclaimed deposits are funds left dormant in bank accounts for an extended period, often due to inactivity or changes in account holder
information. These deposits can include:
Savings accounts
Fixed deposits (FDs)
160/262
Recurring deposits (RDs)
Demat accounts with unclaimed dividends
Matured policies from insurance companies (if channeled through bank)

Portal's key features:


Wide Bank Coverage: While not all banks are currently onboard, UDGAM boasts participation from 30 banks, representing a
staggering 90% of unclaimed deposits in value terms within the RBI's Depositor Education and Awareness (DEA) Fund. More
banks are expected to join in the future, expanding the portal's reach.
Search Functionality: Once registered, users can search for unclaimed deposits using various criteria, including:
PAN (Permanent Account Number)
Account number (if known)
Name (individual or non-individual, like companies)
UDRN (Unique Reference Number): For privacy purposes, each unclaimed account is assigned a UDRN. This protects your personal
information and prevents identification of the specific bank branch or account holder.
User-friendly Interface: The UDGAM portal is designed for ease of use. It offers a clear and intuitive interface, accessible in English and
Hindi, making it convenient for a wider audience.

Que. 285 AMPLIFI 2.0 aims to empower stakeholders in urban development by:

1. Providing grants for local projects.


2. Simplifying the process of obtaining building permits.
3. Making data on Indian cities easily accessible.
4. Standardizing construction regulations across all cities.

Correct Option - 3
The correct answer is option 3.
Key Points
AMPLIFI : AMPLIFI, which stands for Assessment and Monitoring Platform for Liveable, Inclusive and Future-ready Urban India,
is an initiative by the Ministry of Housing and Urban Affairs in India. It's a web portal that serves as a central location for data on Indian
cities.

Launched in an updated version called AMPLIFI 2.0, the scheme aims to:
Provide data for informed decision making: AMPLIFI 2.0 makes raw data from various Indian cities available on a single platform.
This data can be used by researchers, academics, and policymakers to make data-driven decisions on urban development.
Empower stakeholders: By providing easy access to data, AMPLIFI 2.0 empowers stakeholders to participate in the urban development
process.
Focus on data collection: The scheme shifts the focus from rankings or indices to the underlying data itself. This allows for a more
comprehensive analysis of various aspects of Indian cities.

Right now, AMPLIFI 2.0 has data for over 150 cities across 14 sectors, including:
Demographics
Economy
Education
Energy
Environment
Finance
Governance
Health
Housing
Mobility
Planning
Safety and security
Solid waste management
Water and sanitation

Que. 286 Consider the following statements


1. UPI 123pay is a digital payments service designed for feature phone users in India.
2. It uses Near Field Communication (NFC) for contactless payments between phones.
Which of the following statements is/ are correct ?

1. 1 only
2. 2 only
3. Both 1 and 2
4. Neither 1 nor 2

Correct Option - 1
The correct answer is option 1.
161/262
Key Points
UPI 123 pay : UPI 123pay is a service that allows you to make digital payments using UPI (Unified Payments Interface) even if you
don't have a smartphone or internet connection. It's designed especially for feature phone users in India.

UPI 123pay is a service that allows digital payments using UPI even if you don't have a smartphone or internet connection. Hence ,
Statement 1 is correct.
UPI 123pay uses four main methods for transactions: Interactive Voice Response (IVR), missed call approach, app functionality, and
proximity sound-based payments. Proximity sound-based payments use sound waves, not NFC, for contactless payments. Hence
,Statement 2 is incorrect.

Here's a breakdown of UPI 123pay:


How it works: There are four main ways to use UPI 123pay:
Interactive Voice Response (IVR): You call a specific number for a service and follow voice prompts to complete your transaction
(like paying a bill).
Missed call approach: You give a missed call to a number, and the system calls you back to initiate the transaction.
App functionality: Some banks offer a special UPI 123pay app for feature phones.
Proximity sound-based payments: This method uses sound waves to enable contactless payments between phones (typically for
merchant payments).
Benefits:
Makes digital payments accessible to people without smartphones or internet.
Secure with two-factor authentication using UPI PIN

Que. 287 Consider the following statements


1. E-Kuber is a core banking solution developed by the Reserve Bank of India (RBI) that facilitates electronic transactions
between government departments, treasuries, and banks.
2. E-Kuber functions 24/7 to ensure real-time cash flow management for the government.
3. E-Kuber eliminates the need for banks as intermediaries in government financial transactions.
How many of the statements given above is/are correct?

1. Only one
2. Only two
3. All three
4. None

Correct Option - 2
The correct answer is option 2.
Key Points
E-Kuber: E-Kuber is a game-changer for government financial transactions in India. Developed by the Reserve Bank of India (RBI),
it acts as a core banking solution specifically designed to streamline and automate payments and receipts for the government.

E-Kuber is indeed a core banking solution for the Indian government. Hence ,Statement 1 is correct.
E-Kuber typically isn't operational on Sundays and holidays, though exceptions can be made. Hence ,Statement 2 is incorrect
E-Kuber facilitates direct transactions between government entities, bypassing banks as intermediaries. Hence ,Statement 3 is correct.

Key aspects:
Function: E-Kuber facilitates electronic transfers between government departments, treasuries, and banks. This eliminates the need
for manual processes and paper-based transactions, leading to:
Increased Speed: Transactions are processed much faster compared to traditional methods.
Enhanced Efficiency: Streamlined workflows and reduced manual intervention save time and resources.
Improved Transparency: Real-time tracking and electronic records ensure greater transparency in financial activities.
Benefits: E-Kuber offers several advantages for the Indian government:
Faster Disbursement of Funds: Salaries, pensions, and other payments reach beneficiaries quicker.
Reduced Errors: Automated processes minimize the risk of errors associated with manual transactions.
Better Cash Management: Real-time visibility into government finances allows for better cash flow management.
Uniqueness: Unlike traditional banking systems used by individuals and businesses, E-Kuber eliminates the need for banks as
intermediaries in government transactions. The RBI acts as the central authority, handling payments directly between government entities.
Operational Details: While E-Kuber offers significant benefits, it typically follows specific operational schedules:
Regular Days: Generally, the system is not operational on Sundays and other holidays.
Exceptions: There can be exceptions for crucial financial year-ends. For instance, on March 31, 2024 (a Sunday), E-Kuber was made
functional to ensure all government transactions for the financial year 2023-24 were accounted for.

162/262
Que. 288 Project Unnati is aimed at:

1. Upgrading the infrastructure of rural areas in India.


2. Promoting financial literacy among farmers.
3. Equipping MGNREGA beneficiaries with new skills for better livelihoods.
4. Providing scholarships for higher education in rural India.

Correct Option - 3
The correct answer is option 3.
Key Points
Project Unnati : Project Unnati's primary focus is on empowering MGNREGA workers with skills to transition from partial
employment to more sustainable livelihoods. This initiative launched in 2020 by the Ministry of Rural Development aims to empower
beneficiaries of the Mahatma Gandhi National Rural Employment Guarantee Scheme (MGNREGA).
Key features:
Target Group: The project focuses on MGNREGA beneficiaries, particularly those who have completed 100 days of work under the
scheme in the previous year.
Objective: Project Unnati aims to equip these individuals with new skills to:
Increase Employability: Enhance their chances of finding better-paying jobs or pursuing self-employment.
Reduce Dependence on MGNREGA: Move them towards more sustainable livelihoods and reduce their reliance on MGNREGA for
partial employment.
Skill Training: The project leverages existing training programs offered by:
Deen Dayal Upadhyay Grameen Kaushalya Yojana (DDU-GKY)
Rural Self Training Institutes (RSTIs)
Krishi Vigyan Kendras (KVKs)

These programs provide training in various sectors like agriculture, handicraft, and small business management.
Benefits: By equipping beneficiaries with valuable skills, Project Unnati empowers them to:
Earn Higher Income: Secure better-paying jobs or establish successful businesses.
Improve Livelihoods: Transition towards more stable and long-term sources of income.
Become Self-Reliant: Gain the skills and confidence to pursue self-employment ventures.

Que. 289 Consider the following statements


1. Urea Gold is a new type of fertilizer containing nitrogen, sulphur, and other beneficial nutrients for crops.
2. The sulphur in Urea Gold helps plants absorb more nitrogen from the fertilizer, potentially increasing crop yields.
3. Urea Gold releases nutrients slower than traditional urea, reducing fertilizer loss through leaching.
How many of the statements given above is /are correct?

1. Only one
2. Only two
3. All three
4. None

Correct Option - 3
The correct answer is option 3.
Key Points
UREA GOLD :
Urea Gold is a new type of fertilizer that has been developed in India. It is a coated urea product that contains nitrogen, sulphur and other
nutrients that are beneficial for crops.Key Features :

Enhanced Nitrogen Use Efficiency: The sulphur in Urea Gold helps plants absorb more nitrogen from the fertilizer. This is
because sulphur plays a crucial role in plant protein synthesis, which is essential for nitrogen utilization. By improving nitrogen use
efficiency, Urea Gold can potentially increase crop yields.
Slower Nutrient Release: Urea Gold has a slower release of nutrients compared to traditional urea. This is due to the sulphur
coating, which breaks down gradually over time. The slower release helps to reduce fertilizer loss through leaching, a process where
nutrients are washed away from the soil by water.
Reduced Leaching: Leaching can be a major problem for farmers, as it leads to wasted fertilizer and potential environmental
pollution. By reducing leaching, Urea Gold can help farmers save money on fertilizer costs and protect the environment.
Potential Benefits for Farmers: Overall, Urea Gold has the potential to be a valuable tool for farmers. It can improve crop yields,
reduce fertilizer costs, and protect the environment.
Relatively New Product: Urea Gold is a relatively new product, and more research is needed to fully understand its long-term benefits
and effectiveness in different soil and climatic conditions.

Que. 290 Consider the following statements

163/262
1. Space tourism refers to private companies offering suborbital flights for recreational purposes, allowing passengers to experience
weightlessness for a short duration.
2. SpaceX is the only company currently offering orbital space tourism experiences with stays on the International Space Station.
3. Space tourism is a widely accepted concept with minimal environmental impact and safety concerns.
How many of the statements given above is/are correct?

1. Only one
2. Only two
3. All three
4. None

Correct Option - 2
The correct answer is option 2.
Key Points
Space Tourism : Space tourism offers various experiences, from suborbital flights with brief periods of weightlessness to orbital flights
allowing stays in space, like the International Space Station (ISS). Some companies even have plans for lunar tourism in the future.

Space tourism does encompass suborbital flights offered by private companies for recreational purposes. Passengers experience
weightlessness for a short time during these flights. Hence, Statement 1 is Correct.

Currently, SpaceX is the only company offering orbital space tourism experiences. Their Crew Dragon capsule can dock with the
International Space Station (ISS), allowing tourists to stay in space for a limited period. Hence ,Statement 2 is Correct.

Space tourism is a controversial topic. While some see its potential benefits, concerns exist regarding environmental impact and safety.
Hence, Statement 3 is Incorrect

There are a few different types of space tourism experiences available, including:
Suborbital flights: These flights take passengers to the edge of space, where they can experience weightlessness for a short period of
time. Companies like Virgin Galactic and Blue Origin offer suborbital flights.
Orbital flights: These flights take passengers into orbit around the Earth, where they can experience weightlessness for a longer period of
time and see the Earth from space. Currently, the only company offering orbital space tourism is SpaceX. Their Crew Dragon capsule can
dock with the International Space Station (ISS), allowing tourists to stay in space for up to a week.
Lunar tourism: This is still in the early stages of development, but some companies are planning to offer flights to the moon in the future.

Here are some of the potential benefits of space tourism:


It could help to fund the development of new space technologies.
It could inspire people to learn more about science and engineering.
It could create new jobs in the space industry.

Here are some of the potential risks of space tourism:


It could be dangerous, as space travel is inherently risky.
It could be environmentally damaging, as rocket launches produce pollution.
It could exacerbate social inequality, as only the wealthy will be able to afford to travel to space.

Que. 291 Which of the following statements about Tuberculosis (TB) is incorrect ?

1. It is an infectious disease caused by a virus.


2. It primarily attacks the lungs but can spread to other parts of the body.
3. Latent TB infection allows individuals to spread the disease.
4. Early diagnosis and treatment with antibiotics can prevent transmission.

Correct Option - 1
The correct answer is option 1.

In News

At the One World TB Summit in Varanasi, held on March 2023 .Prime Minister instilled fresh energy to the global
tuberculosis (TB) elimination response and reiterated India’s commitment to spearhead this effort.

Key Points
Tuberculosis : Tuberculosis (TB) is an infectious disease caused by bacteria called Mycobacterium tuberculosis. It usually
attacks the lungs, but it can also spread to other parts of the body.

There are two main forms of TB infection:

164/262
Latent TB infection: This is when you have the TB bacteria in your body, but you are not sick and cannot spread the infection to others.
Most people with latent TB infection never develop active TB.
Active TB disease: This is when the TB bacteria multiply and cause symptoms. People with active TB can spread the infection to others
through coughing, sneezing, or spitting.
Tuberculosis in India :

High Burden: India carries the unfortunate distinction of having the highest TB burden globally. According to the WHO's 2023
Global TB Report , India accounted for an estimated 26.3% of all new TB cases worldwide in 2022. This translates to a staggering
number of new cases exceeding 2.6 million annually.
Drug-Resistant Strains: A concerning aspect of TB in India is the emergence of drug-resistant strains. The same WHO report highlights
that India has the highest burden of multidrug-resistant tuberculosis (MDR-TB) globally, with an estimated 187,000 new cases in
2022. This poses a significant challenge as MDR-TB requires a longer and more complex treatment regimen.
Socioeconomic Factors: Poverty, malnutrition, and overcrowded living conditions contribute significantly to the spread of TB in India.
These factors weaken the immune system, making individuals more susceptible to infection. Additionally, limited access to
healthcare and proper diagnosis further complicate efforts to control the disease.

Que. 292 Bioelectronic computers are a type of biocomputer that:

1. Solely relies on biological components for computations.


2. Combines biological and electronic components for functionalities.
3. Uses mechanical properties of cells for calculations.
4. Is the most common and well-developed form of biocomputer

Correct Option - 2
The correct answer is option 2.
Key Points
Bio Computers : Biocomputers are a fascinating concept that uses the power of biology to perform computations. Unlike traditional
computers that rely on silicon chips and electricity, biocomputers use biological molecules like DNA and proteins to process information.
The field of biocomputing is still in its early stages, but researchers believe it has the potential to revolutionize computing in a number of
ways. For example, biocomputers could be used to develop new drugs, design new materials, and even create new forms of artificial
intelligence.

There are three main types of biocomputers:


Biochemical computers: These computers use enzymes and other biological molecules to perform calculations.
Biomechanical computers: These computers use the mechanical properties of cells and tissues to perform calculations.
Bioelectronic computers: These computers combine biological and electronic components to perform calculations.

Que. 293 Consider the following statements about the Megha-Tropiques-1 (MT-1) satellite:
1. It was a joint Indo-French mission launched in 2011.
2. ISRO conducted a controlled re-entry of the satellite in March 2023.
3. This action aligns with UN guidelines for minimizing space debris.
How many of the statements given above is /are correct ?

1. Only one
2. Only two
3. All three
4. None

Correct Option - 3
The correct answer is option 3.
Key Points
Megha-Tropiques-1 :

The Indian Space Research Organisation (ISRO) conducted a controlled re-entry of the Megha-Tropiques-1 (MT-1) satellite on
March 7, 2023 [ISRO to undertake controlled re-entry experiment of decommissioned satellite on March 7, 2023]. This was a joint
Indo-French satellite launched in 2011 that studied weather and climate in the tropics.

ISRO is committed to following space debris mitigation guidelines set by the United Nations. These guidelines encourage de-
orbiting satellites at the end of their lifespan to minimize the amount of space junk in orbit.

Controlled re-entry of a satellite :

Controlled re-entry of a satellite is a process where a spacecraft at the end of its operational life is deliberately de-orbited and brought
back into Earth's atmosphere. This is done in a controlled manner to ensure:
Safety: The re-entry path is carefully targeted to break up the satellite over a remote, unpopulated area like the ocean. This minimizes the
risk of any debris causing damage or injury on the ground.

165/262
Reduced Space Debris: Satellites that don't undergo controlled re-entry become space junk, circling Earth for years or even
centuries. Controlled re-entries help reduce the amount of space debris in orbit, making space safer for operational spacecraft.

Here's a breakdown of the controlled re-entry process:


Deorbit Maneuver: The satellite's thrusters are fired to slow it down and lower its orbit.
Atmospheric Re-entry: The satellite enters the Earth's upper atmosphere.
Break-up: Due to friction with the atmosphere, the satellite heats up significantly and breaks apart into smaller pieces.
Burn-up: Most of the satellite's components vaporize due to the extreme heat generated during re-entry.
Large Debris Impact: In some cases, very massive or dense parts of the satellite may survive and impact the Earth's surface. These re-
entries are carefully targeted to ensure they land in a safe zone, like the ocean.

Que. 294 Consider the following statements


1. Pellet beam propulsion is a new type of rocket engine that uses lasers to propel spacecraft by heating pellets of fuel.
2. Compared to traditional chemical rockets, pellet beam propulsion offers the potential for faster travel but can only propel very lightweight
spaceships.
Which of the following statements are correct?

1. 1 only
2. 2 only
3. Both 1 and 2
4. Neither 1 nor 2.

Correct Option - 1
The correct answer is option 1.
Key Points
Pellet beam propulsion : Pellet beam propulsion is a new and exciting concept for spacecraft propulsion that's still in the early stages of
development.
Concept:
Inspired by light sail propulsion, pellet beam uses a stream of tiny particles instead of light.
A powerful laser beam hits these particles, causing them to vaporize partially and creating hot plasma.
This hot plasma is expelled at high speeds (over 120 km/s) due to a process called laser ablation.
This high-velocity stream of plasma then pushes on a spacecraft, propelling it forward.

Advantages:
Faster Travel: Pellet beam propulsion has the potential to achieve much faster speeds than traditional chemical rockets, potentially
reaching interstellar space within a human lifetime.
Heavy Payloads: Unlike light sail concepts requiring lightweight craft, pellet beam propulsion could propel heavier spaceships (around
1 ton) carrying more scientific equipment or even crew.

Challenges:
Technical Hurdles: Developing powerful and efficient lasers and managing the immense heat generated during pellet acceleration are
significant challenges.
Concept Stage: Pellet beam propulsion is still a theoretical concept. Extensive research and development are needed before it becomes a
reality.

Que. 295 Food irradiation involves exposing food to:

1. Ultraviolet radiation to enhance vitamin content


2. Ionizing radiation to eliminate harmful microbes
3. Microwave radiation to improve texture
4. Infrared radiation to accelerate ripening.

Correct Option - 2
The correct answer is option 2.
Key Points
Food irradiation: Food irradiation is a process of exposing food to ionizing radiation to kill harmful bacteria, viruses, and parasites. It's
a safe and effective way to improve food safety and extend shelf life. Here's how it works:
Food is placed on a conveyor belt and passed through a chamber where it's exposed to a controlled amount of ionizing radiation.
The radiation penetrates the food and disrupts the DNA of harmful organisms, preventing them from reproducing or making them
sick.
The food is not radioactive after being irradiated.

There are several benefits to food irradiation:


Improved food safety: Food irradiation can kill harmful bacteria, such as E. coli, Salmonella, and Listeria, that can cause foodborne
illness.

166/262
Extended shelf life: Irradiation can slow down spoilage and extend the shelf life of food. This can help to reduce food waste and make
food more accessible to people in all parts of the world.
Reduced reliance on pesticides: Irradiation can be used as an alternative to chemical pesticides to control insects in food.

Food irradiation is a safe and effective way to improve food safety. It's endorsed by major scientific organizations, including the World Health
Organization, the Food and Agriculture Organization of the United Nations, and the Centers for Disease Control and Prevention.

However, there are also some potential drawbacks to food irradiation:


Nutritional loss: There is some concern that irradiation may destroy some nutrients in food. However, studies have shown that any
nutrient losses are small and unlikely to have a significant impact on human health.
Consumer acceptance: Some consumers are concerned about eating irradiated food. There is a misconception that irradiated food is
radioactive, but this is not the case. Food irradiation is a safe and effective way to improve food safety.

Que. 296 The UNCTAD's Technology and Innovation Report 2023 focused on:

1. Artificial Intelligence in healthcare


2. Green technologies and developing countries
3. Blockchain applications in governance
4. Impact of robotics on the workforce

Correct Option - 2
The correct answer is option 2.
Key Points
Technology and Innovation Report 2023 : The Technology and Innovation Report 2023, published by the United Nations Conference on
Trade and Development (UNCTAD), focused on green technologies and their potential to benefit developing countries. Here's a summary:
Green Tech Opportunity: The report highlights how innovation in eco-friendly products and services (green tech) can drive
economic growth and improve technological capabilities in developing countries.
Focus on Sustainability: UNCTAD emphasizes that technological advancements should address the Sustainable Development Goals,
helping developing countries reduce poverty and combat climate change.
Unequal Benefits: The report cautions that existing economic inequalities could worsen as developed countries currently capture most
of the value from green technologies like artificial intelligence, electric vehicles, and the Internet of Things.
Catching Up: The report offers recommendations for developing countries to take advantage of the green tech revolution. These
include building technological capacities, creating green jobs, and participating more in global green value chains.

Que. 297 Consider the following statements


1. The VAIBHAV Fellowship is a program designed to promote research collaboration between Indian scientists and researchers
abroad and Indian institutions.
2. Only Indian citizens residing in India are eligible for the VAIBHAV Fellowship.
Which of the following statements is/are correct?

1. 1 only
2. 2 only
3. Both 1 and 2
4. Neither 1 nor 2

Correct Option - 1
The correct answer is option 1.
Key Points
VAIBHAV Fellowship : The Vaishvik Bharatiya Vaigyanik (VAIBHAV) Fellowship is a program launched by the Department of
Science and Technology (DST) of the Government of India. It aims to connect Indian scientists and researchers living abroad (known
as the Indian diaspora) with Indian academic and R&D institutions to collaborate on research projects.
VAIBHAV fellowship program:
Eligibility: Open to Non-Resident Indians (NRIs), Persons of Indian Origin (PIOs), and Overseas Citizens of India (OCIs) with a Ph.D.,
M.D., or M.S. degree in science, technology, engineering, mathematics, or medicine (STEMM) fields, currently living abroad.
Benefits:
Stipend of ₹4,00,000 per month (around USD 5,000) for a minimum of 1 month and a maximum of 2 months per year.
Business class airfare from your work location abroad to India (once per year).
Up to three years of fellowship tenure.
Responsibilities:
Collaborate with a chosen Indian academic institution.
Work on joint research projects in cutting-edge scientific and technological fields.
Share knowledge and expertise with Indian researchers.

Que. 298 Consider the following statements regarding Learning Science via Standards
1. The Ministry of Education launched the Learning Science via Standards Initiative to improve science education in schools.
167/262
2. This initiative uses historical scientific discoveries to explain the functioning of everyday products.
3. The goal is to bridge the gap between theoretical knowledge and practical applications of science.
How many of the statements given above is/are correct?

1. Only one
2. Only two
3. All three
4. None

Correct Option - 1
The correct answer is option 1.
Key Points
Learning Science via Standards :The Learning Science via Standards Initiative is a program launched by the Bureau of Indian Standards
(BIS), the national standards body of India. It aims to improve students' understanding of science by using everyday products as
examples.

The Bureau of Indian Standards (BIS), not the Ministry of Education, launched the Learning Science via Standards Initiative. Hence
,Statement 1 is incorrect
The initiative focuses on the scientific principles behind the functioning of everyday products, not historical discoveries. Hence ,
statement 2 is incorrect.
The program aims to bridge the gap between theoretical knowledge and practical applications of science by using everyday products.
Hence , Statement 3 is correct.

Key aspects :
The initiative provides teachers with lesson plans that focus on the scientific concepts, principles, and laws behind the manufacturing,
functioning, and quality testing of various products. These products are all based on BIS standards, which ensure that they are safe and of
good quality.

By learning about the science behind everyday items, students can gain a better understanding of how science is applied in the real
world. This can help to make science more relevant and engaging for students.

Some of the products covered in the Learning Science via Standards Initiative include:
Cement
Footballs
Gas stoves
Helmets
LED bulbs
LPG cylinders
Paint

The Bureau of Indian Standards (BIS) is a government agency that is responsible for developing and implementing standards for a wide
range of products and services. The BIS standards help to ensure that products are safe, reliable, and of good quality.

Que. 299 Which of the following statements correctly describes a sand battery?

1. It is a method for converting solar energy into electrical energy.


2. It is a type of battery that uses sand as an electrode material
3. It is a thermal energy storage system that uses sand to store heat from renewable sources.
4. It is a device that generates electricity from the movement of sand particles.

Correct Option - 3
The correct answer is option 3.
Key Points
Sand battery : A sand battery is a type of thermal energy storage system that uses sand or sand-like materials to store heat from
renewable energy sources, such as solar and wind power. This heat can then be used to generate electricity or provide heat for buildings
and industrial processes.
Sand batteries are a promising new technology for storing renewable energy. They are relatively cheap and simple to build, and they can store
large amounts of energy for long periods of time. This makes them a valuable tool for helping to integrate renewable energy sources into the
grid.

Here are some of the benefits of sand batteries:


They can store large amounts of energy for long periods of time.
They are relatively cheap and simple to build.
They can use sand, which is a widely available material.
They can help to integrate renewable energy sources into the grid.

However, there are also some challenges associated with sand batteries:

168/262
They are still a new technology, and there is not a lot of experience with them yet.
They can be large and bulky, which can make them difficult to site.
They are less efficient than some other types of energy storage, such as pumped hydro storage.

Que. 300 Consider the following statements regarding the LIGO India project:
1. It is the first ever gravitational wave observatory in Asia.
2. It is a collaborative project between the Indian government and international partners, including the LIGO Laboratory in the USA.
3. The observatory is already operational and collecting data.
How many of the statements given above is/are correct?

1. Only one
2. Only two
3. All three
4. None

Correct Option - 1
The correct answer is option 1.
Key Points
LIGO India project : The LIGO India project is a planned observatory being built in India to detect gravitational waves. It will be part
of a global network of detectors that includes the existing LIGO facilities in the United States.
While LIGO India will be a major observatory, there are other gravitational wave detectors in Asia, such as the Kamioka Gravitational-
wave Observatory (KAGRA) in Japan. Hence ,Statement 1 is incorrect.
The project is indeed a collaborative effort. Hence ,Statement 2 is correct.
As of March 2024, the observatory is under construction and expected to begin scientific operations around 2025. Hence ,Statement 3 is
incorrect.
The project is a collaboration between the Indian government and the LIGO Laboratory in the USA, along with its international partners.
It received the Indian Government's in-principle approval in February 2016.

The observatory will be located in Hingoli District, Maharashtra and is expected to begin scientific runs sometime around 2025.

Here are some of the scientific goals of the LIGO-India project:


To detect gravitational waves from the universe, ripples in spacetime predicted by Einstein's theory of general relativity.
To study the properties of these waves, such as their source and frequency.
To use gravitational waves to learn more about the universe, such as the nature of black holes and neutron stars.

The LIGO-India project is expected to have several benefits for India, including:
Advancing scientific research in astronomy and astrophysics.
Developing cutting-edge technologies.
Training and educating a new generation of scientists and engineers.

Que. 301 The recently created detailed map of dark matter is significant because it:

1. Contradicts the standard model of cosmology.


2. Supports the existence of dark matter as predicted by Einstein's theory of gravity.
3. Allows direct observation of dark matter particles.
4. Provides conclusive evidence for dark energy.

Correct Option - 2
The correct answer is option 2.
Key Points
Dark Matter : Dark matter is a hypothetical form of matter that is believed to exist in the universe but is invisible and does not interact
with light.
Importance of Dark Matter: Dark matter is essential to explaining the observed structure of the universe. It helps to account for the
distribution of matter in galaxies and the cosmic web. Understanding dark matter is important for developing a complete understanding of the
universe and its evolution.
In News
Recently, researchers have created a detailed map of the invisible dark matter that makes up 85% of the universe.
Findings:

The new findings align with the standard model of cosmology based on Einstein's theory of gravity.

The researchers used the Atacama Cosmology Telescope (ACT) to map dark matter using light from the early universe, known as
the cosmic microwave background (CMB) radiation.

169/262
They used the CMB radiation to map dark matter by observing how it interacts with the gravity of massive objects like galaxy
clusters and lumps of dark matter.

The gravitational field generated by these objects bends and distorts the light that passes through them, which helps in
detecting dark matter.

Que. 302 Consider the following statements


1. The National Quantum Mission (NQM) aims to develop quantum computers with 50-100 qubits but does not prioritize research
in quantum communication.
2. A key focus area of the NQM is the development of single-photon sources for secure communication.
Which of the following is/are correct ?

1. 1 only
2. 2 only
3. Both 1 and 2.
4. Neither 1 nor 2.

Correct Option - 2
The correct answer is option 2.
Key Points
National Quantum Mission (NQM) :India's National Quantum Mission (NQM), launched in 2023, is a government initiative to develop
quantum technologies and make the country a leader in this field. Here's a breakdown of the mission's goals:
Research and Development: The NQM aims to fund and promote research in various areas of quantum science, including quantum
computing, communication, sensing, and materials.
Building Quantum Computers: A key objective is developing quantum computers with 50-100 qubits in five years and reaching
1000 qubits or more within eight years.
Quantum Ecosystem Creation: The mission fosters collaboration between academia, industry, and the government to create a vibrant
ecosystem for quantum technology development.
Skilled Workforce Development: The NQM invests in programs to train researchers, engineers, and other professionals with
expertise in quantum technologies.
Focus Areas: The mission prioritizes specific areas like:
Quantum magnetometers for precision timing and navigation.
Design of novel materials for building quantum devices.
Development of single-photon sources for secure communication.

Que. 303 With reference to recent space missions, consider the following statements:
1. JUICE is a mission by NASA to explore the Venusian atmosphere.
2. It aims to study the icy moons of Jupiter for potential habitability.
3. The mission is currently underway and will reach Jupiter in 2023.
How many of the statements given above is/are correct?

1. Only one
2. Only two
3. All three
4. None

Correct Option - 1
The correct answer is option 1 .
Key Points
JUICE : The Jupiter Icy Moons Explorer (JUICE) is a space mission developed by the European Space Agency (ESA) to study Jupiter
and its three largest moons, Ganymede, Europa, and Callisto. Launched on April 14, 2023, it is currently on an eight-year journey to
Jupiter and is expected to arrive in July 2031.

JUICE is a mission by the European Space Agency (ESA) and targets Jupiter's icy moons, not Venus. Hence , Statement 1 is incorrect.
JUICE's primary goal is to investigate the potential for life on Jupiter's moons, particularly Ganymede, Europa, and Callisto, all
believed to harbor subsurface oceans. Hence , statement 2 is correct.
JUICE was launched on April 14, 2023, and is indeed underway. However, its arrival at Jupiter is expected in July 2031, not 2023. It's
currently on an eight-year journey. Hence , statement 3 is incorrect.

The main objectives of the JUICE mission are to:


Characterize the icy moons of Ganymede, Europa, and Callisto as planetary objects and potential habitats for life.
Explore the atmosphere of Jupiter and its interaction with its moons.
Study the magnetosphere of Jupiter and its influence on the moons.
Investigate the formation and evolution of the Jupiter system in the context of giant planet formation.

170/262
JUICE is equipped with a suite of scientific instruments that will allow it to study the moons in detail. These instruments include:
Radar and magnetometers to study the surfaces and interiors of the moons.
Spectrometers to study the composition of the moons' atmospheres and surfaces.
A laser altimeter to measure the topography of the moons' surfaces.
A thermal imager to study the moons' temperatures.

By studying Jupiter and its moons, JUICE will help us to understand how our Solar System formed and evolved, and whether or not there is
life elsewhere in the universe.

Que. 304 Consider the following statements about the World Health Organization's (WHO) Preparedness and Resilience for Emerging
Threats (PRET) Initiative:
1. PRET focuses on developing separate disease-specific preparedness plans for each potential pandemic threat.
2. The initiative emphasizes collaboration between healthcare and non-healthcare sectors for a comprehensive response.
3. PRET aims to improve global health security by aligning with regional health mandates.
How many of the statements given above is/are correct?

1. Only one
2. Only two
3. All three
4. None

Correct Option - 2
The correct answer is option 2.
Key Points
Preparedness and Resilience for Emerging Threats (PRET) : The Preparedness and Resilience for Emerging Threats (PRET)
Initiative is a global program led by the World Health Organization (WHO) to improve our ability to fight future pandemics. Launched in
April 2023, PRET focuses on a shared-learning approach for disease preparedness, recognizing that many pathogens share similar
transmission methods.

PRET focuses on shared systems for pathogens with similar transmission methods, not individual plans for each threat. Hence ,
statement 1 is incorrect.
Collaboration across sectors is a key aspect of PRET. Hence , statement 2 is correct.
Aligning with regional and global health security mandates is an objective of PRET. Hence , statement 3 is correct.

Key aspects of the PRET initiative:


Focus on Shared Systems: PRET emphasizes leveraging the same systems, capabilities, knowledge, and tools for various pathogens
grouped based on their transmission (respiratory, vector-borne, etc.). This optimizes resource allocation and avoids duplication of effort.
Multisectoral Collaboration: The initiative acknowledges that pandemics impact more than just healthcare. It encourages
collaboration across sectors like government, public health, and even non-health entities to ensure a comprehensive response.
Learning from Experience: PRET incorporates valuable lessons learned from past outbreaks like COVID-19 to strengthen existing
pandemic preparedness frameworks.
Global and Equitable Response: The program aims to create a more equitable and robust response to future pandemics by aligning
with regional and global health security mandates.

Here are some benefits of the PRET initiative:


Improved Preparedness: By focusing on shared systems and leveraging existing knowledge, countries can be better prepared to respond
to novel pathogens.
Efficient Resource Allocation: Shared resources and a focus on multisectoral collaboration can optimize resource utilization.
Faster Response: Streamlined planning and coordinated efforts across sectors could lead to quicker and more effective responses during
outbreaks.

Que. 305 The SUPREME scheme is aimed at:

1. Encouraging research collaborations between universities and private laboratories.


2. Funding the purchase of entirely new analytical equipment.
3. Extending the lifespan and improving the functionality of existing research infrastructure.
4. Providing financial assistance for research projects in specific scientific fields.

Correct Option - 3
The correct answer is option 3.
Key Points
Support for Up-gradation Preventive Repair & Maintenance Of Equipment (SUPREME) : The Support for Up-gradation Preventive Repair
and Maintenance of Equipment (SUPREME) is a program launched by the Department of Science and Technology (DST) of India in
2023. It aims to provide financial assistance for the repair, upgradation, maintenance, retrofitting, or acquisition of additional
attachments to improve the functionality of existing analytical instrumentation facilities (AIFs) supported by DST.

Who can apply?

171/262
Universities and institutions recognized by the University Grants Commission (UGC) including Central Universities, State Funded
Universities, Deemed Universities, Private Universities, IITs, AIIMS, NITs, IISERS
Government and private laboratories

What is not eligible for funding?


Equipment or infrastructure funded under self-sustaining programs of other funding agencies like SAIF, SATHI etc.

What kind of support is provided?


Funding to repair, upgrade, maintain, or retrofit existing analytical instrumentation facilities.
Grants to purchase additional attachments to enhance the capabilities of existing equipment.

For how long is the support provided?


The maximum duration of support is three years.

Funding Pattern
75% funding from DST
25% funding from the applicant institution (government or private)

Que. 306 Consider the following statements regarding Tropospheric Emissions Monitoring Of Pollution (TEMPO)
1. TEMPO is the first space-based instrument to monitor air pollutants hourly across North America.
2. TEMPO is a geostationary satellite that provides continuous imagery of North America.
Which of the following is/are correct?

1. Only one
2. Only two
3. Both 1 and 2
4. Neither 1 nor 2.

Correct Option - 1
The correct answer is option 1.
Key Points
Tropospheric Emissions Monitoring Of Pollution (TEMPO) : The Tropospheric Emissions Monitoring of Pollution (TEMPO)
instrument is a space-based spectrometer designed to measure air pollution across greater North America at high resolution and on an
hourly basis. Launched on April 7, 2023, it is the first ever space-based instrument to monitor air pollutants hourly across the North
American continent during daytime.

While TEMPO is attached to a geostationary satellite, it is a spectrometer, not an imaging instrument. It collects data on specific
wavelengths of light to measure air pollution levels. Hence , statement 2 is incorrect.

Features :

TEMPO is an ultraviolet–visible spectrometer that provides hourly data on ozone, nitrogen dioxide, and formaldehyde in the
atmosphere. It is attached to a communications satellite in geostationary orbit, providing a constant view of North America.

This allows TEMPO to capture the variations in pollution levels throughout the day.

TEMPO's measurements will be used to improve air quality forecasts, monitor emission sources, and develop emission control
strategies. The data will also be used to study the effects of air pollution on human health and the environment.

Que. 307 Which of the following statements is incorrect about Electromagnetic Ion Cyclotron (EMIC) waves?

1. They are a type of plasma wave.


2. They are sound waves propagating through Earth's atmosphere.
3. They can affect Earth's ionosphere at high latitudes.
4. They involve oscillating electric and magnetic fields.

Correct Option - 2
The correct answer is option 2 .
Key Points
Electromagnetic Ion Cyclotron (EMIC) : Electromagnetic Ion Cyclotron (EMIC) waves are a type of plasma wave observed in the
Earth's magnetosphere. Plasma is a hot, charged gas that makes up over 99% of the visible universe . EMIC waves are discreet
electromagnetic emissions that can affect Earth's atmosphere and even satellites in space.

172/262
Sound waves require a medium to travel through, and EMIC waves can travel through the near-vacuum of space. Sound waves cannot
propagate through the Earth's magnetosphere. Hence ,Statement 2 is incorrect.
Key terms:
Electromagnetic: EMIC waves involve both electric and magnetic fields oscillating together.
Ion: An ion is an atom or molecule that has lost or gained one or more electrons, giving it a positive or negative electrical charge.
Cyclotron: The cyclotron frequency is the natural frequency at which a charged particle travels in a circular path in a magnetic field.
Magnetosphere: The magnetosphere is the region around Earth that is dominated by Earth's magnetic field.

So, EMIC waves are electromagnetic waves that have a frequency close to the cyclotron frequency of ions in Earth's magnetosphere. These
waves are generated in the equatorial regions of the magnetosphere and then travel along magnetic field lines to reach the ionosphere at high
latitudes. They can also be observed on the ground by magnetometers.

Que. 308 Consider the following statements on Deep Learning Geomagnetic Perturbation (DAGGER)
1. DAGGER is a climate model that uses satellite data to predict long-term weather patterns.
2. DAGGER is a deep learning model that can predict geomagnetic storms with 30 minutes of lead time.
3. Early warning from DAGGER can help protect power grids and communication systems from solar wind disruptions
How many of the statements given above is/are correct ?

1. Only one
2. Only two
3. All three
4. None

Correct Option - 2
The correct answer is option 2.
Key Points
Deep Learning Geomagnetic Perturbation : Deep Learning Geomagnetic Perturbation, also known as DAGGER, is a computer model that
uses artificial intelligence (AI) to predict geomagnetic storms caused by solar activity.

DAGGER deals with geomagnetic storms, which are short-term events, not long-term weather patterns. Climate models focus on
weather trends over months or years. Hence ,Statement 1 is Incorrect.
DAGGER excels at using deep learning to predict geomagnetic storms with a 30-minute lead time. Hence, Statement 2 is Correct.
Early warnings from DAGGER allow critical infrastructure operators to take precautions like powering down sensitive equipment to
minimize damage from geomagnetic storms. Hence ,Statement 3 is Correct.

Breakdown of what DAGGER does:


Function: Predicts geomagnetic perturbations (changes in Earth's magnetic field) caused by solar wind (charged particles from the
Sun) 30 minutes in advance.
Technology: Deep learning, a type of AI that can recognize patterns in data.
Benefits: Provides advanced warning of geomagnetic storms which can disrupt power grids, communication systems, and satellites.

DAGGER is significant because it offers more lead time for preparation compared to traditional methods. This allows critical infrastructure
operators to take measures to mitigate the effects of a geomagnetic storm.

Here are some additional points to consider:


DAGGER is an open-source platform, meaning its code is freely available for anyone to use and improve.
The model is trained on data collected from satellites that measure the solar wind.
By analyzing this data, DAGGER learns to identify patterns that precede geomagnetic storms.

Que. 309 Consider the following statements about the DAGGER Model:
1. The DAGGER Model utilizes artificial intelligence to predict the time of arrival of geomagnetic disturbances triggered by the
solar wind.
2. It offers a lead time of 30 minutes for solar storm predictions, aiming to minimize the impact on critical infrastructure.
3. The model's effectiveness was confirmed using data from geomagnetic storms that happened in 1998 and 2002.
How many of the given statement are correct?

1. Only one
2. Only two
3. All three
4. None

173/262
Correct Option - 2
The correct answer is option 2
Key Points
DAGGER model

It is developed by NASA.
It employs AI techniques to analyze and interpret data from spacecraft observations of the solar wind, indeed aiming to predict
geomagnetic disturbances. Hence statement 1 is correct.
DAGGER model offers a lead time of 30 minutes, which is crucial for taking preventive measures to protect critical infrastructure
against the effects of solar storms. Hence statement 2 is correct.
The model's potential was validated using data from geomagnetic storms that occurred in August 2011 and March 2015, not 1998 and
2002. Hence statement 3 is incorrect.

Additional Information
The continuous stream of charged particles, or plasma, emanating from the Sun is known as the solar wind.
This solar wind, along with other solar phenomena, affects the Earth’s magnetosphere and can lead to geomagnetic disturbances.
The peak of the Sun’s 11-year activity cycle, known as the solar maximum, is characterized by increased solar wind and other space
weather events, which can pose a significant threat to modern technology. The Sun can experience a peak solar storm in 2025 .

Que. 310 The controversy surrounding "three-parent babies" primarily stems from:

1. The high cost of the procedure


2. The potential for unintended side effects
3. The use of genetically modified organisms
4. The ethical implications of altering human germline

Correct Option - 4
The correct answer is option 4.
Key Points
Three-parent baby : A "three-parent baby" is a term used for a baby conceived using a type of assisted reproductive technology
(ART) called mitochondrial replacement therapy (MRT) or three-person in vitro fertilization (IVF). This technique is used to prevent
mothers from passing on mitochondrial diseases to their children.
Breakdown of the process:
Mitochondria are structures inside cells that convert food into energy. They have their own separate DNA besides the DNA in the cell
nucleus.
Mitochondrial diseases are caused by mutations in mitochondrial DNA and can cause serious health problems.
MRT involves using genetic material from three people to create a healthy embryo. The mother and father provide the nuclear DNA,
and a healthy donor egg provides healthy mitochondrial DNA.

There are a few different methods for MRT, but the general idea is to replace the mother's egg cytoplasm, which contains the mitochondria,
with cytoplasm from a healthy donor egg. The nucleus from the mother's egg, containing the nuclear DNA from both parents, is then inserted
into the donor egg. This egg is then fertilized with sperm from the father and implanted in the mother's uterus for pregnancy.
Ethical concerns surrounding germline modification include:
The long-term, unknown effects on future generations.
The potential for unintended consequences or genetic mutations.
Questions about who has the right to choose such modifications for their children.
Whether it opens the door to designer babies with predetermined traits.

The high cost and potential side effects are important considerations, but they are not the primary reasons for the controversy. GMOs are not
typically a concern because the modification only affects the mitochondrial DNA, not the nuclear DNA which makes up the vast majority of
an individual's genetic makeup.

Que. 311 Consider the following statements


1. Generic drugs are cheaper than brand-name drugs because they contain different active ingredients.
2. Generic drugs undergo the same rigorous approval process for safety and effectiveness as brand-name drugs.
Which of the statements given above is/are correct?

1. 1 only
2. 2 only
3. All three
4. None

Correct Option - 2
The correct answer is option 2.

174/262
Key Points
Generic drugs : Generic drugs are medications that contain the same active ingredients as their brand-name counterparts, but they are
typically much cheaper.

Generic drugs are just as safe and effective as brand-name drugs, having to go through a rigorous approval process by the Food
and Drug Administration (FDA) to demonstrate that they meet the same standards for safety, strength, purity and effectiveness as
brand-name drugs .

Why are generic drugs cheaper?

There are a few reasons why generic drugs are cheaper than brand-name drugs. First, the manufacturer of a brand-name drug has to
recoup the costs of research and development. This can be a very expensive process, and the manufacturer is allowed to charge a
higher price for the drug to make back their investment.

Second, generic drug manufacturers don't have to spend money on advertising. Brand-name drug companies spend millions of dollars
on advertising to convince doctors and patients to use their products.

Bioequivalence and Potential Considerations:


Bioequivalence testing: Generic drugs must demonstrate bioequivalence to the brand-name drug. This means they deliver the same
amount of the active ingredient to the bloodstream at a similar rate.
Inactive ingredients: While the active ingredients are the same, generic drugs may have different inactive ingredients (fillers, coloring,
etc.) In rare cases, these inactive ingredients might cause minor side effects like a different taste or upset stomach. It's always best to
consult your doctor if you experience any issues.

Policy and Regulatory Landscape:


Promoting generics: Governments and healthcare organizations often encourage generic use through various policies to improve access
to affordable medications and reduce overall healthcare costs.
Authorized generics: In some cases, brand-name companies may license their drugs to other manufacturers before the patent expires.
These "authorized generics" are essentially the same medication but may be sold at a slightly lower price.

Que. 312 Consider the following statements


1. A quasi-moon is a celestial object that orbits the Earth like a true moon but is not gravitationally bound to it.
2. Asteroids can be classified as quasi-moons if they share Earth's general path around the Sun.
Which of the following statements is/are correct?

1. 1 only
2. 2 only
3. Both 1 and 2
4. Neither 1 nor 2

Correct Option - 3
The correct answer is option 2.
Key Points
Quasi-moon : A quasi-moon, also sometimes called a co-orbital object or a mini-moon, is a celestial object that isn't a true satellite of a
planet but shares a similar orbital path around the Sun.

Key Characteristics:
Orbit: A quasi-moon's orbit around the Sun is similar to a planet's orbit, but not identical. It might have a slightly different eccentricity
(ovalness) or inclination (tilt) compared to the planet.
Gravitational Influence: While a true moon is gravitationally bound to its planet and orbits it directly, a quasi-moon experiences a
weaker gravitational influence from the planet. This allows it to maintain its own distinct orbit.
Temporary Occupancy: Quasi-moons can sometimes enter and exit a planet's orbital zone over long periods.

Examples:
Earth has several known quasi-moons, including asteroids like 2006 FV35, 2013 LX28, and the most stable one, 469219 Kamoʻoalewa.
Mars also has a few quasi-moons, such as the asteroids Phobos and Deimos, which are much closer to the planet than most quasi-moons
and are tidally locked (meaning they always show the same face to Mars).

Distinguishing from True Moons:

The key difference between a quasi-moon and a true moon lies in the gravitational relationship. True moons are firmly bound to their
planets by gravity and orbit them in a relatively fixed path. Quasi-moons, on the other hand, have a looser association and exhibit more
complex orbital patterns.

Significance of Quasi-Moons:

175/262
Studying quasi-moons can provide insights into the formation and evolution of planetary systems. They might be remnants of early
solar system collisions or captured asteroids. They can also be potential targets for future space exploration missions.

Que. 313 Consider the following statements


1. The Titan submersible was a deep-sea submersible designed for scientific research.
2. Submersibles are underwater vehicles used solely for exploration and tourism.
3. Unmanned submersibles (AUVs) are operated remotely and can be used for tasks too dangerous for manned submersibles.
How many of the statements given above is/are correct?

1. Only one
2. Only two
3. All three
4. None

Correct Option - 1
The correct answer is option 1.
Key Points
Titan Submersible and the Fateful Dive : The Titan submersible was a deep-sea submersible designed to carry five people. It was
operated by the American company OceanGate, which specializes in tourism and expedition cruises. On June 18, 2023, the Titan tragically
imploded during an expedition to explore the wreckage of the RMS Titanic in the North Atlantic Ocean. All five people on board died in
the accident. The submersible was found in a debris field, and investigators believe that a catastrophic implosion caused the deaths.

The Titan submersible was designed for tourism and expedition cruises, not scientific research. Hence ,Statement 1 is incorrect.
.Submersibles have various applications, including scientific research. Hence ,Statement 2 is incorrect
AUVs are a type of submersible used for various tasks, including those deemed risky for manned submersibles. Hence ,Statement 3 is
correct.

Submersible Vehicles

Submersibles are underwater vehicles capable of operating at various depths. They are used for various purposes, including scientific
research, exploration, and tourism. There are different types of submersibles, each with its own capabilities and design.
Research Submersibles: These submersibles are designed for scientific research and are equipped with specialized tools and sensors for
data collection.
Deep-Sea Submersibles: These submersibles can withstand the immense pressure of the deep ocean and are used to explore the ocean
depths.
Manned Submersibles: These submersibles carry one or more people inside a pressure hull that protects them from the water pressure.
Unmanned Submersibles (AUVs): These submersibles are operated remotely and are used for tasks that may be too dangerous or
difficult for manned submersibles.
The development of submersibles has revolutionized our understanding of the ocean. They have allowed us to explore the deepest parts of the
ocean and make groundbreaking discoveries about the marine environment.

Que. 314 Which of the following statements is correct regarding the Centralized Laboratory Network (CLN)?

1. It is the world's largest network of laboratories for routine diagnostic testing.


2. It is a global network designed to expedite vaccine evaluation during outbreaks.
3. It consists of laboratories solely focused on cancer research.
4. It is an initiative independent of the Coalition for Epidemic Preparedness Innovations (CEPI).

Correct Option - 2
The correct answer is option 2.
Key Points
Centralized Laboratory Network (CLN) : The Centralized Laboratory Network (CLN) is a global network of laboratories that work
together to test vaccines for use during pandemics and epidemic disease outbreaks. It is the largest group of its kind in the world, and it
is part of the Coalition for Epidemic Preparedness Innovations (CEPI).

The CLN consists of 15 partner facilities in 13 countries. These laboratories are all equipped with the same standardized methods and
materials, which allows them to quickly and accurately evaluate vaccine candidates. This is essential for speeding up the
development of vaccines during public health emergencies.

The CLN is a key part of CEPI's 100 Days Mission, which aims to reduce vaccine development timelines to just 100 days. By
having a global network of laboratories in place, CEPI can ensure that the most promising vaccine candidates are evaluated quickly
and efficiently. This can save lives during a pandemic outbreak.

India joined the CLN in 2021. The only laboratory of this kind in India is located at the Translational Health Science & Technology
Institute (THSTI) in Faridabad.

176/262
Que. 315 Consider the following statements
1. Lithium Iron Phosphate (LFP) is a commonly used cathode material due to its high energy density.
2. Lithium-ion batteries are not suitable for grid storage applications due to their safety concerns.
How many of the statements given above is/are correct?

1. 1 only
2. 2 only
3. Both 1 and 2
4. Neither 1 nor 2

Correct Option - 2
The correct answer is option 2.
Key Points
Lithium-ion batteries : A lithium-ion battery, or Li-ion battery, is a type of rechargeable battery that powers many of our portable
electronics today. They are known for their high energy density, meaning they can store a lot of energy in a small space, and their long
lifespan.

Lithium Iron Phosphate (LFP) is actually known for its high safety and long lifespan, but it has a lower energy density compared to
other cathode materials like Lithium Cobalt Oxide (LCO). Hence, Statement 1 is Incorrect.
Lithium-ion batteries are increasingly being explored for grid storage applications due to their scalability and ability to discharge
quickly. However, safety measures need to be considered during large-scale implementation. Hence ,Statement 2 is Incorrect.

Li-ion batteries are made up of several key components:


Anode: The anode is the negative electrode of the battery. During discharge, lithium ions flow from the anode to the cathode. Common
anode materials include graphite and lithium titanate oxide (LTO).
Cathode: The cathode is the positive electrode of the battery. During discharge, lithium ions flow from the anode to the cathode. Common
cathode materials include lithium cobalt oxide (LCO), lithium nickel manganese cobalt oxide (NMC), and lithium iron phosphate (LFP).
Electrolyte: The electrolyte is a solution that allows lithium ions to flow between the anode and cathode. It is typically a lithium salt
dissolved in an organic solvent.
Separator: The separator is a thin, microporous membrane that prevents the anode and cathode from coming into contact with each other.
It allows lithium ions to flow through but blocks the flow of electrons.
Current collectors: Current collectors are conductive materials that collect current from the anode and cathode and deliver it to the
external circuit.

When a lithium-ion battery is discharging, lithium ions flow from the anode to the cathode. This flow of ions creates a current that can be
used to power a device. When the battery is charging, an external current forces lithium ions to flow from the cathode back to the anode.

Que. 316 The Indian Space Research Organisation's (ISRO's) Chandrayaan-3 mission aimed to:

1. Deploy a communication satellite around the Moon.


2. Study the composition of Martian rocks.
3. Demonstrate soft landing and deploy a rover on the lunar surface.
4. Investigate the possibility of life on Europa, a moon of Jupiter.

Correct Option - 3
The correct answer is option 3.
Key Points
Chandrayaan-3 : Chandrayaan-3 is the third mission in the Chandrayaan programme, a series of lunar exploration missions developed
by the Indian Space Research Organisation (ISRO). It was successfully launched on July 14, 2023, and landed on the lunar surface near
the south pole on August 23, 2023.

The mission consists of a lander named Vikram and a rover named Pragyan. The primary objective of the mission is to demonstrate
the capability of soft landing on the lunar surface and deploying a rover. The rover will carry out scientific experiments on the
lunar surface, including studying the composition of the lunar soil and the lunar atmosphere.
Chandrayaan-3 is a significant achievement for India's space program. It represents a major step forward in India's ability to explore
space and conduct scientific research on other celestial bodies.

Que. 317 Consider the following statements regarding HIV/AIDS transmission:


1. HIV can be transmitted through casual contact, such as sharing utensils or using public restrooms.
2. Sharing needles or syringes contaminated with infected blood is a major mode of transmission.
Select the correct option:

1. 1 only
2. 2 only
3. Both 1 and 2
177/262
4. Neither 1 nor 2

Correct Option - 2
The correct answer is option 2.
Key Points
AIDS : Acquired Immunodeficiency Syndrome (AIDS) is the most advanced stage of infection with the Human Immunodeficiency
Virus (HIV). HIV attacks the body's immune system, progressively weakening its ability to fight off infections and diseases. While there's no
cure for AIDS, effective treatment with antiretroviral therapy (ART) can suppress the virus, allowing people with HIV to live long and
healthy lives.
Transmission:

HIV is primarily transmitted through:


Unprotected sexual contact with an infected person
Sharing needles or syringes contaminated with infected blood
From mother to child during pregnancy, childbirth, or breastfeeding

Symptoms:

Early HIV infection often has no noticeable symptoms. As the virus progresses, symptoms may include:
Fever
Fatigue
Weight loss
Swollen lymph nodes
Opportunistic infections (infections the body normally fights off easily)

Global Impact:
Millions of people worldwide live with HIV/AIDS.
While new infections are declining, significant disparities exist across regions and populations.
Stigma and discrimination remain major barriers to testing and treatment.

The Fight Against AIDS:


Major advancements have been made in treatment, with ART allowing people with HIV to live healthy lives.
Global initiatives focus on:
Increasing access to testing and treatment
Promoting safe sex practices
Reducing stigma and discrimination
Researching a cure for HIV

Que. 318 Consider the following statements


1. All non-sugar sweeteners are calorie-free and do not contribute to blood sugar rise.
2. Sugar alcohols are naturally occurring alternatives to sugar and provide a similar level of sweetness.
3. Non-sugar sweeteners may be beneficial for weight management and dental health but require moderation due to potential digestive
issues.
How many of the statements given above is/are correct?

1. Only one
2. Only two
3. All three
4. None

Correct Option - 1
The correct answer is option 1.
Key Points
Non sugar sweetners : Non-sugar sweeteners are a category of substitutes for table sugar (sucrose) that provide sweetness without the
same caloric impact.

While artificial sweeteners are mostly calorie-free, sugar alcohols contain some calories (less than sugar). Hence ,Statement 1
is Incorrect.
Sugar alcohols are not naturally occurring in high amounts but can be derived from natural sources. Their sweetness level
varies. Hence ,Statement 2 is Incorrect.
Non-sugar sweeteners can be helpful for weight management and some sugar alcohols benefit dental health. However, moderation is
key due to potential digestive issues. Hence ,Statement 3 is Correct.

They can be broadly categorized into two main groups:


1. Artificial Sweeteners: These are synthetically created compounds with high intensity sweetness. They often don't contribute any
calories or carbohydrates to the diet. Some common examples include aspartame, sucralose, saccharin, and acesulfame K.

178/262
2. Sugar Alcohols: These are naturally occurring or man-made sugar alcohols with a sweetness level varying from less to as sweet as
sugar. They provide fewer calories than sugar (around 0.2-3 calories per gram compared to 4 calories per gram for sugar) but still
contribute some calories to the diet. Some popular sugar alcohols include erythritol, xylitol, sorbitol, and maltitol.

Here's a breakdown of some key points to consider with non-sugar sweeteners:


Benefits:
Can be helpful for managing weight or blood sugar levels in individuals with diabetes or prediabetes.
May promote dental health by reducing the risk of cavities (for some sugar alcohols like xylitol).
Potential Drawbacks:
The long-term safety of some artificial sweeteners is still under investigation.
Excessive consumption of sugar alcohols can cause digestive issues like bloating or gas.
May alter taste perception and lead to increased cravings for sweet foods.

Que. 319 Consider the following statements about RoboMapper :-


1. It is used to develop alloys with perovskites materials that can do material testing in a more efficient and sustainable manner.
2. It automates the process of material testing.
Which of the following statements is/are correct?

1. 1 only
2. 2 only
3. Both 1 and 2.
4. Neither 1 nor 2.

Correct Option - 3
The correct answer is option 3.
Key Points
RoboMapper:-

Recently, scientist has developed a new technology named ‘RoboMapper’ which have been able to develop alloys with ‘perovskites
materials’ that can improve solar cells.Hence, Statement 1 is correct
It is able to develop alloys with perovskites materials that can do material testing in a more efficient and sustainable manner.
It also automates the process of material testing, by placing dozens of samples on each chip by miniaturizing the material samples
with the help of modern printing.Hence, Statement 2 is correct.​

​Perovskites:-

It is a naturally occurring mineral of calcium titanate with a chemical formula of CaTiO3.


This mineral was first discovered by German mineralogist Gustav Rose in 1839.​

​Why is it significant of solar energy extraction?

All photovoltaic solar cells rely on semiconductors — materials in the middle ground between electrical insulators such as glass and
metallic conductors such as copper — to turn the energy from light into electricity.
Light from the sun excites electrons in the semiconductor material, which flow into conducting electrodes and produce electric
current
Perovskites is desirable because it can absorb sunlight better than silicon, which means the cells can be lighter.
Silicon has been the primary semiconductor material used in solar cells since the 1950s, as its semiconducting properties align well
with the spectrum of the sun’s rays and it is relatively abundant and stable.
However, the large silicon crystals used in conventional solar panels require an expensive, multi-step manufacturing process that
utilizes a lot of energy.

How it can be processed into cells?

Perovskite solar cells can be manufactured using simple, additive deposition techniques, like printing, for a fraction of the cost and
energy.
Because of the compositional flexibility of perovskites, they can also be tuned to ideally match the sun’s spectrum.
In 2012, researchers first discovered how to make a stable, thin-film perovskite solar cell with light photon-to-electron conversion
efficiencies over 10%, using lead halide perovskites as the light-absorbing layer.
Since then, the sunlight-to-electrical-power conversion efficiency of perovskite solar cells has skyrocketed, with the laboratory record
standing at 25.2%.

Que. 320 Consider the following statements regarding the navigation system Sagar Sampark:
1. Sagar Sampark is an advanced Differential Global Navigation Satellite System (DGNSS) initiative aimed at refining maritime
navigation through enhanced accuracy in positioning information.

179/262
2. The system enhances maritime safety by significantly reducing the risks associated with maritime navigation, such as collisions and
groundings, through its precise positioning capabilities.
3. Sagar Sampark's approach to improving maritime navigation relies solely on utilizing the Global Positioning System (GPS) for positional
corrections, without integrating other satellite constellations.
Which of the statements given above is/are correct?

1. 1 and 3 only
2. 1 and 2 only
3. 2 and 3 only
4. 1,2 and 3

Correct Option - 2
The correct answer is Option 2.
Key Points
SAGAR SAMPARK

It is an indigenous Differential Global Navigation Satellite System (DGNSS).


DGNSS is a terrestrial based enhancement system which corrects the errors and inaccuracies in the Global Navigation Satellite
System (GNSS) allowing for more accurate positioning information. Hence Statement 1 is correct.
The introduction of the DGNSS system will have a profound impact on maritime navigation, enabling safer and more efficient
movement of vessels in port and harbor areas.
Features:
Enhanced Safety: The DGNSS service will assist mariners in safe navigation, reducing the risk of collisions, groundings, and
accidents at sea.
Accuracy Improvement: The DGNSS significantly improves the accuracy of GPS positioning, reducing errors caused by
atmospheric inferences, satellite clock drift and other factors. Hence Statement 2 is correct.
Redundancy and Availability: The DGNSS incorporates multiple satellite constellations like GPS and Global Navigation
Satellite System (GLONASS), ensuring increased availability and redundancy as per international standards. Hence Statement
3 is correct.
Precise Positioning: Mariners can now improve their positioning within 5 meters using the latest DGNSS system, enabling
better navigation and reducing the margin of error.
Meeting International Obligations: The DGNSS fulfills the international obligations of the International Maritime
Organization (IMO), Safety of Life at Sea (SOLAS), and the International Association of Marine Aids to Navigation and
Lighthouse Authorities (IALA

Additional Information
Other Government Initiatives Related to Maritime Security

NavIC: NavIC or Indian Regional Navigation Satellite System (IRNSS) is designed with a constellation of 7 satellites and a network
of ground stations operating 24×7.NavIC coverage area includes India and a region up to 1500 km beyond Indian boundary.
IMO has recognized NavIC as a component of the World-Wide Radio Navigation System (WWRNS).
SAGAR Vision: In 2015, India unveiled it's strategic vision for the Indian Ocean i.e. Security and Growth for All in the Region
(SAGAR) to improve relations with its maritime neighbours on the economic and security fronts.
Indian Ocean Naval Symposium: The IONS is a voluntary and inclusive initiative that brings together navies of Indian Ocean
Region (IOR) littoral states to increase maritime co-operation and enhance regional security.
Maritime India Vision 2030: It aims to accelerate the growth of the maritime sector over the next decade by enhancing infrastructure,
efficiency, services and capacity across ports, shipping and waterways.

Que. 321 Consider the following statements about microsites project:-


1. These are defined geographical regions where focused outreach efforts would be made to onboard small and medium-scale
private healthcare providers.
2. They are divided into two categories depending upon the type of facilities.
3. They are implemented by the central government.
How many of the above statements is/are correct?

1. Only one
2. Only two
3. All three
4. None

Correct Option - 2
The correct answer is option 2.
​Recently, the National Health Authority (NHA) announced 100 Microsites projects for accelerated adoption of the Ayushman Bharat
Digital Mission (ABDM) across the country.: The first ABDM Microsite under the 100 Microsites Project by the National Health
Authority is launched in Aizawl, Mizoram.
Key Points
180/262
Microsites Project:-
These are defined geographical regions where focused outreach efforts would be made to onboard small and medium-scale private
healthcare providers.Hence, Statement 1 is correct.
Types of facilities to be covered under a Microsite:

Private facilities such as standalone clinics, polyclinics, nursing homes, small hospitals (preferably <10 beds), labs, pharmacies, and
any other healthcare facilities wherein health records are generated.
Health Facilities and Health Professionals from all the systems of medicine

Categories of a Microsite: Any State/UT can choose to implement any of the following categories of Microsites as per the regional
priorities.

Category A Microsite: A microsite which consists of at least 1000 facilities, inclusive of all types of health facilities.
Category B Microsite: A microsite that consists of at least 500 facilities but less than 1000 facilities, inclusive of all types of health
facilities.

Implementation:

These Microsites would be majorly implemented by State Mission Directors of Ayushman Bharat Digital Mission (ABDM), while
the financial resources and overall guidance would be provided by NHA. Hence, Statement 3 is incorrect.
This team will spread awareness about the benefits of ABDM. It will help the service providers to join the core registries under
ABDM, besides promoting the use of ABDM to enable digital solutions for regular clinical documentation.
Patients will be able to link the health records generated at these facilities with their Ayushman Bharat Health Accounts
(ABHAs). They will be able to view and share these records using any ABDM-enabled Personal Health Record (PHR)
application on their phones.

Que. 322 Consider the following statements regarding the Short Anti-TB Regimen (BPaL):
1. BPaL stands for a combination of bedaquiline, pretomanid, and linezolid, offering a treatment regimen significantly shorter
than the traditional 18-month protocols.
2. Clinical trials across multiple countries have demonstrated a favourable outcome with BPaL in 84% to 94% of cases involving the most
severe TB patients.
3. As of the statement, BPaL has not yet been incorporated into India's national TB program, but all three component drugs of the regimen
have been approved for use in the country.
How many of the above statements given above is/are correct?

1. Only one
2. Only Two
3. All of the above
4. None of the above.

Correct Option - 3
The correct answer is Option 3.
Key Points
Short Anti-TB Regimen (BPaL)

The BPaL regimen is indeed a significant advancement in the treatment of tuberculosis (TB), specially designed to target the most
severe forms of drug-resistant TB. The regimen consists of a six-month course of treatment that drastically reduces the treatment
period compared to the traditional TB treatment protocols, which can last up to 18 months. This not only has the potential to increase
patient compliance but also to make TB treatment more manageable and less burdensome for patients and healthcare systems alike.
While the BPaL regimen offers a promising advance in TB treatment, it's important to note that, as of the last available information, it
was still under clinical trials in India and had not yet been officially adopted into the national TB programme. However, the individual
drugs comprising the BPaL regimen (bedaquiline, pretomanid, and linezolid) have been approved for use in India. Hence
Statement 1 is correct.
Clinical trials conducted in multiple countries have validated the efficacy of the BPaL regimen, demonstrating success rates of 84%
to 94% among patients with the most challenging TB cases, including those with extensive drug resistance. These high success rates
highlight the potential of BPaL as a game-changer in the global fight against TB, providing hope for those affected by the most
intractable forms of this deadly disease. Hence Statement 2 is correct.
This approval indicates recognition of their safety and effectiveness, paving the way for potential future integration of the BPaL
regimen into India's TB control efforts, pending the outcomes of ongoing clinical trials and regulatory review.
In summary, the BPaL regimen represents a significant step forward in TB treatment, offering a more efficient and potentially highly
effective option for tackling even the most drug-resistant forms of TB. As clinical trials continue and more data becomes available,
there is hope that BPaL could be adopted more widely, including in India's national TB program, to help reduce the burden of this
devastating disease. Hence Statement 3 is correct.

​ Additional Information
Issue with BPaL

181/262
Side-effects: the right proportion of these three drugs is a big worry as some patients developed side-effects affecting the nervous system and
vision to BPaL.
Significance of BPaL

There has been no mortality among BPaL trial patients even though most had advanced disease.
BPaL regimen is approved by the U.S. Food and Drug Administration (FDA) for the treatment of adults with pulmonary extensively
drug-resistant tuberculosis (XDR-TB) or multidrug-resistant TB (MDR-TB) that is treatment-intolerant or non-responsive.
Oral treatment: Patients no longer have to undergo painful daily injections for months.

Tuberculosis
Cause:
It is caused by Mycobacterium tuberculosis (bacteria) and it most often affects the lungs.
Transmission:

TB is spread through the air when people with lung TB cough, sneeze or spit.
A person needs to inhale only a few germs to become infected.
With TB infection, a person gets infected with TB bacteria that lie inactive in the body. This infection can develop into TB disease if
their immune system weakens.

Symptoms:

Prolonged cough, chest pain, weakness/fatigue, weight loss, fever, etc.


Often, these symptoms will be mild for many months, thus leading to delays in seeking care and increasing the risk of spreading the
infection to others.

Diagnosis:

In the case of suspected lung TB disease, a sputum sample is collected for testing for TB bacteria.
For non-lung TB disease, samples of affected body fluids and tissue can be tested.
WHO recommends rapid molecular diagnostic tests as initial tests for people showing signs and symptoms of TB
Other diagnostic tools can include sputum smear microscopy and chest X-rays.

Treatment:
Both TB infection and disease are curable using antibiotics.
It is treated by the standard 6-month course of 4 antibiotics. Common drugs include rifampicin and isoniazid.
In drug-resistant TB, the TB bacteria do not respond to the standard drugs. Its treatment is longer and more complex. It is treated by
Bedaquiline.
In case of infection (where the patient is infected with TB bacteria but not ill), TB preventive treatment can be given to stop the onset of
disease. This treatment uses the same drugs for a shorter time.

Que. 323 Consider the following statements regarding the "zombie drug":
1. Xylazine, often referred to as the "zombie drug," has been approved by the US FDA for human consumption due to its sedative
and painkiller properties.
2. Xylazine overdoses present a unique challenge in medical response as overdose of the drug cannot be reversed by naloxone, the most
commonly used overdose reversal treatment.
Which of the statements given above is/are correct?

1. Only 1
2. Only 2
3. Both 1 and 2
4. Neither 1 and 2

Correct Option - 2
The correct answer is Option 2
Key Points
Zombie Drug

Xylazine, which goes by the street names of “tranq,” “tranq dope,” and “zombie drug,” is a new and deadly substance.
It induces sedative-like symptoms, leading to extreme sleepiness and respiratory depression, resulting in videos of individuals unable
to stand.
The drug can also cause raw wounds on the skin, which rapidly spread with repeated exposure, beginning with ulcers and hardening
into dead skin known as eschar. If left untreated, amputation may become necessary.
Xylazine can also cause amnesia and slow down breathing, heart rate, and blood pressure to extremely low levels.
Initially appearing in Philadelphia, the drug has since spread to San Francisco and Los Angeles.
Xylazine is a central nervous system depressant that acts as a sedative and a painkiller. It has been approved by the US FDA for use in
animals only.

Hence Statement 1 is incorrect.


How is it turning people into ‘zombies’?

182/262
The tranquillizer effects of xylazine can cause users to be completely knocked out at high doses.
When mixed with fentanyl, it may result in users falling asleep and waking up many hours later, in contrast to opioids’ semi-awakened
bliss.
Overdosing on xylazine has little chance of being reversed. There is also a risk of overdose if the “zombie drug” is mixed with other
substances. Hence Statement 2 is correct.
Overdose of the drug prevents one from responding to naloxone, or Narcan—the most commonly used overdose reversal treatment.
According to the New York City Health Department, 2,668 people died in New York from xylazine overdoses in 2021.

Que. 324 Consider the following statements regarding Bluesky:


1. Bluesky, a micro-blogging platform, was launched in 2019 with the vision of fostering an open and decentralized platform for
public conversation, primarily aimed at competing with traditional social networks by utilizing the AT Protocol.
2. The primary goal of Bluesky is to centralize content moderation through a top-down approach, focusing on stringent control over public
discourse.
Which of the statements given above is/are correct?

1. Only 1
2. Only 2
3. Both 1 and 2
4. Neither 1 or 2

Correct Option - 1
The correct answer is Option 1.
Key Points

Bluesky

It is a micro-blogging platform and social web built on the AT Protocol (Authenticated Transport Protocol).
The CEO of Bluesky is Jay Graber, a software engineer with a background in cryptocurrency.
Bluesky was launched in 2019 by former Twitter CEO Jack Dorsey, who chose Ms. Graber to lead the project.
Bluesky might be classified as a Twitter competitor due to its founding team but it is different in terms of its structure, as it is meant to
form part of a decentralised ecosystem.
It aims to “develop and drive large-scale adoption of technologies for open and decentralized public conversation.” Hence
Statement 1 is correct.

Features:

Bluesky is currently in private beta, meaning that only a select group has been allowed to join via invite codes.
Regular Bluesky members are also given a new invite code at periodic intervals that they can share with new applicants they deem
trustworthy.
Platform users also have the ability to set their domains as their handle, making it easier to link their accounts across ecosystems, and
authenticate their identity.
Moderation of content: It is easier to restrict sign-ups than clean up network abuse after quickly letting in a large number of users.
Bluesky will follow automated filtering, manual admin actions, and community labeling to moderate content.
Users spreading hate or bullying others have also been banned from the platform.

Additional Information

Difference between Bluesky and Mastodon

Bluesky and Mastodon both strive to be decentralized social media platforms, Bluesky is still highly controlled by its team of creators,
and entry is based on an invite code.
Mastodon has multiple servers that users can join or apply to join, making it less controlled in terms of entry.
Mastodon is also older, going back to 2016. Its servers saw over 2.5 million active users late last year .
But there are many complaints like Mastodon’s multiple server structure confused users, and that it lacked a significant user base.
Bluesky is newer and opting for a more regimented release of its product.

What is the AT Protocol?

The AT Protocol is an open-source framework for building social apps, meaning people have transparency into how it is built and what
is being developed.
It creates a standard format for user identity, follows, and data on social apps, allowing apps to interoperate and users to move across
them freely. It is a federated network with account portability.
Users of apps built on the AT Protocol would be able to move between platforms without losing their followers, media, work, and
data.

183/262
Que. 325 Consider the following statements about Direct Methanol Fuel Cells:
1. It is lightweight, powerful and long-lasting.
2. It is based on the liquid fuel DMFC technology, which uses direct methanol as the fuel to create electricity.
3. Methanol is easier to transport, store and use than hydrogen.
4. DMFCs fuel cell systems are used for off-grid and mobile applications such as Mission Critical Communication, Information Technology,
Optronics and Sensors.
How many of the given statements are correct?

1. Only one
2. Only two
3. Only three
4. All four

Correct Option - 4
The correct answer is option 4
Key Points
Direct Methanol Fuel Cells (DMFC)

The DMFC fuel cell is a technological marvel that is lightweight, powerful and long-lasting. Hence statement 1 is correct.
These fuel cells are based on the liquid fuel DMFC technology, which uses direct methanol as the fuel to create electricity. Hence
statement 2 is correct.
Storing methanol is easier than hydrogen storage since it does not require high pressures or low temperatures. Hence statement 3 is
correct.
DMFCs are the market leader in fuel cell systems for off-grid and mobile applications such as Mission Critical Communication,
Information Technology, Optronics, Sensors and Auxiliary Power. Hence statement 4 is correct.

Additional Information
Working Principle

The DMFC’s design is a lot like that of a battery.


It has two electrodes separated by a membrane: an anode and a cathode.
When coupled with oxygen, the methanol fuel cell transforms the fuel (methanol) to energy, hence the name.

Que. 326 Consider the following statements regarding Superconductivity:


1. Superconductors exhibit zero electrical resistance and perfect diamagnetism below their critical temperature.
2. The Meissner Effect is characterized by the enhancement of a magnetic field within a superconductor as it transitions into the
superconducting state.
Which of the above statements is/are correct?

1. 1 only
2. 2 only
3. Both 1 and 2
4. Neither 1 and 2

Correct Option - 1
The correct answer is option 1
Key Points
Superconductors

Superconductors are materials that display zero electrical resistance and perfect diamagnetism below a certain temperature known as
the critical temperature. Hence statement 1 is correct.
The Meissner Effect, contrary to the enhancement of a magnetic field within a superconductor, is actually the expulsion of a
magnetic field from the interior of a superconductor as it transitions into the superconducting state. Hence statement 2 is
incorrect.​

Additional Information
Superconductivity: Key Concepts

Superconductivity is observed in certain materials at temperatures below a critical threshold.


Exhibits two primary characteristics: zero electrical resistance and perfect diamagnetism.

Zero Electrical Resistance:

Allows electric current to flow without energy loss.


Revolutionary for electrical systems, offering potential for more efficient power transmission.

Perfect Diamagnetism and the Meissner Effect:

184/262
Superconductors completely repel magnetic fields (perfect diamagnetism).
The Meissner Effect is the expulsion of magnetic field lines from a superconductor upon its transition to the superconducting state.
It's the opposite of enhancing a magnetic field within the material.
Enables applications like magnetic levitation (Maglev) technology.

Applications of Superconductivity:

Energy Transmission: Superconductors can potentially transmit electricity with no power loss.
Medical Imaging: Key component in MRI machines for generating stable, high-intensity magnetic fields.
Research and Development: Utilized in particle accelerators like the LHC for high-energy particle physics experiments.
Transportation: Enables high-speed Maglev trains to float above tracks, minimizing friction.

Que. 327 Consider the following statements about India Stack:


1. India Stack enables the digital storage of crucial documents through the DigiLocker system.
2. UPI (Unified Payments Interface) is a component of India Stack that enables bio-metrically authenticated transactions.
3. Aadhaar, the world's largest biometric ID system, is an integral part of India Stack facilitating digital identity verification.
4. The Goods and Services Tax Network (GSTN) is directly managed and operated as a part of India Stack to streamline tax collection and
compliance.
Which of the above statements are correct?

1. 1, 2, and 3 only
2. 1 and 3 only
3. 2, 3, and 4 only
4. 1, 3, and 4 only

Correct Option - 2
The correct answer is option 2
Key Points
India Stack
It is a set of APIs that allows governments, businesses, startups, and developers to utilize a unique digital Infrastructure to solve
India’s hard problems towards presence-less, paperless, and cashless service delivery. Key components include Aadhaar for
biometric identity, UPI for financial transactions, and DigiLocker for document storage.

DigiLocker is indeed part of India Stack, allowing users to digitally store and share verified documents. Hence statement 1 is correct.
Although UPI is a crucial component of India Stack, it utilizes mobile number-based identity and PINs for authentication rather than
biometric authentication. Hence statement 2 is incorrect.
Aadhaar is central to India Stack, offering a biometric identity that underpins various services including KYC (Know Your Customer)
processes. Hence statement 3 is correct.
The Goods and Services Tax Network (GSTN) is related to India's tax collection and compliance framework but is not a component of
India Stack, as it operates in the domain of tax administration rather than providing the digital infrastructure stack. Hence statement 4
is incorrect.

India Stack represents a pioneering initiative that includes several digital solutions designed to enhance governance, financial inclusion, and
service delivery across the country.
Additional Information
An Application Programming Interface (API) is a set of rules, protocols, and tools for building software and applications. It specifies how
software components should interact, enabling seamless data exchange and functionality sharing between different software applications.
Essentially, APIs allow different programs to communicate with each other, serving as bridges between different software systems to enable
them to work together.
Example: Weather Data API
Imagine a weather data service that collects and processes vast amounts of meteorological data from around the globe. This service provides
an API that allows other developers to access its weather data and integrate it into their own applications or websites. For instance, a travel
app developer wants to include real-time weather forecasts for various destinations in their app. Instead of setting up their own weather
stations, which would be impractical and costly, the developer can use the weather service's API to fetch current weather conditions,
forecasts, and other relevant meteorological data.

Que. 328 Consider the following statements about Advanced Liquid Processing System (ALPS)
1. It is used to remove radioactive contaminants from the water that has been used to cool the damaged reactors at the Fukushima
Daiichi Nuclear Power Plant.
2. It is capable of removing most of the radioactive contaminants from water, except for tritium.
3. The system employs a sophisticated filtration process through multiple stages of treatment to reduce the levels of radioactive substances.
How many of the given statements are correct?

1. Only one

185/262
2. Only Two
3. All three
4. None

Correct Option - 3
The correct answer is option 3
Key Points
Advanced Liquid Processing System (ALPS)

The Advanced Liquid Processing System (ALPS) is a water treatment system designed to remove most radioactive contaminants from
the water that has been used to cool the damaged reactors at the Fukushima Daiichi Nuclear Power Plant. Hence statement 1 is
correct
It removes most of the radioactive elements except for tritium, a hydrogen isotope that is difficult to separate. Hence statement 2
is correct.
The system employs a sophisticated filtration process through multiple stages of treatment to reduce the levels of radioactive
substances. Hence statement 3 is correct.​
The International Atomic Energy Agency (IAEA) is assisting Japan to release the water into the sea.

Additional Information
Tritium is a radioactive isotope of hydrogen, denoted by the symbol "T" or "³H." It is the rarest and heaviest naturally occurring
isotope of hydrogen. Tritium has one proton and two neutrons in its nucleus, whereas the most common form of hydrogen, known as
protium, has only one proton and no neutrons.
Tritium is easily absorbed by the bodies of living beings and rapidly distributed via blood.

Que. 329 Consider the following statements regarding the Green Hydrogen initiatives in India:
1. The Ministry of New and Renewable Energy (MNRE) has defined "Green Hydrogen" as hydrogen produced exclusively
through the electrolysis of water.
2. The well-to-gate emission threshold for hydrogen to be classified as "green" by the MNRE is not exceeding 1 kg of CO2 equivalent per kg
of hydrogen.
3. The Bureau of Energy Efficiency (BEE) Nodal Authority for accrediting agencies that oversee the monitoring, verification, and
certification of green hydrogen production.
4. The National Green Hydrogen Mission aims to produce 5 million metric tonnes of green hydrogen annually by 2030.
Which of the statements given above are correct?

1. 1 and 2 only
2. 3 and 4 only
3. 2 and 3 only
4. 1, 2, and 4 only

Correct Option - 2
The correct answer is option B
Key Points
Green Hydrogen

It is defined as hydrogen produced using renewable energy sources, including but not limited to electrolysis. It also encompasses
hydrogen produced via the conversion of biomass and electricity generated from renewable sources stored or banked according to
regulations. Hence statement 1 is incorrect.
The set emission threshold for "green" classification is not exceeding 2 kg of CO2 equivalent per kg of hydrogen, not 1 kg. Hence
statement 2 is incorrect.
The Bureau of Energy Efficiency (BEE), under the Ministry of Power, has been designated as the Nodal Authority for accrediting
agencies that oversee the monitoring, verification, and certification of green hydrogen production. Hence statement 2 is correct.
India launched the National Green Hydrogen Mission with the objective of producing 5 million metric tonnes of green hydrogen
annually by 2030. Hence statement 4 is correct.

Additional Information
National Green Hydrogen Mission

India launched the National Green Hydrogen Mission with the objective of producing 5 million metric tonnes of green hydrogen
annually by 2030.
The mission aligns with a target of about 125 gigawatts of associated renewable energy capacity.
The program offers financial incentives to promote domestic production of electrolysers and green hydrogen.
These incentives are designed to facilitate rapid scale-up, technology development, and cost reduction.

Que. 330 Consider the following statements related to the 'Graphene-Aurora Program':

186/262
1. The Graphene-Aurora Program aims to promote the production of graphene through chemical synthesis methods exclusively.
2. The 'India Graphene Engineering and Innovation Centre' (I-GEIC) will be established as a for-profit entity to facilitate the
commercialization of graphene innovations.
3. The program is a collaborative effort involving the Digital University Kerala, the Ministry of Electronics & Information Technology
(MeitY), the Government of Kerala, and industry partners.
4. Andre Geim and Konstantin Novoselov, who isolated graphene in 2004, were awarded the Nobel Prize in Physics in 2010 for their work on
graphene.
How many of the given statements are correct?

1. Only one
2. Only two
3. Only three
4. All Four

Correct Option - 2
The correct answer is option 3
Key Points
Graphene-Aurora Program

'Graphene-Aurora Program' does not specifically limit its focus to the promotion of graphene production through chemical synthesis
methods exclusively. The program aims to foster innovation, research, and commercialization across a broad spectrum of graphene
production methods. Hence statement 1 is incorrect.
The 'India Graphene Engineering and Innovation Centre' (I-GEIC) will be established as a section 8 company, which is a not-for-
profit entity. Its purpose is to bridge the gap between research and development and commercialization, fostering a smoother
transition of graphene innovations from the laboratory to the market. Hence statement 2 is incorrect.
It is a collaborative effort between Digital University Kerala, MeitY, the Government of Kerala, and industry partners, including the
budget allocated for the program. The mention of Carborundum Pvt Limited as one of the key industry partners and the overall budget
outlay of Rs 94.85 Crore highlights the program's scale and collaborative nature. Hence statement 3 is correct.
Andre Geim and Konstantin Novoselov, who isolated graphene in 2004, were awarded the Nobel Prize in Physics in 2010 for their
work on graphene. Hence statement 4 is correct.​

Additional Information
Graphene

Graphene is a material that is extracted from graphite and is made up of pure carbon.
It is one of the most important elements in nature which we find in daily objects like the lead of a pencil.
It is the world’s thinnest, strongest, and most conductive material of both electricity and heat.
It conducts electricity better than copper.
It is 200 times stronger than steel but six times lighter.
It is almost perfectly transparent as it absorbs only 2% of light.
It is impermeable to gases, even those as light as hydrogen and helium.

Que. 331 Consider the following statements regarding the Finger Minutiae Record - Finger Image Record (FMR-FIR) Modality :
1. It employs a combination of artificial intelligence (AI) and machine learning (ML) technologies to ensure the authenticity of
fingerprint biometrics during Aadhaar authentication.
2. It can differentiate between a genuine, "live" finger and a cloned or fake fingerprint, thereby preventing spoofing attempts.
3. The implementation of FMR-FIR is aimed at reducing the risk of Aadhaar-enabled Payment System (AePS) frauds by enabling the
detection of cloned or fake fingerprints in real-time.
How many of the given statements are correct?

1. Only one
2. Only two
3. All three
4. None

Correct Option - 3
The correct answer is option 2
Key Points
Finger Minutiae Record - Finger Image Record (FMR-FIR) Modality

FMR-FIR modality employs a combination of AI and ML technologies to enhance the security of the Aadhaar authentication process,
specifically by analyzing finger minutiae and images. Hence statement 1 is correct.
The modality's primary function lies in assessing the liveness of the captured fingerprint. It can differentiate between a genuine,
"live" finger and a cloned or fake fingerprint, thereby preventing spoofing attempts. Hence statement 2 is correct.
FMR-FIR modality's introduction aims to reduce the risk of AePS frauds. It can detect cloned or fake fingerprints in real-time,
thereby preventing spoofing attempts and enhancing transaction security. Hence statement 3 is correct.
187/262
Additional Information
Finger Minutiae Record – Finger Image Record (FMR-FIR) modality is Unique Identification Authority of India (UIDAI) in-house
Artificial Intelligence/Machine Learning (AI/ML) technology.

Que. 332 Which of the following statements about the development of low-pungent mustard in India is correct?

1. The low-pungent mustard variety is developed using traditional breeding methods.


2. The development of low-pungent mustard involves the use of CRISPR/Cas9 gene editing, and the variety is transgene-free.
3. The genetically modified mustard variety was developed without the use of CRISPR/Cas9 and contains high levels of glucosinolates.
4. CRISPR/Cas9 was used to introduce Bacillus thuringiensis genes into mustard to reduce its pungency and increase pest resistance.

Correct Option - 2
The correct answer is option 2.
Key Points
Low Pungent Mustard:-

Indian scientists have developed the first ever low-pungent mustard that is pest and disease-resistant. It is based on CRISPR/Cas9
gene editing, while being non-GM and transgene-free.Hence, Statement 2 is correct.

Implications of Gene Editing on Plant Defense and Pest Resistance:

The modified mustard plants showcased glucosinolate levels in their seeds that were lower than the 30 ppm threshold set for
canola-quality seeds.
Interestingly, the leaves and the walls of the pods around the seeds displayed higher amounts of glucosinolates.
This increase was attributed to a disruption in the transport of these compounds.
This heightened accumulation of glucosinolates in the leaves and pods plays a crucial role in bolstering the plant's ability to resist
pests.
As a result of these genetic modifications, the edited mustard lines exhibited robust defense mechanisms against both fungal and
insect pests.​

Que. 333 Consider the following statements about the "Einstein Cross" phenomenon:
1. The "Einstein Cross" is a rare astronomical event where light from a distant quasar is magnified and replicated four times by a
massive elliptical galaxy.
2.This phenomenon validates Albert Einstein's 1915 prediction under his theory of general relativity, demonstrating how gravity can bend the
light of a distant object when a massive galaxy acts as a lens, perfectly aligning with the object and Earth.
Which of the statements given above is/are correct?

1. 1 only
2. 2 only
3. Both 1 and 2
4. Neither 1 nor 2

Correct Option - 3
The correct answer is Option 3.
Key Points
Einstein Cross

Astronomers have unveiled an extraordinary find: a seldom-seen cosmic occurrence termed the "Einstein cross," in which light from a
distal galaxy is divided and intensified by an intervening elliptical galaxy. Hence Statement 1 is correct
This incredible event, foretold by Albert Einstein in 1915, arises as a manifestation of his theory of general relativity.
In this captivating celestial spectacle, light from a galaxy in the background, probably a quasar and positioned roughly 11 billion light-
years distant, undergoes distortion and is replicated fourfold by the gravitational influence of a colossal elliptical galaxy approximately
6 billion light-years from our planet.

Hence Statement 2 is correct.


Additional Information

Why are these so rare?


It turns out that gravitational lensing happens everywhere in the universe, mostly in the form of so-called “weak lensing.
Creating an Einstein Cross requires a precise alignment of the lensing body and light source and astronomers refer to this as “strong
gravitational lensing”.

188/262
Que. 334 Consider the following statements about ' Agnibaan SubOrbital Technological Demonstrator (SOrTeD):
1. Agnibaan SOrTeD is fully developed by the Indian Space Research Organisation (ISRO) alone, without any support from other
organizations.
2. The rocket is powered by the Agnilet engine, which is notable for being the world's first 100% 3D-printed, semi-cryogenic rocket engine.
3. It has the capability of reaching an altitude of 700 km with payloads up to 100 kg and can be launched in configurations up to five different
settings.
How many of the given statements are correct?

1. Only one
2. Only two
3. All three
4. None

Correct Option - 2
The correct answer is option 2
Key Points
Agnibaan SubOrbital Technological Demonstrator (SOrTeD)

Chennai-based space startup Agnikul Cosmos Private Limited is set to launch its first rocket, the Agnibaan Suborbital Technology
Demonstrator (SOrTeD).
AgniKul Cosmos' journey is supported by the Indian Space Research Organisation (ISRO) and Indian National Space Promotion and
Authorisation Centre (IN-SPACe). Hence statement 1 is incorrect.
The Agnilet engine, a groundbreaking advancement in rocketry, is distinguished by its method of production - being 100% 3D-
printed. This highlights the innovation in its manufacturing process, making it a significant development in the field of rocket engines.
Hence statement 2 is correct.
Agnibaan SOrTeD is capable of carrying payloads up to 100 kg to an altitude of 700 km, and its versatility is showcased through its
ability to be launched in five different configurations. This adaptability makes it a notable entrant in the realm of space launch vehicles,
especially for small payloads. Hence statement 3 is correct.

Additional Information
It is India’s first launch from a private launchpad.
It is India’s first semi-cryogenic engine-powered rocket launch.
The world’s first launch of a rocket with a single-piece 3D printed engine, designed and built locally.
It is India’s first liquid oxygen-kerosene rocket flight.
Agnikul’s rocket is the second privately developed rocket in India.
Vikram-S was the India’s first privately developed rocket. Developed by the Hyderabad-based start-up Skyroot Aerospace
Private Limited. This mission was titled Prarambh.

Que. 335 Consider the following statements about the DRACO program:
1. DRACO stands for Demonstration Rocket for Agile Cislunar Operations, aimed at demonstrating a nuclear thermal rocket
(NTR) in orbit.
2. The goal of the DRACO program is to cut travel time to Mars to one month, making it significantly faster than any current propulsion
methods.
3. DRACO's nuclear-powered propulsion is expected to offer a thrust-to-weight ratio around 10,000 times greater than electric propulsion.
4. The DRACO spacecraft is designed to remain in orbit for over 300 years to ensure the safe decay of radioactive elements.
How many of the given statements are correct?

1. Only one
2. Only two
3. Only three
4. All of the above

Correct Option - 3
The correct answer is option 3
Key Points
DRACO Program

Demonstration Rocket for Agile Cislunar Operations (DRACO). Hence statement one is correct.
While the DRACO program aims to significantly reduce the travel time to Mars, it aims to cut the travel time to about three to four
months, not one month. The substantial decrease in travel time compared to conventional propulsion systems stems from the efficiency
of nuclear-powered propulsion, but the claim of reducing it to one month is inaccurate according to the provided details. Hence
statement 2 is incorrect.
Nuclear thermal rocket (NTR), like the one being demonstrated by the DRACO program, offer a high thrust-to-weight ratio,
approximately 10,000 times greater than that of electric propulsion systems, and a two-to-five times greater specific impulse compared
to in-space chemical propulsion systems. This increase in efficiency and performance is a critical advantage of nuclear-powered
propulsion. Hence statement 3 is correct.

189/262
The DRACO spacecraft is designed to orbit at an altitude of approximately 700 to 1,994 kilometers above Earth. It is expected to
remain in this orbit for over 300 years to ensure the safe and prolonged decay of radioactive elements, thus minimizing the potential
for radioactive contamination. Hence statement 4 is correct.


Additional Information
The goal of the DRACO program is to demonstrate a nuclear thermal rocket (NTR) in orbit. NTRs use a nuclear reactor to heat
propellant to extreme temperatures before exhausting the hot propellant through a nozzle to produce thrust.
Compared to conventional space propulsion technologies, NTRs offers a high thrust-to-weight ratio around 10,000 times greater than
electric propulsion and two-to-five times greater specific impulse (i.e. propellant efficiency) than in-space chemical propulsion.
It aims to leverage nuclear reactions to significantly reduce travel time, making interplanetary missions more efficient and safer.
The spacecraft will orbit at an altitude of approximately 700 to 1,994 kilometers, staying in orbit for over 300 years to ensure
safe decay of radioactive elements.

Que. 336 Consider the following statements about STEREO-A spacecraft:


1. STEREO-A, along with STEREO-B, was launched for the first time to study the Earth’s atmosphere and its impact on climate
change.
2. It is European Space Agency sun mission.
3. The main objective of STEREO-A's Earth flyby is to test a new theory suggesting coronal loops may be optical illusions and to study the
evolution of the magnetic field of Coronal Mass Ejections (CMEs) as they travel towards Earth.
How many of the given statements are correct?

1. Only one
2. Only two
3. All three
4. None

Correct Option - 1

Key Points
STEREO-A (Solar Terrestrial Relations Observatory - Ahead) along with its twin STEREO-B (Behind) was launched to study the
Sun, not the Earth’s atmosphere. Their primary goal was to provide a stereoscopic view of the Sun to enable researchers to study
solar activities and phenomena from multiple perspectives. Hence the first statement is incorrect.
It is NASA's (National Aeronautics and Space Administration) spacecraft. Hence statement 2 is incorrect.
The flyby provides an opportunity to employ stereoscopic techniques to gain 3D information about the sunspot regions and to test the
theory that coronal loops may be optical illusions. It also aims to study the evolution of the magnetic field in Coronal Mass Ejections
(CMEs) as they travel towards Earth, which is critical for understanding the potential impact of CMEs on Earth's technology and
infrastructure. Hence statement 3 is correct.​

Additional Information
By integrating its observations with those from NASA's and the European Space Agency's Solar and Heliospheric Observatory
(SOHO) and NASA’s Solar Dynamics Observatory (SDO), STEREO-A will enhance its capability to study the Sun. As it moves
closer and then farther from Earth during the flyby, it can adjust its observational precision, similar to fine-tuning the focus of a
colossal telescope spanning several million miles. This unique positioning will enable researchers to analyze the progression of the
magnetic fields within coronal mass ejections (CMEs) as they journey towards Earth, offering critical insights into these powerful solar
phenomena.

Que. 337 Consider the following statements about the Ayushman Bharat Digital Mission (ABDM):
1. The ABDM, aimed at developing India’s digital health infrastructure, was launched under the recommendations of the National
Health Policy 2017.
2. ABDM Microsites are geographically unlimited areas targeting the integration of large-scale healthcare providers into the digital
framework of ABDM.
3. A state's ABDM Microsite implementation is primarily driven by the State Mission Directors of ABDM with assistance from the NHA,
including financial resources and guidance.
4. Patients can use digital services like e-prescriptions and e-reports through ABDM-enabled facilities.
How many of the following statements are correct?

1. Only one
2. Only two
3. Only three
4. All four

Correct Option - 3
The correct answer is option 3

190/262
Additional Information
Ayushman Bharat Digital Mission

The Ayushman Bharat Digital Mission (ABDM) is a national initiative that aims to develop the digital health infrastructure of the
country, launched in September, 2021. It is a flagship scheme of India which was launched as recommended by the National Health
Policy 2017, to achieve the vision of Universal Health Coverage (UHC). Hence statement 1 is correct.
ABDM Microsites are geographically defined regions where targeted outreach initiatives are conducted to onboard small and
medium-scale private healthcare providers. Hence statement 2 is incorrect.
The Microsite project aims to ensure that all healthcare facilities, including private clinics, small hospitals, and labs, become ABDM-
enabled and offer digital health services to patients. An interfacing agency under this program will have an on-ground team to reach
out to the healthcare providers in the area.
The implementation of ABDM Microsites is spearheaded by the State Mission Directors of ABDM. They receive pivotal support
from the National Health Authority (NHA), which includes both financial resources and comprehensive guidance, ensuring a cohesive
approach towards the digitization of healthcare services within their state. Hence statement 3 is correct.
Patients can benefit from digital services such as online appointments, e-prescriptions, e-reports, e-bills, e-consent, e-referrals,
and e-feedback at these facilities. Hence statement 4 is correct.
Health records generated within these ABDM-enabled facilities can be seamlessly linked to their Ayushman Bharat Health Accounts
(ABHAs).
Furthermore, these records are accessible and shareable through any ABDM-enabled Personal Health Record (PHR) application on
mobile devices, ensuring that patients have access to their health information at their fingertips.

Additional Information
Mizoram has become the first state to operationalize an ABDM Microsite in its capital city, Aizawl.

Que. 338 Consider the following statements:


1. The first WHO draft text on Good Manufacturing Practices (GMP) was adopted in 1968, establishing a global standard for
pharmaceutical production.
2. GMP compliance in India is regulated by the Drugs and Cosmetics Act and Rules, mandating high standards in the manufacture and
distribution of pharmaceutical products.
3. Schedule M of the Drugs and Cosmetics Act outlines the GMP requirements for pharmaceutical manufacturing in India.
4. Despite the large number of pharmaceutical manufacturing units in India, only a fraction meet the WHO-GMP standards, indicating a gap
in compliance and enforcement.
How many of the following statements are correct?

1. Only one
2. Only two
3. Only three
4. All Four

Correct Option - 4
The correct answer is option 4
Key Points
Good Manufacturing Practices (GMP)

It is a set of guidelines and standards aimed at ensuring the quality, safety, and consistency of pharmaceutical, food, and other
regulated products during their manufacturing process.
The adoption of the first WHO draft text on GMP in 1968 marked a significant step towards establishing a global benchmark for
pharmaceutical manufacturing, aiming to ensure the quality, safety, and efficacy of pharmaceutical products. This initiative
underscored the importance of standardizing manufacturing practices to safeguard public health internationally. Hence statement 1 is
correct.
In 1969, when the World Health Assembly recommended the first version of the WHO Certification Scheme on the quality of
pharmaceutical products moving into the global market, it accepted the WHO GMP as an integral part of the Scheme.
Compliance with GMP in India is not regulated under the Indian Penal Code but rather under the Drugs and Cosmetics Act and Rules.
This act provides a comprehensive framework for the regulation of drugs and cosmetics in India, including the manufacturing, sale,
and distribution of pharmaceutical products, outlining the mandatory adherence to GMP standards. Hence statement 2 is correct.
Schedule M of the Drugs and Cosmetics Act explicitly delineates the GMP requirements for pharmaceutical manufacturing within
India. It includes detailed standards and guidelines covering various facets of production, from infrastructure and equipment to
personnel training and quality assurance mechanisms. This schedule plays a crucial role in ensuring that manufactured pharmaceuticals
meet the highest quality standards. Hence statement 3 is correct.
Although India's pharmaceutical sector is one of the largest in the world, a notable discrepancy exists between the total number of
manufacturing units and those certified as WHO-GMP compliant. Out of over 10,500 manufacturing units, only 2,000 have
achieved this status, highlighting a significant challenge in meeting international quality standards across the industry. This gap
underscores the need for enhanced measures to ensure broader compliance and elevate the industry's overall quality benchmarks.
Hence statement 4 is correct.

Que. 339 Consider the following statements regarding Antimicrobial Resistance (AMR):

191/262
1. Antimicrobial Resistance (AMR) occurs exclusively due to the overuse and misuse of antibiotics, without other contributing factors.
2. The Indian Council of Medical Research (ICMR) has established the AMR Surveillance Network (AMRSN) to monitor and research
trends in drug-resistant infections across India.
3. The National Action Plan on Antimicrobial Resistance (NAP-AMR) in India adopts a One Health approach, emphasizing the involvement
of various stakeholders and ministries.
4. Studies have shown that a 10% increase in PM2.5 air pollution levels is associated with a 1.1% rise in antimicrobial resistance globally.
How many of the given statements are correct?

1. Only one
2. Only two
3. Only three
4. All four

Correct Option - 3
The correct answer is option 3
Key Points​
AMR is not solely caused by the overuse and misuse of antibiotics. While these are significant drivers of antibiotic resistance, other
factors also contribute to the rise of AMR. These include lack of access to clean water, sanitation and hygiene (WASH) for both
humans and animals, poor infection prevention and control measures in healthcare and agricultural settings, limited access to quality
healthcare services, and a deficiency in awareness and knowledge regarding AMR. Hence statement 1 is incorrect.
The AMR Surveillance Network (AMRSN), established by the Indian Council of Medical Research (ICMR), plays a crucial role
in tracking and understanding the patterns of drug-resistant infections within India. This initiative is pivotal for generating data-driven
evidence that informs policy-making and helps in developing strategies for combating AMR effectively. Hence statement 2 is correct.
The National Action Plan on Antimicrobial Resistance (NAP-AMR) in India follows a One Health approach, recognizing the
interconnections between human, animal, and environmental health. This comprehensive strategy calls for the collaboration of various
stakeholders, including different ministries and departments, to address the multifaceted challenges posed by AMR, reflecting a
cohesive and coordinated effort at the national level. Hence statement 3 is correct.
Recent studies published in The Lancet Planetary Journal, have illuminated the relationship between air pollution, specifically PM2.5
particles, and the exacerbation of AMR. The finding that every 10% increase in air pollution correlates with a 1.1% increase in
AMR stresses the environmental dimensions of AMR and suggests that addressing pollution could be a critical component in the
global strategy against the spread of drug-resistant infections. Hence statement 4 is correct.

Additional Information
Antimicrobial Resistance (AMR) is the phenomenon where microorganisms such as bacteria, viruses, fungi, and parasites evolve to
resist the effects of antimicrobial drugs that were previously effective against them. This resistance makes the standard treatments
ineffective, resulting in persistent infections, increased risk of spreading to others, and enhanced severity of illness or mortality.
AMR occurs naturally over time, often through genetic changes. However, it is accelerated and spread by several human actions,
including the overuse and misuse of antimicrobial medicines in humans, animals, and agriculture. Contributing factors also include
poor infection prevention and control in healthcare facilities and farms, inadequate sanitary conditions, and lack of access to clean
water.

Que. 340 Consider the following statements about tidal locking:


I. It occurs when a smaller celestial body's rotation becomes synchronized with its orbital period around a larger body.
II. It is primarily caused by the differential gravitational forces acting on the rotating body.
III. It results in one side of the smaller body always facing the larger body.
How many of the statements given above is/are correct ?

1. Only one
2. Only two
3. All three
4. None

Correct Option - 3
The correct answer is option 3.
Key Points
Tidal locking : Tidal locking is a fascinating phenomenon that occurs between two celestial bodies orbiting each other. It's when one object
becomes "locked" in its rotation relative to another, always keeping the same side facing it.
Statement I is correct: Tidal locking occurs when the rotation of a smaller celestial body becomes synchronized with its orbital period
around a larger body. This means the smaller body takes the same amount of time to spin on its axis as it does to complete one revolution
around the larger body.
Statement II is correct: The primary cause of tidal locking is the differential gravitational forces acting on the rotating body. The closer
side of the smaller body experiences a stronger gravitational pull from the larger body than the farther side. This creates a tidal bulge on
the closer side. Over time, the friction caused by this bulge slows down the rotation of the smaller body until it reaches synchronous
rotation.

192/262
Statement III is correct: As a result of tidal locking, one side of the smaller body always faces the larger body. This is because the tidal
bulge becomes locked in place, unable to overcome the gravitational pull.

How it works:
Gravity plays a key role. The gravitational pull of one body (usually larger) creates a bulge on the other body.
This bulge isn't perfectly aligned with the gravitational pull all the time. As the object rotates, the bulge gets tugged on slightly, causing a
friction-like effect.
Over a long time (millions or billions of years), this tugging slows down the rotation of the smaller object. Eventually, it reaches a point
where it takes exactly as long to spin on its axis as it does to complete one orbit around the larger body.

Examples:
Our moon is tidally locked to Earth. That's why we always see the same side!
Many moons in our solar system, like the larger moons of Jupiter and Saturn, are tidally locked to their planets.
Pluto and its moon Charon are a special case, where both are tidally locked to each other, always facing the same side.

Effects:
Tidal locking can have significant consequences. On a tidally locked moon, one side constantly faces the sun, leading to extreme
temperature variations between the "day" and "night" sides.

Que. 341 Consider the following statements about the Y chromosome:


I. Recent advancements have allowed for the complete sequencing of the Y chromosome for the first time.
II. The Y chromosome is known to have more genes compared to the X chromosome.
III. Studies suggest a possible disappearance of the Y chromosome in the distant future.
How many of the statements given above is/are correct ?

1. Only one
2. Only two
3. All three
4. None

Correct Option - 2
The correct answer is option 2.
Key Points
Y chromosomes : The Y chromosome is one of the two sex chromosomes in humans and many other mammals. It plays a crucial role in sex
determination and development, but it's also a fascinating topic for other reasons.
Sequencing the entire Y chromosome was a challenge due to its repetitive nature. Recent advancements have enabled scientists to achieve
the first complete sequencing of the Y chromosome in multiple individuals. Hence ,Statement I is correct:
The Y chromosome is significantly smaller and contains fewer genes compared to the X chromosome. Hence, Statement II is incorrect
Research suggests the Y chromosome is undergoing a process of losing genes over time, raising the possibility of its eventual
disappearance in the very distant future. Hence ,Statement III is correct:

Function:
Sex Determination: The Y chromosome carries the SRY gene, which triggers the development of testes in males. Without a Y
chromosome, an embryo develops female characteristics.
Sperm Production: Other genes on the Y chromosome are essential for sperm development and function.

Structure and Uniqueness:


Smaller and Simpler: Compared to the X chromosome, the Y chromosome is much smaller and has fewer genes. It's also quite repetitive
in its DNA structure.
Degeneration: Research suggests the Y chromosome might be "degenerating" over time, losing genes at a slow pace. This has sparked
discussions about its potential disappearance in the very distant future.

Recent News:
Complete Sequencing: Recent advancements have allowed scientists to achieve the first complete sequencing of the Y chromosome, not
just from one person, but from multiple individuals. This opens doors for a deeper understanding of its role in biology.

Que. 342 Consider the following statements about gene drive technology:
1. It utilizes engineered genetic elements to introduce new traits into a population and ensure their spread across generations.
2. It is a currently proven and widely deployed technique for controlling insect-borne diseases.
Select the correct option:

1. 1 only
2. 2 only
3. Both 1 and 2
4. Neither 1 nor 2

Correct Option - 1
193/262
The correct answer is option 1.
Key Points
Gene drive technology : It is a powerful and controversial tool in genetic engineering. Here's a breakdown of its key aspects. A method to
alter the inheritance of specific genes within a population. Utilizes engineered genetic elements that spread a desired trait more effectively
than natural inheritance patterns.
Gene drive technology does use engineered genetic elements to introduce specific traits into a population. These elements are designed to
be more likely to be passed on to offspring compared to the natural version of the gene, ensuring the desired trait spreads through
generations. Hence ,Statement 1 is correct.
Gene drive technology is a relatively new and evolving field. While it holds promise for controlling diseases like malaria by modifying
mosquito genes, it is not yet a proven and widely deployed technique. Safety concerns and regulations are still being addressed. Hence
,Statement 2 is incorrect.

How it works:
Involves inserting genetic material with a specific purpose (e.g., disease resistance) into an organism's genome.
This engineered element is designed to "outcompete" the natural version of the gene during reproduction, increasing the likelihood of
offspring inheriting the desired trait.

Applications:
Potential benefits include:
Controlling insect-borne diseases like malaria by modifying mosquito genes.
Eradicating invasive species that harm ecosystems.
Introducing beneficial traits into crops, such as resistance to pests or herbicides.

Concerns:
Potential risks include:
Unintended consequences on ecosystems if the modified genes spread uncontrollably.
Ethical considerations regarding manipulating entire populations.
The possibility of gene drives escaping into unintended environments.

Current Status:
Gene drive research is ongoing, with scientists working on improving control mechanisms and addressing safety concerns.
Strict regulations and open discussions are crucial before widespread use of this technology.

Que. 343 Hyperloop is a proposed transportation system that utilizes:

1. Magnetic levitation and high-pressure tubes for ultra-fast travel.


2. Supersonic jets and elevated tracks for intercity commutes.
3. Self-driving cars and a network of interconnected highways.
4. Underwater tunnels and high-speed electric trains

Correct Option - 1
The correct answer is option 1.
Key Points
Hyperloop : Hyperloop is a futuristic transportation system that promises ultra-fast travel for passengers and cargo.
Magnetic levitation and high-pressure tubes is the defining characteristic of Hyperloop. Magnetic levitation eliminates friction within the
tube, and near-vacuum pressure minimizes air resistance, allowing for high speeds.
It's still in the development stages, but here's the gist of it:
High-Speed Travel: Hyperloop aims to propel pods through near-vacuum tubes at speeds exceeding 700 mph (1,126 km/h). This could
significantly reduce travel times between cities.
Reduced Friction: The secret lies in the tubes. By maintaining a near-vacuum inside, the pods experience very little air resistance.
Additionally, magnetic levitation technology keeps the pods frictionlessly gliding inside the tube.
Pods and Tubes: The system consists of a network of elevated or underground tubes. Passengers or cargo board these pods which zip
through the tubes at high speeds.

While the concept is exciting, there are challenges:


Technical Hurdles: Building and maintaining ultra-high-pressure tubes across long distances is an engineering feat. Safety, evacuation
procedures, and potential emergencies also need to be addressed.
Commercial Viability: The cost of construction and operation is a major unknown. Hyperloop companies are working on making the
system commercially viable.

Current Status:

Several companies are developing hyperloop technology, with some conducting test runs on short tracks. While a functional hyperloop
system is yet to be realized, ongoing research holds promise for the future of transportation.

Que. 344 Which of the following statements correctly describes a nuclear reactor achieving a 'critical' state?

194/262
1. The reactor is operating at unsafe temperature levels, posing a significant risk of meltdown.
2. The power output of the reactor is decreasing, indicating a malfunction
3. The reactor has reached a state where the nuclear fission reaction is self-sustaining, producing constant power.
4. The reactor is shut down safely for maintenance and refueling operations.

Correct Option - 3
The correct answer is option 3.
Key Points
Criticality of a nuclear reactor:-

It is the first step towards power production. A nuclear reactor is said to be critical when the nuclear fuel inside a reactor sustains
a fission chain reaction.Hence, Statement 3 is correct.
Each fission reaction releases a sufficient number of neutrons to sustain a series of reactions. Heat is produced in the event, which
is used to generate steam that spins a turbine to create electricity.
Fission is a process in which the nucleus of an atom splits into two or more smaller nuclei, and some byproducts.
When the nucleus splits, the kinetic energy of the fission fragments (primary nuclei) is transferred to other atoms in the fuel as heat
energy, which is eventually used to produce steam to drive the turbines.
Recently, the fourth unit of Kakrapar Atomic Power Station (KAPS), Gujarat has achieved its first Criticality — the beginning of
the regulated fission reaction — paving the way for its eventual transition to generating power for commercial use.

Que. 345 Consider the following statements regarding hypertension:


1. Hypertension is characterized as having a blood pressure measurement of 140/90 mm Hg.
2. Systolic pressure is indicated by the first number and represents the pressure in the blood vessels when the heart contracts.
3. The World Bank has released its first-ever report on the global impact of hypertension.
How many of the above statements are correct?

1. Only One
2. Only Two
3. All three
4. None

Correct Option - 2
The correct is Option 2.
Key Points
What is Hypertension?

Hypertension, or elevated blood pressure, occurs when the force within your blood vessels exceeds normal levels (140/90 mmHg or
above). Hence Statement 1 is correct.
While it's a frequent condition, lack of management could lead to grave health issues. Blood pressure readings consist of two digits.
The initial (systolic) figure indicates the force in the blood vessels during a heartbeat, while the subsequent (diastolic) figure shows
the pressure while the heart is in a resting phase between beats. Hence Statement 2 is correct.
To raise awareness and motivate individuals to manage and prevent hypertension, World Hypertension Day is observed annually on
May 17.

Global Report on Hypertension


The World Health Organization's latest report on hypertension paints a concerning picture of a growing global health crisis, with significant
implications for millions of people worldwide. Hence Statement 3 is correct.
Drawing upon the key highlights of the report, a multi-faceted, inclusive, and sustainable approach is required to address this crisis
effectively. This solution framework focuses on the prevention, early detection, effective management, and the reduction of disparities
in hypertension treatment. It includes:

Prevention:Public Awareness and Education:


Launch global and national public awareness campaigns to educate people about the risks of hypertension and the importance of
maintaining a healthy lifestyle.Promote dietary approaches to stop hypertension (DASH) and encourage physical activity, smoking
cessation, and moderation in alcohol consumption.
Policy Interventions:Implement policies to reduce salt intake across the population, including reformulating processed foods to
contain less salt and providing clear nutrition labeling.Support urban planning and policies that promote physical activity, such as cycle
lanes, parks, and pedestrian areas.
Early Detection:Screening Programs:Introduce regular, free, or low-cost blood pressure screening programs within communities,
especially targeting at-risk populations.Leverage technology and mobile health units to extend screening reach to remote and
underserved communities.
Integrated Healthcare Services:Integrate hypertension screening with other routine healthcare services to ensure early detection.
Train healthcare providers to recognize the importance of regular blood pressure checks and management advice.
Effective Management:Accessibility to Treatment:Ensure the availability and affordability of essential medicines for hypertension
across all regions, prioritizing the poorest and most vulnerable.Encourage the use of fixed-dose combinations to improve adherence to

195/262
treatment.
Patient-Centered Care:Develop individualized care plans that consider the patient's lifestyle, preferences, and socio-economic status.
Enhance patient education on the importance of adherence to medication and lifestyle changes.
Reducing Disparities in Treatment Coverage:
Global Collaboration:Foster international cooperation to share best practices, innovations, and resources for hypertension control.
Support low- and middle-income countries through funding, technology transfer, and capacity-building initiatives.
Universal Health Coverage:Integrate hypertension control programs into national health strategies towards achieving universal health
coverage.
Ensure that hypertension prevention, detection, and treatment services are included in national health benefit packages without
financial hardship to patients.
Monitoring and Research.
Data Collection and Analysis:Implement robust systems for monitoring and evaluation to track progress on hypertension control
efforts, identify gaps, and adjust strategies accordingly.
Promote research on new and innovative approaches to hypertension control, focusing on cost-effectiveness and scalability.
Stakeholder Engagement:Engage a wide range of stakeholders, including governments, the private sector, non-governmental
organizations, and communities, in a concerted effort to combat hypertension.
Leverage the role of communities and patient groups in advocating for better hypertension care and support.

By adopting a comprehensive approach that includes prevention, early detection, effective management, and reducing treatment disparities,
alongside robust monitoring and stakeholder engagement, the global community can make significant strides in controlling the hypertension
epidemic. This requires a strong commitment from all sectors of society and sustained investment in health systems strengthening toward
universal health coverage.

Que. 346 Gujarat Declaration is related to:

1. Modernization of major and minor Ports.


2. Protection of biodiversity of Gir Forests.
3. It reaffirms global commitment and harness potential of traditional medicine to achieve health and well-being for all
4. Providing instant loans to start ups.

Correct Option - 3
The correct answer is Option 3.
Key Points
Gujarat Declaration:
The WHO Traditional Medicine Global Summit 2023 in Gujarat, India, concluded with the "Gujarat Declaration", a document that
reaffirms the global commitments to indigenous knowledge, biodiversity, and traditional, complementary, and integrative medicine (TCIM).
Hence Statement 3 is correct.
Key Points of Declaration:

Scientific Rigor:Stresses the application of strict scientific methods within traditional medicine to enhance understanding, evaluate
effectiveness, and guide usage.Highlights the necessity of appreciating the holistic, context-specific, and individualized nature of
traditional medicine in improving health and well-being.
India's Role:Recognizes India's significant contribution through hosting the WHO Global Traditional Medicine Centre in Jamnagar,
Gujarat, aiding the global advancement of traditional medicine.Acknowledges India's commitment to promoting traditional medicine
on a global scale.
Action Points:Proposes action points focused on promoting individual and planetary well-being, evidence-based research in
traditional medicine, universal health coverage, strengthening primary healthcare, leveraging digital technologies, preserving
biodiversity, ensuring equity and human rights, and adhering to ethical principles in traditional medicine practices.
Commitment to Traditional Medicine:Highlights India's dedication to advancing traditional medicine systems, emphasizing
collaborative and sustainable integration into modern healthcare.
Harnessing Traditional Medicine:Sees the declaration as a step toward maximizing the potential of traditional medicine through
scientific validation and integration into healthcare systems, as voiced by the WHO Director-General.
Universal Health Coverage (UHC) and SDGs:Aims to expand evidence-based traditional, complementary, and integrative medicine
to support UHC and achieve SDGs, calling for cross-regional and interdisciplinary collaboration.
Research and Standardization:Calls for the integration of validated traditional medicine practices into health policies, urging faster
development and regulation of proven traditional medicine products.
Digital Health and AI:Emphasizes the role of digital health technologies, particularly AI, in enhancing traditional medicine resources
to improve health outcomes.
Biodiversity and Indigenous Rights:Recommends actions for biodiversity conservation, equitable benefit-sharing, and upholding the
rights of Indigenous Peoples as per the UN declaration.
Ethical Practices:Underscores the importance of embedding ethical principles and practices in traditional medicine research and
application.

Que. 347 Consider the following statements about Tasmanian Tiger:-


1. It resembled a wolf, aside from the tiger-like stripes on its back.
2. It is a large carnivorous marsupial now believed to be extinct.
3. It was the only member of the family Thylacinidae to survive into modern times.

196/262
Which of the above statements is/are correct?

1. 1 and 2 only
2. 2 and 3 only
3. 1 and 3 only
4. 1,2 and 3.

Correct Option - 4
The correct answer is option 4.
Key Points
Tasmanian Tiger:-

The Tasmanian tiger resembled a wolf, aside from the tiger-like stripes on its back.Hence, Statement 1 is correct.
The last known Tasmanian tiger succumbed in a Tasmanian zoo in 1936.
It is a large carnivorous marsupial now believed to be extinct.Hence, Statement 2 is correct.
It was the only member of the family Thylacinidae to survive into modern times.Hence, Statement 3 is correct.
It was widespread over continental Australia, extending north to New Guinea and south to Tasmania.
It was an apex predator that hunted kangaroos and other prey. ​

Que. 348 which of the following statement is correct about Hubble constant?

1. It express the rate at which universe is expanding.


2. It express the rate at which Hubble telescope revolve around the earth.
3. It express the rate of moon deceleration.
4. It express the rate of Sun deceleration.

Correct Option - 1
The correct answer is option 1.
Key Points
Hubble Constant

The Hubble Constant is the unit of measurement used to describe the expansion of the universe. Hence statement 1 is correct.

Additional Information
Universe Expansion Dispute

The dispute surrounding the expansion of the universe centres on determining the precise rate at which it is expanding, known as the
Hubble constant.
Multiple methods have been used to calculate this constant, including analyzing the brightness of supernovae, studying the cosmic
microwave background, and observing gravitational waves.
However, these methods have provided conflicting results, with some measurements reporting higher values than others.
Researchers from the International Centre for Theoretical Sciences (ICTS), Bengaluru, the Inter-University Centre for Astronomy and
Astrophysics (IUCAA), Pune, and the University of California, Santa Barbara (UCSB), have proposed a new method to determine the
Hubble constant.
Proposed new methods, such as using lensed gravitational waves, to independently estimate the Hubble constant

Que. 349 Consider the following statements about Mars Oxygen In-Situ Resource Utilization Experiment (MOXIE):
1. It is developed by ISRO.
2. It can generate oxygen in Mars.
3. It was one of the payload of Mangalyaan mission.
How many of the statements are correct?

1. Only one

197/262
2. Only two
3. All three
4. None

Correct Option - 1
The correct answer is option 1
Key Points
MOXIE (Mars Oxygen In-situ Resource Utilization Experiment)

MOXIE is a small box sent by the Massachusetts Institute of Technology(MIT) with NASA’s Perseverance in 2019. Hence
statement 1 and 3 is incorrect.
It has produced oxygen on Mars with components from the planet’s atmosphere. Hence statement 2 is correct.
Mangalyaan Mission is India's Mar mission.

Additional Information
Global Mars missions.

ExoMars rover (2021) (European Space Agency)


Tianwen-1: China's Mars Mission (2021)
UAE’s Hope Mars Mission (UAE’s first-ever interplanetary mission) (2021)
Mars 2 and Mars 3 (1971) (Soviet Union)
NASA’s Perseverance Rover
InSight lander- NASA (2018)
Curiosity-NASA (2011)
Mars Express- European Space Agency (2003)
MAVEN- NASA (2013)

Que. 350 Consider the following statements about NASA's Juno Mission
1.The Juno Mission was launched in 2011 primarily to study the origin and evolution of Jupiter, and it indirectly contributes to our
understanding of the solar system's formation, including Earth.
2.One of Juno's objectives is to understand Jupiter's migration history, which provides insights into the solar system's dynamics and the
positioning of other planets, including Earth.
3.Juno's observations and studies of Jupiter's magnetic field aim to enhance our understanding of magnetic field generation in gas giants.
How many of the above statements are incorrect?

1. Only one
2. Only two
3. All of the above
4. None of the above

Correct Option - 4
The correct answer is Option 4
Key Points
Juno Mission Objectives:

1. Origins of the Solar System: By investigating Jupiter's core structure and atmospheric composition, Juno offers crucial data on the
solar nebula's initial makeup. This insight is vital for piecing together the formation process of all planets, including Earth, providing a
glimpse into the solar system's nascent stage and the building blocks that constituted its early formation. Hence Statement 1 is
correct.
2. Patterns of Planetary Movement: Juno's findings on the internal composition of Jupiter and evidence of its potential migration from
a more distant origin point towards the sun elucidates the early solar system's dynamic phase. This understanding affects our
interpretation of how this movement influenced the arrangement and dynamics of Earth and other planets, contributing to a coherent
narrative of the solar system's development.
3. Dynamics of the Solar System and Protection Measures: Careful planning of Juno's path was essential to avoid the contamination
of Jupiter’s moons, such as Europa with its underwater seas, preserving their untouched state. These moons are critical for exploring
life's potential evolution elsewhere in the solar system, enhancing our understanding of extraterrestrial life conditions. Hence
Statement 2 is correct.
4. Generation of Magnetic Fields: Juno's extensive data on Jupiter's intense magnetic field, the most powerful among the solar system's
planets, contribute to our understanding of magnetic field creation, particularly those generated by liquid metallic hydrogen interiors.
Insights from Jupiter can be paralleled to Earth's magnetic field, underscoring its importance in shielding our planet from solar
radiation. Hence Statement 3 is correct.
5. History of Comet Collisions: Through monitoring comet impacts on Jupiter, Juno offers a window into the pattern and impact of such
cosmic events in the early solar system. This information is pivotal for theorizing the role of comets in delivering essential components

198/262
like water and organic molecules to early Earth, possibly facilitating the advent of life.
6. Planetary Atmospheric Insights: The mission's detailed analysis of Jupiter's atmosphere reveals its composition and weather systems,
providing a deeper understanding of gas giants' atmospheric evolution. This information enriches our knowledge of Earth's
atmosphere, critical for comprehending climatic patterns and life-supporting conditions.
7. Gravitational Insights: The study of Jupiter's gravitational field by Juno unearths important details about its interior structure and the
distribution of its mass. These findings offer a lens through which the gravitational forces that helped sculpt the solar system can be
better understood.

Que. 351 Consider the following statements about K2-18 b:


1. K2-18 b is a super Earth exoplanet.K2-18 b is a super Earth exoplanet that orbits an M-type star.
2. It is within the “habitable zone” of its star
Which of the statements given above is/are correct?

1. 1 only
2. 2 only
3. Both 1 and 2
4. Neither 1 nor 2

Correct Option - 3
The correct answer is option 3.
Key Points
K2-18 b

K2-18 b is a super Earth exoplanet that orbits an M-type star. Hence statement 1 is correct.
Planet k2-18b is within the “habitable zone” of its star, that is conditions are just right – neither too hot nor too cold – for life to exist.
Hence statement 2 is correct.

Additional Information
It is 120 light years from earth and orbits the cool dwarf star K2-18 a.
It is 8.6 times as massive as Earth and the size lies that of between earth and Neptune.
Its discovery was announced in 2015.
NASA’s James Webb Space Telescope in its study of K2-18 b discovered the presence of carbon dioxide and methane.

Que. 352 Consider the following statements related to magnetospheric substorms and their impact on space weather forecasting:
1. The study of energetic ion variations during magnetospheric substorm intervals is essential for enhancing the precision of space
weather predictions.
2. The occurrence of magnetospheric substorms, which have an average duration of about 2-4 hours, depends on factors like the
Interplanetary Magnetic Field (IMF) orientation, solar wind velocity, and solar wind dynamic pressure, with a northward direction of the IMF
being a necessary prerequisite for their occurrence.
Which of the above statements is/are correct?

1. Only Statement 1 is correct.


2. Only Statement 2 is correct
3. Both statement 1 and 2
4. None of the above

Correct Option - 1
The correct answer is Option 1.
Key Points
Magnetospheric Substorm:

Substorms, or brief disturbances in the Earth's magnetosphere, lead to magnetic field dipolarization, which reconfigures the local
magnetic field from a stretched tail-like shape to a quasi-dipole shape.
This process results in increased heavy ion flux in the inner magnetosphere, providing opportunities to understand changes and
improve the accuracy of space weather forecasting, according to a new study. Hence Statement 1 is correct.
The magnetospheric substorm is a short-lived event influenced by the magnitude and direction of the Interplanetary Magnetic Field
(IMF), solar wind velocity, and solar wind dynamic pressure.
A southward direction of the IMF is a crucial condition for substorm occurrence, as it facilitates magnetic reconnection in the
dayside magnetosphere.Hence Statement 2 is correct.
Typically, the average duration of a substorm ranges from 2 to 4 hours.
During this process, a significant amount of energy, approximately 10^15 Joules, is extracted from the interaction between the solar
wind and the magnetosphere, eventually being deposited in the inner magnetosphere.

Que. 353

199/262
Under the Food Safety and Standards (Packaging) Regulations, 2018, the use of newspapers for storing and wrapping food is
prohibited.Which of the following is the reason for this prohibition?

1. Newspapers are readily available and cheap.


2. Newspapers are eco-friendly and recyclable.
3. Newspapers and their ink can lead to health issues such as cancer and liver failure.
4. Newspapers and their ink can lead to skin rashes.

Correct Option - 3
The correct answer is Opiton 3.
Key Points
"Under the Food Safety and Standards (Packaging) Regulations, 2018, the use of newspapers for storing and wrapping food is prohibited
due to the risk of contamination from environmental exposure and toxic substances in the ink, such as lead and naphthylamines."

Newspapers are readily available and cheap. While true about newspapers, does not relate to the reason for the prohibition, which is
focused on health and safety concerns.Statement 1 is incorrect.
Newspapers are eco-friendly and recyclable.This option also does not address the health risks associated with using newspapers for
food packaging, which is the core reason for the prohibition. Statement 2 is incorrect.
Newspapers, when used as food packaging, can pose health risks due to contaminants and toxic substances in the ink. These substances
include lead and naphthylamines, which are linked to serious health issues like cancer and liver failure. The ink may also contain
other harmful chemicals that act as agonists for the aryl hydrocarbon receptor (AhR), a protein known to mediate toxicity. Statement
3 is correct. Statement 4 is incorrect.

Que. 354 Consider the following statements regarding Protein Binders:-


1. These are biological agents which are used to manufacture a wide range of new food products, especially semi-solid or solid
foods.
2. These can affect the biological and nutritive value of milk proteins.
3. Recently, the Food Safety and Standards Authority of India (FSSAI) permitted the addition of protein binders in milk and milk products.
How many of the above statements is/are correct?

1. Only one
2. Only two
3. All three
4. None

Correct Option - 2
The correct answer is option 2.
Key Points
Protein Binders:-

These are biological agents have emerged as an important and required class of ingredients to manufacture a wide range of new
food products, especially semi-solid or solid foods.Hence, Statement 1 is correct.
These are known to affect the digestibility of the protein bound and thus can affect the biological and nutritive value of milk
proteins. Hence, Statement 2 is correct.
Protein binding also influences the bioavailability and distribution of active compounds.​
Recently, the Food Safety and Standards Authority of India (FSSAI) clarified that addition of protein binders in milk and milk
products are not permitted.Hence, Statement 3 is incorrect.

Additional Information
FSSAI:-

It is a statutory body established under the Food Safety and Standards Act, 2006 consolidated various acts & orders that had earlier
handled food-related issues in various Ministries and Departments.
Nodal ministry: Ministry of Health & Family Welfare.

Que. 355 How many of the following under National Mission on Interdisciplinary Cyber Physical Systems (NM-ICPS) are correctly
matched:-
1. Artificial Learning and Machine Intelligence- IIT, Kharagpur.
2. Cyber Security and Physical Infrastructure- IIT Kanpur.
3. Autonomous Navigation System- IIT Delhi

1. Only one
2. Only two

200/262
3. All three
4. None

Correct Option - 2
The correct answer is option 2.
Key Points
Below is the list of 25 Technological Verticals with their host institutes:-
S.No Name Of The TIH And Host Institute Technology Verticals
TIH Foundation for IOT and IOE Technologies for Internet of
1. at Indian Institute of Technology Things & Internet of
Bombay Everything
IIT Kharagpur AI4ICPS I-Hub
Foundation Artificial Intelligence and
2.
at Indian Institute of Technology (IIT), Machine Learning
Kharagpur
IIIT-H Data I-HUB Foundation
Data Banks & Data
3. at International Institute of Information
Services, Data Analysis
Technology, Hyderabad

I-HUB for Robotics and Autonomous Robotics & Autonomous


4.
Systems Innovation Foundation at Indian Systems
Institute of Science, Bengaluru
Cyber Security and Cyber
IHUB NTIHAC Foundation
5. Security for Physical
Infrastructure
at Indian Institute of Technology Kanpur
Computer Vision,
IHUB DRISHTI Foundation
6. Augmented and virtual
reality
at Indian Institute of Technology Jodhpur

Divyasampark iHub Device Material and


Technology Device Technology and
7.
Materials
at Indian Institute of Technology
Roorkee
IIT Patna Vishlesan I-hub Foundation Speech, Video & Text
8.
at Indian Institute of Technology, Patna Analytics
IITM Pravartak Technologies Foundation Sensors, Networking,
9.
at Indian Institute of Technology Madras Actuator & controls
NMICPS Technology Innovation Hub on
Autonomous Navigation
Autonomous Navigation Foundation
10. and Data Acquisition
(TiHAN) at Indian Institute of
systems (UAV, RoVetc)
Technology Hyderabad
I-DAPT-Hub Foundation at Indian
Data Analytics & Predictive
11. Institute of Technology (Banaras Hindu
Technologies
University), Varanasi
IIT Guwahati Technology Innovation and
Technologies for Under
12. Development Foundation at Indian
water exploration
Institute of Technology Guwahati
IIT Mandi IHub and HCI Foundation at Human Computer
13.
Indian Institute of Technology, Mandi Interaction
I-Hub Foundation for Cobotics (IHFC) at
14. Cobotics
Indian Institute of Technology Delhi
IIT Ropar Technology and Innovation
Technologies for
15. Foundation at Indian Institute of
Agriculture & Water
Technology Ropar

Technology Innovation in Exploration &


16. Technologies for Mining
Mining Foundation at Indian Institute of
Technology, Dhanbad
IIT Palakkad Technology Ihub
Intelligent Collaborative
17. Foundation at Indian Institute of
Systems
Technology, Palakkad

201/262
IIITB COMET Foundation, at Advanced Communication
18.
International Institute of Information System
Technology Bengaluru

19. BITS BioCyTiH Foundation, at Birla Bio-CPS


Institute of Technology & Science, Pilani
IDEAS- Institute of Data Engineering, Data Science, Big Data
20. Analytics and Science Foundation at Analytics and Data curation
Indian Statistical Institute Kolkata etc.
System Simulation,
21. IITI DRISHTI CPS Foundation at Indian
Modelling & Visualization
Institute of Technology Indore

IHUB Anubhuti-IIITD Foundation at Cognitive Computing &


22.
Indraprastha Institute of Information Social Censing
Technology, New Delhi

I-HUB Quantum Technology


Foundation,
23. Quantum Technologies
at Indian Institute of Science Education
and Research Pune

IIT Tirupati Navavishkar I-Hub


Positioning and Precision
24. Foundation
Technologies
at Indian Institute of Technology Tirupati

IIT Bhilai Innovation and Technology Technologies for Financial


25.
Foundation at Indian Institute of Sector
Technology Bhilai

Additional Information
National Mission on Interdisciplinary Cyber Physical System (NM-ICPS):-

It is implemented by Department of Science & Technology (DST) .The Mission was approved by the Union Cabinet in 2018 at a
total outlay of Rs.3660 Crores for a period of five years.
It is a comprehensive Mission aimed at complete convergence with all stakeholders by establishing strong linkages between
academia, industry, Government and International Organizations.
The Mission working with all the concerned Ministries/ Departments to identify their technology needs, develop solutions and
technical support.
The Mission aims at development of technology platforms to carry out R&D, Translational Research, Product Development,
Incubating & Supporting Start-ups as well as Commercialization.

Mission Objectives are:

To promote translational research in Cyber-Physical Systems (CPS) and associated technologies.


To develop technologies, prototypes and demonstrate associated applications pertaining to national priorities.
To enhance high-end researchers base, Human Resource Development (HRD) and skill-sets in these emerging areas.
To enhance core competencies, capacity building and training to nurture innovation and start-up ecosystem.
To establish and strengthen the international collaborative research for cross-fertilization of ideas.
To set up world-class interdisciplinary centers of excellence in several academic institutions across the country, that can become
repositories of core expertise in CPS and related areas and serve as focal points for technology inputs for the industry and policy
advice for the government
To involve Government and Industry R&D labs as partners in the collaboration centers. Incentivise private participation to encourage
professional execution and management of pilot scale research projects
To set mission mode application goals and foundational themes for excellence for different centers. Set up CPS test beds at various
centers.
To tie up with incubation centers and accelerators to foster close collaboration with entrepreneurship eco-system
To address some of the National issues and development of sector-specific solutions.

Que. 356 Consider the following statements about Neanderthals based on recent research findings:
1. Neanderthals, identified as Homo neanderthalensis, were a distinct species of humans who lived in Europe and southwest and
central Asia from about 400,000 to 40,000 years ago.
2. Neanderthals were not skilled in tool-making and did not develop any significant cultural technologies.
Which of the following is correct?

202/262
1. Only Statement 1 is correct.
2. Only Statement 2 is correct.
3. Both Statements are correct.
4. Neither Statements are correct.

Correct Option - 1
The correct answer is Option 1.
Key Points
Neanderthals, identified as Homo Neanderthalensis, were a distinct species of humans who lived in Europe and southwest and
central Asia from about 400,000 to 40,000 years ago.
Neanderthals are indeed classified as Homo Neanderthalensis, a distinct species separate from Homo sapiens (modern humans).
They inhabited parts of Europe and Asia during the time period specified in the statement. This is well-supported by archaeological
and paleontological evidence.Statement 1 is correct.
Neanderthals were not skilled in tool-making and did not develop any significant cultural technologies.. Neanderthals were known for
their tool-making skills. They developed the Mousterian tool culture, which is recognized for its refined flake tool techniques,
including scrapers, points, and hand axes. These tools were sophisticated and represent significant technological and cultural
advancements. Statement 2 is not correct.
.This answer is supported by extensive archaeological evidence demonstrating the presence and capabilities of Neanderthals in
tool-making and their distinct classification as a separate species from modern humans. Their contributions to prehistoric tool-
making technology highlight their importance in human evolutionary history.

Que. 357 Consider the following statements about Green Ammonia:


1. Green ammonia, also known as renewable ammonia, is a form of ammonia that is produced using renewable energy sources.
2. To produce green ammonia, green hydrogen must first be obtained through a process of water electrolysis.
3. Haber-Bosch process is used in the production of Green Ammonia.
How many of the given statements are correct?

1. Only one
2. Only two
3. All three
4. None

Correct Option - 3
The correct answer is option 3.
Key Points
Green ammonia

Green ammonia, also known as renewable ammonia, is a form of ammonia that is produced using renewable energy sources and which
is proposed as a sustainable, emission-free alternative with a multitude of applications in industry and other sectors. Hence statement
1 is correct.
To produce green ammonia, green hydrogen must first be obtained through a process of water electrolysis. That is, water is
decomposed into hydrogen and oxygen, using electrical energy generated from renewable sources. Hence statement 2 is correct.
The hydrogen is then combined with atmospheric nitrogen through a process known as Haber-Bosch synthesis, which allows
hydrogen and nitrogen to react at high pressure and temperature in the presence of a catalyst to form ammonia. Hence statement 3 is
correct.
The end result is the production of green ammonia using green hydrogen and atmospheric nitrogen.

Additional Information
Ammonia

It is a colourless, highly irritating gas with a sharp suffocating odor.


It dissolves easily in water to form ammonium hydroxide solution which can cause irritation and burns.
Ammonia gas is easily compressed and forms a clear, colorless liquid under pressure.
It is usually shipped as a compressed liquid in steel cylinders.
Ammonia is not highly flammable, but containers of ammonia may explode when exposed to high heat.

Que. 358 Consider the following features of Wi-Fi 7 technology:-


1. Backward Compatibility
2. Increased Latency
3. Multi-Link Operation
How many of the above statements is/are correct?

203/262
1. Only 1
2. Only 2
3. All three
4. None

Correct Option - 2
The correct answer is
Key Points
Wi-Fi:-

Wireless fidelity, or Wi-Fi, is a wireless technology that enables internet connectivity and inter-device communication.
For high-speed wireless internet and network connectivity, it employs radio waves.
Three mediums requires to broadcast a Wi-Fi signal: base stations, routers, and accessing devices (such as phones, laptops, etc.).
The newest Wi-Fi standard, Wi-Fi , is built on IEEE 802.11be, which stands for exceptionally high throughput (EHT).
The biggest technical professional association in the world, the Institute of Electrical and Electronics Engineers (IEEE) is commits
to developing technology for the good of humanity.

Key Features of Wi-Fi 7:

Backward Compatibility: Allows devices in the 2.4GHz, 5GHz, and 6GHz bands to connect without the need for additional
hardware. Hence, Statement 1 is correct.
It will combine several frequencies from the available spectrum to offer a solution rather than being restricted to just one.
Reduced Latency: Makes cloud-based activities like gaming and file transfers speedier.Hence, Statement 2 is incorrect.
Multi-Link Operation (MLO): Enhances network performance by combining numerous channels on various frequencies.Hence,
Statement 3 is correct.
Wi-Fi 7 has the potential to provide up to 330 gigabits per second of data per access point in terms of speed and capacity. Four
times as fast as Wi-Fi 6’s top speed.

Benefits of Wi-Fi 7:

Alignment with India’s Tech Adoption: Wi-Fi makes it possible for the country to quickly adopt wireless and cloud-based apps, as
well as government digital security and privacy efforts.
Positive Changes in India: Enterprise digital transformation, smartphone technology, fixed wireless connection, and artificial
intelligence (AI), particularly Edge AI, have all significant promise.
Meeting the needs of developing technologies: Wi-Fi may be able to handle the increasing number of wearables, smart home
appliances, and industrial IoT applications.
Telesurgery and telediagnostics in the medical field.
Improving User Experiences with Advanced Applications: Wi-Fi makes it easier to stream media with greater resolutions, such 8K
movies.
Boost efficiency in AR/VR, cloud gaming, and other data-intensive tasks.
Management of shipping and logistics.

Que. 359 Consider the following statements about The Diverse Epigenetic Epidemiology Partnership (DEEP):
1. It is funded by WHO.
2. It is integrated genomics and epigenomics study to understand the genetics behind Non-Communicable Diseases (NCDs) in diverse
populations.
3. The project is to uncover the effects of genomic and environmental diversity in disease risk observed in people across countries in Asia and
Africa only.
4. It is a five-year international project.
Which of the given statement/statements are correct?

1. 1 and 2 only
2. 2 only
3. 3 and 4 only
4. 2 and 4 only

Correct Option - 4
The correct answer is option 4.
Key Points
Diverse Epigenetic Epidemiology Partnership (DEEP)

The Diverse Epigenetic Epidemiology Partnership (DEEP) is a partnership of 20 research groups from around the world that aims to
improve global health by exploring the effects of genomic and environmental diversity on disease risk across the global population.
It is funded by the Medical Research Council in the United Kingdom. Hence statement 1 is incorrect.

204/262
It is a ground-breaking integrated genomics and epigenomics study to understand the genetics behind Non-Communicable Diseases
(NCDs) in diverse populations. Hence statement 2 is correct.
The project is to uncover the effects of genomic and environmental diversity in disease risk observed in people across the world,
including those in Asia, Africa and North and South America. Hence statement 3 is incorrect.
It is a five-year international project. Hence statement 4 is correct.

Additional Information
DEEP study researchers are interested in studying the differences in disease mechanism and disease risk prediction that are associated
with DNA methylation.
DNA methylation refers to modifications to DNA which help in turning genes on or off (transcription).
The DEEP study ultimately aims to identify disease mechanisms and drug targets that work for all.

Que. 360 Consider the following statements about Atmospheric Waves Experiment (AWE):
1. It is ISRO mission to study the interactions between terrestrial and space weather.
2. It is planned under Chandrayaan 3 Mission.
3. It will be launched and mounted on the exterior of the Earth-orbiting International Space Station (ISS).
4. It will measure the airglow at mesopause
How many of the given statements are correct?

1. Only one
2. Only two
3. Only three
4. All four

Correct Option - 2
The correct answer is option 2.
Key Points
Atmospheric Waves Experiment (AWE)

It is NASA experimental attempt aimed at studying the interactions between terrestrial and space weather. Hence statement 1 is
incorrect.
It is planned under NASA’s Heliophysics Explorers Programme. This mission will study the links between how waves in the lower
layers of the atmosphere. Hence statement 2 is incorrect.
It will be launched and mounted on the exterior of the Earth-orbiting International Space Station (ISS). Hence statement 3 is correct.
It will measure the airglow at mesopause (about 85 to 87 km above the Earth’s surface), where the atmospheric temperatures dip to
minus 100 degrees Celsius. Hence statement 4 is correct.
It will perform focused mapping of the colourful airglows in the Earth’s atmosphere.

Additional Information
Airglow

It is a faint luminescence of Earth’s upper atmosphere that is caused by air molecules’ and atoms’ selective absorption of solar
ultraviolet and X-radiation.
Most of the airglow emanates from the region about 50 to 300 km above the surface of Earth.

Que. 361 Consider the following statements about Amyloidosis:-


1. It is a rare disease that occurs when an abnormal protein, called amyloid, builds up in one’s organs, affecting their shape and
functioning.
2. Amyloid deposits can build up in the heart, brain, kidneys, spleen and other parts of the body.
3. They can occur in association with other diseases.
Which of the above statements is/are correct?

1. 1 and 2 only
2. 2 and 3 only
3. 1 and 3 only
4. 1,2 and 3

Correct Option - 4
The correct answer is option 4.
Key Points
Amyloidosis:-

I​ t is a rare disease that occurs when an abnormal protein, called amyloid, builds up in one’s organs, affecting their shape and
functioning. Hence, Statement 1 is correct.

205/262
Amyloid deposits can build up in the heart, brain, kidneys, spleen and other parts of the body, leading to life-threatening
conditions like organ failure.Hence, Statement 2 is correct.
Some varieties of amyloidosis occur in association with other diseases.Hence, Statement 3 is correct.
These types may improve with treatment of the underlying disease. Some varieties of amyloidosis may lead to life-threatening
organ failure.
The general signs or symptoms would include: Severe fatigue, Loss of weight,Swelling in the belly, legs, ankles or feet, Numbness,
pain or tingling in hands or feet and changes in skin colour.
Different amyloidosis that are prevalent:-
Light-chain (AL) amyloidosis: People with conditions such as multiple myeloma or a bone marrow illness are more likely to have AL
amyloidosis.
A amyloidosis: Previously known as secondary amyloidosis, this condition is the result of another chronic infectious or inflammatory
disease, such as rheumatoid arthritis, Crohn’s disease, or ulcerative colitis.

Que. 362 Consider the following statements about the Stratospheric Observatory for Infrared Astronomy (SOFIA):
1. SOFIA was a telescope mounted on a Boeing 747 SP aircraft that studied infrared light, essentially heat, emitted by objects in
the universe.
2. The observatory was a collaborative venture between NASA and the European Space Agency (ESA).
3. It is place in Geosynchronous orbit.
4. Scientists detected water molecules on the surface of two asteroids for the first time ever using data from now-retired SOFIA .
Which of the statements given above are correct?

1. 1 and 4 only
2. 2 and 3 only
3. 3 only
4. 1,2,3 and 4

Correct Option - 1
The correct answer is option1
Key Points
Stratospheric Observatory for Infrared Astronomy (SOFIA)

SOFIA was a telescope mounted on a Boeing 747 SP aircraft that studied infrared light, essentially heat, emitted by objects in the
universe. Hence statement 1 is correct.
SOFIA is operated jointly by NASA and the German space agency. Hence statement 2 is incorrect.
It was a telescope mounted on a Boeing 747 SP aircraft and is the world's largest airborne astronomical observatory, complementing
NASA’s space telescopes as well as major Earth-based telescopes. Hence statement 3 is incorrect.
Scientists recently detected water molecules on the surface of two asteroids for the first time ever, using the data from NASA's now-
retired SOFIA airborne observatory. Hence statement 4 is correct.

Que. 363 Which of the following statement is true about ISI7017 (Part 2/Sec 7): 2023 ?

1. It is charging connector standard tailored for light electric vehicles (LEVs) such as scooters, bikes, and rickshaws.
2. It is India's new building code.
3. It is standard for manufacturing led bulb
4. It is standard for helmet manufacturing.

Correct Option - 1
The correct answer is option 1.
Key Points
ISI7017 (Part 2/Sec 7): 2023, is charging connector standard for Light Electric Vehicles. Hence statement 1 is correct.

206/262
It is a result of collaboration among NITI Aayog, the Department of Science and Technology, Ather Energy (a private firm), and other
stakeholders.

Que. 364 Consider the following statements about the Ayurveda Gyan Naipunya Initiative (AGNI):
1. It is launched by Central Council for Research in Ayurveda Sciences (CCRAS).
2. To promote research for mainstreaming the pragmatic Ayurveda practices through scientific validation and evidence-based appraisal.
3. The initiative is in collaboration with the National Commission for Indian System of Medicine (NCISM) for documenting and publishing
medical practices.
4. The AGNI initiative primarily aims to provide a platform for allopathic practitioners to report innovative practices in managing various
disease conditions.
How many of the given statements are correct?

1. Only one
2. Only two
3. Only three
4. All four

Correct Option - 3
The correct answer is option 3.
Key Points
Ayurveda Gyan Naipunya Initiative (AGNI)

Central Council for Research in Ayurveda Sciences (CCRAS) launched Ayurveda Gyan Naipunya Initiative (AGNI). Hence statement
1 is correct.
Nodal agency – Central Council for Research in Ayurveda Sciences (CCRAS)
Cooperate with National Commission for Indian System of Medicine (NCISM), a statutory body. Hence statement 3 is correct.
Aim
Aim – To promote research for mainstreaming the pragmatic Ayurveda practices through scientific validation and evidence-
based appraisal. Hence statement 2 is correct.
It will provide a platform to Ayurveda practitioners for reporting their innovative practices and experiences in various disease
conditions.
The AGNI initiative is designed for Ayurveda practitioners, not allopathic practitioners, to report their innovative practices and
experiences in various disease conditions. This statement misrepresents the targeted audience of the initiative. Hence statement 4 is
incorrect.


Additional Information
​Central Council for Research in Ayurveda Sciences (CCRAS):

An apex research organization committed for undertaking, coordinating, formulating, development and promotion of research on
scientific lines in Ayurveda.
Ministry – Ministry of AYUSH
Research activities – Medicinal Plant Research, Drug Standardization, Pharmacological Research, Clinical Research, Literary
Research & Documentation and Tribal Health Care Research Programme.
Initiatives –
SPARK – Studentship Program for Ayurveda Research Ken for Undergraduate Scholars
PG-STAR – Scheme for Training in Ayurveda Research for PG Scholars for PG Scholars
SMART – Scope for Mainstreaming Ayurveda Research in Teaching Professionals program for teachers

Que. 365 Consider the following statements regarding Onchocerciasis:


1. It is predominantly transmitted through the bite of infected female Anopheles mosquitoes.
2. Onchocerciasis is a leading cause of blindness worldwide, second only to trachoma in terms of infection-based blindness.
3. The World Health Organization has launched the Global Onchocerciasis Network for Elimination (GONE) to accelerate the elimination of
the disease.
4. It is classified by WHO as an Neglected Tropical diseases.
Which of the statements given above are correct?

1. 1,2 and 4 only


2. 2,3 and 4 only
3. 1,3 and 4 only
4. 2 and 4 only

Correct Option - 2
The correct answer is option 2

207/262
Key Points
Onchocerciasis

The infection is spread by black flies that live near fast-flowing rivers; Caused by the parasitic worm Onchocerca volvulus. It causes
severe skin irritation, itching and, eventually, irreversible blindness. Hence statement 1 is correct.
Onchocerciasis, also known as river blindness, is indeed the second leading infectious cause of blindness after trachoma. Hence
statement 2 is correct.
GONE was launched in January 2023 by WHO, its Member States and partners whose goal is to empower countries to accelerate
progress towards the achievement of the road map targets for onchocerciasis elimination. Hence statement 3 is correct.
It is classified by WHO as NTD. Hence statement 4 is correct.​

Additional Information
Neglected tropical diseases (NTDs)

NTDs constitute a varied collection of 20 diseases, primarily occurring in tropical region.


They are caused by a variety of pathogens such as viruses, bacteria, protozoa and parasitic worms.
They predominantly impact impoverished communities, with women and children being the most adversely affected.
Over one billion individuals suffer significant health, societal, and financial repercussions due to these diseases.

Que. 366 Consider the following statements about Vikram-S:-


1. It is the first privately developed rocket of India.
2. It is developed by Agnikul Cosmos.
3. Recently, India has allowed private sector participation in space sector.
How many of the above statements is/are correct?

1. Only one
2. Only two
3. All three
4. None

Correct Option - 2
The correct answer is option 2.
Key Points
Rise of Space tech Startups in India:

In November 2022, the first privately developed rocket, Vikram-S was launched by the Indian space startup Skyroot Aerospace.
Hence, Statement 1 is correct, Statement 2 is incorrect.
Agnikul Cosmos, a private Indian company set up India’s first launch pad at the Satish Dhawan Space Centre.
Bellatrix Aerospace, another Indian space startup, is engaged in advanced in-space propulsion systems and rocket propulsion
technologies.

Efforts in Privatisation of Space Sector:

Indian Space Policy, 2023: The Indian Space Policy, 2023 has provided much-needed clarity on the role of private sector
participants and removed stern barriers to their entry into the sector.Hence, Statement 3 is correct.
IN-SPACe: The Indian National Space Promotion and Authorisation Centre (IN-SPACe) is a single-window, independent,
nodal agency that functions as an autonomous agency in the Department of Space (DOS).
New Space India Limited (NSIL): It was incorporated in 2019, as a wholly-owned Government of India Undertaking / Central Public
Sector Enterprise (CPSE), under the administrative control of the Department of Space (DOS) to commercially exploit the research
and development work of Indian Space Research Organisation (ISRO).

Que. 367 Consider the following statements about e-Cigarettes:-


1. They are battery powered devices that work by heating a liquid into an aerosol that the user inhales and exhales.
2. The liquid typically contains nicotine, propylene glycol, glycerin, flavorings, and other chemicals.
3. Mere possession of e-Cigarettes is an offence in India.
How many of the above statements is/are correct?

1. Only one
2. Only two
3. All three
4. None

Correct Option - 3
The correct answer is option 3.

208/262
Key Points
e-Cigarettes:-

They are battery powered devices that work by heating a liquid into an aerosol that the user inhales and exhales.Hence,
Statement 1 is correct.
The e-cigarette liquid typically contains nicotine, propylene glycol, glycerin, flavorings, and other chemicals. Hence, Statement 2
is correct.
There are many different types of e-cigarettes in use, also known as electronic nicotine delivery systems (ENDS) and sometimes
electronic non-nicotine delivery systems (ENNDS).
In India, the possession of e-cigarettes and similar devices is a violation of the Prohibition of Electronic Cigarette Act (PECA)
2019.Hence, Statement 3 is correct.
Recently, the World Health Organization (WHO) has urged governments to treat e-cigarettes similarly to tobacco and ban all
flavors, threatening cigarette companies' bets on smoking alternatives.

Que. 368 Consider the following provisions about the recent Organ Transplantation Guidelines:-
1. It has put a cap above which organ donation is prohibited.
2. There is no domicile requirement in case of organ donation.
3. There is a minimal registration fee payable to states.
How many of the above statements is/are correct?

1. Only one
2. Only two
3. All three
4. None

Correct Option - 1

The correct answer is option 1.

Key Points
National Organ Transplantation Guidelines:-

Removed Age Cap:The upper age limit has been removed as people are now living longer. Hence, Statement 1 is incorrect.
Earlier, according to the NOTTO (National Organ and Tissue Transplant Organization) guidelines, an end-stage organ failure patient
above 65 years of age was prohibited from registering to receive the organ.
No Domicile Requirement:The ministry has removed the domicile requirement to register as an organ recipient in a particular state
under a ‘One Nation, One Policy’ move.Hence, Statement 2 is correct.
No Fees for Registration:There will be no registration fee that states used to charge for this purpose, the Centre has asked states that
used to charge for such registration to not do so.Hence,Statement 3 is incorrect.

Que. 369 Consider the following statements about the Global Partnership on Artificial Intelligence (GPAI)?
1. India is one of the founding members of GPAI.
2. GPAI's secretariat is located in Geneva, Switzerland.
3. There are 28 member countries in GPAI, including the European Union.
How many of the given statements are correct?

1. Only one
2. Only two
3. Only three
4. None

209/262
Correct Option - 2
The correct answer is option 2
Key Points
Global Partnership on Artificial Intelligence

India is one of the founding members of GPAI. Hence statement 1 is correct.


Its secretariat is hosted at the Organisation for Economic Co-operation and Development (OECD), Paris. Hence statement 2 is
incorrect.
At present it has 28 member countries including the European Union. Hence statement 3 is correct.

Additional Information
It is a multi-stakeholder initiative which aims to bridge the gap between theory and practice on AI by supporting cutting-edge research
and applied activities on AI-related priorities.
It was launched in June 2020.
Membership open to all countries, including emerging and developing countries.
The working groups will initially focus on four themes:
Responsible AI
Data Governance
the Future of Work
Innovation and Commercialization

Que. 370 Which of the following statement is correct about Dark Fibre?

1. It refers to fibre optic cables with no service or traffic running on them – an unused point-to-point connection.
2. It refers to fibre optic cables used for criminal activities.
3. It refers to fibre optic cables running along the bottom of the ocean.
4. It refers to fibre optic cables used exclusively for government to government data transfer.

Correct Option - 1
The correct answer is option 1.
Key Points
Dark Fibre

A Dark Fibre network, also known as unlit fibre, refers to fibre optic cables with no service or traffic running on them – an unused
point-to-point connection. It’s called ‘dark’ because the cables aren’t lit by a light signal. Hence statement 1 is correct.
Dark Fibre gives you maximum control and flexibility to create your own high capacity network with almost limitless scalability.
The main advantage of Dark Fibre connectivity is that you have complete control over your network.
It’s available for purchase or rent from network service providers.
Dark fibre is different from “lit fibre” which refers to cables that are actively being used.
Dark fibre is inactive because no information is passed through it, so no light pulse is transmitted.

Que. 371 Consider the following statements:


1. NASA's Lucy mission is designed specifically to study comets in the Kuiper Belt.
2. Selam, discovered by NASA’s Lucy mission, is the first contact-binary satellite observed in the solar system.
3. Selam orbits the asteroid Dinkinesh.
4. Jupiter Trojan asteroids, explored by NASA’s Lucy mission, share Jupiter's orbit around the Sun.
Which of the statements given above are correct?

1. 1 and 2 only
2. 2 and 3 only
3. 2, 3, and 4 only
4. 1, 3, and 4 only

Correct Option - 3
The correct answer is option 3.
Key Points
Selam contact-binary satellite

National Aeronautics and Space Administration (NASA ‘Lucy’ mission is to explore the Jupiter Trojan Asteroids. Hence statement 1
is incorrect.
Discovered by NASA’s Lucy mission, Selam is the first contact-binary satellite ever observed. Hence statement 2 is correct.

210/262
Contact binary system is where two space bodies gently rest against each other.
International Astronomical Union has named the satellite of asteroid Dinkinesh as “Selam (meaning peace)”.Selam orbits the asteroid
Dinkinesh. Hence statement 3 is correct.
Jupiter Trojan asteroids, explored by NASA’s Lucy mission, share Jupiter's orbit around the Sun. Hence statement 4 is correct.​

Additional Information
Mission Lucy: NASA​

The solar-powered mission is estimated to be over 12 years long, during which the spacecraft will visit eight asteroids covering a
distance of about 6.3 billion km to deepen the understanding of the “young solar system”.
The spacecraft’s first encounter will be with an asteroid that lies in the main belt that can be found between Mars and Jupiter. This
asteroid is named ‘Donald Johnson’.

Que. 372 Which of the following statement is correct about sub-Neptunes ?

1. Planets with radii between that of the Earth and Neptune are referred to as ‘sub-Neptunes’
2. Planet closest to Neptunes.
3. Planet between Neptune and Mars.
4. Planets beyond Neptune.

Correct Option - 1
The correct answer is option 1.
Key Points
Sub-Neptune planets

Planets with radii between that of the Earth and Neptune are referred to as ‘sub-Neptunes’. Hence statement 1 is correct.
These are the most commonly observed types of planets in our galaxy.
They could be rocky worlds with thick atmospheres of hydrogen and helium gas, or perhaps composed of rock and ice with warm,
water-rich atmospheres.

Que. 373 Consider the following statements about the ‘Krutrim AI’:
1. It is a homegrown AI assistant developed by the Center for Development of Advanced Computing (C-DAC).
2. It can converse in regional languages of India.
Which of the given statements are correct?

1. Only 1
2. Only 2
3. Both 1 and 2
4. Neither 1 nor 2.

Correct Option - 2
The correct answer is option 2.
Key Points
Krutrim AI:-

It is Ola’s homegrown AI assistant, designed to cater to the diverse needs and nuances of Indian consumers, bridging the gap
between conventional AI and specific cultural contexts.Hence, Statement 1 is incorrect.
Multilingual Support: Krutrim boasts the ability to converse in English, Hindi, Tamil, Telugu, Malayalam, Marathi, Kannada,
Bengali, Gujarati, and Hinglish, catering to the linguistic diversity of India.Hence, Statement 2 is correct.
Multi-Functionality: Users can leverage Krutrim for a range of tasks, including writing emails, seeking information, learning new
skills, planning travel, discovering recipes, etc.
211/262
Technology behind Krutrim:-

Sophisticated AI Model: Krutrim operates on a sophisticated AI model trained on vast datasets encompassing Indian languages, social
contexts, and cultural references.
Natural Language Processing (NLP): Utilizes NLP to comprehend human language nuances, including colloquialisms and cultural
contexts, enhancing user interactions.
Machine Learning (ML): ML algorithms enable Krutrim to learn from datasets, continuously improving responses and understanding
user intent.
Deep Learning: Leverages Deep Learning to recognize patterns and analyze complex data, crucial for contextual responses and
performance enhancement.​

​Application and Benefit for Users:-

Enhanced User Experience: Krutrim AI enhances user experiences across various sectors by offering culturally sensitive interactions,
personalized learning in education, and automating administrative tasks.
Support for Content Creators: Content creators can leverage Krutrim for ideation and localization, making content more relatable
and engaging.
Automating Repetitive Tasks: Krutrim’s capabilities extend to automating repetitive administrative tasks across industries, boosting
efficiency and productivity.

Que. 374 Consider the following statements about Hydrogen for Heritage Project:-
1. It is a plan to operate hydrogen-powered trains on select heritage and hilly routes that are environmentally sensitive.
2. It involves the retrofitment of hydrogen fuel cells on diesel electric multiple unit (DEMU) rakes.
3. It is based on Hydrogen Fuel Cells Technology.
How many of the above statements is/are correct?

1. Only one
2. Only two
3. Only three
4. None

Correct Option - 3
The correct answer is option 3.
Key Points
Hydrogen for Heritage Project:-

Announcement in Union Budget: The Finance Minister in her Budget speech in February 2023 announced the ‘Hydrogen for
Heritage’ scheme, a plan to operate hydrogen-powered trains on select heritage and hilly routes that are environmentally
sensitive. Hence, Statement 1 is correct.
Proposed Plan: India’s plan to develop the technology involves the retrofitment of hydrogen fuel cells on diesel electric multiple
unit (DEMU) rakes. This prototype is expected to run on the Jind-Sonipat section in Haryana initially.Hence,Statement 2 is correct.
Indian Railways has proposed to run 35 hydrogen trains at an estimated cost of Rs 80 crore per train, with ground infrastructure costing
Rs70 crore per route.
Manufacturing cost: one rake of six coaches with hydrogen propulsion is estimated at Rs 80 crore; for ground infrastructure, it is Rs
600 crore.
Thirty-five train-set rakes (with six coaches each) have been sanctioned for the ongoing financial year for eight sections–
Matheran Hill Railway, Darjeeling Himalayan Railway, Kalka-Shimla Railway, Kangra Valley, Bilmora Waghai, Patalpani
Kalakund, Nilgiri Mountain Railways, and Marwar-Goram Ghat.
Challenges: According to, the Ministry of Railways the initial running cost of hydrogen-powered trains would be higher, but it
would subsequently reduce with an increase in the number of such trains.
Hydrogen trains basically run on hydrogen fuel cells. These fuel cells combine oxygen and hydrogen to create electricity, which
drives the train’s motors.Hence, Statement 3 is correct.
Environmental Benefits: Hydrogen trains are more environmentally beneficial than traditional diesel trains as they do not emit
dangerous pollutants like nitrogen oxides, carbon dioxide, and particulate.

Que. 375 Consider the following statements about Anthrobots:


1. They are constructed from human tracheal cells which are bio-robots that possess self-assembly capabilities.
2. They can spontaneously fuse together to form a larger structure called a superbot.
3. Xenobots is its another name.
4. They hold promise for regenerative medicine, wound healing, and disease treatment.
How many of the given statements are correct?

1. Only one
2. Only two

212/262
3. Only three
4. All four

Correct Option - 3
The correct answer is option 3.
Key Points
Anthrobots

They are miniature robots developed by using human cells. These are constructed from human tracheal cells which are bio-robots
that possess self-assembly capabilities. Hence statement 1 is correct.
They can spontaneously fuse together to form a larger structure called a superbot, which was able to encourage the growth of neurons.
Hence statement 2 is correct.
Anthrobots are different from Xenobots, which are created from embryonic stem cells of frog. Hence statement 3 is incorrect.
They hold promise for regenerative medicine, wound healing, and disease treatment. Hence statement 4 is correct.
They are capable of both movement and healing neurons within a laboratory setting.
They measured between the width of a human hair and the tip of a sharpened pencil.

Additional Information
Tracheal cells

These are from the lining of the bronchi/trachea, the network of tubes used to convey air to the lungs.
They are responsible for producing lubricating mucus to keep the airways functional.
They are a type of epithelial cell, a term used generally to refer to cells lining the inside or outside of the body.

Que. 376 Consider the following statements about AKTOCYTE:


1. Its a tablet used during cancer treatment to minimize the side effects of radiotherapy.
2. It is developed by pharma company Bristol Myers Squibb.
3. AKTOCYTE has received approval from the FSSAI.
How many of the given statements are correct?

1. Only one
2. Only two
3. All three
4. None

Correct Option - 2
The correct answer is option 2.
Key Points
AKTOCYTE ​

Its a tablet used during cancer treatment to minimize the side effects of radiotherapy. Hence statement 1 is correct.
Scientists from the Department of Atomic Energy (DAE) and M/s. IDRS Labs Pvt. Ltd. in Bengaluru have collaboratively
developed a breakthrough nutraceutical named AKTOCYTE tablets. Hence statement 2 is incorrect.
AKTOCYTE has received approval from the FSSAI. Hence statement 3 is correct.
AKTOCYTE Tablets show promise in pelvic cancer patients suffering from radiotherapy-induced Cystitis (Blood in urine).
Beyond a supplement, they serve as an adjuvant to cancer radiotherapy, a regenerative nutraceutical, an immunomodulator, and an
antioxidant, highlighting their diverse applications in cancer care.

Additional Information
FSSAI

Food Safety and Standards Authority of India (FSSAI) is a statutory body established under the Food Safety and Standards Act, 2006
(FSS Act).
FSSAI is an autonomous body under the Ministry of Health & Family Welfare.

Que. 377 Consider the following statements about Aarogya Maitri Aid Cube:
1. It is the world’s first portable hospital.
2. It is designed indigenously under Project BHISHM (Bharat Health Initiative for Sahyog Hita and Maitri).
3. It has operation theatre.
4. It integrates Artificial Intelligence (AI) and data analytics to facilitate effective coordination, real-time monitoring, and efficient
management of medical services.

1. Only one

213/262
2. Only two
3. Only three
4. All four

Correct Option - 4
The correct answer is option 4.
Key Points
Aarogya Maitri Aid Cube

It is the world’s first portable hospital. Hence statement 1 is correct.


Designed indigenously under Project BHISHM (Bharat Health Initiative for Sahyog Hita and Maitri), the modular trauma
management and aid system is made up of 72 detachable mini-cubes, each being a specialized station for emergency response and
humanitarian efforts. Hence statement 2 is correct.
It contains medical equipment and supplies such as a mini-ICU, an operation theatre, cooking station, food, water, a power
generator, blood test equipment, an X-ray machine, and more. Hence statement 3 is correct.
It integrates Artificial Intelligence (AI) and data analytics to facilitate effective coordination, real-time monitoring, and efficient
management of medical services in the field. Hence statement 4 is correct.
It can treat as many as 200 patients.
These cubes are light and portable, and can be rapidly deployed anywhere, from airdrops to ground transportation.
It consists of 72 cubes that can be combined to form a specialized cage capable of accommodating 36 mini-cubes. The mini-cubes are
packed with everything essential for the survival of 100 individuals for duration of 48 hours.

Que. 378 Consider the following statements about XPosat:


1. It is the first dedicated satellite from ISRO to carry out research in space-based polarisation measurements of X-ray emission
from celestial sources.
2. It is NASA mission.
3. XPoSat comprises two payloads POLIX and XSPECT.
How many of the statements are correct ?

1. Only one
2. Only two
3. All three
4. None

Correct Option - 2
The correct answer is option 2.
Key Points
X-ray Polarimeter Satellite (XpoSat)

It is the first dedicated satellite from ISRO to carry out research in space-based polarisation measurements of X-ray emission from
celestial sources. Hence statement 1 is correct and statement 2 is incorrect.
It is designed to study X-ray polarization in the medium X-ray band, offering insights into celestial sources' radiation mechanisms and
geometry.
The satellite carries two main payloads, POLIX (Polarimeter Instrument in X-rays) and XSPECT (X-ray Spectroscopy and
Timing). Hence statement 3 is correct.
POLIX will observe about 40 bright astronomical sources, while XSPECT will study the electromagnetic spectrum generated by
different matter.​

Additional Information
Built by two Bengaluru-based institutes—ISRO’s UR Rao Satellite Centre and Raman Research Institute.
The mission is propelled by the PSLV-C58 rocket in Low Earth Orbit.
XPoSat is world's second mission dedicated to X-ray polarization in the medium X-ray band. NASA's Imaging X-ray
Polarimetry Explorer (IXPE), launched in 2021 is the first to do so.

Que. 379 Consider the following statements about space habitation:

Space habitation involves establishing human settlements beyond Earth, such as on the Moon and Mars, and can include habitation on
celestial bodies in the Goldilocks zones or building space habitats like capsule modules.
The Stanford Torus, proposed by NASA, and the O'Neill Cylinder are designs for self-sustaining habitats in space, both of which rely
on rotation to create artificial gravity for agriculture and human activities in a microgravity environment.
Only national space agencies like NASA (USA), CNSA (China), and ISRO (India) are exploring space habitation and sending
astronauts to the Moon.

How many of the above statements are correct?

214/262
1. Only One
2. Only Two
3. All three
4. None of the above

Correct Option - 2
The correct answer is Option 2.
Key Points
Space Habitation:
Space habitation encompasses the development of living quarters for humans beyond Earth, such as on the Moon and Mars.
Methods of Establishing Space Habitats

Habitation on celestial bodies within the Goldilocks zones, like Mars.


Construction of capsule module-based space habitats that can be positioned anywhere within the solar system, a prime example
being the International Space Station. Hence Statement 1 is correct.

Proposals for Self-Sustaining Space Habitats

The Stanford Torus, envisioned by NASA, is a proposed toroidal (doughnut-shaped) space station featuring a central hub and a
rotating outer ring.
This rotation is designed to generate artificial gravity on the inner surface of the ring, allowing for human habitation.
The O'Neill Cylinder, conceptualized by physicist Gerard K. O'Neill, involves a pair of large cylinders rotating in opposite
directions.
This rotation aims to create artificial gravity on their inner surfaces, facilitating agriculture and other activities in a microgravity
environment.

Hence Statement 2 is correct


Global and Private Initiatives in Space Habitation
Nations such as the USA, China, and India have initiated programs to send astronauts to the Moon and investigate the feasibility of space
habitation.
Additionally, private sector visionaries like Elon Musk are advocating for the establishment and support of permanent human
settlements on Mars.Hence Statement 3 is incorrect.

Que. 380 Consider the following statements about the National Quantum Mission (NQM) and choose the correct option:

1. The Mission Coordination Cell (MCC) established under NQM will function independently without any supervision.
2. One of the objectives of NQM is to develop quantum computers with the processing capacity of 500 qubits.
3. NQM aims to establish a secure and high-bandwidth Quantum Communication Network spanning 2,000 kilometers.

1. 1 and 2 only
2. 2 and 3 only
3. 1 and 3 only
4. 3 only

Correct Option - 4
The correct answer is Option 4.
Key Points
Nationa Quantum Mission:
The National Quantum Mission (NQM) held its first Mission Governing Board (MGB) meeting, focusing on the action plan, implementation
timelines, and the formation of the Mission Coordination Cell (MCC).
Key Points on Mission Coordination Cell (MCC):

Establishment: The MCC will serve as the main coordinating body for NQM, in collaboration with the Mission Secretariat and the
Department of Science of Technology (DST). Hence Statement 1 is incorrect.
Location and Supervision: It will be established within a DST-identified institution based on merit and available infrastructure and
will operate under the guidance of the Mission Technology Research Council (MTRC).
Role of MTRC: The council is tasked with guiding the NQM, overseeing its progress, and selecting projects for funding.
International Outer Space Governance Agreements under UNCOPUOS:
Outer Space Treaty: Addresses state activities in outer space exploration, including the Moon and other celestial bodies.
Rescue Agreement: Outlines agreements on the rescue and return of astronauts and space objects.
Moon Agreement: Also known as the Agreement Governing the Activities of States on the Moon and Other Celestial Bodies, it
regulates state activities on the Moon and celestial bodies.
Liability Convention: Deals with liability issues arising from damage caused by space objects.
Registration Convention: Focuses on the registration of objects launched into outer space.

National Quantum Mission (NQM):

215/262
Overview: A flagship endeavor introduced by the Government of India for 2023-2031, aimed at positioning India as a leader in
quantum technologies.
Purpose: To foster and expand scientific and industrial R&D, thereby creating a dynamic and innovative quantum technology
ecosystem.

Key Objectives:
Quantum Computing Power: To create quantum computers with the capability of 1,000 qubits, significantly enhancing computational
power. Hence Statement 2 is incorrect.
Magnetometry and Atomic Clocks: To advance the development of sensitive magnetometers and reliable atomic clocks for precise
measurements.Quantum Communication Network: To develop a secure, high-bandwidth communication network covering 2,000
kilometers. Hence Statement 3 is correct.
Quantum Materials Design: To synthesize and analyze new quantum materials with specific properties for advanced device manufacturing.
Initiatives: Plans include establishing four thematic research hubs (T-Hubs) to support the mission's objectives.

Que. 381 Consider the following statements about the Centre for Industrial Revolution (C4IR) set up in Hyderabad and the Fourth
Industrial Revolution (4IR):

The agreement to establish the Centre for Industrial Revolution (C4IR) in Hyderabad was signed between the Government of
Telangana and the World Bank.
The concept of the Fourth Industrial Revolution, coined by Klaus Schwab in 2016, aims at the digital transformation of industries
through technologies like artificial intelligence, Internet of Things (IoT), and smart factories.
One of the anticipated benefits of the 4IR for India includes transforming the country into a global manufacturing hub, leveraging
demographic advantages, demand, and decisive governance.

How many of the above statements are correct?

1. Only One
2. Only Two
3. Only Three
4. None of the above

Correct Option - 2
The correct answer is Option 2
Key Points

The Government of Telangana has entered into an agreement with the World Economic Forum to establish the Centre for the Fourth
Industrial Revolution (C4IR) in Hyderabad. Hence Statement 1 is incorrect.
Fourth Industrial Revolution (4IR):

Introduced by Klaus Schwab, the founder of the World Economic Forum, in 2016, the notion of the Fourth Industrial Revolution
encapsulates the comprehensive digitization of the manufacturing sector. This includes the integration of advanced technologies like
artificial intelligence (AI), additive manufacturing (3D printing), augmented/virtual reality, and the Internet of Things (IoT). Hence
Statement 2 is correct.
Additionally, this revolution emphasizes the emergence of "smart factories." These are sophisticated cyber-physical systems that blend
the attributes of the physical and digital worlds seamlessly.

Advantages of 4IR for India

A significant uplift in digital infrastructure leading to a surge in productivity, spearheaded by advancements in:
Digital communication, which fosters new forms of connectivity between people and devices on a global scale.
Digital energy systems, which ensure efficient matching of energy production with consumption through smart grids.
Digital health solutions that permit healthcare delivery from remote locations.
The potential for India to position itself as a leading global manufacturing hub, using its demographic edge, strong demand, and
assertive policy direction in areas such as 3D printing, machine learning, data analytics, and IoT. Hence Statement 3 is correct.
The formation of a future-ready workforce, where India leverages cutting-edge technology and methodologies for skill enhancement,
re-skilling, and up-skilling to become the world's primary source of skilled labor.
An enhancement in supply chain efficiency and resilience, as 4IR technologies streamline operations, minimize waste, and boost
productivity.
The digital boon's spillover effects are expected to unlock opportunities across diverse sectors—including healthcare, transportation,
and energy—thus enhancing industry digitalization.

Que. 382 Consider the following statements regarding India's first graphene center, the India Innovation Centre for Graphene (IICG):

1. The India Innovation Centre for Graphene is a collaboration between the Digital University of Kerala, Centre for Materials for
Electronics Technology (C-MET), and Reliance Industries Limited, and it receives funding from the Department of Science and
Technology (DST).

216/262
2. The IICG aims to bridge the gap between research and development (R&D) and commercialization in the field of graphene and two-
dimensional materials, supporting the Graphene-Aurora program initiated by the Ministry of Electronics and Information Technology
(MeitY).
3. Graphene, known for its exceptional properties such as being 200 times stronger than steel yet six times lighter and having higher
thermal and electrical conductivity than copper, is derived from graphite, the material used in pencil lead.

How many of the statements given above is/are correct?

1. Only One
2. Only Two
3. Only Three
4. None of the above

Correct Option - 2
The correct answer is Option 2.
Key Points
India's First Graphene Centre: India Innovation Centre for Graphene (IICG) in Kerala

The IICG stands as India's pioneering Graphene Centre. It represents a collaborative initiative among the Digital University of
Kerala, the Centre for Materials for Electronics Technology (C-MET), and Tata Steel Limited, with funding from the Ministry
of Electronics and Information Technology (MeitY). Hence Statement 1 is incorrect.
The centre's focus encompasses the promotion of research and development, product innovation, and capacity building in the
fields of Graphene and two-dimensional materials (2DM). Hence Statement 2 is correct.
In particular, IICG is set to bolster the Graphene-Aurora program of MeitY, aimed at bridging the gap between R&D and
commercialization by equipping startups and the industry with comprehensive facilities.

Graphene:

Originating from graphite (found in pencil lead), graphene is essentially a one-atom-thick layer of carbon atoms, organized in a
hexagonal honeycomb lattice, and represents a two-dimensional form of carbon.
A millimeter of graphite can yield roughly 3 million layers of graphene.
Prepared from Intercalated Graphite, Coal tar, Shellac, or a mix of these, in an inert atmosphere.

Properties of Graphene:

Strength: Considered the strongest material, being 200 times more robust than steel, yet six times lighter.
Thermal Conductivity: Exhibits unprecedented thermal conductivity, reaching up to 5000 W/m/K at room temperature.
Electrical Conductivity: Surpasses even copper in terms of electrical conductivity. Hence Statement 3 is correct.
Surface Area: Offers an exceptionally high surface area owing to its two-dimensional structure.
Impermeability: Proves impermeable to gases, encompassing those as light as hydrogen or helium.
Transparency and Flexibility: Almost perfectly transparent, absorbing just 2% of light, and is incredibly flexible, making it ideal for
wearable electronics.
Chemical Stability and Biocompatibility: Chemically stable and inert; does not readily react with other chemicals. Its
biocompatibility renders it suitable for biomedical applications.

Additional Information
Applications of Graphene:

Electronics: Crafting graphene-based transistors, circuits, and conductive components to enhance the speed and efficiency of
electronic devices.
Energy Storage: Developing batteries and supercapacitors with high energy density and swift charging features.
Conductive Films: Deploying transparent conductive films for touchscreens, flexible displays, and solar cells.
Materials Reinforcement: Reinforcing composites to boost mechanical strength whilst diminishing weight, particularly in aerospace
and automotive sectors.
Thermal Management: Utilizing heat sinks and thermal interface materials for effective heat dissipation in electronic gadgets.
Sensors: Facilitating high-sensitivity sensors for the detection of gases, chemicals, and biological substances.
Biomedical Devices: Employing in drug delivery systems, biosensors, and imaging solutions courtesy of graphene’s biocompatibility
and unique properties.
Water Filtration: Applying graphene-based membranes for advanced water filtration, desalination, and purification.
Coatings and Films: Crafting anti-corrosion coatings, anti-fogging films, and protective layers for various surfaces.
Lightweight Materials: Creating lightweight yet strong materials for sports equipment, automotive parts, and structural components.
This concise reformulation offers a clear and structured overview of the India Innovation Centre for Graphene and the remarkable
attributes and applications of graphene, underscoring its transformative potential across a multitude of industries.

Que. 383 Consider the following statements regarding the transmission of electricity for wireless charging:

217/262
1. Capacitive coupling involves the transmission of electricity through magnetic fields using inductive coupling between coils of wire.
2. Inductive charging, the most widely used wireless charging technology, operates on a principle similar to a transformer, based on the laws
of magnetic induction.
3. To initiate charging in vehicles with inductive charging technology, drivers must connect their electric vehicle (EV) to a charging station
using a physical cable.
Which of the statements given above is/are correct?

1. 1 only
2. 2 only
3. 1 and 2 only
4. 2 and 3 only

Correct Option - 2
The correct answer is Option 2.
Key Points
Induction/Wireless charging

Induction Charging (IC) also known as wireless charging is method of charging in which electromagnetic fields are used to transfer
energy between two coils without the need of a physical connection. Hence Statement 1 is correct.
Processes Involved/Working

The transmission of electricity generally takes place by either of the two methods:

By electric fields using capacitive coupling between metal electrodes (capacitive charging).
By magnetic fields using inductive coupling between coils of wire (inductive charging).

The most extensively utilised wireless technology is inductive charging.


The operating principle is similar to that of a transformer and is based on the laws of magnetic induction. Hence Statement 2 is
correct.
One coil is in the charging station and the other is in in the EV and the energy is transferred from transmitting coils to the receiving
coils. Hence Statement 3 is incorrect.
A primary circuit, called a transmitter, generates a time varying magnetic field.
A secondary circuit receives this field, called the receiver, which is connected to the device to be powered.
To start charging, drivers only need to park their EV above the charging pad.

Types of Inductive charging

Static or stationary charging (used at homes, office): In this method of charging, the EV does not move while being charged. It is
used when EV is parked for a long time.Used at homes, workplaces, public parking, garages etc.
Quasi-dynamic or opportunistic charging: This process permits an EV to get charged while traveling at low speeds and during
momentary pauses.Used at intersections, traffic signals, bus stops etc.
Dynamic EV Charging (Roads and Highways) In this method, owners can charge their EVs continuously as they go. It allows
owners to drive long distances without having to stop for charging or risk running out of power. Used for long distances, inter-city
trips, highways, expressways with restricted accessibility to stops etc.

Que. 384 Consider the following statements regarding NASA's Peregrine Lunar Lander mission:

1. The mission's primary objective includes the analysis of the Moon's exosphere.
2. Peregrine Lunar Lander aims to land in the Mare Tranquillitatis, better known as the Sea of Tranquility.
3. It is part of NASA's Artemis program, focused on returning humans to the Moon by 2024.

Which of the above statements is/are true?

1. 1 only
2. 1 and 2 only
3. 1 and 3 only
4. All of the above

Correct Option - 1
The correct answer is Option 1
Key Points
Peregrine Lunar Lander mission

1. One of the scientific goals of the Peregrine Lunar Lander mission includes the analysis of the lunar exosphere among other
objectives such as assessing thermal properties and hydrogen content of lunar regolith and studying magnetic fields. Hence
Statement 1 is correct.
218/262
2. The Peregrine Lunar Lander's target for exploration is the Moon's Bay of Stickness (also known as Sinus Viscositatis region), not
the Mare Tranquillitatis (Sea of Tranquility). The Bay of Stickness is described as lying adjacent to the Gruithuisen Domes
near Oceanus Procellarum, or Ocean of Storms, which differentiates the mission's landing site from the Sea of Tranquility.Hence
Statement 2 is incorrect.
3. The Peregrine Lunar Lander mission is not a part of NASA's Artemis program. Instead, it specifically notes that the mission is a
component of NASA's Commercial Lunar Payload Services initiative, aimed at partnering with commercial entities to deliver
payloads to the Moon. The Artemis program, which aims to return humans to the Moon, is separate from this particular initiative.
Hence Statement 3 is incorrect.

Additional Information
The Artemis Program
The Artemis Program is NASA's ambitious plan to return humans to the Moon and establish sustainable exploration by the end of the decade.
It marks a significant step in expanding humanity's presence beyond Earth, setting the stage for further exploration of Mars and beyond.
Below are key details about the Artemis Program, its goals, components, and current progress:

Objective and Vision:Return Humans to the Moon: One of the primary objectives of the Artemis program is to return humans to the
Moon for the first time since the Apollo 17 mission in 1972.
Sustainable Presence: Beyond just landing astronauts on the lunar surface, Artemis aims to establish a sustainable human presence on
the Moon by the end of the 2020s.
Gateway Outpost: Part of the sustainability goal involves the construction of the Gateway, a lunar orbiting space station that will
serve as a multi-purpose outpost supporting long-term human lunar exploration and serving as a model for future missions to Mars.

Key Components and Missions

Artemis I: An uncrewed test flight intended to demonstrate the Orion spacecraft and Space Launch System (SLS) rocket capabilities.
It lays the foundation for human deep space exploration.
Artemis II: Planned as the first crewed mission of the Artemis program, Artemis II will orbit the Moon with astronauts onboard,
testing key systems.
Artemis III: This mission aims to land "the first woman and the next man" on the Moon, specifically targeting the lunar South Pole, an
area believed to contain water ice that could be crucial for in-situ resource utilization.
Lunar Gateway: The Gateway will be a vital component for sustaining human exploration, serving as a staging point for missions to
the lunar surface and, eventually, expeditions to Mars.

Partnerships and International Collaboration

Commercial and International Partners: NASA is leveraging partnerships with commercial space companies and international
space agencies to achieve the goals of the Artemis program. This cooperation will manifest in various capacities, including providing
lunar landing services, developing the Gateway, and contributing scientific instruments and technology.

Scientific Goals and Exploration

Lunar South Pole Exploration: Artemis missions will focus on exploring the lunar South Pole, with its permanently shadowed
regions, to study water ice deposits. These deposits are crucial for understanding the Moon's water cycle and potential in-situ resource
utilization.
Geological and Environmental Studies: The program aims to conduct extensive geological studies of the Moon's composition,
structure, and processes. This includes understanding the lunar exosphere, magnetosphere, and surface radiation environment.
Technological Innovation: Advanced Space Suits: Artemis astronauts will be equipped with the next-generation spacesuits that
provide greater mobility and flexibility, making exploration and scientific tasks more efficient.
In-Situ Resource Utilization (ISRU): Technology developments under Artemis will focus on using lunar resources for life support,
fuel, and building materials, key aspects for sustaining long-term presence and reducing dependency on Earth.

Que. 385 Consider the following statements about FEAST software:-


1. It is developed by NASA.
2. It is a computerized method to predict how a structure reacts to real-world forces.
Which of the above statements is/are correct?

1. 1 only
2. 2 only
3. Both 1 and 2
4. Neither 1 nor 2

Correct Option - 2
The correct answer is option 2.
Key Points
FEAST:-
219/262
FEAST (Finite Element Analysis of STructures) is the structural and heat transfer analysis software based on the finite element
method realised by Vikram Sarabhai Space Centre.Hence, Statement 1 is incorrect.
It is supported by state-of-the-art pre/post processor - PreWin.
Its sub-structured and multi-threaded solver implementation ensures high performance by exploiting the multi-core architecture of
modern computing platforms.
The software can be deployed in Linux and Windows operating systems.
It is available in three versions. The classification is based on the number of finite element nodes viz; Academic (for students and
educational institutions), Premium (for small and medium scale industries) and Professional (for general large scale
applications).

Significance of FEAST:

It has real-life applications in structural engineering, solid mechanics and heat transfer problems of Aerospace, Automobile,
Civil, Mechanical and Marine engineering.Hence,Statement 2 is correct.
It performs Finite Element Analysis (FEA) of various structures, including rockets, aircraft, satellites, buildings, etc.
The cost of owning the software is a fraction of the price compared to similar non-indigenous software packages.
So far, users have mainly depended on expensive software versions from foreign firms for this function.
With FEAST, ISRO hopes to take this indigenous FEA software to the Indian user base.
The software can be customized for specific user requirements.
Operable in Windows and Linux operating systems, the software can run on systems with minimal hardware configurations.

Que. 386 Consider the following statements regarding Kilonova radiation:


1. A Kilonova event is observed when a neutron star collides with either another neutron star or a black hole.
2. The aftermath of a Kilonova includes the emission of heavy metals such as gold, silver, and selenium at extremely high speeds, along with
radiation.
3. It is associated with a gamma-ray burst, which is recognized as one of the most powerful events in the universe.
How many of the above statements are correct?

1. Only One
2. Only Two
3. All Three
4. None of the above

Correct Option - 3
The correct answer is Option 3
Key Points
Kilonova Radiation:

Researchers have confirmed the detection of a star system that will one day end in a kilonova.
A newly discovered star system, identified as CPD-29 2176, has captured attention located approximately 11,400 light-years away
from Earth.
Presently, CPD-29 2176 consists of a neutron star and another star that are in orbit around each other.
A kilonova represents an astronomical phenomenon that takes place when a neutron star collides with either another neutron star or a
black hole. Hence Statement 1 is correct.
This event results in the expulsion of heavy metals such as gold, silver, and selenium into space at significant speeds, alongside the
emission of radiation. Hence Statement 2 is correct.
Furthermore, radiation from a kilonova is often accompanied by a gamma-ray burst, one of the universe's most intense energetic
events.Hence Statement 3 is correct.

Additional Information
Neutron stars:

Definition: Neutron stars are the dense remnants of massive stars that have undergone a supernova explosion. They are composed
almost entirely of neutrons.
Formation: They form when the core of a massive star collapses under gravity during a supernova explosion.
Size and Mass: Despite being only around 20 kilometers in diameter, neutron stars have masses about 1.4 times that of the Sun. This
makes them incredibly dense.
Gravity: The gravity on the surface of a neutron star is about 2 billion times stronger than the gravity on Earth.
Magnetic Field: Neutron stars have extremely strong magnetic fields, millions to billions of times stronger than Earth's magnetic field.
Types: There are different types of neutron stars, including pulsars, which emit beams of radiation that can be observed as pulses when
they rotate, and magnetars, known for their extremely powerful magnetic fields.
Rotation: Neutron stars can spin at incredibly high speeds, with some rotating hundreds of times per second.
Significance in Astrophysics: The study of neutron stars helps astronomers understand the extremes of physical laws, including the
behavior of matter under the most extreme conditions.
Accretion and Binary Systems: Neutron stars can accrete material from a companion star if they are in a binary system, leading to
phenomena such as X-ray bursts.

220/262
Importance for Science and Technology: Observations of neutron stars, especially pulsars, have led to applications in areas such as
the testing of general relativity and the development of highly accurate timekeeping systems (pulsar clocks).
Future Research: Research into neutron stars continues to challenge our understanding of physics, with potential implications for the
study of quantum mechanics, nuclear physics, and gravitational waves.

Que. 387 With reference to the Chameleon Trojan, a malware affecting Android devices, consider the following statements:
1. The Chameleon Trojan primarily targets disabling biometric authentication methods to access sensitive user information,
including but not limited to credit card passwords and login credentials.
2. Once it infects a device, it is capable of stealing on-screen content and employing user gestures to capture PINs and passwords, utilizing
stolen PINs to unlock devices and extract sensitive information.
3. Prevention strategies against the Chameleon Trojan include regularly updating the device’s operating system, refraining from installing
apps from unofficial sources, and being cautious about granting 'Accessibility service' permissions to unknown applications.
Which of the statements given above is/are incorrect?

1. 1 and 2 only
2. 2 and 3 only
3. 1 and 3 only
4. 1,2 and 3

Correct Option - 4
The correct answer is Option 4.
Key Points
Chameleon Trojan Description:
This malware possesses capabilities to bypass security methods like fingerprint and facial recognition to covertly obtain confidential data.

Functionality: Chameleon Trojan seamlessly integrates with legitimate Android apps, for example, the widely utilized Google
Chrome app, thereby remaining undetected at the outset. It operates covertly, with a focus on deactivating biometric securities such
as PIN validation while avoiding detection during its operation.
This enables the malware to evade Google Protect notifications and bypass antivirus software on the affected device, operating
under the radar and sidestepping direct security responses. The behavior of this Trojan varies with different versions of Android.
Hence Statement 1 is correct.
Once it becomes operational, it captures the content displayed on the screen, amplifies its access permissions, and may even mimic
user actions to record PINs and passwords used by individuals to access their devices.
The gathered PIN is then used covertly to unlock the device, allowing the malware to collect critical data like credit card
information and account passwords. Additionally, it monitors the application usage patterns of the users, choosing to strike when the
device is presumably idle. Hence Statement 2 is correct.
Protective Measures: To defend against the Chameleon Trojan, it is essential to only download Android applications from reputable
sources. Users should also be cautious about granting 'Accessibility service' permissions to applications that are not recognized, as
a measure to enhance security. Hence Statement 3 is correct.

Que. 388 What feature distinguishes the first batch of Starlink satellites launched by SpaceX as having "Direct-to-Cell" capabilities?

1. Utilization of advanced eNodeB modems onboard, acting as space-based cellphone towers.


2. Limitation to connect exclusively with non-IoT devices.
3. They solely enhance GPS accuracy on smartphones.
4. Deployment of mobile land-based communication towers in remote areas of earth.

Correct Option - 1
The correct answer is Option 1
Key Points
Direct-to-Cell technology.

SpaceX, an American corporation, initiated the deployment of the inaugural series of Starlink satellites endowed with "Direct-to-
Cell" technology.
These satellites are equipped with an advanced onboard eNodeB modem, functioning essentially as a space-based cell tower. Hence
Statement 1 is correct.
Additionally, this innovation facilitates the connection of Internet of Things (IoT) devices, utilizing standard LTE (Long-Term
Evolution) protocols. Hence Statement 2 is incorrect.
The IoT ecosystem comprises interconnected physical objects equipped with sensors, software, and network capabilities, enabling the
aggregation and exchange of data. Hence Statement 3 is incorrect.
Traditionally, signal transmission has relied on terrestrial cell towers.
Such technology enables Starlink to provide uninterrupted satellite broadband connectivity directly to smartphones across the
globe.Hence Statement 4 is correct.

221/262
Additional Information
Direct to Cell technology represents a transformative advancement in telecommunication, bridging the gap between satellite networks
and cellular devices without the need for intermediary infrastructure. This innovative approach enables satellites to communicate
directly with cell phones, radically altering the landscape of mobile connectivity, especially in remote and underserved areas.
Core Functionality

Direct to Cell technology utilizes satellites equipped with advanced communication systems, such as eNodeB modems, effectively
transforming these satellites into space-based cell towers. This setup allows mobile devices to connect directly to satellites
overhead, bypassing the traditional cellular towers and ground-based network infrastructure.

Advantages
Global Coverage: One of the most significant benefits is the potential for truly global mobile coverage, including in rural,
remote, and maritime environments where traditional cell tower infrastructure is either sparse or non-existent.
Enhanced Reliability: By relying on satellites, Direct to Cell technology can provide more reliable service during natural
disasters or other scenarios where ground-based infrastructure might be compromised.
Cost-Effectiveness: For regions without existing cellular infrastructure, this technology offers a cost-effective solution to
extending mobile connectivity without the substantial investment required for building traditional towers and network systems.
Application
Smartphones: Direct to Cell technology enables smartphones to access broadband internet services and maintain connectivity
for calls and messaging anywhere on the planet.
IoT Devices: It also promises to revolutionize the Internet of Things (IoT) by connecting devices directly to satellite networks,
facilitating real-time data exchange and monitoring in even the most remote locations.
Challenges

While Direct to Cell technology offers numerous benefits, there are also challenges to consider, such as regulatory hurdles,
ensuring compatibility across a range of devices and networks, and managing the increased demand on satellite networks.

The Future
As companies like SpaceX deploy Direct to Cell capable satellites, we stand on the brink of a new era in telecommunications.
This technology holds the promise not only of universal connectivity but also of fostering global inclusivity by connecting
underserved communities to the wider world.
By addressing the technical and regulatory challenges, Direct to Cell technology could fundamentally reshape how we think
about communication in the 21st century.

Que. 389 Consider the following statements regarding the SMART 2.0 program:
1. The SMART 2.0 program aims to generate evidence demonstrating the efficacy and safety of Ayurveda interventions.
2. CCRAS functions exclusively under the Ministry of Health and Family Welfare, Government of India, for promoting Ayurveda and Sowa-
Rigpa system of medicine.
3. One of the primary functions of the NCISM is Framing policies for regulating medical institutions and medical professionals of Indian
System of Medicine.
4. The NCISM Act, 2020, facilitates the constitution of a statutory body to ensure the availability of qualified medical professionals of Indian
System of Medicine.
How many of the following statements are correct?

1. Only one
2. Only two
3. Only three
4. All four

Correct Option - 3
The correct answer is option 3
Key Points
SMART 2.0' program
The objective of ‘SMART 2.0’ is to generate a tangible evidence to demonstrate efficacy and safety of Ayurveda interventions using
interdisciplinary research methods and translating it into public health care. Hence statement 1 is correct.
CCRAS is an autonomous body of the Ministry of AYUSH (Ayurveda, Yoga & Naturopathy, Unani, Siddha and Homeopathy),
Government of India. Hence statement 2 is incorrect.
One of the primary functions of the NCISM is Framing policies for regulating medical institutions and medical professionals of Indian
System of Medicine. Hence statement 3 is correct.
The NCISM Act, 2020, facilitates the constitution of a statutory body to ensure the availability of qualified medical professionals of
Indian System of Medicine. Hence statement 4 is correct.

Additional Information
SMART 2.0 program​

cope for Mainstreaming Ayurveda Research among Teaching professionals (SMART) program promotes robust clinical studies in
priority areas of Ayurveda with Ayurveda academic institutions/hospitals across the country through mutual collaboration.

222/262
The objective of ‘SMART 2.0’ is to generate a tangible evidence to demonstrate efficacy and safety of Ayurveda interventions using
interdisciplinary research methods and translating it into public health care.
The study aims at safety, tolerability and adherence to Ayurveda formulations in the priority research areas of Bal Kasa, malnutrition,
insufficient lactation, Abnormal Uterine Bleeding, Osteoporosis in post-menopausal women and Diabetes Mellitus (DM) II.

Central Council for Research in Ayurvedic Sciences (CCRAS)

It is an autonomous body of the Ministry of AYUSH (Ayurveda, Yoga & Naturopathy, Unani, Siddha and Homeopathy), Government
of India.
It is an apex body in India for the formulation, coordination, development and promotion of research on scientific lines in Ayurveda
and Sowa-Rigpa system of medicine.

National Commission for Indian System of Medicine (NCISM​)

It is the statutory body constituted under NCISM Act, 2020. An Act to provide for a medical education system that improves access to
quality and affordable medical education, ensures availability of adequate and high quality medical professionals of Indian System of
Medicine in all parts of the country.
Framing policies for regulating medical institutions and medical professionals of Indian System of Medicine.
Assessing the requirements of healthcare related human resources and infrastructure.
Ensuring compliance by the State Medical Councils of Indian System of Medicine of the regulations made under the Bill.
Ensuring coordination among the autonomous boards.

Que. 390 Consider the following statements about the World Health Organization (WHO)'s launch of the ICD-11 Traditional Medicine
Module 2

1. The ICD-11 Traditional Medicine Module 2 incorporates data and terminology from Ayurveda, Siddha, and Unani Medicine.
2. The Ministry of AYUSH's collaboration with WHO in this endeavor aims to dilute the effectiveness of India’s traditional medicine
sector.
3. The goal of including traditional medical terminologies in ICD-11 is to bridge the gap between traditional medicine and international
healthcare standards.

How many of the above statements are incorrect?

1. Only One
2. Only Two
3. All Three
4. None of the above

Correct Option - 1
The correct answer is Option 1
Key Points
ICD 11 TM Module 2 Launch

The World Health Organization's (WHO) International Classification of Diseases 11th Revision (ICD-11) now includes Module 2
(ICD-11 TM-2).
Module 2, a supplemental chapter within ICD-11, specifically focuses on Traditional Medicine, encompassing Ayurveda, Siddha,
and Unani (ASU) data and terminology. Hence Statement 1 is correct and Statement 2 is incorrect.
The module features over 500 codes, covering a wide range of conditions, including infectious diseases like Malaria and lifestyle-
related diseases such as chronic insomnia.
The ICD serves as the global standard for the systematic recording, reporting, analysis, interpretation, and comparison of
mortality and morbidity data.
Among its chapters, a dedicated section is presented for traditional medicine.
ICD-11's Module-1, which was introduced earlier, centers on traditional medicine conditions that have origins in ancient China and
are widely recognized and used in countries such as Japan and Korea.
ICD-11 officially came into effect in January 2022.
The significance of ICD-11 lies in its provision of a comprehensive list of diagnostic categories. This list enables the collection and
reporting of traditional medicine conditions in a manner that is comparable internationally. Hence Statement 3 is correct.

Que. 391 Consider the following statements:


1. The WHO Framework Convention on Tobacco Control (WHO FCTC) is the first international treaty negotiated under the
auspices of WHO.
2. India is not a party to WHO FCTC.
3. One drawbacks of WHO-FCTC is that it focus only on demand side and not on supply side keeping in mind the interest of tobacco
farmers.
4. In 2008, the WHO established MPOWER, a plan consisting of the six most important and effective tobacco control methods.
223/262
How many of the statements are correct?

1. Only one
2. Only two
3. Only three
4. All four

Correct Option - 2
The correct answer is option 2
Key Points
The WHO Framework Convention on Tobacco Control (WHO FCTC) is the first international treaty negotiated under the auspices of
WHO. Hence statement 1 is correct.
Govt. of India ratified the WHO Framework Convention on Tobacco Control (WHO FCTC) in 2004. Hence statement 2 is incorrect.
WHO-FCTC provides for various measures to reduce the demand as well as supply of tobacco. Hence statement 3 is incorrect.
In 2008, the WHO established MPOWER, a plan consisting of the six most important and effective tobacco control methods. Hence
statement 4 is correct.

Additional Information
WHO Framework Convention on Tobacco Control ​

The WHO FCTC was developed by countries in response to the globalization of the tobacco epidemic.
It aims to tackle some of the causes of that epidemic, including complex factors with cross -border effects, such as trade liberalization
and direct foreign investment, tobacco advertising, promotion and sponsorship beyond national borders, and illicit trade in tobacco
products.
COP7: Ministry of Health & Family Welfare, Government of India organized the Seventh Session of the Conference of Parties (COP7)
under the World Health Organization Framework Convention on Tobacco Control (WHO FCTC) from 7th—12th November 2016 in
Greater Noida.

MPOWER
MPOWER, a plan consisting of the six most important and effective tobacco control methods. The six MPOWER strategies include:

M: Monitor tobacco use and prevention policies


P: Protect people from tobacco smoke
O: Offer help to quit smoking
W: Warn about the dangers of tobacco
E: Enforce bans on tobacco advertising, promotion and sponsorship
R: Raise taxes on tobacco

Que. 392 Which of the following statements is/are true regarding Regarding Bubble Baby Syndrome (BBS), ?

1. BBS is another name for a highly infectious viral condition that mainly affects infants.
2. A baby with BBS has an enhanced immune system that makes them immune to most infections.
3. Babies with Bubble Baby Syndrome (BBS) do not have a functional immune system, making them highly susceptible to severe
infections.
4. The most common treatment for diseases affecting the skeletal system in infants, like BBS, is physical therapy.

Correct Option - 3
The correct answer is Option 3.
Key Points
Bubble Baby Syndrome
Bubble Baby Syndrome (BBS), also medically known as Severe Combined Immunodeficiency (SCID). This disorder significantly impacts
the immune system of those affected.
BBS/SCID is not an infectious disease itself but a genetic disorder that affects the immune system, making the individual highly vulnerable
to infections.Hence Statement 1 is incorrect.
Children with BBS lack a functional immune system. Their immune system is compromised, not enhanced, leaving them unable to fight off
infections effectively.Hence Statement 2 is incorrect.
Bubble Baby Syndrome. Babies affected by this condition do not have a functional immune system. The disorder is characterized by the
absence or malfunctioning of critical immune cells, such as T-cells and B-cells, leading to severe susceptibility to infections. Hence
Statement 3 is correct.
BBS affects the skeletal system and that physical therapy is a treatment, which is incorrect. BBS primarily affects the immune system, not the
skeletal system. The most common treatment for BBS is a stem cell transplant (often a bone marrow transplant), not physical therapy. Hence
Statement 4 is incorrect.

Que. 393 Which of the following statements is/are correct regarding the new treatment regimen for Leprosy introduced by the Ministry?

224/262
1. The Ministry has decided to replace the two-drug regimen with a three-drug regimen for Pauci-Bacillary (PB) cases of leprosy, to be
administered over six months.
2. The Multi-drug therapy (MDT) recommended by WHO for leprosy consists of the three drugs: dapsone, rifampicin, and clofazimine.

1. Only 1
2. Only 2
3. Both 1 and 2
4. None of the above

Correct Option - 3
The correct answer is Option 3.
Key Points
New Treatment of Leprosy:

The Ministry has announced the introduction of a three-drug regimen for treating Pauci-Bacillary (PB) leprosy cases.
This new regimen replaces the previous two-drug treatment and is to be administered over six months. Hence Statement 1 is
correct.
According to the World Health Organization's (WHO) recommendations, the Multi-Drug Therapy (MDT) for leprosy should consist of
three drugs: dapsone, rifampicin, and clofazimine. Hence Statement 2 is correct.

Recent Steps Taken:

The launch of the National Strategic Plan (NSP) & Roadmap for Leprosy (2023-27) aims at achieving zero transmission of
leprosy by the year 2027.
The National Leprosy Eradication Programme, a centrally sponsored scheme, is part of the National Health Mission.
Introduction of the Nikusth 2.0 Portal, an integrated platform for case management of leprosy, marking a significant advancement in
the effort to control and eradicate the disease.

Additional Information
About Leprosy (Hansen’s Disease):

Cause: Leprosy is a chronic infectious disease caused by the bacterium Mycobacterium leprae.
Affects: The disease primarily affects the skin, peripheral nerves, mucosa of the upper respiratory tract, and eyes.
Transmission Mode: Leprosy is transmitted through droplets from the nose and mouth, primarily during close and frequent contact
with untreated cases.
Treatment: The disease is curable with multidrug therapy (MDT).
Classification for Treatment Purposes:
Pauci-Bacillary (PB) Cases: Characterized by having fewer bacteria that are not visible and showing no signs of advanced
disease in biopsies.
Multi-Bacillary (MB) Cases: Characterized by the presence of visible bacteria and may exhibit signs of more advanced
disease in biopsies.

Status of Leprosy in India:

India reached the milestone of eliminating leprosy as a public health concern according to the World Health Organization (WHO)
criteria, which is defined as having less than 1 case per 10,000 population at the national level, in 2005.
The national Prevalence Rate of leprosy has declined to 0.45 per 10,000 population in the fiscal year 2021-22, compared to 0.69 per
10,000 population in 2014-15.

Que. 394 The term ANEEL recently in news is related to

1. Use of State of Art Technology to tap wind energy.


2. Installation of wind mills along the coastal countries.
3. An American company has developed a nuclear fuel (named after India’s scientist, Dr Anil Kakodkar).
4. Promote the use of renewable energy among least developed countries.

Correct Option - 3
The correct answer is Option 3.
Key Points
ANEEL (ADVANCED NUCLEAR ENERGY FOR ENRICHED LIFE)

ANEEL is fuel developed by an American company, named in honor of Indian scientist Dr. Anil Kakodkar.
It is a mix of Thorium and Uranium, enriched to the level of High Assay Low Enriched Uranium (HALEU), with an enrichment
level of 5-20%.
Most current nuclear reactors use fuel enriched up to 5%, making HALEU more enriched.

225/262
Potential benefits of ANEEL include compatibility with existing Pressurized Heavy-Water Reactors (PHWRs) in India and a
reduction in nuclear waste.

Additional Information
Institutions of Nuclear Energy in India:

NITI Aayog and Central Electricity Authority are focusing on nuclear power to help India in its clean energy transition.
Atomic Energy Commission (AEC): Governs the Department of Atomic Energy (DAE), under the Prime Minister. Organizes atomic
research and promotes nuclear research in India.
Atomic Energy Regulatory Board (AERB): Formed in 1983 under AEC, it oversees nuclear safety regulations.
Nuclear Power Corporation of India (NPCIL): Owns and operates India's nuclear power plants (except PFBR), responsible for their
design, construction, and operation.
Bhabha Atomic Research Centre: Falls under DAE, focuses on peaceful applications of nuclear energy and oversees nuclear power
production aspects.

Advantages of Nuclear Energy in India:

Energy security: Offers a reliable power supply compared to intermittent wind and solar energy.
Reduction in GHGs: Nuclear power doesn't emit greenhouse gases, aiding in reducing India's GHG contribution.
Sustainable future: Aiming to increase atomic power's contribution to meet long-term energy security.
Weather-independent power: Provides consistent power supply and requires a smaller land footprint than renewable sources.

Disadvantages of Nuclear Energy in India:

Limited private sector participation: Operations and fuel management are controlled by the public sector.
Nuclear liability issues: The Civil Liability for Nuclear Damage Act 2010 deters foreign companies due to potential high liabilities.
Challenges for AERB: Regulating scattered nuclear/radiation facilities and meeting modern safety expectations present significant
challenges.
Uranium scarcity and import dependency: Regular uranium imports are needed, causing supply uncertainties and dependency on
foreign suppliers.

Que. 395 Landslide Atlas of India is released by which of the following?

1. National Remote Sensing Centre (NRSC)


2. Geological Survey of India
3. IIT Mumbai
4. IIT Delhi

Correct Option - 1
The correct answer is option 1.
Key Points
Landslide Atlas of India

National Remote Sensing Centre (NRSC) under the Indian Space Research Organisation (ISRO) has released the Landslide
Atlas of India. Hence statement 1 is correct.

Additional Information
Scientists did a risk assessment on the basis of 80,000 landslides recorded between 1998 and 2022 in 147 districts in 17 states and
two Union Territories to build a "Landslide Atlas" of the country.
The pan-India landslide database classifies landslides into – seasonal (2014, 2017 monsoon seasons), event-based and route-based
(2000 – 2017).
Mizoram topped the list, recording 12,385 landslide events in the past 25 years, of which 8,926 were recorded in 2017
alone, followed by Uttarakhand (11,219) and Kerala.
The number of districts with the maximum landslide exposure are in Arunachal Pradesh (16), Kerala (14), Uttarakhand and
Jammu and Kashmir (13 each), Himachal Pradesh, Assam and Maharashtra (11 each), Mizoram (8) and Nagaland (7).
Rudraprayag and Tehri Garhwal districts of Uttarakhand have the highest landslide density and landslide risk exposure in the
country.

Que. 396 "CAIPEEX" recently seen in news is related to

1. Stock Market
2. RBI reforms
3. Cloud seeding
4. Biotechnology

Correct Option - 3
226/262
The correct answer is option 3.
In News
The Hindu: IITM Pune demonstrates cloud seeding can produce rainfall.

Key Points
Cloud Seeding:

Cloud seeding is the process of spreading either dry ice or more commonly, silver iodide aerosols, into the upper part of clouds
to try to stimulate the precipitation process and form rain.
Cloud seeding uses planes to spray clouds with chemicals to condense smaller particles into larger rain droplets.
Cloud seeding increases rainfall rates by approximately 10% to 30% per year and cloud seeding operations cost much less than the
desalination process.
Cloud Seeding Methods:
Hygroscopic Cloud Seeding: Disperses salts through flares or explosives in the lower portions of clouds. The salt grows in size
as water joins with them.
Static Cloud Seeding: It involves spreading a chemical like silver iodide into clouds. The silver iodide provides a crystal
around which moisture can condense. The moisture is already present in the clouds, but silver iodide essentially makes rain
clouds more effective at dispensing their water.
Dynamic Cloud Seeding: It aims to boost vertical air currents, which encourages more water to pass through the clouds,
translating into more rain.
The process is considered more complex than static cloud seeding because it depends on a sequence of events working properly.
A cloud seeding experiment carried out in Solapur city, which falls on the leeward side of the Western Ghats and hence gets low
rainfall — 384 mm and 422 mm of total rainfall during the period June to September 2018 and 2019, respectively — was able to
achieve 18% relative enhancement in rainfall, which is approximately 8.67mm more rainfall. The relative enhancement of accumulated
rainfall was seen over two hours after seeding the clouds. In all, the total enhancement of water availability through cloud seeding
experiments was 867 million litres.
The experiment — Cloud Aerosol Interaction and Precipitation Enhancement Experiment (CAIPEEX phase-4) — was a
scientific investigation conducted in Solapur city during the summer monsoon period of 2018 and 2019. The primary objective was
to investigate the efficacy of hygroscopic seeding in deep convective clouds and to develop a cloud seeding protocol. The
experiment used two aircraft for studying various cloud parameters and for seeding the clouds. Hence option 3 is correct.

Que. 397 Which of the following is correct about LIQUID 3 ?

1. It is new battery developed as alternative to Lead batteries.


2. It is new type of rocket fuel developed by SpaceX.
3. It is third generation ethanol brand.
4. It is photo-bioreactor which produce pure oxygen through photosynthesis by using microalgae.

Correct Option - 4
The correct answer is option 4.
Key Points
Liquid tree also called LIQUID 3

Liquid tree also called LIQUID 3 is Serbia’s first urban photo-bioreactor, a solution in the fight for clean air. It contains six hundred
litres of water and works by using microalgae to bind carbon dioxide and produce pure oxygen through photosynthesis. Hence
statement 1 is correct.
The advantage of microalgae is that it is 10 to 50 times more efficient than trees.
Its functioning is similar to that of trees and grass, but it is more efficient than trees because of the ability of microalgae to survive in
polluted environments.

227/262
Que. 398 Consider the following statement about South Atlantic Anomaly :
1. It is a weak spot in Earth’s magnetic field, which protects the planet from high doses of solar wind and cosmic radiation.
2. It leads to the penetration of solar energetic particles deep into Earth’s atmosphere.
3. It exists because the Earth's inner Van Allen radiation belt comes closest to the planet’s surface.
How many of the above given statements are correct.

1. Only one
2. Only two
3. Only three
4. None

Correct Option - 3
The correct answer is option 3.
Key Points
South Atlantic Anomaly (SAA)

The South Atlantic Anomaly is a weak spot in Earth’s magnetic field, which protects the planet from high doses of solar wind and
cosmic radiation. Hence statement 1 is correct.
This leads to the penetration of solar energetic particles deep into Earth’s atmosphere, posing severe problems for airplanes and
ships' positioning systems as well as spacecraft electronic systems. Hence statement 2 is correct.
This anomaly exists because the Earth’s inner Van Allen radiation belt comes closest to the planet’s surface, causing an increased
flux of energetic particles. Hence statement 3 is correct.
It stretches out between South America and southwest Africa.

Additional Information
Van Allen radiation belts

It t is a zone of energetic charged particles, most of which originate from the solar wind.
The particles are captured by and held around a planet by that planet’s magnetic field.
Acts as a shield, blocking highly energetic electrons from reaching Earth.
Outer belt consists of high-energy particles trapped by Earth's magnetosphere, originating from the Sun.
Inner belt formed by cosmic rays interacting with Earth's atmosphere.

Que. 399 Consider the following statements about First Waterbodies Census:
1. It is released by Ministry of Jal Shakti.
2. West Bengal has the highest number of ponds and reservoirs.
3. The census also includes swimming pool.
How many of the given statements are correct?

1. Only one
2. Only two
3. All three
4. None.

228/262
Correct Option - 2
The correct answer is option 2.
Key Points
First Waterbodies Census

The Ministry of Jal Shakti has conducted the first-ever census of water bodies in India. Hence statement 1 is correct.
The census enumerated a total of 24,24,540 water bodies across the country, with West Bengal accounting for the most (7.47 lakh)
and Sikkim the least (134). Hence statement 2 is correct.
It defines a waterbody as "all-natural or man-made units bounded on all sides with some or no masonry work used for storing water for
irrigation or other purposes."
Ocean, lagoons, rivers, Stream, springs, waterfalls, canals, etc which are free-flowing without any bounded storage of water,
swimming Pool etc are excluded from the census. Hence statement 3 is incorrect.

Additional Information
The top 5 States in terms of the number of water bodies- West Bengal, Uttar Pradesh, Andhra Pradesh, Odisha, and Assam which
constitute around 63% of the total water bodies in the country.
States/UTs with the least water bodies: Sikkim, Chandigarh, Delhi.
The report reveals that:
West Bengal has the highest number of ponds and reservoir.
The top district in terms of waterbodies is South 24 Parganas in West Bengal.
Andhra Pradesh has the highest number of tanks.
Tamil Nadu has the highest number of lakes
Maharashtra leads in water conservation schemes.

Que. 400 Upwards lightning best refers to which one of the following situations?

1. It is a phenomenon whereby a self-initiated lightning streak develops from a tall object that travels upward toward an overlaying
electrified storm cloud.
2. It is a military terminology for launching air defence system.
3. It refers to artificial light pollution from the ground causing disorientation to birds.
4. It refers to the bright lights emitted by rockets.

Correct Option - 1
The correct answer is option 1.
Key Points
Upwards lightning

Upward lightning is a phenomenon where lightning streaks develop from tall objects that travel upward toward electrified storm
clouds (opposite to normal Lightening). Hence statement 1 is correct.

Additional Information
​ pward lightning occurs in the presence of tall structures like skyscrapers or lightning rods under storm clouds charged with
U
electricity. These high objects intensify the electric field at ground level, which can lead to the formation of an ionized air path, known
as an upward leader, that moves upward toward the cloud. At the same time, a downward-stepped leader may extend from the cloud.
When these two leaders meet, they create a complete electrical circuit, allowing for the discharge of electricity from the cloud to the
ground

Que. 401 Consider the following statements about The SATAT (Sustainable Alternative Towards Affordable Transportation) scheme:
1. It promote the use of Compressed Biogas (CBG) as a clean and alternative fuel for transportation.
2. The initiative is exclusively aimed at providing a Sustainable Alternative Towards Affordable Transportation which will benefit only the
users.
3. The scheme is under the aegis of the Ministry of Petroleum and Natural Gas.
How many of the given statements are correct?

1. Only one
2. Only 2
3. All three
4. None

Correct Option - 2

229/262
The correct answer is option 2.
Key Points
The SATAT (Sustainable Alternative Towards Affordable Transportation) scheme

It is an initiative launched by the Government of India to promote the use of Compressed Biogas (CBG) as a clean and alternative
fuel for transportation. Hence statement 1 is correct.
The initiative is aimed at providing a Sustainable Alternative Towards Affordable Transportation (SATAT) as a developmental effort
that would benefit both vehicle-users as well as farmers and entrepreneurs. Hence Statement 2 is incorrect.
The scheme is under the aegis of the Ministry of Petroleum and Natural Gas. Hence statement 3 is correct.

Additional Information
Compressed Bio-Gas plants are proposed to be set up mainly through independent entrepreneurs.
CBG produced at these plants will be transported through cascades of cylinders to the fuel station networks of OMCs for marketing as
a green transport fuel alternative.
The entrepreneurs would be able to separately market the other by-products from these plants, including bio-manure, carbon-dioxide,
etc., to enhance returns on investment.
It is planned to roll out 5,000 Compressed Bio-Gas plants across India in a phased manner.
This initiative is expected to generate direct employment for 75,000 people and produce 50 million tonnes of bio-manure for crops.
This initiative is expected to generate direct employment for 75,000 people and produce 50 million tonnes of bio-manure for crops.

What is CBG?

Compressed Biogas (CBG) is a renewable and sustainable fuel produced from the anaerobic digestion of organic materials such as
agricultural residue, food waste, animal dung, and sewage.
Compressed Biogas (CBG), which is similar to Compressed Natural Gas (CNG) in terms of its properties and usage.
CBG has pure methane content of over 95%.

Que. 402 Consider the following statements about UN High Seas Treaty:
1. It is the first-ever treaty to protect the world's oceans that lie outside national boundaries.
2. It is not a legally binding treaty.
3. The treaty is aim at protecting the rights of free and safe passage of vessels in high seas.
How many of the statements are correct?

1. Only one
2. Only two
3. All three
4. None

Correct Option - 1
The correct answer is option 1.
Key Points
UN High Seas Treaty

The High Seas Treaty, also known as the agreement on Biodiversity Beyond National Jurisdiction is the first-ever treaty to
protect the world's oceans that lie outside national boundaries. Hence statement 1 is correct.
t is also known as the ‘Paris Agreement for the Ocean.’
It is a legally binding treaty, new agreement contains 75 articles that aim at protecting, caring for, and ensuring the responsible
use of the marine environment, maintaining the integrity of ocean ecosystems, and conserving the inherent value of marine
biological diversity. Hence statement 2 and 3 is incorrect.​
Adopted by the Intergovernmental Conference on Marine Biodiversity of Areas Beyond National Jurisdiction (BBNJ), the “high
seas” treaty aims at taking stewardship of the ocean on behalf of present and future generations, in line with the Convention on the
Law of the Sea.

Additional Information
Targets setting aside 30% of marine areas as protected zones by 2030, in line with commitments from the 2022 UN biodiversity
conference.
Introduces a legal structure for creating extensive Marine Protected Areas (MPAs) to safeguard against wildlife loss and distribute
high seas' genetic resources fairly.
Mandates environmental assessments for understanding the impacts of commercial ventures, like deep-sea mining, on marine
environments.
Focuses on enhancing the resilience of ocean ecosystems, incorporating the polluter-pays principle for environmental restoration
and setting up dispute resolution mechanisms.
Advocates for an ecosystem-based approach to ocean management that increases resilience against climate change effects and ocean
acidification, and emphasizes the preservation and restoration of ecosystem services, including those related to carbon sequestration.
Acknowledges the importance of indigenous peoples' and local communities' knowledge rights, supports unfettered scientific
research, and promotes the equitable sharing of marine benefits.
Pays special attention to the unique challenges of small island states and landlocked developing countries.

230/262
Plans the creation of a Conference of the Parties (CoP) for regular meetings and accountability mechanisms concerning governance
and marine biodiversity conservation.
Commits its members to cooperative sharing and management of oceanic resources.

Que. 403 Consider the following statements about short-lived halogens:


1. It helps in cooling the planet by depleting ozone in the troposphere.
2. It increase methane’s lifetime in the atmosphere.
3. It increase the level of water vapour, a greenhouse gas, in the atmosphere.
How many of the given statements are correct?

1. Only one
2. Only two
3. All three
4. None

Correct Option - 3
The correct answer is option 3.
Key Points
Short-Lived Halogens

Oceans do more than just absorbing carbon dioxide and moderating the climate. They also cool the planet by releasing short-lived
halogens such as chlorine, bromine and iodine.
The short-lived halogens from the ocean reduce warming by depleting ozone, ozone is a greenhouse gas that traps outgoing
radiation, leading to warming. Hence statement 1 is correct.
It increase methane’s lifetime in the atmosphere by destroying hydroxyl radicals (OH), OH is a sink as it is known to break down
this greenhouse gas. Hence statement 2 is correct.
Short-lived halogens increased global methane burden by 14 per cent and 9 per cent for pre-industrial and present-day conditions,
respectively.
It increase the level of water vapour, a greenhouse gas, in the atmosphere. Hence statement 3 is correct.
Though these halogens drive an increase in warming by influencing methane, water vapour and aerosols, they compensate this
by destroying ozone, which exerts a cooling effect.
Halogen emissions from the ocean are not the same across the world. Over continents, the emissions are small while it is bigger in
polar regions and some places with higher ozone levels.
Currently, these halogens contribute 8-10 per cent of cooling. This could increase to 18-31 per cent by 2100.

Que. 404 Consider the following statements about Commission for Air Quality Management :
1.The CAQM is a statutory body.
2. It has been set up for Air Quality Management in National Capital Region and Adjoining Areas.
3. The Chief Minister of Delhi is the ex-officio chairman of CAQM.
How many of the given statements are correct?

1. Only one
2. Only two
3. All three
4. None

Correct Option - 2
The correct answer is option 2.
Key Points
CAQM

The CAQM was formed under the Commission for Air Quality Management in National Capital Region and Adjoining Areas,
Act 2021 as a statutory body. Hence statement 1 is correct.
The Commission has been set up for Air Quality Management in National Capital Region and Adjoining Areas for better co-
ordination, research, identification and resolution of problems surrounding the air quality index and for matters connected therewith or
incidental thereto. Hence statement 2 is correct.
CAQM to be chaired by a government official of the rank of Secretary or Chief Secretary. He will hold the post for three years or
until s/he attains the age of 70 years. Hence statement 3 is incorrect.

Additional Information
CAQM has replaced the Environment Pollution (Prevention and Control) Authority (EPCA) which had been appointed by the Supreme
Court and was operational for 22 years.

231/262
Non-compliance or contravention with the Commission’s orders or directions can lead to a prison term of up to five years
and/or a fine of up to Rs. 1 crore.
Only the National Green Tribunal will have the authority to hear cases involving the CAQM, excluding the civil courts.

Que. 405 Consider the following statements about Resource Efficiency Circular Economy Industry Coalition (RECEIC):
1. Resource Efficiency Circular Economy Industry Coalition (RECEIC) is launched by Union Minister of Commerce and
Industry.
2. It is envisaged to be industry-driven and a self-sustaining initiative.
3. The coalition is structured around the three guiding pillars of partnerships for impact, technology cooperation and finance for scale.
How many of the given statements are correct?

1. Only one
2. Only 2
3. All three
4. None

Correct Option - 2
The correct answer is option 2.
Key Points
Resource Efficiency Circular Economy Industry Coalition (RECEIC)

Union Minister Bhupender Yadav launch the Resource Efficiency Circular Economy Industry Coalition (RECEIC) on the sidelines
of the 4th G20 Environment and Climate Sustainability Working Group (ECSWG) and Environment and Climate Ministers
meeting in Chennai. Hence statement 1 is incorrect.
The RECEIC, conceptualised by the India’s G20 Presidency is envisaged to be industry driven and a self-sustaining initiative
continuing to function even beyond India’s G20 Presidency. Hence statement 2 is correct.
The coalition is structured around the three guiding pillars of partnerships for impact, technology cooperation and finance for
scale. Hence statement 3 is correct.

Additional Information
Mission of RECEIC
to facilitate and foster greater company-to-company collaboration,
build advanced capabilities across sectors and value chains,
bring learnings from diverse and global experiences of the coalition members, and
unlock on-ground private sector action to enhance resource efficiency and accelerate circular economy transition.
Circular Economy
The circular economy is a model of production and consumption, which involves sharing, leasing, reusing, repairing,
refurbishing and recycling existing materials and products as long as possible.

Que. 406 Consider the following statements about Carbon Credit Trading Scheme:
1. It is a new initiative by the Ministry of New and Renewable Energy .
2. Carbon credit certificates will be issued by the Bureau of Energy Efficiency (BEE).
3. Grid Controller of India Limited (GCIL) acts as the registry for the scheme, undertaking the registration of entities and maintaining a
record of transactions.
How many of the given statements are correct?

1. Only one
2. Only two
3. All three
4. None

Correct Option - 2
The correct answer is option 2.
Key Points
Carbon Credit Trading Scheme

The term “Carbon Credit” refers to a tradable permit that represents the right to emit a specific amount of carbon dioxide or an
equivalent amount of another greenhouse gas.
The Carbon Credit Trading Scheme, 2023, is an initiative by the Ministry of Power to regulate and incentivize the reduction of
greenhouse gas emissions in India. Hence statement 1 is incorrect.
Carbon credit certificates will be issued by the Bureau of Energy Efficiency (BEE) to entities that surpass their assigned emission
reduction target. Hence statement 2 is correct.
These certificates will be traded on power exchanges registered with the Central Electricity Regulatory Commission (CERC).

232/262
Grid Controller of India Limited (GCIL) acts as the registry for the scheme, undertaking the registration of entities and
maintaining a record of transactions. Hence statement 3 is correct.​

Additional Information
The Ministry of Power, based on recommendations from the Bureau of Energy Efficiency, notifies entities obligated to comply with
the trading scheme.
The Ministry of Environment, Forest, and Climate Change sets the emission intensity targets for obligated entities, upon the
recommendation of the Ministry of Power.

Que. 407 The term " Infochemicals'' is recently seen in the news. It is best described as

1. Chemical compounds that carry information.


2. Chemicals formed by AI based chemical lab using voice command.
3. Chemical compounds with detailed information on its packaging.
4. Chemical Compound that should mandatorily include precautionary warning during packaging.

Correct Option - 1
The correct answer is option 1.
Key Points
Infochemicals

A chemical compound carrying information that is employed by organisms. Hence option 1 is correct
Climate change is impacting chemical communication in the animal world, with significant consequences for ecosystems and
human well-being.
Climate change is altering the production of these chemicals, such as hormones, pheromones, and allelochemicals, affecting various
species and disrupting vital functions and behaviours
Example- Warming temperatures cause a decay in the pheromones used by some ant species for trail-following, affecting their ability
to communicate effectively.
This Chemical signals, known as infochemicals, regulates interactions between organisms, supports ecosystem equilibrium, and
underpins critical ecological processes affecting food provision and clean water for humans.

Que. 408 The term "Cryptobiosis" is best described as

1. a state of extreme inactivity in response to adverse environmental conditions.


2. Crypto currency in developmental process.
3. Crypto currency generated using green energy.
4. an experimental economy which uses cryptocurrency for day to day transaction.

Correct Option - 1
The correct answer in option 1.
Key Points
Cryptobiosis

Cryptobiosis is a state of extreme inactivity in response to adverse environmental conditions. Hence statement 1 is correct.
In the cryptobiotic state, all metabolic procedures stop, preventing reproduction, development, and repair.
In cryptobiosis, an organism can live almost indefinitely while it waits for environmental conditions to become better.
The most common type of cryptobiosis is desiccation or drying out.

Additional Information
Types of Cryptobiosis

Anhydrobiosis- Anhydrobiosis means “life without water”.


Anoxybiosis- A biological response triggered by a lack of oxygen in which an organism takes in water and becomes turgid and
immobile.
Chemobiosis- It is the suspension of metabolic activities in response to high levels of metabolic toxins.
Cryobiosis- Cryobiosis is a form of cryptobiosis that takes place in reaction to decreased temperature.
Osmobiosis- Osmobiosis is a form of cryptobiosis that results from elevated osmotic pressures.

Que. 409 The term "Shelf Cloud" is recently seen in the news. It is best described as

1. Cloud computing in nearby local data centres.


2. It is a type of low-lying, horizontal cloud formation characterized by a clearly defined line of solid clouds.
3. Decentralized and distributed data storage in nearby personal computers.

233/262
4. Cloud storage used to hide sensitive information.

Correct Option - 2
The correct answer is option 2.
Key Points
Shelf Cloud

It is a type of low-lying, horizontal cloud formation characterized by a clearly defined line of solid clouds. Hence statement 2 is
correct.
A “shelf cloud” or “Arcus cloud” is generally forms along the leading edge of thunderstorms.
It has distinctive wedge-shaped formation.
They usually appear as a broad arc across the sky that can sometimes appear to be rotating horizontally.

Additional Information
It is form when cold and dense air is forced into a warmer air mass by wind.
This rush of cold air often occurs in a thunderstorm’s downdraught, where cold air rushes towards the ground before spreading out to
create a gust front.
Shelf clouds produced by thunderstorms are always preceded by a rush of dry and cold air ahead of the cloud, with rain arriving after
the shelf cloud has passed overhead.

Que. 410 Consider the following statements about Pacific Decadal Oscillation (PDO):
1. The PDO waxes and wanes approximately every 20 to 30 years.
2. PDO shares similarities with the El Niño-Southern Oscillation (ENSO) yet it functions over a more extended period and influences distinct
outcomes.
3. The 'cool' phase is characterized by a cool wedge of lower than normal sea-surface heights/ocean temperatures in the eastern equatorial
Pacific.
How many of the given statements are correct?

1. Only one
2. Only two
3. All three
4. None

Correct Option - 3
The correct answer is option 3.
Key Points
Pacific Decadal Oscillation (PDO)

The Pacific Decadal Oscillation (PDO) is a long-term climate pattern that affects the temperature of the Pacific Ocean and the weather
patterns around it.
The PDO is a naturally occurring phenomenon that shifts between warm and cool phases, with each phase lasting around 20-30 years.
Hence statement 1 is correct.
The PDO shares similarities with El Niño-Southern Oscillation (ENSO) but operates on a longer timescale and has different effects.
Hence statement 2 is correct.

Additional Information
The PDO oscillates between positive and negative phases.
From ocean surface topography data, together with other ocean and atmospheric data, scientists can determine whether we are in
a ‘cool’ phase or a ‘warm’ phase.
The 'cool' phase is characterized by a cool wedge of lower than normal sea-surface heights/ocean temperatures in the eastern
equatorial Pacific and a warm horseshoe pattern of higher than normal sea-surface heights connecting the north, west and
southern Pacific.
In the 'warm' or 'positive' phase, the west Pacific Ocean becomes cool and the wedge in the east warms.
However, more recently, the ‘warm’ and ‘cold’ phases have been much shorter.

Positive Phase Impacts

It is often associated with milder winters in the western United States, increased rainfall in the Pacific Northwest, and changes in
marine ecosystems.
In the northern hemisphere wintertime, much of Asia is usually cooler than normal, with above-normal temperatures more likely
over India.
At the same time, China and Japan are likely to be drier than normal, while India often has a winter that is wetter than normal.

​Negative Phase Impacts

It can lead to colder winters in the western U.S., drier conditions in the Pacific Northwest, and different marine ecosystem dynamics.

234/262
Many parts of the US will see drier than normal weather, but there are some notable exceptions, winter rainfall is usually above normal
for the Ohio and Tennessee Valleys as well as the Northern Rockies and Plains.
In the northern hemisphere wintertime, much of India and China is usually cooler than normal, But Japan has warmer than normal
weather, especially in the North.
Japan and Northwest China are usually wetter than normal, while Southwest China and much of India are usually wetter than
normal.

Que. 411 Which of the following measures was taken by the Sikkim State Disaster Management Authority to reduce the risk of a Glacial
Lake Outburst Flood (GLOF) at South Lhonak Lake?

1. Increasing the depth of the lake to store more water.


2. Using High Density Polyethylene (HDPE) pipes to drain water from the lake.
3. Building a large earthen dam to replace the moraine dam.
4. Completely freezing the lake to prevent any outburst.

Correct Option - 2
The correct answer is option 2.
Key Points
Glacial Lake Outburst Floods:-

A GLOF (Glacial Lake Outburst Flood) is a sudden and potentially catastrophic flood that occurs when water stored behind a
glacier or a moraine (a natural accumulation of ice, sand, pebbles, and debris) is released rapidly.
These floods happen when glacial lakes formed by melting ice accumulate water behind weak moraine dams.
Unlike sturdy earthen dams, these moraine dams can fail abruptly, releasing large volumes of water in minutes to days, leading to
devastating downstream flooding.

​Disaster Management of GLOF:-

Such was the potential risk at South Lhonak Lake in Sikkim, situated at an elevation of approximately 5,200 meters above sea
level.
The lake had been expanding due to ice melt, exacerbated by factors like climate change, which increases the rate of glacier melting,
and seismic activities, particularly a notable earthquake in 2011.
Recognizing the augmented risk of a GLOF, the Sikkim State Disaster Management Authority, with the leadership of innovator
Sonam Wangchuk, undertook a mitigation effort in 2016.
The innovative approach involved employing High Density Polyethylene (HDPE) pipes to siphon off water, successfully
lowering the lake's volume by about 50%.Hence, Statement 2 is correct. This action aimed to reduce the pressure on the moraine
dam and mitigate the GLOF risk, showcasing a proactive approach to disaster risk reduction.
Other measures like early warning systems, glacial lake monitoring, public awareness, and international cooperation are also
critical for comprehensive GLOF risk management.

Que. 412 Consider the following statements about Sunderbans Region:-


1. It is located between Indian and Bangladesh
2. It was designated as a Biosphere Reserve by UNESCO in 2001.
3. Sunderbans Delta is the only mangrove forest in the world inhabited by tigers.
Which of the following statements is/are correct?

1. Only one
2. Only two
3. All three
4. None

Correct Option - 3
The correct answer is option 3.
Key Points
Sunderbans Region:-

I​ t is a vast contiguous mangrove forest ecosystem in the coastal region of Bay of Bengal spread over India and Bangladesh on the
delta (world’s largest) of the Ganges, Brahmaputra and Meghna rivers.Hence, Statement 1 is correct.
It contains the world’s largest mangrove forests.
Much of the area has long had the status of a forest reserve, but conservation efforts in India were stepped up with the creation of the
Sundarbans Tiger Reserve in 1973.
Sundarbans National Park, established in 1984, constitutes a core region within the tiger reserve; it was designated a UNESCO
World Heritage site in 1987.
Sunderbans was designated as a Biosphere Reserve by UNESCO in 2001.Hence, Statement 2 is correct.

235/262
Sundarban Wetland, India was recognised as the ‘Wetland of International Importance’ under the Ramsar Convention in January
2019.
Sunderban National Park is known for its wide range of fauna, including 260 bird species and is home to many rare and globally
threatened wildlife species such as the Estuarine Crocodile, Royal Bengal Tiger, Water Monitor Lizard, Gangetic Dolphin and Olive
Ridley Turtles.
The Sunderbans Delta is the only mangrove forest in the world inhabited by tigers.Hence, Statement 3 is correct.
For its preservation, Discovery India and World Wide Fund (WWF) India partnered with the Government of West Bengal and local
communities in the Sundarbans in 2019.

Additional Information

Que. 413 Consider the following statements about phosphorous:-


1. It is highly reactive and can spontaneously combust in air, producing a red smoke.
2. It is commonly found in the Earth's crust.
3. India is the world's largest importer of phosphorus.
How many of the above statements is/are correct?

1. Only one
2. Only two
3. All three
4. None

Correct Option - 2
The correct answer is option 2.
Key Points
Phosphorus:-

It is a chemical element with the symbol "P" and atomic number 15.
It is an essential element for life and has various important properties and applications.

Chemical Properties:

Phosphorus readily forms compounds with other elements, especially oxygen, forming various phosphates.
It is highly reactive and can spontaneously combust in air, producing a white smoke. Hence, Statement 1 is incorrect.
Phosphorus compounds are crucial in biology, as they are a fundamental component of DNA, RNA, and ATP (adenosine
triphosphate).

Natural Occurrence:

Phosphorus is commonly found in the Earth's crust in the form of various phosphate minerals, such as apatite.Hence, Statement
2 is correct.

Industrial Uses:

Phosphorus compounds are used in the production of fertilizers, as they are necessary for plant growth.
It is also used in detergents, where phosphate compounds help break down and remove stains.
Phosphorus is utilized in the production of steel and other metallurgical processes.

Phosphorus in India:
236/262
India is deficient in Apatite (group of phosphate minerals) & Rock Phosphate availability.
According to Indian Minerals Yearbook 2018, in case of apatite, the country is fully dependent upon imports, while the Rock
Phosphate production is only from two states namely, Rajasthan and Madhya Pradesh.
India is the world's largest importer of phosphorus, primarily sourcing it from West African deposits contaminated with
cadmium. Hence, Statement 3 is correct.
Paddy, a staple crop in India, is particularly prone to cadmium uptake, and Indian farmers extensively use fertilizers on paddy fields.

Que. 414 Consider the following statements about Pangolin:-


1. They are nocturnal mammals that dig burrows and feed on ants and termites.
2. They have scales made of keratin that cover their entire body.
3. Indian Pangolin is listed in the Critically Endangered (EN) category under Red List of IUCN.
How many of the above statements is/are correct?

1. Only one
2. Only two
3. All three
4. None

Correct Option - 2
The correct answer is option 2.
Key Points
Characteristics of Pangolin:-

​ hey are nocturnal mammals that dig burrows and feed on ants and termites, and play a vital role in ecosystem management,
T
mostly in aerating and adding moisture to the soil.Hence,Statement 1 is correct.
They are known for their unique appearance.
They have scales made of keratin that cover their entire body.Hence, Statement 2 is correct.
When threatened, they can roll into a ball to protect themselves.

Pangolin Species: There are eight species of pangolin:

4 Species in Africa: Black-bellied pangolin, White-bellied pangolin, Giant Ground pangolin and Temminck's Ground pangolin.
4 Species in Asia: Indian pangolin, Philippine pangolin, Sunda pangolin and the Chinese pangolin.

Habitat:

It is adaptable to a wide range of habitats including primary and secondary tropical forests, limestone and bamboo forests,
grasslands and agricultural fields.
The Indian Pangolin is found across the Indian subcontinent; Bihar, West Bengal, and Assam also have the presence of Chinese
pangolin.

Protection Status:

In the red list of animals published by the International Union for Conservation of Nature (IUCN), Indian Pangolin is listed in the
Endangered (EN) category.Hence, Statement 3 is incorrect.
The Chinese pangolin has been listed as “critically endangered”.
In India, pangolins, both Indian and Chinese, are protected under Schedule 1 of the Wildlife (Protection) Act 1972 that prohibits its
hunting, trade or any other form of utilisation.
All pangolin species are listed in Convention on International Trade in Endangered Species (CITES) Appendix I.

Additional Information

237/262
14/05/2024, 13:51 lms.testbook.com/genpdf/fromques.php?qids=65f93cf52fe08b6c70dc4d11,65f93e6eb5431ddfb31dfe15,65f96fd18acd632ab8e39282,65f97…

Que. 415 Kamlang Tiger Reserve recently in news is located in which state?

1. Arunachal Pradesh
2. Assam
3. Nagaland
4. Meghalaya

Correct Option - 1
The correct answer is option 1
Key Points
Kamlang Tiger Reserve:-
Location:

It is located in the southeastern part of Lohit District in Arunachal Pradesh near the border with Myanmar.
It is named after the Kamlang River, which flows through the reserve and joins the Brahmaputra.
It is spread between the famed Namdapha National Park on its south and Lang River on its north.
The local inhabitants are Hishmi, Digaru and Mizo.

Landscape: It encompasses a diverse range of landscapes, including dense tropical forests, grasslands, rivers, and hills.
Climate: It enjoys a sub-tropical climate as it falls within the sub-tropical zone.
Vegetation:

The vegetation may broadly be classified into tropical, temperate and alpine.
The lower reaches and foothills of the region are covered with tropical wet evergreen forests, while the alpine vegetation dominates the
higher altitudes.

Flora: Common species of trees in the reserve include Ammora wallichi (Amari), Gmelina arborea (Gamari), Terminalia chebula
(Hillika), Canarium resigniferum (Dhuna) and many others.
Fauna:

All four big cats, including the Snow Leopard, Clouded Leopard, Leopard and Tiger, are the common inhabitants.
Certain endangered species, such as Himalayan Palm Civets, Leopard Cat, Slow Lories and Hoolock Gibbon, can also be spotted
in the sanctuary.
Other species commonly found here are Giant and Flying Squirrels, Hornbills, Deer, Civets, Boar, Capped Langur, Stump Tailed
Macaque etc.

Que. 416 Climate Services Report is released by which of the following organisations?

1. United Nations Environment Program


2. World Meterological Organisation
3. International Union for Conservation of Nature
4. United Nations Framework Convention on Climate Change.

Correct Option - 2
The correct answer is option 2.
Climate Services Report:-

238/262
​ ess than 25 per cent of health ministries across the world utilise climate information and services to monitor health risks related
L
to climate sensitivity.
It highlights health protection as a priority in almost all countries and identifies the need for high-quality information for better
decision making.
While extreme heat causes the greatest mortality of all extreme weather, heat warning services are provided to health decision makers
in only half of the affected countries;
Climate change mitigation leading to reducing air pollution can save lives; and
Investments to improve the capabilities of the health sector to safeguard the most vulnerable are insufficient.

World Meterological Organisation:-

It is originated from the International Meteorological Organization (IMO), which was founded in 1873.
It was established in 1950.
It is the specialized agency of the United Nations for meteorology(weather and climate), operational hydrology and related
geophysical sciences.
Currently it has a membership of 187 countries.
Headquarters: Geneva, Switzerland.​
Recently, the World Meteorological Organization (WMO) released Climate Services report 2023. Hence, Statement 2 is correct.

Que. 417 The Global Cooling Watch report is released by which of the following?

1. World Meteorological Organization


2. UN Environment Programme
3. NewClimate Institute
4. International Energy Agency

Correct Option - 2
The correct answer is option 2.
Key Points
The Global Cooling Watch report

The Global Cooling Watch report, Keeping it Chill: How to meet cooling demands while cutting emissions, is released by the UN
Environment Programme led Cool Coalition. Hence statement 2 is correct.
The report is released in support of the Global Cooling Pledge, a joint initiative between the United Arab Emirates as host of
COP28 and the Cool Coalition.
It focuses on three key measures: passive cooling, higher-energy efficiency standards, and a faster phase-down of climate-
warming refrigerants.
It presents pathways to get to near-zero emissions in the key cooling sectors and provides a call to action for countries to pursue the
policies and strategies that have the greatest impact in reducing cooling-related emissions and advancing sustainable cooling for all.

Additional Information
Global Cooling Pledge

An initiative of the United Arab Emirates as host of the 2023 United Nations Climate Change Conference (COP28), the Pledge is one
of nine non-negotiated declarations, pledges, and charters that constitute key outcomes for the COP28 Presidential Action Agenda.
It aims to raise ambition and international cooperation through collective global targets to reduce cooling related emissions by 68%
from today by 2050, significantly increase access to sustainable cooling by 2030, and increase the global average efficiency of
new air conditioners by 50%.
India had declined to sign the pledge.

Que. 418 Consider the following statements about Global Green Credit Initiative (GGCI):
1. It is launched by UNEP.
2. It is platform for dialogue, collaboration and the exchange of innovative environmental programs and instruments.
3. The Green Credit Initiative serves to incentivize voluntary tree planting nationwide.
How many of the given statements are correct?

1. Only one
2. Only two
3. All three
4. None

Correct Option - 2
The correct answer is option 2.
Key Points
Global Green Credit Initiative (GGCI)

239/262
The Global Green Credit Initiative, launched by India’s Prime Minister at COP28. Hence statement 1 is incorrect.
It is platform for dialogue, collaboration and the exchange of innovative environmental programs and instruments. Hence statement 2
is correct.
The Green Credit Initiative serves to incentivize voluntary tree planting nationwide, issuing Green Credits and creating a land
inventory for Afforestation programs. Hence statement 3 is correct.

Additional Information
The Green Credit Initiative aims to register land on a web portal for plantation activities, open to various entities like individuals,
communities, and mining companies with degraded land.
This land bank is accessible for voluntary, CSR-driven, and Accredited Compensatory Afforestation plantations, encompassing various
land types such as degraded forests, watersheds, waste land, and more.

Green Credits

Green credits, assigned to specific actions, will be tradable commodities, allowing them to be sold on domestic market platforms.
It has been notified through Green Credit Rules, 2023 by the Ministry of Environment and Climate Change under the
Environment Protection Act, 1986.
The initiative operates as a market-based mechanism, encouraging individuals, communities, private sector industries, and companies
to participate in environmental activities.

Que. 419 Consider the following statements about National Landslide Susceptibility Map:-
1. It used ensemble machine learning methods to analyse the data.
2. It is accessible to the public, and anyone can engage with the data without requiring technical expertise.
3. It was prepared with the help of IIT Madras.
How many of the above statements is/are correct?

1. Only one
2. Only two
3. All three
4. None

Correct Option - 2
The correct answer is option 2.
Key Points
National Landslide Susceptibility Map:-

In the wake of severe monsoon triggered heavy floods and landslides, IIT Delhi team has developed India’s first national landslide
susceptibility map. Hence, Statement 3 is incorrect.
The map has been devised using 1.5 lakh known landslides event from sources like Geological Survey of India (GSI) along with 16
landslide conditioning factors such as soil cover, the number of trees covering the area, and how far it is from any roads or
mountains, etc.
The team used ensemble machine learning methods to analyse the data.Hence, Statement 1 is correct.

Note- Ensemble machine learning is when multiple machine learning models are used together to average out an oversize
impact from any one model.

High resolution mapping- The map offers a detailed (100 sqm.resolution) overview of landslide susceptibility across India, including
previously unrecognized areas.
Revealing new risk zones- It identified high landslide susceptibility in familiar areas like the Himalayan foothills, Assam-Meghalaya
region, and Western Ghats. Additionally, it uncovered new high-risk zones, including parts of the Eastern Ghats north of Andhra
Pradesh and Odisha.
Map available online- The map is accessible to the public, and anyone can engage with the data without requiring technical expertise.
Hence, Statement 2 is correct.

Additional Information
National Landslide Susceptability Mapping Programme:-
1) The Geological Survey of India initiated the Macro Scale (1:50,000) National Landslide Susceptibility Mapping (NLSM) program in
2014.
2) The goal is to map the 0.42 million sq. km landslide-prone areas across the country.
Aim and objectives:
a. To create a dynamic National Landslide Susceptibility Geodatabase for India.
b. To prepare GIS – based seamless Landslide Susceptibility Maps of India on 1:50,000 scale.
c. To prepare a nation-wide repository on GIS-based Landslide Inventory.

Que. 420 Consider the following statements about PRITHvi VIgyan Scheme:-

240/262
1. It is an overarching scheme of the Ministry of Earth Sciences (MoES) for the period from 2021 to 2026.
2. It addresses the five components of Earth System Sciences: atmosphere, hydrosphere, geosphere, cryosphere, and biosphere.
3. It provides advanced warning services for natural disasters.
Which of the above statements is/are correct?

1. 1 and 2 only
2. 2 and 3 only
3. 1 and 3 only.
4. 1, 2 and 3

Correct Option - 4
The correct answer is option 3.
Key Points
PRITHvi VIgyan (PRITHVI) Scheme:-

I​ t is an overarching scheme of the Ministry of Earth Sciences (MoES) for the period from 2021 to 2026.Hence, Statement 1 is
correct.
It encompasses five ongoing sub-schemes, namely:
ACROSS: Atmosphere and Climate Research-Modelling Observing Systems & Services.
O-SMART: Ocean Services, Modelling Application, Resources and Technology.
PACER: Polar Science and Cryosphere Research.
SAGE: Seismology and Geosciences
REACHOUT: Research, Education, Training and Outreach.
PRITHVI scheme comprehensively addresses the five components of Earth System Sciences: atmosphere, hydrosphere,
geosphere, cryosphere, and biosphere.Hence,Statement 2 is correct.

This holistic approach aims to enhance understanding and deliver reliable services for the country.
Objectives:

To augment and sustain long-term observations of the atmosphere, ocean, geosphere, cryosphere and solid earth to record the
vital signs of the Earth System and change
Development of modelling systems for understanding and predicting weather, ocean and climate hazards and understanding the
science of climate change.
Exploration of polar and high seas regions of the Earth towards discovery of new phenomena and resources;
Development of technology for exploration and sustainable harnessing of oceanic resources for societal applications.
Translation of knowledge and insights from Earth systems science into services for societal, environmental and economic benefits.

Benefits for India:

PRITHVI provides advanced warning services for natural disasters like cyclones, floods, heatwaves, and earthquakes,
facilitating prompt and effective disaster management.Hence, Statement 3 is correct.
Additionally, the scheme ensures precise weather forecasts for both land and oceans, enhancing safety and minimizing property
damages in adverse weather conditions.
PRITHVI extends its reach to explore the three poles of the Earth; Arctic, Antarctic, and Himalayas facilitating valuable insights
and knowledge about these regions.
The scheme encourages the development of technology for exploration and sustainable harnessing of oceanic resources, aligning
with modern advancements in Earth Science.

Que. 421 Kuril Islands recently in news lies between which two countries?

1. China and Japan


2. China and Taiwan
3. Japan and Korea
4. Japan and Russia.

Correct Option - 4
The correct answer is option 4.
Key Points
Kuril Islands:-

These are a set of four islands situated between the Sea of Okhotsk and the Pacific Ocean near the north of Japan's, Hokkaido.
Japan refers to them as Northern territories, Russia calls them the Kuril Islands and South Korea named them as Dokdo
islands.
These are part of the Pacific Ring of Fire belt and have over 100 volcanoes, of which 35 are said to be active volcanoes along with
hot springs.
Both Russia and Japan claim sovereignty over them though the islands have been under Russian control since the end of World
War II.

241/262
The Soviet Union had seized the islands at the end of World War II and by 1949 had expelled its Japanese residents.
Tokyo claims that the disputed islands have been part of Japan since the early 19th century.​

Que. 422 Greenhouse Gas Bulletin is released by which of the following?

1. Earthjustice
2. Greenpeace
3. World Meteorological Organization
4. United Nations Environment Programme

Correct Option - 3
The correct answer is option 3.
Key Points
Greenhouse Gas Bulletin

World Meteorological Organisation (WMO) released Greenhouse Gas Bulletin. Hence statement 3 is correct.

Additional Information
Greenhouse Gas Bulletin analyses data from the WMO Global Atmosphere Watch (GAW) Programme.
It shows globally averaged surface mole fractions for carbon dioxide (CO2), methane (CH4) and nitrous oxide (N2O) and compares
them with the mole fractions during the previous year and with ‌preindustrial levels.
It also provides insights into the change in radiative forcing by long-lived GHGs (LLGHGs) and the contribution of individual gases to
this increase.
Key Highlights
Carbon dioxide concentrations in 2022 were 50% above the pre-industrial era
Global average C02 concentrations reached 417.9 ppm in 2022, representing a 150% increase from pre-industrial levels.
Methane (CH4) and Nitrous Oxide (N20) concentrations increased by 264% and 124% respectively from pre-industrial levels in 2022.
From 1990 to 2022, radiative forcing by LLGHGs increased by 49%, with CO2 accounting for about 78% of this increase.

Radiative forcing

Radiative forcing is defined in the IPCC Sixth Assessment Report as follows: "The change in the net, downward minus upward,
radiative flux (expressed in W/m2) due to a change in an external driver of climate change, such as a change in the concentration of
carbon dioxide (CO2), the concentration of volcanic aerosols or the output of the Sun."

World Meteorological Organisation

WMO is the United Nations system's authoritative voice on the state and behaviour of the Earth's atmosphere, its interaction with the
land and oceans, the weather and climate it produces and the resulting distribution of water resources.
HQ- Geneva Switzerland
WMO provides the framework for such international cooperation for its 193 Member States and Territories. India is a member of
WMO.

Que. 423 'The Bletchley Declaration', recently seen in news is related to which of the following?

1. Global collaboration to tackle future Pandemic.


2. Combat desertification.
3. Removal of space debris
4. Tackle risks of advanced AI.

Correct Option - 4

242/262
The correct answer is option 4.
Key Points
Bletchley Declaration

The Bletchley Park Declaration is the first global pact on tackling frontier AI risks. Hence statement 4 is correct.
Britain, along with 28 other countries, including India and the European Union, published the “Bletchley Declaration” at Bletchley
Park, England..
This declaration aims to enhance global collaboration on artificial intelligence (AI) safety.
It has a two-fold focus: identifying shared AI-related risks and enhancing scientific understanding of these risks, as well as creating
cross-country policies to address them.
Frontier AI: Frontier AI refers to highly advanced generative AI models with potentially dangerous capabilities that can pose
significant risks to public safety.

Historical Importance of Bletchley Park

The site played a pivotal role in breaking the ‘unbreakable’ Enigma code, which was used by the Nazis, the site is also known for
creating the Turing Bombe, a device that expedited code-breaking efforts.
Bletchley Park contributed to the development of the Colossus, often considered the world’s first programmable electronic
computer.

Additional Information
Important Initiatives and Summits Related to Artificial Intelligence

Global Partnership on Artificial Intelligence (GPAI)


It is an alliance of 28 countries; the European Union adopted the ‘New Delhi Declaration’ of the GPAI.
India is the lead chair of GPAI in 2024
G7 AI Summit: The summit took place in Hiroshima, Japan, and concluded in December 2023.
marked the beginning of the Hiroshima AI Process (HAP), representing a major advancement in the regulation of AI.

Que. 424 Drone Country


1 MQ-9b Sky United States of America
2 TAI Aksungur Turkey
3 Drishti 10 Starliner India

How many of the above given pairs are correct?

1. Only one
2. Only two
3. All three
4. None

Correct Option - 3
The correct answer is option 3
Key Points
The MQ-9B

Country of origin - USA


Developed by General Atomics Aeronautical Systems (GA-ASI), primarily for the United States Air Force (USAF).
High-altitude, long-endurance drones armed with strike missiles that can take out enemy targets with high accuracy.
The MQ-9B has two variants — SkyGuardian and its sibling SeaGuardian.
The Indian Navy has been operating the MQ-9B Sea Guardian since 2020.

TAI Aksungur

Country of origin- Turkey


Belongs to the Medium Altitude Long Endurance (MALE) class of Unmanned Aerial Vehicles (UAVs).
Designed for a range of missions, including Intelligence, Surveillance, and Reconnaissance (ISR), ground attack, and maritime patrol.
Manufactured by Turkish Aerospace, this MALE UAV has dimensions of 11.6 m in length, a wingspan of 24 m, and a height of 3.1 m.
Can reach a maximum service ceiling of 40,000 feet, has an endurance of 50 hours, a datalink range exceeding 250 km, and a payload
capacity exceeding 750 kg, all while maintaining a maximum take off weight of 3,300 kg.

The Drishti 10 'Starliner'

It is India's inaugural domestically developed unmanned aerial vehicle (UAV), based on the Elbits Hermes 900 MALE (Medium
Altitude Long Range) platform.
Jointly manufactured by Elbit Systems Ltd and Adani Defence and Aerospace.
Drishti 10 Starliner represents a cutting-edge Intelligence, Surveillance, and Reconnaissance (ISR) platform.

243/262
It boasts an impressive 36-hour endurance, a payload capacity of 450 kg, and holds the distinction of being the sole all-weather
military platform with STANAG 4671 certification.
It boasts an impressive 36-hour endurance, a payload capacity of 450 kg, and holds the distinction of being the sole all-weather
military platform with STANAG 4671 certification.
It is authorized for flight in both segregated and unsegregated airspace.

Que. 425 Consider the following statements about Indian Space Association (ISpA) :
1. ISpA is an apex non-profit industry body, setup exclusively for the successful collaborative development of the Private Space
Industry in India.
2. Larsen & Toubro, Bharti Airtel and OneWeb are members of ISpA.
3.It will partner with the Union Government and other stakeholders in this segment to make India the leader in space services as well as self-
reliant for all future space missions.
How many of the above given statements are correct?

1. Only one
2. Only two
3. All three
4. None

Correct Option - 3
The correct answer is option 3
Key Points
Indian Space Association (ISpA

ISpA is an apex non-profit industry body, setup exclusively for the successful collaborative development of the Private Space
Industry in India. Hence statement 1 is correct.
ISpA was created to be the single Voice of the Private Space Industry and act as a bridge between the Government and the Private
Industry.
Indian Space Research Organisation, Nelco (It belongs to the Tata conglomerate), Larsen & Toubro, Bharti Airtel, OneWeb, Alpha
Design Technologies, MapmyIndia , Walchandnagar Industries, Godrej are some of its members. Hence statement 2 is correct.
It will partner with the Union Government and other stakeholders in this segment to make India the leader in space services as well as
self-reliant for all future space missions. Hence statement 3 is correct.


Additional Information
Given below are the aims and objectives of the Indian Space Association:

It aims to become the premier forum where private companies operating in this field can raise issues and discuss important policy
matters.
It will partner with the Union Government and other stakeholders in this segment to make India the leader in space services as well as
self-reliant for all future space missions.
The Indian Space Association shall strive to become the collective voice of India’s space industry.
Another objective of the Indian Space Association is to provide guidance and incubation services to emerging startups in this field.
It shall tie up with various educational institutions and shall conduct regular workshops to encourage future generations to take up a
career in commercial space exploration.

Que. 426 Consider the following statements regarding Operation Samudragupt :-


1. To stop the trafficking of heroin and other drugs in the Indian Ocean region.
2. It is a joint operation of Indian Navy and Narcotics Control Bureau.
Which of the above statements is/are true?

1. 1 only
2. 2 only
3. 1 and 2 both
4. Neither 1 nor 2

Correct Option - 3
The correct answer is option 3.
Key Points
Operation Samudragupt:-

The Director General of Narcotics Control Bureau(NCB) started Operation Samudragupt to stop the trafficking of heroin and other
drugs in the Indian Ocean region.Hence, Statement 1 is correct
244/262
Officers from the Operations Branch of NCB Headquarters participated in the operation,which was led by Sanjay Kumar Singh,
Deputy Director General (Ops), NCB.
The operation’s main goal was to gather actionable intelligence that could result in the
The Narcotics Control Bureau (NCB) and the Indian Navy, in a joint operation off the Kerala coast, have seized approximately
2,500 kg of methamphetamine valued at about 15,000 crore and originating from Pakistan.Hence, Statement 2 is correct.

Que. 427 Consider the following statements about Thermobaric bomb :


1. It is also known as an aerosol bomb or Fuel Air Explosive.
2. It uses oxygen in the surrounding environment to generate a high-temperature explosion.
3. It is a new generation bomb and has not been used so far in any war.
How many of the given statements are correct?

1. Only one
2. Only two
3. All three
4. None

Correct Option - 2
The correct answer is option 2
Key Points
Thermobaric or vacuum bomb

A thermobaric bomb (also called a vacuum or aerosol bomb - or fuel air explosive) consists of a fuel container with two separate
explosive charges. Hence statement 1 is correct.
This can be launched as a rocket or dropped as a bomb from aircraft. When it hits its target, the first explosive charge opens the
container and widely scatters fuel mixture as a cloud.
It uses the oxygen in the surrounding environment to generate a high-temperature explosion. Hence statement 2 is correct.
Thermobaric munitions can be traced back to World War Two, when they were initially used by the German army. They were not
widely developed until the 1960s, when the US used them in Vietnam. Hence statement 3 is incorrect.

Additional Information
There are no international laws specifically banning their use, but if a country uses them to target civilian populations in built-up
areas, schools or hospitals, then it could be convicted of a war crime under the Hague Conventions of 1899 and 1907.​

Que. 428 Consider the following statements about ASEAN-India Maritime Exercise (AIME):
1. AIME-2023 is the first time India has been involved in exercises with the ASEAN.
2. The ‘Harbour Phase’ was conducted at Changi Naval Base, while the ‘Sea Phase’ was conducted in the South China Sea.
3. Two major Indian Navy warships-- the INS Delhi and INS Satpura are part of the exercise.
How many of the given statements are correct?

1. Only one
2. Only 2
3. All three
4. None

Correct Option - 3
The correct answer is option 3
Key Points
ASEAN-India maritime exercise (AIME-2023)

245/262
AIME-2023 is the first time India has been involved in exercises with the ASEAN though there have been exercises with ASEAN
nations separately. Hence statement 1 is correct.
The ‘Harbour Phase’ was conducted at Changi Naval Base, while the ‘Sea Phase’ was conducted in the South China Sea. Hence
statement 2 is correct.
Two major Indian Navy warships-- the INS Delhi, an indigenously built destroyer, and INS Satpura, a Shivalik-class, recently
built modern stealth frigate-- are part of the exercise. Hence statement 3 is correct.
With AIME-2023, India becomes the 4th ASEAN dialogue partner, after Russia, China and the US, to hold the ASEAN+1 maritime
exercise.

Que. 429 Bab-al-Mandeb Strait, a strategic chokepoint connects which two regions of the world?

1. Red Sea with the Gulf of Aden and the Indian Ocean.
2. Mediterranean Sea and Atlantic Ocean.
3. Black Sea and Sea of Marmara.
4. Tasman Sea and Pacific Ocean

Correct Option - 1
The correct answer is option 1.
Key Points
Straits:-

A strait is a narrow, naturally formed waterway that connects two larger bodies of water, typically seas or oceans.
They can be formed by a variety of geological processes, such as tectonic activity, erosion, or the submersion of land.There are

various straits:-

Hence, option 1 is the correct answer.

Que. 430 Consider the following statements regarding Red Sea crisis:-
1. Houthi rebels belonging to Saudi Arabia are behind the attacks
2. USA has launched Operation Prosperity Guardian to safeguard the sea lanes.
3. The countries surrounding Red Sea region include Yemen, Oman and Iran
How many of the above statements is/are correct?

1. Only one
2. Only two
3. Only three
4. None

Correct Option - 1
The correct answer is option 1.
Key Points
Houthis: Roots, Movement, and Regional Alliances in Yemen’s Civil War

Houthis: The Houthis are a large clan belonging to the Zaidi Shia sect, with roots in Yemen’s northwestern Sa’dah province. They
are a militant group that has been fighting the civil war in Yemen for a decade. Hence, Statement 1 is incorrect.
Houthi Movement: It began in the 1990s against the dictatorship of Yemeni President Ali Abdullah Saleh. They now control
northern Yemen, and have a presence in most regions of the country.
Iran’s backing: The Houthis are Shia, and are backed with arms and finances by the Shia regime in Iran.
Support for Hamas: Though Hamas is a Sunni organisation, it is backed by Iran because of their mutual opposition to the US and
Israel. The Houthis’ support for Palestine and the Yemen conflict are a manifestation of existing regional rivalries. ​

246/262
Operation Prosperity Guardian: Multinational Response to Houthi Threats Lead the Red Sea Crisis

Reason for Genesis: In response to the Houthi attacks on ships in the Red Sea, the U.S. Secretary of Defense announced the
formation of Operation Prosperity Guardian in December 2023.Hence Statement 2 is correct.
About: It is a multinational security mission to ensure safe transit through the Red Sea region, under the aegis of the existing
Combined Maritime Forces’ Task Force 153.
Allies Participation: A united effort made up of 20 nations, with almost half preferring to remain unnamed.
The Netherlands, Australia, Canada, and Bahrain provided logistical and intelligence support.
While it is expected that the coalition will grow over time, the group lacks named participation from several key allies, including
Turkey, Germany, Egypt, South Korea and Japan.
India’s Decision: Countries like India, Italy and France have sent ships to the region on their own initiative, distancing themselves
from the U.S. umbrella

Red Sea Region:-.

Hence, Iran doesnot border Red Sea Region. Hence, Statement 3 is incorrect.

Que. 431 Consider the following statements about Suez Canal:-


1. It is an artificial sea-level waterway running north to south across the Isthmus of Suez in Egypt, to connect the Mediterranean
Sea and the Red Sea.
2. It is one of the world’s most heavily used shipping lanes, carrying over 12% of world trade by volume.
3. It is under the administrative control of Isreal.
How many of the above statements is/are correct?

1. Only one
2. Only two
3. All three
4. None

Correct Option - 2
The correct answer is option 2.
Key Points
Suez Canal:-

It is actually the first canal that directly links the Mediterranean Sea to the Red Sea. It was opened for navigation in November
1869.
The Suez Canal is an artificial sea-level waterway running north to south across the Isthmus of Suez in Egypt, to connect the
Mediterranean Sea and the Red Sea.Hence,Statement 1 is correct.
The canal separates the African continent from Asia.
It provides the shortest maritime route between Europe and the lands lying around the Indian and western Pacific oceans.
It is one of the world’s most heavily used shipping lanes, carrying over 12% of world trade by volume.Hence,Statement 2 is correct.
The canal is a major source of income for Egypt’s economy, with the African country earning USD 5.61 billion in revenues from it last
year.
In 2015, Egypt announced plans to further expand the Suez Canal, aiming to reduce waiting times and double the number of
ships that can use the canal daily by 2023.Hence,Statement 3 is incorrect.

247/262
Que. 432 Which of the following statements about Interpol is/are correct?
1. It was established in the year 1923.
2. India is one of the founding members.
3. Recently 91st Interpol General Assembly was held in Vienna.

1. 1 and 2 only
2. 1 and 3 only
3. 2 and 3 only
4. 1,2 and 3.

Correct Option - 2
The correct answer is option 2.
Key Points

248/262
India became a member of Interpol in 1956, which was formed in 1923.Hence,Statement 1 is correct and Statement 2 is incorrect.

Recently, the Indian delegation, led by the Central Bureau of Investigation (CBI) and National Investigation Agency (NIA), urged
member countries to deny safe havens to crime, criminals, and the proceeds of crime at the 91st Interpol General Assembly held in
Vienna, Austria. Hence Statement 3 is correct.

Que. 433 Consider the following statements


1. Small modular reactors (SMRs) are a new generation of nuclear reactors with a power capacity exceeding traditional plants.
2. SMRs are designed for inherent safety features and potentially lower proliferation risks compared to traditional reactors.
3. Due to their modular design and smaller size, SMRs are expected to be more expensive to construct and operate.
How many of the statements given above is/are correct?

1. Only one
2. Only two
3. All three
4. None

Correct Option - 1
The correct answer is option 1.
Key Points
Small Modular Reactors : Small modular reactors (SMRs) are a new generation of nuclear reactors that are designed to be smaller,
simpler, and more affordable than traditional nuclear power plants. SMRs are typically defined as having a power capacity of up to 300
megawatts (MW) per unit, which is about one-third of the generating capacity of traditional nuclear power reactors.

SMRs are designed to be smaller than traditional reactors, typically with a capacity of up to 300 MW compared to traditional plants
exceeding 1000 MW. Hence ,Statement 1 is incorrect.
SMRs have inherent safety features like natural convection cooling and potentially use proliferation-resistant fuel. Hence ,Statement 2
is correct.
SMRs are expected to be cheaper due to their modular construction and smaller size. Hence, Statement 3 is incorrect.

Potential advantages

Safety: SMRs are designed with inherent safety features that make them less likely to experience accidents. For example, some
SMRs use natural convection to circulate coolant, which means that they do not require pumps or other active systems to keep the
reactor cool.
Security: SMRs can be designed to be more resistant to proliferation than traditional nuclear power plants. For example, some SMRs
use fuel that is more difficult to reprocess into weapons-grade material.
Cost: SMRs are expected to be cheaper to build and operate than traditional nuclear power plants. This is due to their smaller
size, modular construction, and simpler design.

249/262
Flexibility: SMRs can be deployed in a variety of locations, including remote areas and near population centers. They can also be used
to supplement existing power grids or to provide power for new developments.

SMRs are still under development, but there are a number of companies and governments around the world that are working on SMR
designs. The first SMRs are expected to be deployed in the coming years.

Que. 434 Consider the following statements about facial recognition technology:
1. It offers improved security and convenience compared to traditional methods.
2. It can be susceptible to bias and lead to inaccurate identification of certain demographics.
3. Its widespread use raises concerns about potential privacy violations.
How many of the statements given above is/are correct?

1. Only one
2. Only two
3. All three
4. None

Correct Option - 3
The correct answer is option 3.
Key Points
Facial Recognition Technology : Facial recognition technology is a type of biometric security system that can identify or verify a person's
identity by analyzing their face.
How it works:
1. Facial Capture: An image or video is captured using a camera.
2. Facial Analysis: Software identifies and measures key facial features like the distance between your eyes, the shape of your jaw, and
the prominence of your cheekbones. This creates a unique "facial signature."
3. Matching: The facial signature is compared to a database of known faces to find a match and verify your identity.

Applications:
Security and Law Enforcement: Facial recognition is used to identify suspects in security footage, verify identities at border
crossings, and unlock secure areas.
Consumer Devices: Many smartphones and laptops use facial recognition for unlocking devices and secure transactions.
Social Media: Some social media platforms use facial recognition to suggest people you may know in photos.

Benefits:
Convenience: Facial recognition can be a faster and more convenient way to verify identity compared to traditional methods like
passwords or PINs.
Security: It can be a powerful tool for security and law enforcement.
Accuracy: Facial recognition technology is becoming increasingly accurate, especially in controlled environments.

Concerns:
Privacy: The widespread use of facial recognition raises privacy concerns, as it allows for constant tracking and monitoring of
people in public places.
Accuracy: Facial recognition technology can be less accurate in certain situations, such as with poor lighting or facial variations due
to age or expressions.
Bias: Facial recognition algorithms can be biased, leading to inaccurate identification, particularly for people of color.

Que. 435 Consider the following statements


1. Supercomputers achieve high performance through a single, powerful central processing unit (CPU).
2. Supercomputers are crucial for scientific simulations in diverse fields like climate change and protein folding.
3. Due to their high cost and specialized applications, supercomputer technology has no impact on advancements in personal computers.
How many of the statements given above is/are correct?

1. Only one
2. Only two
3. All three
4. None

Correct Option - 1
The correct answer is option 1.
Key Points
Supercomputers : Supercomputers are a class of computers designed for exceptionally high performance. They can handle massive
amounts of data and complex calculations at speeds far exceeding traditional computers. Their performance is measured in FLOPS
(Floating-point Operations Per Second), reaching quadrillions (thousands of trillions) of calculations per second.

250/262
Supercomputers rely on numerous processors working in parallel, not a single CPU. Hence ,Statement 1 is Incorrect.
Supercomputers play a vital role in complex scientific simulations. Hence ,Statement 2 is Correct.
Advancements in supercomputer technology eventually lead to improved processors used in personal computers. Hence ,Statement 3
is Incorrect.
How do they work?

Unlike your personal computer with a single CPU, supercomputers utilize numerous processors, often arranged in clusters or nodes.
These processors work together in parallel, dividing tasks and tackling them simultaneously for faster results.

Additionally, they have high-speed interconnects for efficient communication between processors and massive memory capacity to
store enormous datasets.

What are they used for?

Supercomputers power various scientific and technological advancements:


Scientific Simulations: Simulating complex phenomena like climate change, protein folding, or the early universe's formation.
Engineering Design: Optimizing designs for airplanes, cars, or spacecraft, considering factors like aerodynamics and stress analysis.
Medical Research: Analyzing genomic data, developing new drugs, and creating personalized medicine approaches.
Artificial Intelligence: Training complex AI models for tasks like image recognition, natural language processing, and autonomous
vehicles.
Weather Forecasting: Creating highly detailed weather models to predict atmospheric conditions with greater accuracy.

Que. 436 In the context of cryptocurrencies, what is the primary purpose of crypto mining?

1. To generate transaction fees for users


2. To validate transactions and secure the blockchain network
3. To track ownership of digital assets
4. To control the overall supply of a particular cryptocurrency

Correct Option - 2
The correct answer is option 2.
Key Points
Crypto Mining :Crypto mining is the process of validating transactions and adding new blocks to a blockchain, which is the public
ledger that underlies cryptocurrencies like Bitcoin. Miners use powerful computers to solve complex mathematical puzzles, and the first
miner to solve the puzzle gets to add the next block to the blockchain.
How crypto mining works:
Securing the Network: Miners compete to solve a complex math problem. The winner gets to add a new block of transactions to the
blockchain, which validates those transactions and keeps the network secure.
Earning Rewards: As a reward for their work, miners are awarded cryptocurrency. This is how new coins are created in some
cryptocurrencies.
Mining Difficulty: The difficulty of the math problems adjusts based on the computing power dedicated to the network. This ensures
that new blocks are added at a steady pace.
Cryptocurrency :

Cryptocurrency is a digital payment system that combines several key features:


Decentralized: Unlike traditional currencies controlled by governments or central banks, cryptocurrencies operate on a decentralized
network like blockchain. This means there's no single authority managing the system.
Secured by cryptography: Cryptocurrencies use cryptography, a complex system of codes, to secure transactions. This makes them
nearly impossible to counterfeit or double-spend.
Unit of exchange: They can be used to buy and sell goods or services, similar to traditional currencies. However, their acceptance as a
form of payment is still evolving.
Digital ledger: Transactions are recorded on a public ledger called a blockchain. This distributed ledger ensures transparency and
immutability, meaning records cannot be tampered with.
Limited supply: Many cryptocurrencies have a limited total supply programmed into their code. This, in theory, helps control inflation.

Que. 437 With reference to India's space programme, consider the following statements:
1. NavIC is a regional navigation system with an operational footprint covering the entire Indian subcontinent.
2. It utilizes a constellation of geostationary and inclined geosynchronous satellites for positioning services.
3. NavIC offers higher positioning accuracy compared to GPS in all weather conditions.
How many of the statements given above is/are correct?

1. Only one
2. Only two
3. All three
4. None
251/262
Correct Option - 1
The correct answer is option 1.
Key Points
NavlC (Navigation by Indian Constellation) : NavIC, or Navigation with Indian Constellation, is India's independent regional satellite
navigation system developed by the Indian Space Research Organisation (ISRO). It is designed to provide accurate real-time positioning
and timing services for users in India and the surrounding region, extending up to 1,500 kilometers (930 mi) from its borders.

NavIC's primary coverage area is India and extends 1500 km around it. Hence ,Statement 1 is incorrect.
NavIC utilizes a combination of geostationary and inclined geosynchronous satellites. Hence ,Statement 2 is correct.
While NavIC offers good accuracy in India, it might not always outperform GPS globally. Hence ,Statement 3 is incorrect.

NavIC offers two types of services:


Standard Positioning Service (SPS): This is a free service for civilian use, providing positioning accuracy of up to 20 meters.
Restricted Service (RS): This is an encrypted service for authorized users, including the Indian military and government agencies,
offering higher accuracy and additional features.
Satellites:
Currently consists of 8 satellites, with plans to expand to 12.
Three satellites are in geostationary orbit.
Five satellites are in inclined geosynchronous orbit.

Here are some of the benefits of NavIC:


Reduced Reliance on Foreign Systems: NavIC provides India with an alternative to foreign satellite navigation systems like GPS, giving
the country more control over its navigation infrastructure.
Improved Accuracy: NavIC uses two frequency bands, which can help to improve accuracy in areas with signal interference.
Disaster Management: NavIC can be used for disaster management applications, such as tracking the movement of cyclones and floods.
Vehicle Tracking: NavIC can be used for vehicle tracking applications, which can help to improve logistics and fleet management.

Que. 438 A recent discovery by the James Webb Space Telescope has revealed the presence of a system of dust and debris around a young
star. This finding is significant because it represents the:

1. First confirmation of a rocky planet outside our solar system


2. First detection of an active black hole in a nearby galaxy
3. First evidence of an asteroid belt beyond our solar system
4. First image of a star forming nebula

Correct Option - 3
The correct answer is option 3.
Key Points
Asteroid Belt : The asteroid belt is a vast ring of rocky debris located between the orbits of Mars and Jupiter. It's leftovers from the
solar system's formation about 4.6 billion years ago.

There aren't any new asteroid belts found within our solar system, but thanks to the James Webb Space Telescope, we were able to
discover the first ever asteroid belt outside our solar system in May 2023! This incredible discovery is located around a young star
called Fomalhaut, which is 25 light-years away.
The alien asteroid belt consists of three distinct rings of dust and debris circling the star at a distance 150 times greater than that
between Earth and the Sun. This is significantly farther out than our own asteroid belt, which resides between Mars and Jupiter.
Scientists believe that the presence of these rings may indicate the existence of hidden planets within the system. The gravitational
forces exerted by such planets would help shape and constrain the dusty asteroid belts.
This discovery is a major breakthrough in our understanding of planetary system formation outside our own solar system!

Que. 439 The phenomenon of dark, radial streaks appearing across Saturn's rings during specific times is known as:

1. Ring Shadow
2. Cassini Division Gap
3. Spoke Season
4. Equatorial Ridge

Correct Option - 3
The correct answer is option 3.
Key Points
Saturn's majestic rings :Saturn's majestic rings are a constant source of wonder, but they also hold some perplexing secrets. One such
mystery is the phenomenon known as "spoke season." During this time, dark, radial streaks, like spokes on a wheel, appear across the
252/262
rings, seemingly defying explanation.

Saturn's ring spokes:


Appearance: These fleeting shadows appear dark against the backdrop of the brighter, icy particles that dominate the rings. They
can be short-lived, lasting from a few hours to a few days.
Seasonality: Spoke season coincides with Saturn's equinoxes, which occur roughly every seven Earth years. During equinoxes, the tilt
of Saturn's axis causes the rings to be more edge-on towards the Sun.
The Cause? A Scientific Debate: Scientists are still piecing together the exact mechanism behind spoke formation. A leading theory
involves Saturn's magnetic field interacting with the solar wind, a stream of charged particles emanating from the Sun.
Here's how it might work: The interaction could electrify the tiniest icy particles within the rings, causing them to clump together
temporarily. These denser clumps then cast shadows on the surrounding ring material, creating the appearance of dark streaks.
Unanswered Questions: While the solar wind interaction is a promising theory, there are still gaps in our understanding.
For instance, why do the spokes only appear during equinoxes?
What specific properties of the icy particles allow them to be affected by the electrical forces?
Future Observations: Ongoing observations with telescopes like Hubble and the recently launched James Webb Space Telescope,
combined with data from the Cassini spacecraft mission, are expected to shed more light on this intriguing phenomenon.
Features :
Ring shadow refers to the shadow cast by Saturn itself onto the rings, not dark streaks appearing on them.
Cassini Division Gap is a specific permanent gap between two of Saturn's rings.
Equatorial Ridge is a permanent and prominent feature on Saturn, not a fleeting phenomenon.

Que. 440 In the context of promoting technological advancements, which state launched a framework specifically focused on Robotics?

1. Gujarat
2. Tamil Nadu
3. Karnataka
4. Telangana

Correct Option - 4
The correct answer is option 4.
Key Points
Telangana's Robotics Policy Framework :
Telangana's Robotics Policy Framework, launched in 2023, is a pioneering initiative aimed at establishing the state as a leader in the field
of robotics in India. Here's a breakdown of its key aspects:
Vision:
To create a robust robotics ecosystem that fosters innovation, entrepreneurship, and research & development in robotics.
Pillars of the Framework:

Infrastructure Access: Establishing a "Robo Park" with facilities like testing grounds, co-working spaces, and manufacturing units
at competitive rates. This could be on government land or in partnership with industry, academia, and incubators.
Business Enablement: Streamlining regulations and providing financial incentives to attract businesses and startups in the robotics
sector.
Fostering Research & Innovation: Setting up a world-class robotics accelerator to provide startups with incubation support,
infrastructure, guidance on obtaining necessary approvals, market insights, connections to investors, and mentorship.
Human Capital Enhancement: Encouraging skilling and training initiatives to develop a workforce equipped with the necessary
expertise in robotics.
Responsible Deployment: Ensuring the ethical and responsible development and deployment of robotics technology.
Implementation Body:
Telangana Robotics Innovation Center (TRIC) will be the nodal agency responsible for overseeing the framework's implementation.

Que. 441 Consider the following statements


1. Open RAN deployment by QUAD nations reduces reliance on a single vendor, mitigating security risks.
2. Open RAN architecture inherently creates a larger attack surface compared to traditional RAN.
3. The QUAD report downplays security concerns surrounding Open RAN deployments.
How many of the statements given above is/are correct?

1. Only one
2. Only two
3. All three
4. None

Correct Option - 2
The correct answer is option 2.
Key Points

253/262
QUAD's Open Radio Access Network (RAN) Security : The security of Open Radio Access Networks (Open RAN) deployed by QUAD
(Quadrilateral Security Dialogue) has been a topic of discussion. Open RAN promotes diversification by allowing components from
various suppliers. This can potentially mitigate security risks associated with trusting a single vendor for the entire network.
Open RAN allows for diversification of vendors, reducing reliance on a single supplier and potentially mitigating security risks associated
with a monoculture. Hence ,Statement 1 is correct.
Disaggregation of functionalities in Open RAN can introduce more potential vulnerabilities compared to a traditional RAN where
everything is managed by a single vendor. Hence, Statement 2 is correct.
The QUAD report acknowledges security concerns surrounding Open RAN deployments but emphasizes that these can be addressed with
strong security practices. It does not downplay the concerns. Hence ,Statement 3 is incorrect.
Identified Security Concerns:
Incomplete Security Specifications: A QUAD working group identified gaps in the current Open RAN security specifications. These
gaps relate to how security best practices are chosen and implemented.
Increased Attack Surface: Disaggregation of functionalities in Open RAN might introduce more vulnerabilities compared to
traditional RAN setups where a single vendor manages everything.

Ongoing Efforts:
The QUAD report acknowledges these concerns but highlights that many of the risks are also present in traditional RAN deployments.
The report emphasizes that these risks can be addressed through adopting strong security best practices.
The onus lies on industry groups like the O-RAN Alliance to develop and promote clear security guidelines for Open RAN.

Overall, securing Open RAN requires collaboration between governments, industry, and network operators. The QUAD report serves as a
springboard for further development and adoption of robust security practices for Open RAN deployments

Que. 442 Consider the following statements


1. The World Health Organization's (WHO) "Health for All" Report, released in May 2023, emphasizes the need for economic
reforms to prioritize healthcare access.
2. The report highlights the COVID-19 pandemic as a reason to refocus on preventative healthcare measures.
Which of the following is/are correct ?

1. 1 only
2. 2 only
3. Both 1 and 2
4. Neither 1 nor 2.

Correct Option - 3
The correct answer is option 3.
Key Points
Health for All Report :"Health for All: Transforming economies to deliver what matters", published by the World Health Organization
(WHO) in May 2023.

Summary of the report's key points:


Main Goal: This report focuses on achieving "Health for All" by transforming economies to prioritize health and well-being.
Background: The COVID-19 pandemic highlighted the need for a new approach to health, emphasizing preventative measures and
accessible healthcare for all.
Key Recommendations:
Rethink Economic Value: Redefine how economies value health and well-being, considering them as long-term investments.
Revamp Financing Systems: Advocate for increased and efficient health financing, with a focus on universal health coverage.
Foster Innovation for Good: Encourage collaborative innovation focused on solving global health challenges and ensuring
equitable access to advancements.
Strengthen Government Capacity: Invest in effective and well-resourced governments to implement health policies effectively.
Significance: This report offers a roadmap for countries to transform their economies and prioritize health outcomes for all citizens,
leading to a more sustainable and equitable future.

Que. 443 Which of the following statements about Respiratory Syncytial Virus (RSV) is/are correct?

1. It primarily affects children above 6 years old.


2. There is a vaccine readily available to prevent RSV infection.
3. Infants and older adults are considered high-risk groups for RSV complications.
4. Frequent handwashing is an ineffective method of preventing RSV infection.

Correct Option - 3
The correct answer is option 3.
Key Points
254/262
Respiratory Syncytial Virus (RSV) : Respiratory syncytial virus (RSV) is a common respiratory virus that typically causes mild, cold-like
symptoms. Most people recover within a week or two, but RSV can be serious for infants and older adults.

Who is most at risk of RSV complications?


Infants younger than 6 months old
Older adults, especially those 65 and older
People with chronic heart or lung conditions
People with weakened immune systems

RSV symptoms can vary depending on the person's age and overall health. In infants, RSV symptoms may include:
Runny or stuffy nose
Cough
Wheezing
Fever
Decreased appetite
Listlessness

In older adults, RSV symptoms may be similar to those of a cold or flu and may include:
Runny or stuffy nose
Cough
Congestion
Fever
Muscle aches
Fatigue

How to prevent RSV infection

There is no vaccine to prevent RSV infection, but there are steps you can take to help prevent the spread of RSV, including:
Frequent handwashing
Avoiding close contact with sick people
Covering your cough or sneeze with a tissue
Disinfecting surfaces that may be contaminated with RSV

Que. 444 Consider the following statements


1. The International Pathogen Surveillance Network (IPSN) is a World Health Organization (WHO) initiative launched in 2023 to
focus on pathogen genomics for infectious disease prevention.
2. The IPSN relies on data from pathogen sequencing to understand how diseases emerge and spread.
3. The primary function of the IPSN is to develop new vaccines and treatments for existing diseases.
How many of the statements given above is/are correct ?

1. Only one
2. Only two
3. All three
4. None

Correct Option - 2
The correct answer is option 2.
Key Points
International Pathogen Surveillance Network (IPSN) : The International Pathogen Surveillance Network (IPSN) is a new global
initiative launched by the World Health Organization (WHO) in May 2023. Its primary focus is on preventing and detecting infectious
disease threats before they evolve into widespread outbreaks or pandemics.

The IPSN is indeed a WHO initiative focused on pathogen genomics for infectious disease prevention. Hence, Statement 1 is correct
The network utilizes data from pathogen sequencing, analyzing the genetic makeup of pathogens, to understand disease emergence and
spread. Hence, Statement 2 is correct.
While the IPSN's work can contribute to future vaccine and treatment development, its primary function is focused on early detection and
prevention of outbreaks, not directly developing treatments. Hence, Statement 3 is incorrect.

Goal:
To create a globally connected network for pathogen genomics, a field that analyzes the genetic makeup of disease-causing organisms.

Function:
The IPSN leverages pathogen genomics to:
Understand how diseases spread and evolve.
Identify and track pathogens for better outbreak prevention and response.

255/262
Develop more effective treatments and vaccines.

Structure:
The network operates through three main components:
Communities of Practice (COPs): These groups bring together various public health stakeholders to collaborate on advancing
pathogen genomics and addressing challenges in disease surveillance.
Country Scale-Up Accelerator (CSUA): This arm focuses on enabling and empowering countries to build their capacity for
pathogen genomics and disease surveillance.
Funder's Forum: This group coordinates financial resources from donors to support the IPSN's activities.

Significance:
The IPSN plays a vital role in strengthening global public health preparedness. By promoting collaboration and resource sharing in
pathogen genomics, it aims to create a world where all countries have equitable access to tools for early detection and response to
infectious disease threats.

Here are some additional points to consider:


The IPSN relies on data collected through pathogen sequencing, which involves analyzing the genetic material of pathogens.
This information is crucial for understanding the emergence and spread of new diseases.
The network is particularly relevant in the wake of the COVID-19 pandemic, highlighting the importance of global cooperation in tackling
infectious disease threats.

Que. 445 Molybdenum-99 (Mo-99) used in medical imaging is primarily produced through:

1. Natural mining processes


2. Fission of enriched uranium in nuclear reactors
3. Chemical separation of naturally occurring Molybdenum-98
4. Activation of natural Molybdenum with sunlight

Correct Option - 2
The correct answer is option 2.
Key Points
Molybdenum-99 (Mo-99) : Molybdenum-99 (Mo-99) produced through fission is the primary method for obtaining this isotope used in
medical imaging.
Fission Process:
Molybdenum-99 is not naturally abundant. It's artificially produced in nuclear reactors through the fission of uranium-235 (U-235).
When U-235 absorbs a neutron, it splits into two smaller nuclei (fission fragments) along with the release of neutrons and energy.
Mo-99 is one of the many fission products created during this process, though in small quantities.

Target Irradiation:
Aluminum targets containing enriched U-235 are placed inside the reactor core.
These targets are bombarded with neutrons for several days (typically 100-200 hours).
The fission process within the U-235 isotopes creates Mo-99 along with other radioactive isotopes.

Chemical Separation:
After irradiation, the targets are removed from the reactor and transported to hot cells for processing.
Chemical separation techniques are used to isolate Mo-99 from the other fission products and target material.

Benefits of Fission Mo-99:


High Specific Activity: Fission-produced Mo-99 has a high specific activity, meaning a larger amount of radioactivity is concentrated in a
smaller amount of material. This is crucial for its use in medical applications.
Established Production Method: Fission Mo-99 production is a well-established and reliable method with a long history.

Challenges of Fission Mo-99:


Reliance on Enriched Uranium: Fission Mo-99 production necessitates enriched uranium, raising concerns about nuclear proliferation.
Short Half-Life: Mo-99 has a short half-life of about 66 hours. This necessitates frequent production and rapid transportation to ensure its
availability for medical procedures.

Que. 446 Consider the following statements


1. Radiometric dating relies on the predictable rate of decay of radioactive isotopes to estimate the age of objects.
2. Carbon-14 dating is a suitable method for determining the age of dinosaur fossils.
Which of the following is/are correct ?

1. 1 only
2. 2 only

256/262
3. Both 1 and 2
4. Neither 1 nor 2.

Correct Option - 1
The correct answer is option 1.
Key Points
Radiometric dating : Radiometric dating is a scientific technique used to determine the age of rocks, minerals, and even some
biological materials. It relies on the natural process of radioactive decay, where an unstable isotope (a specific form of an element with a
particular number of neutrons) of an element spontaneously breaks down into a more stable form, releasing energy and often a subatomic
particle in the process.
Radiometric dating measures the ratio of a parent radioactive isotope to its stable daughter product, considering the parent's half-life, to
determine the age of a material. Hence ,Statement 1 is correct.
Carbon-14 dating has a limited range of about 50,000 years. Dinosaur fossils are typically much older, making this method unsuitable for
dating them. Hence ,Statement 2 is incorrect.
Key principles of radiometric dating:
Radioactive Decay: Certain isotopes have a characteristic rate of decay, known as their half-life. This is the time it takes for half of the
original radioactive isotope to decay into its daughter product. Half-lives can range from fractions of a second to billions of years.
Isotopic Ratios: When a rock or mineral forms, it incorporates certain radioactive isotopes along with their stable daughter products in a
specific ratio. Over time, the radioactive isotope decays at a predictable rate, while the daughter product accumulates.
Dating Process: By measuring the current ratio of the parent (radioactive) isotope to the daughter product (stable isotope) in a sample,
and knowing the half-life of the parent isotope, scientists can calculate the age of the material.

Applications of Radiometric Dating:


Geologic Timescale: Radiometric dating has been instrumental in establishing the geologic timescale, providing a chronology of
Earth's history.
Archaeological Dating: This technique helps determine the age of ancient artifacts, providing valuable insights into human history.
Meteorite Analysis: By dating meteorites, scientists can gain clues about the formation of our solar system.

Common Radiometric Dating Techniques:


Potassium-Argon Dating: This is a widely used method for dating igneous rocks and minerals. Potassium-40 (parent) decays into
Argon-40 (daughter) with a half-life of 1.25 billion years.
Carbon-14 Dating: This technique is suitable for dating organic materials up to about 50,000 years old. Carbon-14 (parent) decays
into Nitrogen-14 (daughter) with a half-life of about 5,700 years.
Uranium-Lead Dating: This method is useful for dating very old rocks and minerals. It involves multiple decay chains within the
Uranium-238 and Uranium-235 radioactive decay series.

Limitations of Radiometric Dating:


Accuracy: The accuracy of radiometric dating depends on several factors, including the initial ratio of isotopes in the sample and the
possibility of contamination.
Applicability: This technique is most effective for materials containing suitable radioactive isotopes.

Que. 447 Consider the following statements


1. Gravitational waves are distortions in spacetime caused by the movement of massive objects, similar to ripples in water.
2. The first confirmed detection of gravitational waves came from the collision of two galaxies.
Which of the following statements is/are correct ?

1. 1 only
2. 2 only
3. Both 1 and 2
4. Neither 1 nor 2

Correct Option - 1
The correct answer is option 1.
Key Points
Gravitational Waves : Gravitational waves are ripples in the fabric of spacetime, caused by the acceleration of massive objects. Imagine
dropping a pebble in a pond and observing the waves traveling outward. In spacetime, instead of water, these waves are distortions caused
by energetic cosmic events.
Gravitational waves:
Predicted by Einstein: The existence of gravitational waves was first theorized by Albert Einstein in his theory of general relativity
in 1916. However, due to their incredibly weak nature, it took over a century to directly detect them.
How They Form: Gravitational waves are generated by violent cosmic phenomena like:
Merging black holes or neutron stars: As these incredibly dense objects spiral inwards and collide, they release enormous amounts
of energy in the form of gravitational waves.
Rapidly spinning neutron stars: An unevenly spinning neutron star can also distort spacetime enough to emit gravitational waves.
Direct Detection: The first confirmed detection of gravitational waves occurred in 2015 by the Laser Interferometer Gravitational-Wave
Observatory (LIGO) in the United States. LIGO uses incredibly sensitive instruments to detect the minute distortions in spacetime caused
by passing gravitational waves.

257/262
Significance: The study of gravitational waves opens a new window into the universe. It allows us to observe events that wouldn't be
visible with traditional telescopes, such as black hole mergers. This provides valuable insights into the nature of gravity, black holes,
and the violent processes occurring in the cosmos.

Que. 448 Consider the following statements


1. Fixed-dose combination (FDC) drugs combine several active ingredients into a single pill, improving patient adherence.
2. FDCs are always the most cost-effective option for treatment compared to individual medications.
3. FDCs offer reduced flexibility for dosage adjustments as the ingredients are present in a fixed ratio.
How many of the statements given above is/are correct?

1. Only one
2. Only two
3. All three
4. None

Correct Option - 2
The correct answer is option 2.
Key Points
Fixed-dose combination (FDC) drugs : Fixed-dose combination (FDC) drugs are a type of medication that combines two or more active
ingredients into a single pill, tablet, or capsule. The ingredients are present in a fixed ratio and are meant to be taken together.

FDC drugs do combine multiple ingredients into a single pill, making it easier for patients to take their medication as prescribed, thus
improving adherence. Hence, Statement 1 is correct.
While FDCs can be cost-effective in some cases, it's not always true. The price depends on various factors, and individual medications
might be cheaper sometimes. Hence ,Statement 2 is incorrect.
A fixed dose of each ingredient limits the flexibility of adjusting the dosage for individual needs. Hence ,Statement 3 is correct.

FDCs are commonly used to treat a variety of conditions, including:


High blood pressure
High cholesterol
HIV/AIDS
Diabetes
Mental health conditions

There are a number of potential benefits to taking FDC drugs. These include:
Improved adherence: By taking fewer pills, patients are more likely to stick with their treatment plan.
Convenience: FDCs can be easier to take than multiple medications, especially for people who have difficulty swallowing pills.
Reduced cost: In some cases, FDCs may be less expensive than taking multiple individual medications.

However, there are also some potential drawbacks to FDCs. These include:
Lack of flexibility: Because the ingredients are in a fixed ratio, FDCs may not be appropriate for all patients. For example, a patient may
need a higher dose of one medication than is available in the FDC.
Increased risk of side effects: Taking multiple medications together can increase the risk of side effects.
Potential for drug interactions: The ingredients in an FDC can interact with each other or with other medications that a patient is taking.

Que. 449 Which of the following is a disadvantage associated with phage therapy?

1. Improved patient adherence due to targeted treatment


2. Reduced risk of developing antibiotic resistance in bacteria
3. Difficulty in isolating and identifying specific phages for treatment
4. Broad-spectrum action, eliminating both harmful and beneficial bacteria

Correct Option - 3
The correct answer is option 3.
Key Points
Phage therapy : Phage therapy is a type of treatment that uses bacteriophages, or viruses that infect bacteria, to treat bacterial
infections. Bacteriophages (phages for short) are naturally occurring viruses that specifically target and kill bacteria. They are found in soil,
water, and even in the human gut.
Phage therapy has been around for over a century, but it was largely overshadowed by the development of antibiotics in the mid-20th
century. However, with the rise of antibiotic-resistant bacteria, phage therapy is gaining renewed interest as a potential alternative treatment.

Here's how phage therapy works:


Phages are isolated from the environment or from a patient's own bacteria.
The phages are then identified and matched to the specific type of bacteria causing the infection.
The phages are applied directly to the infected area or taken orally or intravenously.
Once they reach the bacteria, the phages inject their genetic material into the bacteria and replicate.
This replication process kills the bacteria.

258/262
Phage therapy has several potential advantages over antibiotics:
Specificity: Phages only target specific bacteria, leaving healthy bacteria unharmed. This can help to reduce the side effects of
treatment.
Reduced risk of resistance: Bacteria are less likely to develop resistance to phages compared to antibiotics. This is because phages
can evolve alongside their bacterial hosts, and there are many different types of phages that can target a single type of bacteria.
Ability to treat biofilms: Phages can penetrate biofilms, which are communities of bacteria that can be difficult to treat with antibiotics.

Phage therapy is still in its early stages of development, but there is growing evidence that it can be an effective treatment for a variety of
bacterial infections. More research is needed to determine the safety and efficacy of phage therapy for different types of infections.

Here are some of the challenges associated with phage therapy:


Regulatory hurdles: Phage therapy is not yet widely approved for use in many countries, including the United States.
Difficulty in isolating and identifying phages: There are millions of different types of phages, and it can be difficult to find the right
phage to treat a specific infection.
Limited research: More research is needed to determine the safety and efficacy of phage therapy for different types of infections.

Que. 450 Phonons are most analogous to which of the following phenomena?

1. Electrons in an atom
2. Light waves traveling through a vacuum
3. Quantized packets of sound waves in solids
4. Movement of planets in their orbits

Correct Option - 3
The correct answer is option 3.
Key Points
Phonons : Phonons are quanta (discrete packets) of vibrational energy in a crystal lattice. Imagine the atoms in a solid arranged in a
regular pattern like a 3D grid. These atoms aren't completely still; they vibrate due to thermal energy. Phonons describe these vibrations in a
quantized way, meaning the vibrational energy comes in specific, indivisible units.

How are Phonons Related to Sound Waves?


In essence, phonons are the quantized version of sound waves in solids. Sound waves are vibrations that travel through a medium (like
air or water). In solids, these vibrations are quantized as phonons.
The frequency and wavelength of a phonon determine its energy and momentum. Higher frequency phonons correspond to higher
energy vibrations.

Types of Phonons:
There are two main types of phonons:
Acoustic phonons: These involve vibrations of the atoms where the entire lattice expands and contracts. They are responsible for
sound propagation in solids.
Optical phonons: These involve vibrations where the atoms move against each other. They interact with light and influence a
material's optical properties.

Applications of Phonons:
Phonons play a crucial role in many physical phenomena in solids, including:
Thermal conductivity: Phonons are the primary carriers of heat in many insulating materials.
Electrical conductivity: In some materials, interactions between phonons and electrons can affect electrical conductivity.
Lasers: Certain types of lasers rely on the interaction of light with phonons to achieve specific properties.

Que. 451 Consider the following statements


1. Project Taara utilizes radio frequencies for data transmission to deliver high-speed internet in remote areas.
2. Project Taara aims to bridge the digital divide by providing internet connectivity in geographically isolated regions.
Which of the following statements is/are correct?

1. 1 only
2. 2 only
3. Both 1 and 2
4. Neither 1 nor 2.

Correct Option - 2
The correct answer is option 2.
Key Points
Project Taara : Project Taara is a moonshot initiative from Alphabet's X lab that aims to deliver high-speed internet access to remote and
underserved areas using lasers.
Technology:

259/262
Project Taara utilizes wireless optical communication (WOC) technology. It transmits data using narrow, invisible beams of light
instead of traditional radio frequencies.

Benefits:
Fast Speeds: Taara offers high-speed internet connectivity comparable to fiber optic cables, reaching speeds of up to 20 gigabits per
second.
Remote Areas: This technology is particularly beneficial for reaching areas where laying fiber optic cables is impractical or expensive
due to geographical constraints or low population density.
Scalability: Taara systems are relatively easy to deploy and can be rapidly scaled up or down as needed.

How it Works:
Project Taara utilizes pairs of terminals equipped with lasers and high-precision pointing and tracking systems.
These terminals are installed on rooftops or towers within a line of sight of each other.
The laser beam transmits data between the terminals, bridging the connectivity gap in remote areas.

Current Status:
Project Taara is still under development but has progressed beyond pilot stages.
Alphabet's subsidiary, Google, has partnered with Bharti Airtel, a leading Indian telecom company, for a large-scale deployment of Taara
in India.
This deployment aims to bridge the digital divide in rural parts of India by providing high-speed internet access.

Que. 452 The Hiroshima AI Process, launched by the G7 nations, aims to:

1. Develop and deploy autonomous weapons systems.


2. Establish global norms for safe and ethical Artificial Intelligence (AI).
3. Promote unrestricted competition in AI research and development.
4. Limit access to advanced AI technologies for developing countries.

Correct Option - 2
The correct answer is option 2.
Key Points
Hiroshima AI Process : The Hiroshima AI Process is a significant international initiative launched by the G7 nations in May 2023
under Japan's presidency. The primary goal of the Hiroshima AI Process is to promote the development and use of safe, secure, and
trustworthy Artificial Intelligence (AI) technologies worldwide.

Focus Areas:
The process focuses on establishing international cooperation and guidelines for:
Identifying and mitigating risks associated with advanced AI systems.
Fostering responsible development and deployment of trustworthy AI.
Upholding democratic values like fairness, accountability, and transparency in AI development and use.

Key Outcomes:
The primary outcome of the Hiroshima AI Process is the Hiroshima AI Process Comprehensive Policy Framework. This framework
consists of four pillars:
Analysis of priority risks, challenges, and opportunities: This involves identifying potential risks and benefits associated with
advanced AI.
International Guiding Principles for all AI actors: These principles establish a set of best practices for all stakeholders involved in
AI development and use.
International Code of Conduct for Organizations Developing Advanced AI Systems: This code outlines specific guidelines and
ethical considerations for organizations creating advanced AI systems.
Project-based cooperation: This fosters collaboration on projects that promote responsible and beneficial AI development.

Significance:
The Hiroshima AI Process represents a critical first step towards establishing a global framework for governing AI. It brings together
governments, industry leaders, academics, and civil society to address the challenges and opportunities presented by rapidly evolving AI
technologies.

Que. 453 Consider the following statements


1. The existing framework for submarine cable landings in India is handled solely by the Department of Telecommunications
(DoT).
2. TRAI's proposed framework introduces a categorization of Cable Landing Stations (CLS) into "Main CLS" and "CLS Point of Presence".
3. The new framework prioritizes economic benefits over security considerations for submarine cable infrastructure.
How many of the statements given above is/are correct?

1. Only one
2. Only two
3. All three

260/262
4. None

Correct Option - 1
The correct answer is option 1.
Key Points
Submarine Communications Cable :

It is a cable laid on the seabed between land-based stations to transmit telecommunication signals across stretches of ocean and
sea. Modern submarine cables use fiber-optic technology. The optical fibre elements are typically coated with plastic layers
and contained in a protective tube suitable for the environment where the cable will be deployed.

Previously, DoT issued ILD/ISP licenses that included permission for CLS setup. However, TRAI proposes a separate framework. Hence
,Statement 1 is incorrect.
The proposed framework categorizes CLS for streamlined approvals. Hence, Statement 2 is correct.
The framework emphasizes security measures and a legal framework. Hence, Statement 3 is incorrect.

Licensing Framework and Regulatory Mechanism for Submarine Cable Landing in India

The current system for submarine cable landings in India is undergoing changes, with the Telecom Regulatory Authority of India (TRAI)
proposing a new framework. Here's a breakdown of the key points:

Background:
Traditionally, the Department of Telecommunications (DoT) granted International Long Distance/Internet Service Provider
(ILD/ISP) licenses, which included permission to set up Cable Landing Stations (CLS) for submarine cables.
However, this system lacked clarity and faced challenges in addressing security concerns and streamlining the approval process.

TRAI's Recommendations:
In December 2022, TRAI issued a consultation paper proposing a new framework for submarine cable landings.
The key recommendations include:
Categorization of CLS: Introducing two categories - "Main CLS facility" and "CLS Point of Presence (PoP)".
Main CLS would handle core landing and interconnection functions, requiring a separate license.
PoP would be a distribution point within India, requiring simpler authorization.
Security Measures: Emphasizing robust security measures at CLS facilities to safeguard the integrity of cables and data flow.
Legal Framework: Recommending amendments to include definitions for "submarine cable" and "Cable Landing Station" in the
Indian Telecommunication Bill, 2022. This would provide a stronger legal foundation for the regulations.
Licensing Process: Streamlining the licensing process for establishing and operating CLS facilities.

Current Status:
TRAI's recommendations are currently under consideration by the Indian government.
The final framework might differ slightly based on further consultations and stakeholder feedback.

Benefits of the Proposed Framework:


Clarity and Transparency: A clearer and more transparent regulatory environment for submarine cable landings.
Enhanced Security: Emphasis on robust security measures to protect critical infrastructure.
Streamlined Approvals: Potentially faster and more efficient approval processes for CLS establishment.
Increased Investment: A more attractive environment for investment in submarine cable infrastructure.

Que. 454 The Geminid meteor shower, known for its fast and bright meteors, is associated with which of the following celestial objects?

1. Comet Hale-Bopp
2. Asteroid 3200 Phaethon
3. Dwarf planet Pluto
4. Neptune

Correct Option - 2
The correct answer is option 2.
Key Points
Geminid meteor shower: The Geminid meteor shower is one of the most popular meteor showers to observe because it's known for its
fast, bright meteors. It peaks around December 13 and 14 of each year.

The meteors in the Geminid shower come from the object 3200 Phaethon, which is thought to be an asteroid composed of rock
and ice. As Phaethon orbits the Sun, it leaves behind a trail of dust and debris.

When Earth passes through this trail of debris, the debris collides with Earth's atmosphere and burns up, creating the meteors
that we see as the Geminid shower.
261/262
The Geminid meteor shower is usually visible for a few days before and after its peak. The best time to view the shower is typically
from midnight to dawn local time, when the constellation Gemini is high in the sky.

262/262

You might also like